Location via proxy:   [ UP ]  
[Report a bug]   [Manage cookies]                

Untitled

Download as pdf or txt
Download as pdf or txt
You are on page 1of 572

CRA CK THE

CORE EXAM
VOLUME 1

9 ^ » (2 0 2 2 ) E D I T I O N
WRITTEN & ILLUSTRATED BY:

PROMETHEUS LIONHART, M.D.


L eg a l S t u f f

R e a d e rs a r e a d v is e d - t h i s b o o k is NOT t o b e u s e d f o r c l i n i c a l
D EC ISIO N M A KING . H UM A N E R R O R D O E S O C C U R , A N D IT I S Y O U R
R E SPO N SIB IL IT Y TO D O U B L E CHECK ALL FACTS PR O V ID ED . TO THE FULLEST
EXTENT O F T H E LAW, TH E A U T H O R A S S U M E S NO R E S P O N S IB IL IT Y FO R ANY
IN JU R Y A N D /O R DAMAGE TO P E R S O N S O R P R O P E R T Y A R IS IN G O U T O F OR
R E L A T E D T O A N Y U S E O F T H E M A T E R I A L C O N T A I N E D IN T H I S B O O K .

A re R e c a l l s In T h is B o o k ? ABSOLUTELY NOT.

T he A u th o r h a s m a d e a c o n s id e r a b l e e f f o r t ( i t ’s t h e o u t r i g h t
P U R P O S E O F T H E TEX T), TO S P E C U L A T E H O W Q U E S T IO N S M IGH T BE A SK E D .
A P h D in B IO C H E M IS T R Y CAN FA IL A MED S C H O O L B IO C H E M IS T R Y T E S T OR
B IO CH EM SE C T IO N ON TH E USMLE, IN S P I T E O F C L E A R L Y K N O W I N G MORE
B IO CH EM T H A N A M ED ICAL S T U D E N T . TH IS IS B E C A U S E T H E Y A R E N O T U S E D
TO M E D IC IN E STY LE Q U E S T IO N S . T H E AIM O F T H I S T E X T IS T O E X P L O R E T H E
LIKELY ST Y L E O F BO A R D Q U E S T IO N S A N D IN C L U D E M ATERIAL LIK ELY TO BE
C O V E R E D , IN F O R M E D BY T H E A B R ’S STUDY G U ID E.

T h r o u g h o u t th e text th e a u th o r w il l a ttem pt to fa tho m t h e m a n n e r

O F Q U E S T IO N IN G AND IN C LU D E TH E C O R R E S P O N D IN G H IG H Y IELD M ATERIAL.


A C O R R EC T ESTIM ATION W IL L BE W H O L L Y C O IN C ID EN TA L.

HUM OR / PR O FA N ITY W A R N IN G

I U S E P R O F A N I T Y IN T H I S B O O K . I MAKE “ G R O W N U P ” JO K ES. PR O B A BLY NOT


A G O O D IDEA TO REA D T H E BOO K O U T L O U D TO S M A L L C H IL D R E N O R E L D E R L Y
MEMBERS OF YOUR CHURCH / TEMPLE / MOSQUE. NOW IS N O T T H E O P T I M A L
T I M E T O B E R E C R E A T I O N A L L Y O U T R A G E D - I’ M J U S T T R Y I N G T O M A K E T H E B O O K
READABLE AND FU N.

I A LSO TALK A M E SS O F S H IT A B O U T D IF F E R E N T M ED ICAL S P E C IA L T IE S . 1 DO


TH IS B EC A U SE R A D IO L O G ISTS ARE TRIBA L, AND STRO K IN G THAT U R G E T E N D S
TO CALM P E O P L E . P R O B A B L Y NOT A G O O D IDEA TO REA D T H E BO O K O U T L O U D
T O M E M B E R S O F Y O U R F A M I L Y T H A T A R E IN S P E C I A L T I E S O T H E R T H A N
R AD IO LO GY . THE TRUTH IS I R ESPECT ALL THE O TH ER SU B -SPEC IA LTIES OF
M E D IC IN E (E V E N FA M ILY M E D IC IN E ) - T H O S E A RE DIRTY J O B S (PO O P, P U S , &
NOTE W R IT IN G ) BU T S O M E O N E N E E D S TO DO TH E M .

If YOU S T IL L F E E L T H E D E S IR E TO EM A IL ME A B O U T H O W I
H U R T Y O U R F E E L I N G S 8e R U I N E D Y O U R L I F E -

P L E A S E D IR EC T T H O S E C O R R E S P O N D E N C E S TO:

Id o n tg iv etw o sh its@ farts.n et


W h a t M a k e s Th is Book U n iq u e ?

S hort Answ er — F u c k in g Ev e r y t h in g

How ? — W e l l f o r s t a r t e r s , M y b o o k s a r e n o t w r i t t e n by 30 re sid e n ts,


2 0 FELLOW S, a n d lO TURBO NERD ATTENDINGS.

In s t e a d t h e b o o k s w e r e w r i t t e n by M e. I do ev er y th in g .
I WRITE EVERYTHING. 1 DRAW EVERYTHING. I U SE NO SLAVES.

F u l l D is c l o s u r e : W h il e w r it in g t h e tex ts I do ty pic a ll y ca ll f o r t h th e
A v a t a r o f K h a i n e (t h e b l o o d y h a n d e d g o d o f w a r a n d s t r i f e ) t o p o s s e s s
my c o r p o r e a l fo r m o n t h e m ortal pl a n e o f e x is t e n c e . T h is may see m
e x c e ssiv e , b u t I ’v e f o u n d i t t o b e t h e m o s t e f f i c i e n t w a y t o p u r g e t h e
h e r etic s , launch my v e n g e a n c e , an d d e liv er m e r c il e s s ju s t ic e u p o n th e
E N EM IES O F FREEDOM AND LIBERTY. IT A L S O A L L O W S A U N I F O R M W R I T I N G S T Y L E
TO D ER IV E A S I N G U L A R V IS IO N F O R T H E U LT IM A T E T E S T P R E P A R A T IO N R E S O U R C E .

T he im petu s fo r th is book w as not to w r ite a r e f e r e n c e text or sta n d a rd


REVIEW BOOK, B U T INSTEA D, A STRATEGY MANUAL FO R S O L V IN G M U L T IP L E CHO ICE
q u e st io n s fo r R a d io l o g y . T he A u t h o r w is h e s to c o n v ey th at th e m u l t ipl e
C H O I C E T E S T I S D I F F E R E N T T H A N O R A L B O A R D S I N T H A T Y O U C A N ’T A S K T H E S A M E
K I N D S O F O P E N - E N D E D E S S A Y - T Y P E Q U E S T I O N S . “ W H A T ’S Y O U R D I F F E R E N T I A L ? ”

Q U E S T I O N I N G T H E C O N T E N T S O F O N E ’S D I F F E R E N T I A L W A S T H E O N L Y R E A L
Q U ESTIO N ON ORAL B OA R DS. N O W THAT S IM P L E Q U E S T IO N B EC O M ES NEARLY
IM P O S S I B L E TO FO R M A T INTO A M U L T I P L E C H O IC E T E S T . IN S T E A D , T H E F O C U S F O R
T R A IN IN G F O R S U C H A T E S T S H O U L D BE ON T H I N G S THAT CAN BE A S K E D . F O R
E X A M P L E , A N A T O M Y F A C T S - W H A T IS IT? . .. OR... TRIVIA FA CTS - W H A T IS T H E MOST
COMMON LOCATION, OR AGE, OR A SSO C IA TIO N , OR S Y N D R O M E ? ... OR... W H A T ’S T H E
NEX T S T E P IN M A N A G E M E N T ? T H I N K B AC K TO M E D I C A L S C H O O L U S M L E S T Y L E , T H A T
IS W H A T Y OU A R E D E A L I N G W I T H O N C E A G A I N . IN T H I S B O O K , T H E A U T H O R T R I E D TO
COVER ALL THE MATERIAL THAT C O U L D BE ASKED (R E A S O N A B L Y ), A ND T H E N
APPROXIMATE HOW Q U E S T IO N S MIGHT BE ASKED ABOUT THE V ARIO US TOPICS.
T h r o u g h o u t t h e b o o k , t h e a u t h o r w il l in t im a t e , “t h is c o u l d b e a s k e d like
T H I S , ” A N D “ T H I S FACT L E N D S I T S E L F W E L L TO A Q U E S T I O N . ” I N C L U D E D IN T H E
S E C O N D V O L U M E O F T H E S E T IS A S T R A T E G Y C H A P T E R F O C U S I N G O N H I G H Y IE L D
“ B U Z Z W O R D S ” THAT L E N D W E L L TO C E R T A IN Q U E S T I O N S .

T h i s i s NOT a r e f e r e n c e b o o k .
T h is b o o k is N O T d e s ig n e d f o r p a t ie n t c a r e .
T h is b o o k is d e s ig n e d f o r s t u d y in g s p e c i f ic a l l y f o r m u l t ip l e c h o ic e
T E S T S , C A S E C O N F E R E N C E , A N D V I E W - B O X PI M PI N G / Q U I Z I N G .
Radiology Master of Sport - National Champion
Americans love to fight. Americans love the champion.

This right here is about recognizing a commitment to being the best. Being the best, standing on the
top stand - and more importantly the quest to stand on the top stand and wear the yellow medal is
what I want to recognize.
The quest is very lonely. They say it is lonely at the top - that is not true. I know that on my quest, 1
didn’t have any friends on the way there - nobody was getting up at 4am with m e... nobody, not one
person. It was just me - alone. It is lonely getting to the top and that is where most people fall off.
It’s Friday and everyone else is going out, or there is a party or whatever - I’m tired and I want to
sleep in. I was on call last night- it is just not worth it. For me that is not what life is. Life is about
being the best person you can be in whatever it is you are doing. That does not have to be Radiology,
but because you are reading this book well that is what it means for you now.
You make the quest. If you get the top score on the exam - I'll be putting your name in this book next
year and you can motivate the next legend of tomorrow. Plus, I ’ll give you some money and have an
enormous lion trophy made for you.
The most important thing is not actually winning. People think I only care about winning because I
rant and rave about how only the gold medal count - but the quest is what I really value. I'm not
training for silver. I'm training for gold. The quest for gold is more important than someone handing
you a gold medal. The katana sword of the black dragon society cannot be stolen - it can only be
earned. Outcomes really don’t matter, because the truth is in this life you can do everything 100%
perfect and still fail and you can do almost everything wrong and still win. If you don’t focus on
outcomes and instead the quest itself you will get something much greater — besides a high
probability of passing the test - what you can gain from the quest, the ability to really dedicate
yourself to something 100% - it is a skill you can use for the rest of your life. This is the way to
always achieve victory. Once you know the way, you can see it in all things - as the samurai say.

2 0 1 7 C h a m p io n 2 0 1 8 C h a m p io n 2 0 1 9 C h a m p io n

Dr. Gary Dellacerra, D.O. Dr. Thomas Pendergrast, M.D. Dr. Nick Broadbent, D.O.
- Hofstra - - Wake Forrest - - University of Illinois
(formerly North Shore -LIJ) in Peoria -

2020 C h a m p io n 2021 C h a m p io n 2022 C h a m p io n

Dr. Oskar Kizhner Dr. Matthew Kluckman May the Mightiest


- US Navy - - US Air Force - Warrior Prevail
Bethesda, Maryland San Antonio Uniformed Services

6
I FIGHT FOR THE USERS

-TRON1982
fROMCTHEUS
Liomhart, m.t>.
Preparing for boards will be one of the most difficult and stressful times in your life. During your
preparation it is very easy to become overwhelmed with despair. Especially, when you start to come
to terms with how much information is potentially testable. So much fucking trivia...

( .......................... T

T ^ 7 .......
.. . .......

1
(>
■ .. -
1
i I
T"
f T ^.. 1
1 Jc_ .....I

i ' 1
T T .... n r
.. 1 I

■'if

My idea for this first chapter is to help you manage this great challenge. I’m going to share with you
all my secrets. All the strategies that 1 employ in my life to battle the forces of evil. You will learn to
embrace the daunting difficulty and repel the forces of evil. I’ll share with you my understanding of
emotional utility as you harness your intense venomous hatred not only for the external forces
responsible and those indifferent to your plight, but also for your own internal weakness.

Do not underestimate the difficulty of the task.

Your focus must be relentless. We will not train to be merciful here. Mercy is for the weak.
Here, in the streets, in competition. A multiple choice question confronts you, it is the enemy.
An enemy deserves no mercy. You will become the corporal manifestation of vengeance.

‘‘First, don’t wait - attack as soon as possible. ”


“Second, even i f the fight isn ’t going to plan - never show your anxiety. ”
“Third, .... Never Give Up. ” - Nomura Tadahiro , 3x Olympic Gold Medalist
S E C T IO N 1:
DETAIL A S S E S S M E N T
^ AND P L A N N IN G

Before we get started with the “when” and “how” o f the exam you need to have a clear
understanding o f the “what. ” Let us begin with the most important topic - “passing. ”

The official stance is that everyone can theoretically pass the examination - which sounds good if
you are simply testing for safety and competence. The problem occurs when you bring in
professional statisticians and test metric experts (which all professional exams do). These people
insist on creating a normalized curve to validate the exam. An examination with 100% pass rate
is statistically invalid (it wasn’t sensitive enough to detect the “dangerous radiology trainees”).

If they passed everyone they would totally invalidate their own exam. They would risk public
perception perceiving the exam as a cash grab disguised as a formality. If they failed more than
20 or 25% they would have a full revolt to deal with (lead by Program Directors who want you
reading nodule follow ups, not studying for a repeat exam). Let us not forget the entire reason the
US boards were restructured was to eliminate off service study time for the more senior residents
(oh sorry, I meant “trainee” or maybe “learner”). They can’t set a hard number or they risk one
o f these two scenarios. So, like every other standardized exam you’ve ever taken, it’s curved.
They may not “curve” the exam in the traditional sense, but questions will be removed until the
statistics provide an acceptable yield o f passing and failing scores. This is standard operating
procedure on any exam o f this magnitude.

Which leads me to my second point. Frequently I am asked “what do I need to score to pass ? ”
Or “how many can I miss ? ’’ You can’t think about this exam as requiring a certain raw score
(even though they will give you one - constructed via space magic). You can’t say I got 9/10
correct so 1 passed. What if everyone else got 10/10 right ? You just got the lowest score in the
country. If anyone is gonna fail it is you - despite getting 90% o f the questions right. You need
to think about it as your score relative to the rest o f the room. Historically around 10% o f people
fail, and 1-2% o f people condition physics. So, if you score in the 15th percentile or higher you
are probably safe.... probably.

Another common question is, “how many subjects can I condition ? ” On the old oral boards
there was a certain number o f subjects that you could condition (fail) without having to retake the
entire exam. You would just have to come back in a few weeks and be tested on that subject (or
subjects) to meet criteria for a pass. The original official statement on the CORE exam was
something similar, that you could fail up to five categories. However, when no one ever
conditioned anything other than physics after 5-6 years people started to notice that was probably
bullshit. Now, there has been an official statement that onlv physics can be conditioned. Your
clinical totals are lumped into a raw score and then compared to a “predetermined” minimal score
(that is later presumably adjusted to fail around 10% o f people). This may sound bad, but it
presents a potential opportunity to bolster weaker sections with stronger ones. As such, I ’d
strongly advise you to capture all low hanging fruit (don’t ignore the silly parts like “non­
in terpretive skills”).

You may be thinking to yourself “That doesn’t sound hard. I ’ve never scored bottom 10% on
anything. ” Sports psychologists will often say “the greatest enemy o f future success is previous
success.” People are easily seduced by the fruits o f their labor without remembering the labor
that was originally necessary.

Do Not Underestimate Your Opponent - this is a central tenet in the art o f war.

10
I’ll repeat this critical point — the art o f war cautions greatly against underestimating your enemy.

You haye neyer competed at this level before. Your shelf exam scores , USMLE, etc... don’t
mean shit because you were competing against Family Medicine , Peds, Psych, etc.... This time
you are competing against Radiology Residents only. Yes, it is less competitive compared to the
“glory days” but Radiology still has a lot o f smart people in it. Smart enough to realize how to
avoid the worst parts o f medicine anyway. Plus, you are dealing with an extremely motivated
group. Most reasonable people are terrified to fail this exam and will therefore be putting
considerable effort into passing.

But Prometheus!? I heard someone say "allyou need to do is take call to pass. ”

Yes... someone did famously say that. Everyone has an agenda in this life. My agenda is to help
you pass, and perhaps find happiness in this world. Other people... perhaps they want you to take
call for them.

In previous editions o f this text, I wasted words making a plea to those in power to simply write a
fair test on intermediate level topics. Unfortunately, people that are bom square rarely die round.
It requires focus to break free o f your intrinsic programming. These people aren’t capable o f that.
They can’t help themselves - and so the spirit o f the warrior will continue to call on me. I have no
choice but to answer.

The Goddess o f Rhamnous isn’t finished with me y e t....

She isn’t finished with you yet either my friends. We have much work to do. So let us waste no
time pleading with the enemy. We will ask for no mercy and we will show no mercy.

Let us switch gears and discuss the structure o f the exam. The exam is a multiple choice test
consisting o f around 600 questions. The punishment is administered over two days, the first is
typically longer than the second (7.5 hours and 6 hours respectively). They give you 30 mins
“break time” but that is built into the exam. You can take this 30 mins in any interval or ratio you
want - (example — you could take thirty 1 min breaks, six 5 min breaks, etc...).

The CORE Exam covers 18 categories (sometimes 17 - in Chicago). The categories include:
breast, cardiac, gastrointestinal, interventional, musculoskeletal, neuro, nuclear, pediatric,
reproductive/endocrinology, thoracic, genitourinary, vascular, computed tomography, magnetic
resonance, radiography/fluoroscopy, ultrasound, physics, and safety. This book is outlined to cover
the above sections, with the modalities o f CT, MRI, Radiography, Fluoroscopy, and Ultrasound
integrated into the system based chapters as one would reasonably expect. Radioisotope Safety
Exam (RISE) trivia is also distributed appropriately in my various texts.

On the exam. Physics questions are integrated into each category with no distinct physics
examination administered. However, the physics section is still considered a virtual section, and
you can fail / condition it. In fact, the physics portion is actually the overall largest section. My
book “The War Machine” is dedicated to review o f physics and the various non-interpretive skills.

II
S E C T IO N 2:
W aging W ar ^

In the previous section, I attempted to introduce the idea that the exam should not be viewed as
a test in which a certain “minimum score” is required to pass. Instead, I want you to look at the
exam as a contest in which you must finish in the top 85% to achieve victory.

I ’m not trying to scare you. I ’m just trying to help you understand that you must take this
exam seriously. It’s tough and smart people fail it every year.

Over the next 6-12 months, when you are out partying and horsing around - remember that
someone else out there at the same time is working hard. Someone is learning, getting smarter,
and preparing to win.

Life is a struggle. Everybody fights - and that doesn’t mean getting punched in the face. That
means not hitting the snooze button, managing your personal relationships, dealing with
disability, trying to do the right thing for your children - e tc ... e tc ... and everyone gets asked
the same question - "Can I move on, or am I going to give up on my dreams ? ”

Transform a threat into a challenge. A life free o f struggle is a life free o f meaning. We are
bom for battle. Battle against the beasts o f the jungle. Battle against the rival tribes. A n d ...
y e s... battle against each other on the glorious field o f multiple choice questions. I encourage
you to embrace this as a great opportunity to weaponize your will.

When I was little my mother would tell the story o f how my Father, how do you do that ?
father worked 72 hours straight in the limestone quarry. Do what ?
Stay awake for 3 days and work
They kept offering overtime shifts, so he just kept staying.
I ’ve done it fo r 4 days.
He seemed to have a super natural ability to endure fatigue How?
and discomfort. It's not that hard.

You must leam to “harness y o u r hate. ” Then he leaned in real close and
whispered in my ear
"'Hate was all I knew, it built my world, it imprisoned me,
taught me how to eat, how to drink, how to breathe. “what makes you angry?’*
Heroes are not born from happiness.
I thought I'd die with all my hate in my veins. ” - V

I ’m trying to teach you to access the awesome utility o f activating your adrenaline pathways. In
times o f crisis there is a tendency towards sadness. Sadness is the vampire o f energy. Sadness is
not useful. It does nothing. If you are feeling overwhelmed then focused rage is the emotion you
want to cultivate. Turn off the Coldplay. Turn on the DMX (start with "Intro" to It's D ark A n d H ell Is
Hot.) This is the fiael you need to drive the engine o f domination. You must become one o f those
science fiction radiation monsters that only gets stronger the more it is attacked. Every bomb they
drop on you just makes you stronger. The capacity to suffer is perhaps the greatest superpower.

The time has come my brothers (and sisters) to show no mercy.


Retribution is at hand.
Let fiiry guide you and let vengeance be your song.
Thank all the Gods o f War for providing you with this opportunity to prove your greatness.

12
S E C T IO N 3:
Attack B y str a ta g em
“Victorious warriors win first and then go to war ”

As we begin our discussion of specifics, I will now share with you my 3 Promethean Laws for success
on this exam and in life. These laws are the beginning of understanding “the way,” and once you
know “the way” you will begin in see it in all things.

m LAW 1 - You must do anything your opponents are willing to do.

Example: If everyone is reading a certain book, then you must also. If everyone is using a particular
q-bank (or banks) then you must also. If everyone is studying a certain amount per day then you must
also.

m LAW 2 - You must do things your opponent is NOT willing to do.

Terry Brands once told me the secret to being a world champion is actually very simple - all it takes is
doing something no one else is doing - and then do it every day. It doesn’t even matter what it is, as long
as no one else is doing it. This principal has more to do with developing a mental edge than cultivating a
specific skill set.

Examples - Setting your alarm at 2 am to study for 20 minutes while everyone else is asleep. Listening
to lectures while driving to work, while in the shower, while pooping. Taking short showers. I time my
showers - never longer than 3 mins. I turn the water on and get in (while it’s freezing cold) - 1 play the
game of enduring the cold shower as it heats up. If my 3 min timer goes off and I have shampoo in my
hair... I’m going to work with shampoo in my hair. This may sound crazy but it is a method for
callusing the mind. It helps me remember my priorities. I want to win more than I want to enjoy a
warm shower. Warm showers don’t help me win. Less extreme examples would be never touching the
snooze button. Not watching any tv, movies, news, comics, etc... until victory is achieved.

This is not punishment - and should not be looked at as such. You are simply reminding your mind
who is actually in charge. Later in the chapter we are going to discuss “The 4th Way” and the idea of
the many “I”s. By subjecting yourself to discomfort you are letting the weaker members of your
consciousness know they are not in charge.

These are habits of the ultra-successful. People who are competing for Olympic gold medals tend to
know dick about anything other than their sport. I remember seeing an interview with a female gold
medalist (I don’t recall the sport) but the reporter asked her who she was supporting in the presidential
election and she didn’t even know who was running. She acted embarrassed and people watching that
probably thought to themselves “Dumb jock! Doesn’t even know who is running for President!” The
people thinking that will never understand what it’s like to achieve an elite level of success at
anything. Knowing political trivia doesn’t help her win at gymnastics (or whatever) and time spent on
anything other than her craft is wasted. Consider treating your preparation for this exam as if you were
in training camp for an Olympic title (or world championship boxing / MMA fight). This mentality is
very useful for maintaining your priorities. So remember - while you are wasting time the enemy is
training and you can’t allow them to work when you are not (see Law 1). Minimize distractions -
Maximize productivity.

13
m LAW 3 - Perform ance on “Gam e D ay” must equal perform ance in practice

Let us take a few m inutes and discuss


“test anxiety. ”

"A nxiety” is a dishonest w ord though isn’t it?


Let’s use a more accurate word. FEAR

N o ... I’m not afraid. I ’m way too macho to


be afraid o f things. There is a difference
between being afraid, and being a coward.
Only crazy people don’t experience fear. It’s
ok to be afraid. If you let that fear dominate
your decision m aking... that is when you run In brightest day, in blackest night,
into problems. No evil shall escape my sight
Let those who worship evil's might.
The inability to control fear is the root cause o f the Beware my power, Green Lantern's light

“text anxiety ” problem. Fear more than anything Members o f the elite Green Lantern Carp
else keeps people from achieving their full potential harness the power o f “w ill’’ to combat the
evil forces o f ‘fe a r ’’
in sports, life, business - in everything.

Fear o f what?? Fear o f failure right? Nope... That is not it. That is not w hat you are afraid of.
It’s not faihng. It is the fear o f a perceived threat to your ego, your self esteem. The fear o f
looking bad and getting em barrassed in front o f your peers. The fear o f being exposed as a
fraud. N ot really smart enough to be a Radiologist. “Im poster Syndrome ”

Agree?

N o w ... let us talk about how to conquer fear and channel all that psychogenic energy into a
powerful m otivated force.

14
S E C T IO N 4:
W eaknesses and S trengths
“C o n fro n tin g fe a r is the destiny o f the Jedi. ”

First we must understand that fear in itself is not something to be ashamed of or embarrassed by. Without
fear there can be no courage. Courage is only possible when one confronts and overcomes fear. It is no
coincidence that the Lion is the symbol for courage. It is also no coincidence that you will find the
iconography of courage throughout these texts.

“You may think that you are scared. But you are not. That is your sharpness. That s your power ” - Drummer

Now 1 w ant to take a few m inutes taking some trouble with w ords, to help explain the various
strategies for controlling fear, and obliterating the negative causatum o f anxiety on your game
day performance.

Controlling fear involves two things: (1) choice, and (2) strategy.

The “choice” is the conscious decision to confront your fear. Once you have made the decision
to confront it, you can then begin to im plem ent a “strategy” . The strategy that I am going to
suggest is to not consider this as an effort to elim inate fear. Fear is normal. Only crazy people
don’t have fear. The goal is to increase courage.

^ M echanism 1: Generating Courage Through Purpose

One powerful mechanism to increase courage is to have a goal or purpose that is worthy. For
example, a m other may run into a burning building to save her child. Does she do this because
she is no longer afraid o f fire? Or does she do it because her purpose as a m other out weighs the
fear and allows her to generate extreme courage?

Purpose ► Courage

^ M echanism 2: Games are Best P layed as Games.

Another mechanism for overcom ing fear is to begin to view the w orld from a certain point o f
view. A point o f view sim ilar to that o f the Court Jester.

The point o f view o f the Jester (or Joker) is to view not ju st the various social institutions, but all
formations o f the natural w orld as gam es. You m ust be careful o f the w ord “gam e.” W hen
people use the word “gam e” they often m ean “frivolous.” There can be im portant games.

Imagine that you were playing a game o f Super M ario Brothers. W hat if you believed that if you
fell into a cavern or were bit by one o f those plant things (or a turtle) that you w ould actually
die ? Do you think this w ould make you better at the gam e ? You’d be terrible at it. You
w ouldn’t be able to relax, you w ould second guess every m ovem ent, and ultim ately your
performance w ould be awfiil. All because you believed this gam e was real and the dangers
w ithin it were real.

15
^ M echanism 2: The Perspective o f the Joker - continued...

Hopefully you can begin to understand the benefits o f this strategy. In viewing stressful
challenges as games you can dim inish the effects o f fear. This is not the only benefit though.
The sincerity that exists in the nature o f play - the act o f doing things simply for themselves
actually improves your performance. Later we will discuss methods to improve focus and
concentration - the ability to “enjoy” the thing you are doing is also very useful in com bating
procrastination. Games are best p la yed as games.

^ M echanism 3: “The Art o f N ot Giving a Fuck” .

In discussing the perspective o f the Joker, people will often becom e upset and insist the
consequences o f failure are very real. That the fear o f failure is justified. Rem ember that your
fear lives in ju st one place - your head. Fear o f a thing that h asn ’t yet occurred - “Negative
Imagination ” is the cardinal sin o f the 4th Way (discussed later).

Confront the fear rationally. Are you afraid o f being em barrassed? I can only be em barrassed if
the people I think I’ve em barrassed m yself in front o f have my respect. If I don’t respect you -
your opinion means very little to me. As the Lebowski says “th at’s ju st like your opinion m an.”
Someone worth respecting would understand the difficulty o f the task and not judge you poorly
for failure. The only people who never fail are the people who d o n ’t try.

Remember, the Joker sees the whole w orld as game playing. That's why, when people take their
games seriously - the people who make stem and pious expressions - the Joker can’t help but
laugh. It’s hard to not laugh w hen you see people pretending that w hat they do is so very
serious. As Bill Hicks w ould say “It’s Just a Ride.”

I also hear this a lot: “ if I fail the test I ’ll lose my fellowship.” First o f all, you don’t need one -
the necessity o f a fellowship is a lie propagated by the institution that wants you as a slave for an
extra year. Even if you really w ant to do one - you w on't lose it. They w on’t even know you
failed, and passing is not required to be accredited at hospitals or w ork as an Attending.
Probably some o f your Attendings are international grads who haven’t even sat for the CORE
exam yet - I know some o f mine were. No one gives a shit - they ju st want you as a slave. If
they “take” your fellowship away they m ight have to do your work - and that w ould defeat the
entire purpose o f having fellows. There is nothing really on the line other than your ego. The
ego is the enemy o f excellence. I w ouldn’t waste time defending it.

This idea couples well with John D anaher’s “parable o f the plank. ” Danaher tells the story o f a
Daredevil who w alked across a plank positioned between two tall buildings. Those who
w itnessed the perform ance were am azed by the D aredevil’s agility. A t the end o f the show the
same plank was brought out and placed on the ground. Spectators were allow ed to w alk it and
found it to be very easy, though none were willing to try it when it was again placed upon the
two tall buildings. It was the same plank. The only thing that changed was the perception o f
danger. There is no danger. It is ju st a game. Games are best p la y e d as games.

16
S E C T IO N S:
Var iatio n s a n d A daptability

I have many hobbies. Disco Dance, Ping Pong, Sunbathe.

But my primary hobby is trying to be the best version of myself - maximizing my potential as a human
being. Since this is a competition, I thought it might be helpfijl to share with you some of my ideas on
this topic.

I’ll start off by saying that I can’t necessarily help you be “the best.” What we are talking about here is
being “the best you.” We are talking “the quest” - the struggle to be the best version of ourselves. I
like to imagine that people are bom as various shaped and sized canisters. Not all of these canisters
will hold the same amount of liquid. “All men were NOT created equal.” It’s just a fact that some
people are better at certain things than others. Some people have greater potential than others. We
can’t all be Tony Stark. The point is to try and be the best version of yourself The best you. You
want to fill that canister all the way up to the brim. Don’t be the half full bottle, that is only half a life.

M ethod 1: M ental Optim ization - The Sly Man

Gurdjiefif’s idea of “The Sly Man” - sometimes referred to as “The Forth Way,” is a complex framework
that can help you understand yourself and maximize your potential. I’ll give you an introduction and then
we can discuss how this might be helpfiil for your preparation. There is a lot to this - I’m just covering
the basics.

Gurdjieff has several basic tenets that are necessary to discuss before you can apply the teaching.

1. Nearly every human exists in a semi-hypnotic waking sleep. If you have played video games you
may be familiar with the term “NPC” or non-player-character. This would be a character controlled
by the computer who simply reacts to its environment. It might seem alive because it chases you
around, but it is really just reacting and following a pre-recorded program. It isn’t consciously
making any choice, it simply reacts. People in this state can be thought of as “mechanical.” Just
think about the last time something “made” you angry or sad. It just sorta happened didn't it ? You
had a stimulus and you reacted reflexively with a response.

• Stimulus; Someone disagrees with your well informed political opinions.


• Response: You become angry and compare them to the bad guys from WW2.

2. While in this state of semi-hypnotic waking sleep you have no free will. Everything that you do is
simply a reflex. You are attracted or repelled by influencing forces in your environment. Things
happen to you - and you react. The culmination of these actions and experiences makes you the
person you are today. You had no choice in the matter. When it rains, the sidewalk becomes wet.
You are the sidewalk.

3. Humans are liars. The study of man is the study of the lying animal. No other animal practices
dishonesty in the way we do. We are dishonest to others, but mostly to ourselves. We lie to ourselves
about how we are perceived and how others perceive us - the defense of the ego is paramount. We lie
to ourselves about our influence over things and even the true nature of our reality. The entire concept
of cognitive dissonance, holding contradictory beliefs simultaneously, is a great example.

17
Once you accept those things, only then are you capable o f becoming awake. You cannot wake up
until you realize that you are asleep. Unfortunately, many people have no desire to wake up - they
enjoy the lies. The lies bring them comfort. As Morpheus said in the Matrix most people are not
ready to be unplugged. They are so hopelessly dependent on the system, that they will fight to
protect it. It is understandable, I ’m still pissed off that Santa Claus wasn’t real.

The forth way offers a pathway to free will, but you have to wake up first and that is not an easy
thing. There is a constant pull back to the sleeping state. Don’t believe me ? Let us try the self
remembering exercise. Concentrate on being 100% present in the moment. Feel your socks. Feel
the chair under your butt. Feel the air in your lungs. Now - while being present in this moment,
detach yourself - try and imagine yourself from the third perspective. See yourself as a fly would see
you. When you try and do this - you will feel yourself become unstuck - “separate” from the rest o f
the world. This is self remembering. The sensation o f existing separate from the world. Like
changing from first person to third person perspective. N o w .... try and hold that state. It's difficult.
You will start to lose focus and become distracted by the chattering of your mind. Constant fucking
daydreaming. A constant pull to sleep.

The capacity to maintain this state requires “work.” In fact, Gurdjieff refers to this as “the work.”
You must apply constant resistance against the impulse to sleep. It takes practice - you have to
really try and do it. The more you do it the easier it gets (like everything). Be present in the moment
and observe your own thoughts. You are a machine - a reactive machine. Study yourself without any
need to defend your actions or ideas. They aren’t actually yours anyway, so there is no need to
defend them.

The second exercise necessary to “wake” is the ability to recognize in yourself the many versions o f
you. For most people they think o f themselves as a single unit. In fact, you refer to yourself as “I.”
But ask yourself which “I” are you referring to ? The “I” that wants to get up early ? Or the “I” that
wants to stay up late ? The “I” that wants six pack abs ? Or the “I” that wants to eat ice-cream. The
“I” that wants to pass this exam ? Or the “I” that wants to watch youtube and not study ? Notice that
we are full o f contradictions - we are liars. We aren’t even honest about how we think. We exist as
many competing versions o f ourselves, all with separate agendas. There are many “I”s.

If you attempt to “self observe” what you are thinking you will be able to begin to recognize which
one o f these voices is doing the talking. You may even be able to recognize why certain things give
you anxiety or make you angry. Is this my limbic system talking ? Is this my frontal lobe ? What am
“I” reacting to ?

There are so many benefits to this understanding. It can free you o f guilt and shame. It can help you
control your emotions. It can help you improve your interpersonal dynamics. It can allow you to
enter the flow state and concentrate deeper and longer (that’s what she said). When you approach
yourself as a machine you can use the same logic an engineer would use in identifying problems and
appropriate solutions. Understanding your various internal agendas and reflexes to align them
towards a single goal. Even 15 minutes a day practicing the self remembering exercise will
massively improve your ability to focus for longer periods o f time.

There are three traditional ways to develop “super powers,” assuming you can’t get your hands on
some compound V. (1) Extreme mental control (the way o f the Yogi), (2) Extreme emotional control
(the way o f the Monk), (3) Extreme physical control (the way o f the Fakir). All three traditional
masteries require decades o f training and social isolation. This forth way (the way o f the Sly Man)
can give you access to the same abilities in shorter time and without the need for isolation. The
capacity to truly control your mind and your body. To free yourself o f all the reflexive actions and
negative imagination. With the appropriate amount o f “work” you may also find the fourth way is a
path to many abilities some consider to be unnatural.

18
There is another facet to this - which I touched on earlier in the chapter. Some people refer to this as the
way of the Joker. Once you understand that people are machines which simply react, you may begin to
become fairly amused by the serious nature that many people take. The way they try to force and control
things they have no control over really can be very funny. The way they take credit for things that simply
happen to them. People love to brag about things. People will even brag about being tall. It is hilarious to
witness the total lack of self awareness. This was the original purpose of the court Jester - to keep the
King from taking himself too seriously.
As you start to see the “code in the matrix” you will notice that life begins to take on the appearance of a
series of games. Games that are to be played. Through self observation you will see that when you play
you are doing things only fo r the purpose o f doing them.
For example, when you listen to music you aren’t trying to get to the end of the song, or count the notes,
or derive meaning in the lyrics. You are listening to the music only for the purpose of listening. The other
thing that happens as you play, or dance, or listen to music is that you become present (mentally) for the
process. This in itself is a method to increase focus, and energy in daily life. This is a method for
minimizing day-dreaming and absent mindedness. This is the way of the Sly Man.
The Joker does not participate in the cardinal sin of the forth way - negative imagination. This is the
process of imaging a dreadful outcome in the future — worrying about things that have not happened yet.
This behavior must be self observed and halted immediately. The Joker never does this. He is fully in the
moment - he plays for the sake of playing.
Study only for the purpose of studying. Avoid the mentality of “getting through the material.” Read the
material the same way you would if you were reading a novel for pleasure. If done correctly you will find
your studying endurance and recall ability improve. Games are best played as games.

A Method 2: M ental Optim ization - N europhysiological M anipulation

Dopamine controls your ability to exert


Classic Experiment (they really did this):
effort. People think about dopamine as the
chemical that causes pleasure, but it has Rats in a cage are given a lever. If they push it they get
more to do with effort. We aren’t interested a delicious snack. The rats like to push the lever.
in pleasure. We are interested in increasing Rats in a different cage are injected with a chemical
our capacity for effort. It is true that that destroys dopamine producing cells in their brains.
Dopamine is released in large amounts They hurt these rats for science (and fun). These rats
during sex or drug use - things that feel were given a similar lever. They also liked to push the
good. But when you actually look at the lever (despite having no dopamine).
science of how it works in the brain N ow ... the real cruelty begins.
dopamine is more about motivation to The Rats were moved into a larger cage. The healthy
pursue more of something in order to relieve rats (the ones who didn’t get neurotoxin injected into
or exclude pain. This is in line perfectly their brains for science / fun) walked across the cage
with my father’s philosophy on life - that and pressed the food lever - to get a delicious snack.
there is no such thing as pleasure or
The Rats who were injected with a Dopamine
happiness in this world , only times when it neurotoxin (for science / fun) just looked at the lever.
hurts less. Our pleasure seeking behavior is They wouldn’t even move one rat length to push it.
actually motivated to relieve pain - and this They said - “fuck it dude, let's go bowling.’
is even more obvious if you’ve been around The scientists had created lazy, lazy
anyone with a true addiction. They (presumably communist) rats. ^
experience agony in the absence of the thing Point: Dopamine is Not about __
they are addicted to and seek to relieve that Pleasure, it is about
pain through that dopamine releasing Motivation.
activity (drug use, eating candy, pom, etc...).

19
Dopamine creates a bias where we are focused on “what we don’t have.

Let us take a slightly deeper look at this - so that we can


weaponize our will; and not live our lives as communist rats -
hoping someone will push the lever of Radiology Board exam
victory for us. There are two basic networks that govern how we TASK
function. "Default Mode Network” - The network of circuits in
your brain that is active when you aren’t doing anything. ‘‘Task
Mode Network” - The network of circuits that allows you (and
D e fa u l^ ^ ^ ^ ^
motivates you) to focus on a specific task. r S l
These networks should work in anti-correlation -like a see-saw. „ t .
Dopamine regulates the switching between these. Dopamine

There are 2 Types of Procrastinators:


Weaponized Anxiety
(1) Those who leverage internal stress: The people who “enjoy”
or “require” the stress of a deadline. These are the people who are Set achievable (but difficult)
goals in your study blocks -
using stress to tap into the fight or flight epinephrine system.
something you can barely get
Remember when I was talking about harnessing anger to give you
done in time. This creates an
energy to combat fatigue ? This is the same biochemical mechanism artificial deadline to cultivate
- but instead of requiring a deadline you activate this system by enhanced focus.
focusing yourself first on any number of cruelties bestowed upon Example: Complete 5 pages
you by this hatefiil world. Remember that your revenge is success. of reading with the
If you have no room for anger in you heart and you’ve lived a life construction of an
full of good times (and noodle salad) you can try to access this abbreviated outline (or
pathway by hyper-oxygenating with 20-30 deep breaths - this tends flashcard set) within 30
to active the same pathway. If you are prone to panic attacks then minutes.
maybe don’t try this. Caffeine can also help - more on this later.
(2) Those who perhaps have general apathy and don’t have enough dopamine on board. These are the
communist rats. We can’t blame everything on Karl Marx (and the liberal media), in many cases you may
have become dopamine resistant in the same way fat people become insulin resistant. Assuming you
haven’t been injected with a dopamine destroying neurotoxin (for science / fun) the solutions here are
probably two fold.

• First you can do things to increase dopamine production. There are ways to supplement this with
precursors (L-Tyrosine or Mucuna Pruriens / Velvet B ean). Dietary tyrosine is found in high protein
foods (tyrosine is an amino acid). Examples: sesame seeds, cheese, soy beans, meat, poultry - etc...
If you have schizophrenia or other dopamine sensitive conditions maybe don’t try this.
• Second strategy, and probably the more effective one, is to go on a dopamine fast. It isn’t actually a
fast, it is more like impulse training. Things that produce dopamine spikes include social media,
gambling, pornography, stress eating, etc... You can try and employ the Pareto 80/20 rule of figuring
out what 20% of your behaviors are causing 80% of your problems - with regards to compulsive
monkey mind behaviors. I discussed in the 1st method of mental optimization the way of the sly man
and the capacity to self observe. This can be put into practice here to help you figure out what to
eliminate or restrict. In most cases this is compulsively checking your phone every 5 minutes and
wondering onto YouTube to watch cat videos or argue about complex geopolitical topics you don’t
really understand on social media.

20
Method 3: M ental O ptim ization - Im pulse Control

Cognitive-Behavioral Therapy (CBT) is considered the gold standard treatment for impulse control
disorders. In particular exposure and response prevention types of CBT. Lets us take a look at a practical
application.

Pre-Game — set a timer for 5 minutes

Step 1: Get your phone out and place it on the table in front of you.

Step 2: Notice when the impulse to watch cat videos on YouTube / TikTok arises, and what thoughts and
feelings you’re experiencing in that moment.

Step 3: Practice self observing the desire to engage in the conditioned response come and go without
giving into it. People with PhDs in the pseudo-science of psychology call this “urge-surfing.”
Do this shit multiple times a day — every day.

Almond Meditation

This can also be done with a raisin if you have a Kryptonian weakness to tree nuts. This is a technique
described as “Visceral Awareness” which I find to be very helpful in training impulse control.

Pre-Game — set a timer for 5 minutes

Step 1: Place a single almond (or raisin) in your mouth

Step 2: Notice the impulse to chew it up and grab another one (or spit in out because almonds are vile).

Step 3: Practice self observing the desire to engage in the conditioned response come and go without
giving into it.

Step 4: After 5 minutes chew it up or spit it out.

Do this shit once a day — every day. Morning time is an excellent time to practice this.

“Self Remembering Exercise”


Remember most people exist in a state of semi-hypnotic "waking sleep.” Studying or performing like this
is not ideal. The "Self Remembering Exercise ” is a tool to harness power and increase your focus, but
simply practicing “being presenf ’ is still very effective.
How this works is that in the morning (or prior to entering the study hall, or exam room) you must spend
5-10 mins being present in the moment. Close you eyes and repeat to yourself “I am here in this place.”
Generate the sense of being present in the current moment. Concentrate only on your breathing. Do not
force or push thoughts from your mind. Thoughts are “sticky” if you touch them. Let them float by and
continue to concentrate on your breathing.
M e d ia te until nothing c a n disrupt you, so that you see a n d feel noth in g - e x c e p t your o w n energy.
— Shidoshi Senzo T a n a k a

21
Why waste time doing this? M editation makes my penis soft!

I agree, it’s not alpha and typically I only endorse alpha male behavior.. .but this thing really works.
Samurai meditate — it’s really not that beta. Remember how I said that all your feeling of dread or
anxiety are focused on the future - the consequences which result after you have failed ? If you center
your mind on the present, all that shit melts away. It is really just that simple. Anytime you feel anxiety
and dread building inside yourself perform the breathing exercise. Eyes closed - say “I am here in this
place,” build a sense of presence in the current moment, focus on your breathing, then return to the exam /
practice room and proceed to kick ass.
Repeat as needed. If necessary add the Then I heard the voice o f the L ord saying, “Whom
phrase “retribution is at hand” or “only in shall I sen d and who w ill go fo r us? ” A n d I said,
death does duty end” or my personal ,,,, . r. .
favorite “happiness is a delusion of the Here am I. Send m e.”
weak,” after centering yourself in the - Isaiah 6:8
present moment.

M ethod 4: M ental Optim ization - The W ay o f the Slot M achine

If you want to know the best way to trick yourself into doing something all day long, then consider the
addictive behavior of gambling. The “intermittent reward” is the ultimate method for keeping that task
oriented pathway lit up. If you won every game it would be boring. If you lost every game it would also
be boring. But... winning sometimes and not knowing when it will happen - that locks you right in. It’s
the same reason people can sit in a fishing boat for 12 hours straight (besides being drunk).
Method for applying this; Later in the chapter I’m going to provide you with a potential study work block
of 3 hours. If you are able to complete the block without distraction (getting onto YouTube to watch cat
videos), then you flip a coin. You could use this for any difficult task (a set of 100 MCQs, going through
a deck of flash cards, consistently waking up early to study, etc...). If you do the difficult thing you get
to flip the coin. If the coin is heads — you are allowed some form of reward (whatever you are into —
watch a TV show, eat a bowl of ice cream, sexual favor for your partner... etc..). This only works if you
actually deprive yourself of these rewards at baseline.
If you fail at completing the block - you don’t flip the coin. BUT, just because you completed the block
doesn’t mean you get your ice cream and / or sexual favor — it is still random and not predictable. This
taps into the cave person brain and motivates. The same way a cave man doesn’t know if he will find
apples in the next valley if he looks — and the not knowing / intermittent reward creates a drive. Casinos
figured this shit out along time ago — now it is time to make it work for you. Addict yourself to doing
difficult things.

M ethod 5: M ental O ptim ization - Blinks & Visual Focusing

“Attentional Blinks” is the term used to describe lapses of attention. This doesn’t just apply to the lack
of impulse control and wondering mind but also in the idea of “satisfaction of search.” When you make a
finding and are so pleased with your effort that you miss all the other findings. This can occur in the
realm of multiple choose as well — it is why I stress to always read every choice before answering.

There is literature to support the use of Vipassana meditation / mindfulness of breath practiced 17 minutes
a day to reduce the number of attentional blinks. Meditation is made overly complicated - but just sitting
and trying to focus on your breathing is necessary if you are serious about getting better at concentration
and having an edge on the exam over those un willing to put the time in to improve.

22
M ethod 5: M ental Optim ization - Blinks & Visual Focusing - continued

“IntentionalBlinks” is the term used to describe exactly what you’d think it describes - how often you
bhnk your eyes. There is research to support the act of intentional blinking (closing your eyes) allows
you to “reset your perception of time.” The rate of blinking is controlled by dopamine. You can hack this
relationship through “fixation focused training activity” - this is a real thing, published in literature. A
short time focused on a specific visual target, has been shown to improve the ability to focus on other
types of information.

Application:

Step 1; Physical Activity first - do a few body weight squats (hindu squats are my preference)

Step 2: Focus on an object close to you (your hand) for 30-60 seconds. You can blink *but try not to.

Step 3: Focus on an object at an intermediate distance for 30-60 seconds.

Step 4: Focus on an object far away for 30-60 seconds.

This is training - this is done daily. — Less blinking = More focus

“Open Monitoring” - When thinking about your visual system, you can focus intensely on the thing
directly in front of you or you can widen your focus and take in the entire room (the side walls, the
ceiling, etc..) - sometimes called “panoramic vision.” This visual change can be used to hack your
overall sensory process. By deliberately opening up your peripheral vision and then changing it to your
coned in central vision you can experience improved mental focus. I would recommend doing this before
every question block or prior to each study session.

“Visual Focusing” - Training to improve greater visual focus


also gives you the capacity to establish and sustain
concentration over longer periods of time.
Application:

Step 1: Sit upright in a chair. Gaze at the spiral for 1 minute.


Step 2: Close your eyes and see how long you can hold the
image of the spiral before it fades. Use a timer to track your
progress.
Step 3: Repeat the process 3-5 times daily.

** More advanced training - when this becomes too easy

Step 2: Close you eyes and hold the image of the spiral.
Rotate the spiral in your mind - first clockwise then
counterclockwise.
Step 3: Continue to rotate the image in your mind, then change the spiral to white and the background to
black. Continue to hold the image. Continue to rotate it with the colors inverted.
Once that is easy - then you can try catching a fly with chopsticks. A man who can catch fly with
chopsticks can do anything.

23
A Method 6: M ental Optim ization - The A ccountability M irror

This is a mechanism I learned from David Goggins. Goggins is the fucking man - if you d on’t
know about him, h e ’s worth a google. Similar to John Wick, he is a man o f focus,
commitment, and sheer will. He really hates him self - in a w ay that only someone who shares
the psychopathy can understand. That w eak person in the m irror is the enemy and he m ust be
held accountable.

The accountability m irror works like this. In the m orning you put sticky notes o f the things
you w ant to accomplish in that day on the mirror. We will talk later about the different types
o f goals and how to set them - but as a concept you should not lim it yourself to individual
tasks but also include goals on how you handle yourself and behave. At the end o f the day
when you are getting ready for bed, brushing your teeth (or tooth - if you are from West
Virginia), you have to look in that m irror and answ er for your progress. You either
accomplished your goals or you didn’t. You have to look at it and face it down. The only way
you can improve outcomes is to be 100% honest about the effort that you put in and the
outcomes that resulted. Hearing “you did great!” even when you didn’t isn ’t helping. Be
honest with y o u rself M ake the changes you need to make and get better. Be brutally fucking
honest — show no mercy. Hurting feelings is sometimes necessary if you want to stimulate
change, and who better to do the hurting than you.

Failing is ok. N ot adapting to failure is not ok.

W hen you fail to reach your goals you need to spend time figuring out why (self observation).
Make the necessary adaptations and try again. Repeat as necessary. Be accountable to y o urself

Method 7: Physical Optim ization - Sunlight

I know that nerds bum easily when exposed to the sun and the prim ary benefit o f being a
Radiologist is to w alk in shadow and move in silence (to guard against extraterrestrial
violence).

Seriously - you only need like 5 mins a day. Your pineal gland demands it. I don’t care if it’s
freezing cold outside - get your ass out there and look at the sun for 5 mins. Be mindful in
the mom ent and play the standing outside in the sun game. I ’ll discuss the use o f a wakeup
light later in this section - that doesn’t replace the need for actual sunlight. You don’t have
X eroderm a pigm entosum - get your ass outside.

If you are unable to get outside or you’ve been telling people “ I ’m allergic to the sun” for so
long that you have started to believe it - at the very m inim um consider a Vitamin D
supplement. A few pearls on vitam in D. Take it in the m orning (some people think it binds
up natural m elatonin and can fuck up your sleep). Take it with oil or fat (it is better absorbed
that way). Take it with a vitamin K pill (supposedly it is synergistic).

24
A Method 8: Physical O ptim ization - Sleep

I have trained m yself to sleep 4.5 hours a night by m apping and understanding my REM cycles.
People will say they "c a n ’t do that. ” People use the phrase “c a n ’t do th a t” w hen they really
mean “I d o n ’t w ant to do that because it will hurt too much. ” Callous your mind.

This takes some effort and practice. If I get 3.5 hours sleep I feel terrible. I d o n ’t really care
about “feeling terrible.” A w arrior does not com plain o f physical discom fort. The problem is I
can’t concentrate. I also feel this w ay with 5.5 hours sleep. A t 4.5 hours I find that I am readily
able to transform my w ater sw o rd .... into a fire sword.

The trick is to map your sleep patterns. There are apps that can do this. Alternatively, you can
go full retard and buy a ‘‘sleep shepherd, ” which is my endorsem ent if you really w ant an
accurate map plus the added benefits o f binaural beats.

Take two weeks and create a sleep journal. W hat you are looking for is how long it takes you to
go through two REM cycles. Then you can plot your bed time and w ake up tim e to maximize
productivity. If you do this right you can potentially get 4 extra hours a day to study.

Get Up Here
(the enemy is weak and still sleeping
earn your tactical edge)

AWAKE

Deepest

10

Hours of Sleep

After I mastered this and freed up m ultiple additional hours o f study time in the m orning I really
began to break away from the pack in m orning case conference. I ’m not going to say how many
asses I kicked, only that I kicked every single ass. W hen people ask m y secret — I w ould tell
them - “ Sleep journaling bitch. D on’t make me show you m y fucking sleep journal. I keep it in
a drawer with my other creepy journals (my poop journal, m y ear w ax journal, my belly button
lint journal).” They thought I was jo k in g ...

25
A M ethod 8: Physical Optim ization - Sleep - Continued

Sleep Induction: A nother com ponent to getting the most out o f your sleep is the abihty to
fall asleep rapidly. All that time you m ight be spending staring at the ceiling is time wasted.
Here are a few strategies:

• Temperature Optimization: Your hypothalamus will lower your body temperature in the natural sleep
induction process. You can help optimized this temperature shift by first taking a warm bath or shower
- followed by entering a cool bedroom ~ typically 60-67°F (15.6-19.4°C). This nightly hot shower /
bath ritual is also an excellent time to practice the various meditation and impulse control exercises
discussed earlier I also find it to be a good time to do some stretching - just don’t slip and bust your
ass, that would be counter productive to sleep induction. I’m actually serious about being careful if you
are going to stretch or exercise in the shower. My Uncle Jamal told me that he was doing some
exercises in the shower and he slipped and a shampoo bottle got stuck up his butt. He had to go the ER
to get it out. Uncle Jamal is a bit clumsy - because that same thing happen a few times. Like another
time he told me he was doing the vacuum cleaning and the phone rang and he slipped on an orange and
the vacuum extension went up his butt. A lot of people didn’t believe that story when he tells it - but he
said these things happen a lot (like 4-5 times). So... just be careful.

• Avoid looking at vour phone in bed and if you must install one of those blue light filters.

• 1 recommend the following nightly supplements:

• Chamomile tea has sleep inducing properties- or you can take 50 milligrams of apigenin (which is a
derivative of camomile)
• Magnesium Glycinate: You want to take it 1-2 hours before bedtime, with a dose ~ 350 mg. Taking
it with some food may decrease nausea if you have a sensitive belly. Also make sure you are getting
the correct magnesium. Magnesium citrate is a great laxative... but not so good at inducing sleep.
• 5-HTP (5-hydroxytryptophan): Doses up to 600 mg per day
• Melatonin. Doses of 0.5-5 mg taken 2 hours before your desired bedtime — this tends to give people
vivid dreams - so get ready to wake up hoping you didn’t really take a shit on your program director’s
desk (this was a recurring dream / fantasy of mine — 1 assume everyone has similar thoughts around
boards).
• L-Theanine: Doses of 400 mg per day
• GABA (gamma-aminobutyric acid): Doses of 250-500 mg

• The use of a “sleep induction mat.” It’s basically a porcupine. This is most helpful if you have a
“busy mind” - full of distracting thoughts. How this works: It’s a bunch of sharp (dull) plastic
points. You take your shirt off and lay on them. It hurts. The pain distracts you from all the random
shit you might be thinking about. Practice the self remembering exercise. In about 5 mins you will
notice that it doesn’t hurt anymore. That is because your body is releasing a low level of
endorphins. Sit up - roll the thing up - and go to sleep. Some people will tell you that it takes 30
mins to work. That's a bunch of baloney ~ 5 mins tops.

• Reveri is a free app on Android and Apple that has short (10-15 min) hypnosis protocols. This may be
helpful if you are having a lot of anxiety and that is preventing you from sleeping. It sounds like hocus-
pocus but supposedly this thing is backed by peer-reviewed research.

26
But Prometheus?! The Codex Astartes Does NOT support this action!

Every year I get at least one belligerent turbo nerd email quoting me sleep research and telling
me that I must have the p.Tyr362 His m utation - a variant o f the BHLHE41 gene - and that my
advice to sleep deprive violates both the teachings o f the Em peror Leto II Atreides and described
will o f the Em peror o f M an as transcribed by the Im perial Ecclesiarchy / Adeptus M inistorum.

Before the writing o f the annual “I need my teddy bear and 10 hours o f sleep em ail”- allow me to
clarify my points:

• This is about getting a mental edge


• This is about getting a tactical advantage from w eaponized will pow er
• It is possible to maximize your sleep efficiency through REM mapping
• I can do it - and so can other people (you m ay be one o f them)
• Reduced sleep quantity can be augm ented with im proved sleep quality - through the methods
described on the prior page - in addition to optim ized nutrition, hydration, exercise e tc ...

If my provided m ethodology (sleep m apping, diet, hydration, exercise, etc..), doesn’t w ork for
you - consider the following:

Alternative approach to overcom ing fatigue:


(Step 1) M ake a fist with your brain
(Step 2) Punch sleepy time in the face

M ethod 9: Physical Optim ization - W aking Up Ready to Fight the Dragon

I wrote a lot of words describing methods to reduce the quantity and improve the quality of your sleep.
The second part of this equation is a morning ritual that allows you to optimize productivity while the
enemy is sleeping cuddling their teddy bears.
Step 1: The morning always begins the night before. You should lay out your clothes and have your
morning study area cleaned / organized with clear goals written out. If you are going to do some early
morning cardio (which I strongly recommend) you should have your gear set out ahead of time.
Step 2: Get a bright light. They make wake up lights or you can get
something like one of those LED drawing pads (they tend to be cheap).
You need something bright - like 900 lux, and you need to shine that shit
in your face. Obviously the sun would work, but if you are serious
about summoning the Avatar of Khaine (bloody handed Aeldari God of
War) - that sun isn’t rising for several hours when you wake up — so
you need a bright light.
Step 3: Movement — if you aren’t gonna do some light cardio (you
pussy), you can at least give me 10-20 Hindu squats.
Step 4: Study First — eat later. I always earn my breakfast with
movement and productivity - build that into your ritual as well. o f Khaine craves only
battle (and reading about

27
A Method 10: Physical Optim ization - Diet

The human body is designed to operate perfectly ... in a w orld that existed 10,000 years ago.
Despite what the prim itive “rat brain” may tell you, sugar is the devil. Foosball, school, girls,
and Ben Franklin are also the devil - but that is unrelated to this discussion.

Avoiding highs and lows is key to optimal function. Avoiding IBS cramps and constant
shitting is also ideal. I ’m not going to endorse a particular diet by brand name. I ’ll ju st say
there is a lot o f data showing that a Ketogenic diet (or something similar) helps to maximize
m ental function. Having said that, Ketogenic diets take some getting used to and if you are
going to go that route consider using your allocated carbs prior to studying.

Foods with Tyrosine (foods with protein) are useful as Tyrosine is a precursor to dopamine —
which we discussed earlier is im portant to regulating your drive.

Avoid sugar - nothing good comes from sugar. Dehydration is bad - drink water.

A Method 11: Physical Optim ization - M ovem ent

You need 15 mins a day o f movement. This can be actual alpha male gym stuff, or beta male
yoga - but any form o f m ovem ent works and is necessary to m axim ize your m ental health.
W hen people say they don’t w ork out — this is like saying “I don’t brush m y teeth.” It is not
optional. No excuses. M ake your body stronger. M ake your m ind stronger.

I will say this - the more vigorous your training the more sleep your body will require. It is
super im portant that you m ove (especially during this stressfiil time in your life) - and I ’m not
suggesting that you avoid working out to sleep less — so don’t get it twisted. I ’m instead
advising you to adjust your study schedule to allow more sleep on nights after you have
worked out super hard. Optim izing physical training and sleep m anagem ent requires
flexibility, patience, and balance. Having said that — d o n ’t use exercise as an excuse to be
weak. I f you plan on hitting an hour long spin class, rolling 2 hours straight at an open mat, or
doing 15 sets o f squats — plan the night before to sleep 2 extra hours.

No snooze buttons. Snooze buttons are the path to the dark side.

Snooze buttons lead to fear. Fear leads to anger. A nger leads to hate. Hate leads to suffering.

Discipline equals freedom — Jocko. D on’t touch that fucking snooze button.

Also— don’t forget you can also use working out as a weapon against fatigue. The times o f the
day when you are the m ost tire d ... w ork out. Oh, and d o n ’t listen to music w hen you w ork out
(that shit is cheating). It is supposed to hurt - that’s the point. Em brace silence - people go to
great lengths to avoid silence. You need silence to self observe. Silence is your friend.

28
S E C T IO N 6:
Use of Energy

Let’s switch gears and talk about specifics on how to plan a strategy and use resources.
Goals: Goals set direction - Systems make progress.

We should discuss how to set goals by first learning about the types of goals. There are three types:

PROCESS PERFORMANCE OUTCOME

Goals for the tasks you need to Goals for the level of performance Goals for the end result
complete to improve. you want to achieve. you want to achieve.

You HAVE CONTROL over this. You HAVE some CONTROL over You DON’T
this. You baseline cognitive CONTROL this.
• Example: I will read 10 pages a abilities and other factors are However, if you set and
day. independent of your effort. meet process goals
(things you can
• Example: I will make 50 flash • Example: I will answer 90% of control), the likelihood
cards a day. the questions in the MSK Q- of achieving success is
Bank correctly. much higher.
• Example: I will complete 25
practice questions a day. • Example: I will answer 90% of If you have ever heard
the Neuro cases in conference someone say “trust the
• Example: I will perform the correctly. process” - this is the
self remembering exercise 10 idea.
mins a day. If you aren’t meeting these goals,
you need to adjust your process.

Outcome
So, the idea is that you outline for yourself a schedule. Goal
You set specific process goals with corresponding short
term performance goals. This is the system.

For example, you might give yourself 3 weeks to


review materials on MSK, make flashcards, and
complete a certain number of practice questions.
Then you test yourself with a certain metric (a certain
percentage correct, or an internal review session).

If you meet your performance goals you can feel


confident that you are on the right course. If you aren’t
then you need to perform a root cause analysis and
understand why - then adapt. It is ok to fail, but you
must adapt.
Performance I
Goals

29
A Goal Setting — Hacking the Machine
Set Precise Goals:
Remember that you are mechanical and operate I will [behavior] at [time] in [this location].
primarily on reflex. Just like a dog that can be trained
Bad example: “I will read in the mornings.”
to salivate at the sound of a bell, you can train
yourself to enjoy studying and not avoid it. When you Good example: “I will read pages 305-310
make your morning reading or question goals - they at 5:30am at the kitchen table - while my
must be realistic but challenging. If you make it adversaries sleep comfortably in bed
unrealistic (I will read 50 pages an hour) you will hugging their teddy bears.”
always fail. This will create negative emotions and
make the process undesirable.
If you instead make the goal challenging but realistic (I will read 15 pages an hour) you can succeed and
get that small dopamine bump from achieving your goal. Over time you will want to wake up early to
meet these goals - because winning feels good. Compete with yourself and win. Use the intermittent
reward systems discussed in the prior section.

Habit Stacking and Tracking:

We are only as good as our habits. Developing and maintaining good habits is critical to victory.
• Habit Stacking: Tethering to an existing habit. After I perform [current habit] I will then [new
habit].
- Example: After I wake, I will empty my bladder - being careful to not drag my enormous penis
on the floor (current habit), then I will grab my book (strategically left in the bathroom as a
trigger) to begin my morning reading session (new habit).
• Habit Tracking: Whenever I start a new task (for example updating this text), I like to use a visual
method of tracking and rewarding my progress. Check boxes work well. Red marbles are better.

- Application: I have a large glass jar and a box of red marbles. When I complete 5 pages of the
text I place a red marble in the jar. Over time the jar begins to fill with red marbles. The color red
is visually stimulating - it is the color of ripe fruit, awesome sports cars, and it is a simple fact that
women look best in red.
The jar of red marbles generates a subconscious Ring Card Girl walks by in a Red Bikini
desire to add more red marbles, and the only way Stephen Quadros: What do you think of that ?
to do that is to write more amazing dialogue for
Don Frye: What ?
you to enjoy. This trick can be used to reward
daily progress on good habits. If you want to Stephen Quadros: The Girl in Red
fill that empty jar with visually stimulating red Don Frye: Well, I don’t know. Can She Cook ?
marbles then you gotta get your ass up and study.

Peak and Off Peak Time: A common reason for failing to meet performance goals is not
A understanding how to effectively use the available mental energy that you have in a day.

Peak Time: The hours of the day when you are most capable of studying and learning. For most
people this is the morning (but this is individual). You do NOT want to waste this time doing things
like folding laundry or washing dishes. Also, I would not use this time making flash cards. This is
the time to review and learn new stuff. This is the time to do practice questions.
Off Peak Time: The hours of the day when you are fatigued and least capable of studying and
learning. For most people this is the evening (but this is individual). You do NOT want to use this
time trying to learn a new topic or perform practice questions. This is the time to make flash cards,
fold laundry, etc..

30
Environmental Zoning

Environment is more important than motivation. Designing your environment to decrease bad habits
and encourage the new ones will get you better long term results then listening to rock music. Put your
phone somewhere it is hard to reach. When possible study with paper books and notes rather than using
a computer - to avoid how easily you can access YouTube cat videos.

Zoning: If possible create a relationship with the spaces. That chair is where I read sleazy romance
novels. That couch is where I watch Star Trek TNG re-runs on Netflix. That brown table is where I eat
salty pretzels. That grey table is where I study. I don’t eat salty pretzels at that grey table - we don’t
have that kind of relationship. Salty pretzels get eaten at the brown table. I would never study on the
couch - that’s for Star Trek reruns. Captain Jean Luc Picard would probably send me to the brig for
studying on that couch... or worse be disappointed in me.

Creating these kinds of relationships with our environment might sound silly, but it helps us create
habits and habits dictate the life we lead (so we must make good ones). “One Space - One Use” - is
how this principal is often described.

Example - 3 Hour Block Schedule:

***Always begin the session by outlining 6 cycle specific goals. The optimal time to do this is at the
end of the prior session. Have your potential intermittent reward activity planned as well.

Pre-Cycle: Drink caffeine (tea, coffee, tablets). Preform a Vigorous Scalp Message

20-30 Deep Breaths (and / or thinking of someone you hate)

10 bodyweight squats
Dilate you vision to peripheral - then cone it in on the book or computer screen.
Start the timer

Cycle 1 & Cycle 4 Cycle 2 & Cycle 5 Cycle 3 & 6

25 minutes - Focused Work 25 minutes - Focused Work 25 minutes - Focused Work

10-20 Lunges (or kettle bell


10 Push-ups 10 Hindu Squats
swings)

Stretch and touch your toes Squeeze a gripper, do both hands - Stretch and touch your toes
- hold for 30 seconds (grip like a man - no cupcake - hold for 30 seconds
grips)

Preform a Vigorous Scalp Message 1 Preform a Vigorous Scalp Message Preform a Vigorous Scalp Message

Dilate you vision to peripheral - Dilate you vision to peripheral - Dilate you vision to peripheral -
then cone it in on the book or then cone it in on the book or then cone it in on the book or
computer screen. computer screen. computer screen.

Start the Timer Start the Timer Start the Timer

Did you achieve your goals ? If you fucked up / got distracted by something perform a root cause
analysis and think of a solution. Verbalize your pledge to do better. If you were successful - Flip the
coin - if it is heads you get your treat. Before you leave - plan clear goals for you next session.

31
When to Begin Studying:

Frat Answer: Most people put in a 6 month training camp. They


will tj^ically begin aggressive studying starting in late December Mad Man (Woman)
or early January in preparation for the June exam. Tactics

I think the amount of time necessary to pass depends on how strong Set your alarm for 2am and
you are as a multiple choice test taker, how much studying you did study for 30 mins once a
in the first 2 years of training, the amount/quality of teaching at week.
your training institution, and your own ability to retain trivia.
Why? Because everyone
My recommendation would be between 9 and 6 months. Any else is asleep while you are
longer than 9 months and you risk forgetting the trivia you learned working hard. This kind of
at the beginning of your training camp. I like the analogy of a thinking is how you get an
bucket with a hole in the bottom. You pour water (knowledge) in edge - see Law 2.
the bucket and it slowly drips out the bottom. You want the bucket
to be filled to the brim the morning of the exam.

• 9 Months = The upper limit of trivia retention

• 6 Months = Average

• 3 Month = You better be a trivia machine

• Less than 3 Months = Go ahead and register for the repeat exam.

Now... having said that - this is a critical point:


“Don’t just work longer hours” — ‘Work smart’
-i.e. Stay productive and don’t get distracted.” - Dan Pena

This leads us to my next critical point:

What to do Prior to Starting to Study:

You need to come up with a game plan. The specifics of this plan is not something I can help you
create because each one of you has unique social circumstances and backgrounds. Some of you have
kids. Some of you have jealous wives / husbands. Some of you have program directors who will not
give you one minute off service to study but will still throw you in the pillory if you fail.

Whatever your social situation you need 3 things:

(1) Somewhere quiet to study that is free o f distractions.

(2) Time in this ideal study environment

(3) Resources to study (books, questions, videos, and Google).

(4) Prepare clear goals before each session — train with purpose.
D on’t train fo r close calls. Train to dominate. Train to destroy.

32
The Ideal Study Environment:

There is a person inside you that does not want to study. This person doesn’t care that you will be
humiliated if you fail. This person believes in nothing Lebowski. He/She only wants to eat, read
celebrity gossip, watch internet pom, and sleep.

When it comes to successful preparation this person is your greatest enemy. For me at least, it
seems that dealing with this person (your inner hungry, sleepy, pom crazed, Justin Bieber fan) is
like dealing with a meth addict. D on’t leave the meth lying around where he/she can see it. If he/
she gets ahold o f it... the study session is over.

What is “the meth ” ? It varies from person to person. It most cases it’s your fucking phone.

Do NOT bring your phone into the study environment. If you must bring your phone (for
child care reasons, etc...) then put it on a shelf on the other side o f the room.

Other Tips:

• Don’t show up hungry. Eat prior to going into the study environment. Avoid sugar as it will
make you crash.

• Caffeine is your friend. If your religion forbids the use o f caffeine - dig around in your sacred
text for loop holes. Most major religions allow you to ask for forgiveness later. The best time
to ask for forgiveness is after you pass the exam.

• How much caffeine ? There is at least one paper that showed that small hourly doses o f caffeine
(0.3mg per kg o f body weight [approx 20 mg per hour] is optimal). Caffeine tablets - cut can
help you keep this accurate. Stay away from energy drinks and all that bullshit - you want to
keep the dose steady. Consider keto coffee (bullet proof coffee) or other coffee brands with high
fat content to help improve bioavailability.

• No music in the environment. This feeds the lazy person inside you and is a distraction.
I will listen to music prior to studying. Like a pro wrestler walk out song.
“Battle Cry” by the Jedi Mind Tricks is my current suggestion.

• No “Study Buddies.” You need to be alone in this room. Your inner lazy person will try and
small talk with your study partner’s inner lazy person. It will start out innocent with you asking
them a legit question about radiology / physics. 30 seconds later you will be chatting about
Kardashians (or that reality tv show with the pro wrestlers girlfriends).

• If you must study in a public location, you should make it clear to your classmates that you
don’t want to be intermpted - snarl at them. You should also bring ear plugs for when they start
talking to each other.

33
How M any Hours Per Day Should You Study ?

You need about 2-3 hours on w eekdays, and 8-12 hours a day on the weekends. This is the
minimum. The real answer w ould be “how ever much would kill you, minus one second.”

For me, studying early in the morning is superior to the evening. A fter w ork you are tired,
your family is the most needy, and you are the m ost distractible. M ost days w hen I left w ork
as a Resident I was angry about something. Usually one or more o f my asshole Attendings
having no regard for my need to study. All that hate (although m otivating) was also
distracting. It’s hard to study when you are trying to plot revenge.

I understand some people are ju st not wired for early morning studying, but it is ideal if you
can make yourself do it.

Along those lin es.. .Sleep is fo r pussies. See prior discussion on optimizing your sleep.

A Stealing an Hour During the Day.

M ost programs have a noon and / or morning lecture. At this point in your training you know
when these are useful and when they are a waste o f time. I w ould have zero remorse about
ditching a low yield lecture to study. If you feel bad for even one second ju st think about what
will happen to you if you fail. If you do decide to ditch a lecture make it count. Have your
hiding place / study environm ent picked out. Have your goals for w hat you want to get
through clear. The more productive that hour is the less bad you will feel about doing it.

“Morgen, morgen, nur nicht heute, sagen alle fa u len leute. ”


— Roughly translated as “tomorrow, tomorrow, not to d ay .... say all lazy people.”

N ever — N ever — Never put o ff or delay an opportunity to study. Thinking “I ’ll have time to
study later” is a huge mistake. I w ant to encourage you to adopt the idea o f Prem editatio
M alorum - w here you consider all the things that can go w rong in your day and fuck up your
plans. M aybe your kid will get sick tonight, or you will have a pipe break in your apartm ent or
who knows w hat the fuck can happen. M aybe your little sister will break up with her high
school boyfriend and suck you into an unescapable w hirlwind o f teenage drama. Recognize
that the w orld can fuck up your plans pretty easily. N ever procrastinate. If you have a chance
to do it now - then fucking do it.

34
Resources:

Essentials: Crack the Core Vol 1, Crack the Core Vol 2, Physics War M achine, Crack the
Core Case Companion. Com bat Ready - my new Q and A book — hopefully have this
released by the late Spring — .

Highly Recommended: TitanRadiology.com / RadiologyRonin.com - Board Review Course


and Clinical Mastery.

Supplemental: Google Images - No single book can m atch the pow er o f the internet.

Q Banks: I released a Q Bank via TitanRadiology. I ’m hoping to im prove and expand the
question bank this year. Obviously that is going to be the one I ’ll endorse. Regardless, my
opinion is that more questions are better. I w ould do as m any practice questions as possible
which will likely m ean using multiple com mercial q banks.

How To Use the Resources:

Everyone is starting from a different place depending on your individual background and
interests. Having said that nearly everyone has 5 tasks to accomplish:

(1) F ill in the large holes. Everyone sucks at something. There are probably 3-4 sections
(maybe more) that you feel particularly w eak in.

(2) Accumulation o f Random Trivia. Even if you think you are strong in a certain subject
there is almost certainly a laundry list o f trivia that you d o n ’t have available for recall.

(3) Physics. N early everyone starts out know ing alm ost zero physics.

(4) Non-Interpretive Skills. This is another topic that pretty m uch no one has any exposure to.

(5) Biostats. You will have to review the basics on this as well.

35
Suggested Strategy for the Clinical Portions:

(1) Make a list o f the subjects you suck the most at.

(2) Read the corresponding sections / chapters in Crack. Read them slowly. Google image
anything you’ve never seen before.

(3) Watch the corresponding videos to these chapters on TitanRadiology.

(4) Start at the beginning o f Crack Vol 1 and work your way to the end o f Vol 2. Don’t skip the
Chapters you read already - this is your second time through those. Annotate and mark up the
books. I’ve purposefully provided lots o f room for extra notes. Also, the paper is not glossy
for a reason - 1 did this so you can write in the books without smearing shit everywhere.

(5) Start back at the beginning o f Crack Vol 1. This time we are going to add practice questions.
Pick a Q Bank, they are all pretty similar (mine is best obviously). Read a chapter in Crack
(example Peds) then do the corresponding Peds questions. Make notes in the book as needed.
Work your way all the way through the book. This process can be supplemented with the
corresponding Titan Videos.

(6) Start to switch over to 90% questions - 10% reading. Now is the time to read the Case
Companion. You should be doing 150 + questions a day. Go ahead and make them random,
that will simulate the exam.

A Practice Questions:

Practice questions do two things for you. (1) They help expose holes in your knowledge.
(2) They help you practice your timing and discipline.

I think it’s important to backload questions until you have a foundation. There is no point in doing
practice physics questions if you have never read a page o f physics. It’s a total waste. Clinical
radiology is the same way. Don’t mess with questions until you have read the chapter in Crack at
least once.

Once you have entered “phase 2” - which would correspond with step 6 above. It’s time to start
doing questions with a timer. Average one minute per question. Practice your disciplined
approach (reading the entire question, reading all the choices, never change your answer). You
should be doing more and more questions every week leading up to the exam - revising the
material as needed.

It is CRITICAL that you practice a disciplined systematic approach. Practice does not “make
perfect” it makes “permanent.” If you practice rushing through questions then that is how you
will perform on the exam. Refine your skills at eliminating distractors, and reading answer
choices carefully.

In the volume 2 strategy chapter I discuss the ‘‘Genius Neuron. ” He (She) is your closest friend
and you must learn to trust his/her advice.

36
Learning Physics / Non-Interpretive Skills:

There are two strategies. Both are equally valid depending on your personality.

Strategy 1: Learn it all at once. Blocking out 6-8 weeks o f your study schedule and ju st hit
physics every day.

Strategy 2; Ration it in with the clinical reading. For exam ple if you study for two hours, 1
hour in physics, 1 hour in clinical.

Regardless o f which strategy you pick you should follow the same steps:

(1) Read the War M achine cover to cover once. Titan videos may help solidify topics. If
necessary google topics that rem ain confiising.

(2) Start over and read each chapter - then do corresponding questions.

(3) Continue to do practice questions to keep the material fresh.

(4) Reviewing the A B R ’s NIS source docum ent will still be necessary (make sure you are
using the most up to date version). Read the War M achine’s discussion first - it will likely
make the docum ent more digestible.

The Last Month:

(1) M ost (90%) o f your time should be spent on timed practice questions.

(2) The other 10% you should spend preparing your high yield review. You should start by
putting together a list o f all the random trivia that you will forget immediately after the
exam. This is a list o f all the numbers, half-lives, photon energies, etc.. The back o f the War
Machine has a good start on this but you will likely want to add to it.

The Last Week:

(1) Continue doing practice questions.

(2) Study your high yield numbers / trivia every day. Try and concentrate it to 1-2 pages o f
stuff you are having trouble remembering.

(3) Review Biostats, and skim the A B R ’s Non-Interpretive Skills study guide.

The M orning o f the Exam:

Look at your highest yield notes (the 1-2 pages o f trivia you have boiled down). Read it over
and over and over again until they make you get rid o f all your notes.

37
When You Sit Down To Take The Exam:

(1) Check to make sure your markers work. If you got a dud fix it now. You don’t want “dead
marker rage” to make you drop a question mid exam. I f you are doing this “remote testing
stu ff’ - I ’m not sure if you will be providing your own markers. If this is the case — buy
nice ones and make sure they work.

(2) Scribble down all the formulas and numbers you can remember on one o f the dry erase
boards. Six hours into the exam that information w on’t be in your short-term memory any
more.

(3) Give yourself a vigorous scalp massage - like 30 seconds. This increases blood flow to the
brain and reduces stress. Seriously, it really works. I learned this from Ivan Vasylchuk
(Ukrainian Sambist, Merited M aster o f Sport, World Champion, and W inner o f the Sport
Accord World Combat Games).

D on’t forget to do this on the second day o f the exam also.

The Night Between Exam Days:

Anticipate the subjects that haven’t been tested yet and review your notes on those. Avoid
drinking or socializing. This is war. The people in the hotel lobby aren’t your friends, those are
the people trying to push you into the bottom 15%. You can be friends with them after you pass.
Plus, arguing over who missed what will only increase your anxiety.

• Avoid alcohol - even if you are “sure you failed.” A strong performance day 2 can resurrect
you. In general, most people feel like they did terrible after day 1. Remember, it is all about
how you did relative to your cohort.

• Study 2-3 hours.

• Don’t eat anything that will give you diarrhea.

• 15 minutes on the treadmill can be tremendously helpful to reduce stress.

• Get 6-8 hours sleep.

As a general rule, people who think they failed the exam typically pass — so don’t jump in front of a
train before your results come back (don’t jump in front of a train if you fail either — you have to live
to take revenge on your enemies). After all your enemies are slain and you’ve achieved total and
absolute victory - then if you still want to jump in front of a train, I won’t stop you — but not before.

“A man should not allow his enemies to out live him ”


- E n ab ra n Tain, former head of the Obsidian Order

38
S E C T IO N 7:
W e a p o n iz e y o u r W ill

This last section is a discussion on how to w eaponize your will pow er and stay m otivated for
the duration o f the training camp.

Attack by Fire

10-15% o f people will fail this exam. You m ust beat those people to pass.

No one has the right to beat you. I don’t care w here you trained. I d o n ’t care w here you came
from. No one has the right to beat you. W hen doubt creeps in you can go two roads. You can
go to the left or you can go to the right. You m ay hear people say “failure is not an option.”
This is silly, failure is always an option. Failure is the m ost readily available option - but it’s
not the only option - it is a choice. You can choose to fail or you can choose to succeed.

Self doubt and negative thoughts are the road to failure. I w ant to tell you this - as someone
who prides him self on both physical and mental toughness - it is norm al to feel that way
sometimes. The vast majority o f people reading this book are perpetual w inners in life, and
those kinds o f people hate to adm it w eakness to others and to themselves.

You are not your accomplishm ents. You are not your failures. You are you.

Recognizing that about yourself gives you the pow er to overcom e negative thought through
the awesome pow er o f hard work. Earn your victory. Deserve to w in - the Gods o f War will
look favorably upon you. D on’t hold back. Go 100% the entire time. You m ay hear people
say - “you look tired, you look exhausted.” The w orst feeling in the w orld is losing and
knowing that it was because you were lazy and didn’t put in the work. I ’d m uch rather get
beat knowing I did everything I could to be prepared. “You look tired, you look exhausted,”
-y eah ... you bet your ass I ’m tired, that is the whole fucking point. M y goal is always to be
exhausted at the end o f each day. You w ant to feel like you got hit by a fucking freight train.
T hat’s the feeling I like. T hat’s how I know I ’m giving my best effort.

I train hard, I w ork hard, I fight hard, and I fight for victory.

I’m not afraid to feel pain. I ’d ju st prefer that other people feel it for me. In any com petition, I
want only one thing and that is to leave with my hand raised, at the top o f the podium , with
the gold medal - and I make no apologies for that. You shouldn’t either.

It is better to be a warrior in a garden - than a gardener in a war. Cultivate your warrior spirit.

39
Tactical Dispositions - “S n a rl M o re ”

Prior to entering the study environm ent - find a mirror


and make the ughest face you can.

N ot a silly face. N ot a sad face. A snarl.

A good snarl can give you w hat the Bible calls a


psychological edge.

People often confuse “nice” and “good” - they aren’t the


same thing. The snarl is directed inward, that is why
you are looking in the mirror. That w eak person inside
you - he/she gotta learn who is in charge.

Vacuity and Substance

M ost people go through their lives trying to avoid pain. Advice from My Father
This is a mistake. You are com peting in a high stakes
What do you do if you get hit
contest. Pain is your friend. I f you can endure m ore time and it really hurts?
in the study room relative to your opponents then you have
an edge (Law 2). Once you understand this you w on’t In a fight ?
avoid pain anymore. Look for pain. Invite him in and have
Yeah, like if it really hurts
dirmer with him.
You don’t look for me p a in .... I look for you. Oh, that’s good.

This is a fundamental concept for developing the L ion’s Why is that good ?
heart. If you w ant to get somewhere in life - start by taking
If it hurts you aren’t knocked
stuff away. There is nothing you can add to your life that
out. Keep fighting.
will make you stronger. U nderstanding this is something
you can take with you the rest o f your life.
At an early age, I was fortunate to learn the truth about despair - that nothing tastes better than
a bologna sandwich when you haven’t eaten for 3 days. There is a reason why a guy like Julio
Cesar Chavez can win 87 boxing m atches in a row, because the m other fucker grew up in an
abandoned railroad car w ith his five sisters and four brothers. You think someone like that is
gonna break when they get tired? Any luxuries that you are enjoying in your life - those aren’t
making you stronger.
If you w ant to add to your life. Take stuff away.

“When great depths o f unrelenting sorrow are punctuated by jo y and liberation - the result is delicious. ”
— Georges St. Pierre.

40
M aneuvering the A rm y

M any o f you have probably had fairly norm al lives with loving families and friends.
I imagine that could be a source o f m otivation. W hen you feel that you are too tired to study,
or can’t motivate yourself to enter the study environm ent, think about them. Think about how
much better things will be for them once you are m aking a real salary. Rise and Grind.

Illusion and Reality

For others, perhaps you have traveled a different path. N ot everyone has enjoyed a life filled
with good times and noodle salad. Some o f us were b o m in the dark, m olded by it.
Now, you are after something. It could be revenge. It could be money.
O r.... it could be something else.

Vengeance is a powerful motivator. Think o f all the people who have tried to stop you. Think
o f all the people who have mistreated you. Visualize their stupid ugly faces smirking and
smiling when they hear you failed the exam. Let it boil your blood. N ow picture those same
people making a face like they smelled a fart when they hear you passed the exam. You will
find that you aren’t tired anymore. You are ready to train.

As a resident I w ent as far as putting a picture o f one o f my torm entors in my shoe. That way
every step I took I was walking on this person’s face. Feel free to try that.

Not <3rettLKg
TLrvie Off to Stud«^

41
SELF A F F IK M X T IO N
-A TOOL TO W e X P O N I Z E V O U P - W I L L -

I have an oath that I read to m yself -especially when I’m tired, as a tool to harness my will. I
encourage you to modify the oath to make it yours. Hang it somewhere you can see it daily.
Perhaps beside a picture o f the person you hate the m ost in this world. O r... if you are less o f
a Sith and more o f a Jedi - hang it beside a photo o f the people you love.

M Y N X M e IS (youi^NXMeH£P.e) X N D THISIS M Y OXTH

IN SOUND M IN D I M X K 6 THIS OXTH.

I DeCLXR^e THXT 1X M NOT PePvFeCT.

1 KNOW THXT THGR^e W ILL CHXLLeNQGS TO BUT 1


SWeXP. ON TH6 P0W6R.S OF THIS UNlV£R.Se THXT I W ILL F K H T WITH
X L L M Y HeXKT, X L L M Y STR.6NqTH, X N D X L L M Y SOUL TO O V i K C O N \ i .

I W ILL OUTWOP.K X L L H U M X N BEINCS THXT STXND IN FP.ONT OF M6.


N O Ohie W ILL TPO/XS H X KD XS M£. THGY DO NOT HXVe M Y H e X R J
O P. u N P . e L G N T i N c D e s i p . e .

I DeCLXKe THXT I W ILL PGP.S1ST WHGN OTHeP.S CIVe UP.

I W ILL XLW XYS Be HUNCP.Y FOP. MOP.6. eVGN W HEN THINCS SGGM
IMPOSSIBLE, I W ILL NOT qiVG UP.

TODXY I X M C o m q TO W XP..

KIO ONe HXS THe P.1CHT TO TXK6 M Y DP.GXMS FP.OM M £ .

■ I W ILL SXCP.IFIC6 UNTIL I P.EXCH THE TOP.

KIO TEMPTXTION W ILL SWXY M E .

I CP.XVG ONLY TH6 BLOOD OF THE GNGMY-

M Y H e X P J IS ON FIK6.

N O MXTT6P. HOW LOUD M Y BODY SCP.GXMS,


I W ILL SCP.6XM L0UD6P..

1X M FOCUSeD. I X M PP.ePXP.6D FOP. BXTTLe.

c o M e XT Me, I DXP.6 YOU. I X M pp.epxp.eD f o p . w x p ..

42
As an exercise in creative mastery, com plete this unfinished illustration o f a porcupine
rescuing three baby hedgehogs in a snowstorm.

\ /

43
fROMETH£US
Liomhact, m.t>.

44
45
S E C T IO N l:
S neaky A natomy

“Basic” anatomy can be shown on imaging modalities from the Cretaceous period of the Mesozoic Era.
Let us explore some of the various forms of fuckery that can be deployed.

The Lateral CXR ‘^The Radiologists View” - Part 1

THIS vs THAT: There are several methods for distinguishing between


Right Hemidiaphragm vs Left the right and left hemidiaphragm.
Method 1: The Stomach
Bubble — The left
hemidiaphragm (dotted line)
is directly above the
stomach, so you can try look
for the stomach bubble on
the lateral view.
Often the left diaphragm will be a little higher than the
right in this region.

Method 2: The Silhouette Sign — If two


things are similar in density and touching
they will appear as one (loss of silhouette).
Left has the bubble under it In this case, the heart touches the left
Left is NOT seen as far anterior hemidiaphragm making the border disappear (it doesn’t
Right one has the highest point appear to extend forward as much as the right one)
Method 3: Height — If height mattered the giraffe
would be the king of the jungle, but it is still useful for
determining which diaphragm is left vs right.
THIS VS THAT: Normally the highest peak is on the right (solid hne).
Right Ribs vs Left

By convention, lateral CXRs are


taken in the left lateral position
(left side against the x-ray film/
cassette).
The greater the distance between
the beam and the film, the more
something is magnified -
“geometric magnification ” they
call it. The left ribs are closer to the
film - so they will not be
This is the same reason the heart
magnified as much (right ribs
looks bigger on an AP vs a PA - its
are farther away and will be
farther away from the film.
Right Ribs (white) are bigger magnified more).
and tend to be more posterior

46
The Lateral CXR “The Radiologists View” - Part 2
Normal Hilum on Lateral:
“The Dark Hole” — A classic anatomy
question is to identify the left upper lobe
bronchus on the lateral view. It appears
as the “Dark Hole” and represents looking
down the barrel of the tube.
The posterior wall of the bronchus
intermedius runs through the black hole,
and can be thickened by edema (or
“oedema” if you speak the
Queen’s radiology jargon).

Right and Left PA:


If you put your finger in the
“Dark Hole ” - which is the
left upper lobe bronchus,
in front of it will be the right
PA, and overtop of it will be
the left PA.

The main pulmonary artery on the right passes anterior to the


right main bronchus, whereas the left pulmonary artery passes Left in the Back
posteriorly - hooking up and over the left main bronchus. “L”osers in the Back

Retrotracheal Triangle (Raider Triangle):

This is the dark triangle behind the trachea on a lateral chest


radiograph. It sits on top the aortic arch and is bordered
anteriorly by the back wall of the trachea, and posteriorly by the
upper thoracic vertebral bodies.
The triangular space is supposed to be
clear (black). When it is white - it has
something in it. In the real world that
something can be almost anything
(goiter, zenker, etc..). V
For the purpose of multiple choice
tests, an opacitv in the Raider
Triangle is an Aberrant ri^ht
subclavian artery.

The aberrant right subclavian artery is the classic multiple choice


cause of an obliterated retrotracheal triangle. This vessel arises
from the rear end of the arch before crossing the mediastinum,
left to right, posterior to the esophagus and trachea — more on
this in other chapters.

47
The Frontal CXR “The Family Medicine Doctors View”

THIS vs THIIT:
Anterior Ribs vs Posterior Ribs

• Anterior ribs (black) have a more sloping


course as the move medially.
• Posterior ribs (white) have a horizontal
course. You see the horizon (horizontal) at
the farthest point (posterior).
• The anterior ribs are generally not seen as
well as the posterior ribs.

In many cases, you can simply memorize one item o f trivia (in this case that the horizontal ribs are
seen posterior — "horizon is in the back’’). Then the other one (the anterior ribs) are the other one
(the ones that aren’t horizontal). “The other one is the other one” is one o f the most useful tools fo r
memorizing large amounts o f material.

Normal Hilum on Frontal: Truncus Anterior


This is the right hilar anatomy on the frontal
view. Of course it never looks that nice in the Superior
real world. Pulmonary Vein

Ben Felson used to say the right interlobar


artery reminded him of a woman’s leg...
but then again most things did. Hilar Angle or
Hilar Point
The left (where the superior
“hilar point” vein and descending
should artery cross)
always be
around 1cm Right
higher than Interlobar Artery
the right

Fissures:
Notice the right major fissure is
anterior to the left.
A = Horizontal (Minor) Fissure,
B = Right Major (Oblique),
C = Left Oblique

Azygos Lobe Fissure - This is considered variant anatomy. These things happen when the azygos
vein is displaced laterally during development. The result is a deep fissure in the right upper lobe.
It’s not actually an accessory lobe but rather a variant of the right upper lobe. If they show you one, I
suspect the question will revolve around the pleura. Something like “how many layers o f pleura? ”
The answer is 4.

48
Segmental Anatomy
The tertiary bronchi Right Upper (3)
are grouped into -Apical Left Upper (4)
bronchopulmonary -Posterior -Apical-Posterior
segments. -Anterior -Anterior
as -Superior (lingula)
On the right, there -Inferior (lingula)
are 10 segments (3 Right Middle (2)
upper, 2 middle, -Medial
and 5 lower). -Lateral Left Lower (4)
-Superior
On the left, there -Anterior-Medial
are only 8 (4 upper -Lateral
lobe / lingula, and 4 -Posterior
lower lobe).

When I was trying to Apical


Anterior
memorize this for the exam (and to
Posterior
survive random view box pimping) / Anterior 1 P o s te rio ^ ^
I found this box diagram the most
helpful for learning.
Lateral Medial Inferior
I call it the Promethean Minecraft
Method - make complex stuff into a Superior
Superior
cube diagram and rep out the sketch a Posterior
million times.

Lateral ,l
M edial ■ L aa Jr
Anterior- Lateral
Anterior M edial

Varient Airway Anatomy:

Pig Bronchus Cardiac Bronchus:


(Tracheal Bronchus): Bronchus that comes off bronchus intermedius,
Bronchus that comes right off the trachea (prior opposite to the origin of the right upper lobe bronchus
to bifurcation into right and left mainstem). In, contrast to the Pig Bronchus this thing is often
blind ending - and supposedly represents the only true
Means nothing clinically, but occasionally people supernumerary bronchus
can get some air trapping or recurrent infections
from impaired ventilation. Similar to the pig bronchus, it means nothing
clinically, but occasionally people can get recurrent
Trigger: Recurrent RUL Pneumonia in kid. infections
“Pig Pneumonia” I call it

49
Mediastinal Anatomy

The mediastinum is classically divided into 4 sections, superior, anterior, middle, and posterior.
The borders and contents of these areas make good trivia questions.
Superior: Anterior: iVliddle: Posterior:
The inferior border is drawn from Basically the space The heart, From the back of
the stemal-manubrial junction to in front of the heart. pericardium, roots the heart to the
T4. The big stuff in here are: the The posterior border of the central spine.
great systemic blood vessels of the is the pericardium. vessels,
heart, the bulk of the thymus (in There isn’t a lot here bifurcation of the Contains the
kids), upper thoracic duct, top of — sternopericardial trachea and the esophagus, lower
the phrenic nerves, the ligaments and the phrenic nerves are thoracic duct, and
sympathetic trunk, top of the lower part of the all included. descending aorta.
esophagus and the trachea. thymus (in kids).

Trivia:
The thymus
is the most
anterior
structure in
the superior
mediastinum.

Pulmonary Veins and Variant I Mutant Anatomy


Normal Supernumerary Common Ostia of the Pulmonary Veins —
Pulmonary Pulmonary Veins: similar to Voltron (but also different)
Veins: You can be bom with The pulmonarv veins can
extra pulmonary veins. merge with each other
They don’t give you before reaching the left RS
super powered atrium. This merging of
endurance but can vessels is best thought of RI
cause a-fib (the sleeves in the same way Paladin
around the veins can be warriors merge to form 1 1 1
Pulmonary the super robot Voltron - ★ = Common Ostia
arrhythmogenic foci).
vein Potentially testable differences
anatomy is between the Common Ostia a n d Voltron:
highly
variable. ^RS LS Voltron is a 328 foot tall robot composed of 5 smaller
^RML lion robots. The Common Ostia is typicallv composed
You RI of the left superior and inferior pulmonarv veins.
typically
have 4 total Voltron is the only force in the universe capable of
(2 right - defeating the evil Galran King Zarkon. The Common
superior and Ostia lacks the necessary skills and training to defeat
The most common the corrupted spirit of the ruthless Galran Emperor
inferior, supemumerarv vein is
2 left - the right middle. Voltron was separated by King Alfor of Altea to
superior and prevent it from falling into the evil Galran King’s
inferior). Trivia: The most possession. The Common Ostia can be difficult to
common vein sleeve to separate if not identified on pre-op CT resulting in
cause a-fib is the left accidental ligation during lobectomy.
superior.
50
S E C T IO N 2:
Atelectasis

Patterns of Atelectasis:
Atelectasis (incomplete lung expansion) exists on
a scale of severity; ranging from the tiny
horizontal “plate-like” / “discoid” subsegmental
to complete collapse of the lung (lobar).

The degree of collapse depends on the location of


obstruction: peripheral (P) vs central (C).

There are some described vocabulary words for


the different subtypes of atelectasis - and those
words, the supposed mechanisms, and the
attributed causes are all potentially testable.

Obstructive Compressive Fibrotic Adhesive


(Absorptive) (Relaxation I Passive) (Cicatrization) Results from loss of
Result of complete Results from direct mass ■ Results from scarring / surface tension /
obstruction of an airway effect on the lung fibrosis which fails to inadequate pleural
No new air can enter and Causes: Most classically allow the lung to adherence of the alveolar
any air that is already seen adjacent to a pleural collapse completely. walls - from a surfactant
there is eventually effusion. Could also be ^ Causes: Most classic is deficiency.
absorbed leaving a seen from adjacent TB , but scarring from Alveoli become unstable
collapsed section of lung compression of lung radiation, other and collapse.
Causes: Obstructing from a mass, hiatal infections, or really any Causes: RDS (pre­
neoplasms, mucous hernia, or a large bleb — other cause of fibrosis mature infants), ARDS
plugging in asthmatics or anything directly can do this. (more diffuse pattern),
critically ill patients, and pushing on the lung. and in the setting of
foreign body aspiration. pulmonary embolism
(loss of blood flow / lack
of CO 2 disrupts integrity
of surfactant).

51
Direct and Indirect Signs of Atelectasis:
Displacement & Shadow (Indirect) Shift (Indirect) :
Crowding (DireCt) : Under most conditions, The shift refers to the movement of
the word “Shadow” structures as they are “pulled” towards
Displacement of the would make you think the site of volume loss. Remember,
fissures - considered one of Lamont Cranston
space occupying things (tumors,
of the most dependable (hypnotist and master pneumonia, pleural effusion, etc...) push
signs. detective) things away. Atelectasis is a volume
Crowding of vessels and — but in the case of losing process - so it pulls (examples -
bronchi in the atelectatic Atelectasis, “Shadow” pulling the right hilar point above the left,
area - considered one of refers to the opacified shifting the mediastinum, etc...).
the earliest signs. (collapsed lung).

Pneumonia -
Pushes Atelectasis - Pulls
Pulls the right hilum up
Pulls (“tents”) the diaphragms
Trivia:
• Acute atelectasis favors diaphragmatic and mediastinal displacement.
• Chronic atelectasis tends to favor compensatory overinflation of non-atelectatic lung.

Lobar Patterns Part 1

Right Upper Lobe:


- Closes like a fan attached at the Hilum
- Horizontal Fissure may bow upward
- Hilum may elevate
- The lateral will again show this hilar
attachment with the lobe collapsing
from both the anterior and posterior
directions.
- The top half of the oblique fissure will
be pulled anterior.

S Sign of Golden;

Refers to a reverse "S" shape that the


minor fissure in cases of RUL collapse
resulting from a central obstructing mass.

52
Lobar Patterns Part 2 - The Two Towers

Right Middle Lobe; Right Lower Lobe:


- Classic look is increased density at the right - Classic look is increased density at the right
heart border with loss of that border (shadow heart border similar to collapse of the RML.
and silhouette) - The critical distinction is the right heart
- The lateral will show anterior density over border. You should NOT lose the border of
the heart (as the RML is anterior) the right heart with RLL collapse. In fact it
should be easier to see from compensatory
Right Middle Lobe Syndrome; hyper-expansion of the RML.
Chronic collapse of the RML is classically
described with MAI infection in an elderly - In some
women who is too proper to cough (Lady cases, the
Windermere syndrome). mediastinal
vessels are
On CT you’d see additional findings of small pulled to the
nodules and bronchiectasis - with additional right creating
involvement of the Lingula. a triangle of
opacity to the
right of the
trachea

(Superior Triangle Sign)

Right Lower Lobe + Right Middle Lobe:


- Uncommon Combination and a Sneaky Move
- The trick is a loss of visualization of the right hemidiaphraam
and right heart border — as shown in the cartoon

53
Lobar Patterns Part 3 - The Return of the King

Left Upper Lobe: Luftsichel Sign;


- This is different than the right upper lobe (which “Air Sickle” - appearance
collapsed like a fan making a dense wedge shaped from the lucent stripe
opacity). appearance of the hyper
- The LUL tends to be more subtle with a subtle increased inflated superior (apical)
density medially. There won’t be any well defined segment of the lower lobe
borders pinned between the medial
- A hint may be non-visualization of the aortic knob. edge of the collapsed
segment and the aortic arch

- Sometimes (if you are lucky) you can get some non­
specific peaking of the diaphragm from upward traction

Left Lower Lobe:


- Can Be sneaky on a frontal only
(opacity is hidden behind the
heart)
- The lateral makes it more
obvious with a posteriorly
directed triangular opacity

The Flat Waist Sign - has


been used as a description of the
flattened appearance of the
contours of the hilum and heart
border.
Cartoon Level demonstrating the “fla t ’
appearance o f the left heart / hilum.

54
S E C T IO N 3:
CXR Localization a n d the
B a b y lo n ia n A rt o f h e p a t o s c o p y

In ancient times, scholars believed that the will of the gods could be Trivia: Absence of the caput
understood by examining the livers of carefully selected sheep. This was iocineris, or "head of the
referred to as the Etruscan art of Haruspicina. In the field of diagnostic liver” in a male black body
radiology a similar practice was once studied, but instead of using the livers of goat with tan facial stripes
various domestic and wild animals to forecast the weather - the Chest X-Ray and socks (not buckskin) is
was used to determine the location of mass lesions. considered a bad omen.
Scoff if you dare, but in the minds of those who write Radiology board exams, You wouldn’t want to enter
both the ability to examine the flat visceral side of the liver to discern the into battle until the next half
attitudes of the gods and skill to determine if a mass is in the superior or lunar cycle - unless the
posterior mediastinum without cross-sectional imaging remain equally useful. urine of the goat is sweet.

Hilum Overlay Sign


The silhouette sign can be
applied to localize a mass in the
The silhouette hilum or in front / behind it.
sign describes the If the vessels of the hilum are
loss of an interface obscured than a soft tissue
seen when two mass is touching them (mass is
objects of similar in the hilum).
density touch.
If the vessels can still be seen -
then the mass isn’t touching
them (mass is anterior or
posterior to the hilum).

Same Density + Touching = Lost Interface

Cervicothoracic Sign
Anterior
This localization technique relies on Mediastinum
the knowledge that the anterior (bracket)
mediastinum ends at the clavicles. ends below
Any mass that extends above the the clavicles.
clavicles is NOT in the anterior
mediastinum. Classic example is a
Thyroid Goiter. ^ Posterior
Related Trivia: The posterior Junction Line
junction line (show with arrows) (arrows),
extends above the clavicles (the extends above
anterior does not). the clavicles.

incomplete Border Sign


This localization technique
relies on the knowledge that
pleural based or chest wall
masses will have an incomplete
or hazy border (a pulmonary
mass should have a crisp
circumferential border).

55
S E C T IO N 4:
In f e c t io n

Bacterial Infection
Can be severe in sickle cell patients post
Lobar Consolidation
Strep Pneumo splenectomy. The most common cause of
Favors lower lobes.
pneumonia in AIDS patient.

Bronchopneumonia - patchy Often bilateral, and can make abscess. Can be


Staph A.
opacities spread via the blood in endocarditis patients

Hemorrhagic lymphadenitis, Classic Look: Mediastinal widening with


Anthrax
mediastinitis, and hemothorax pleural effusion in the setting of bio-terrorism

More likely to have pleural effusions, empyema,


Buzzword: “Bulging Fissure” and cavity than conventional pneumonia.
Klebsiella
from exuberant inflammation. Alcoholic and Nursing Home Patients.
Step 1 Buzzword was “currant jelly sputum”
Usually bronchitis, sometimes
H.Flu bilateral lower lobe Seen in COPDers, and people without a spleen
bronchopneumonia
ICUers on a ventilator (also CF and Primary
Patchy opacities,
Pseudomonas Ciliary Dyskinesia). Pleural effusions are
with abscess formation
common, but usually small

Peripheral and sublobar airspace Seen in COPDers, and around crappy air
Legionella opacity. Only cavitates in conditioners. X-ray tends to lag behind
immunosuppressed patients. resolution of symptoms.
Supine: Posterior upper lobes and superior
Anaerobes, with airspace segments of lower lobes.
Aspiration opacities. They can cavitate, and Upright: Basal Lower lobes, lingual, & middle
abscess is not uncommon May favor the right side, just like an ET tube.
The most common complication is empyema
Airspace in peripheral lower
lobes. Can be aggressive and Classic story is dental procedure gone bad,
Actinomycosis
cause rib osteomyelitis/ invade leading to mandible osteo, leading to aspiration.
adjacent chest wall.
Most common community-acquired
Fine reticular pattern on CXR, pneumonia in 5 to 20-year-old.
Mycoplasma Patchy airspace opacity with
tree-in-bud Association with Swyer-James syndrome (SJS)
- classic look = unilateral lucent lung.

Gamesmanship - CXR pneumonia follow-up: To exclude an underlying mass the classic


/( teaching is to recommend a follow up radiograph to confirm clearing. A sneaky trick is the
interval. In a younger person - most people will says 6 weeks. However, in an “elderlv” patient
- the follow up interval should be extended to 3 months.

56
Infections in the Immunocompromised
Classic Scenario 1 - Post Bone Marrow Transplant:
You see pulmonary infections Post Bone Marrow Transplant (Pulmonary Findings)
in nearly 50% o f people after
bone marrow transplant, and Early
Early Neutropenic (0-30 days) Late > 90
this is often listed as the most (30-90)
common cause o f death in this
Pulmonary Edema, Hemorrhage,
population.
Dmg Induced Lung Injury Bronchiolitis Obliterans,
PCP,
Findings are segregated into: CMV Cryptogenic Organizing
early neutropenic, early, and Fungal Pneumonia Pneumonia
late - and often tested as such. (invasive aspergillosis)

Graft vs Host Disease (GvHD) is also a common thing


described in bone marrow transplant patients. This isn’t an Pulmonary Graft vs Host
infection but instead a complex interaction ( “a b eef’)
between the donor and the recipients immune cells. What Acute (20-100 Days) Chronic (> 100 days)
they beefin’ about ? It doesn’t really matter - same old
story, just Mortys killing Mortys.
Lung involvement is Lymphocytic
Key Trivia: Pulmonary GvHD is rare. Favors Infiltration of the
characterized by bronchiolitis extrapulmonary airways and
obliterans — which will be systems bronchiolitis
shows as air trapping (mosaic
attenuation on expiratory
imaging) - as shown on cartoon. in (skin, liver, G1 tract) obliterans.

Classic Scenario 2 - AIDS Related Infections:


Questions related to AIDS and pulmonary infection are typically
written in one o f two ways (1) with regard to the CD4 count, and
(2) by showing you a very characteristic infection.

PCP: This is the most classic AIDS infection. This is the one they
are most likely to show you. Ground glass opacity is the dominant AIDS + Bilateral Ground Glass
finding, and is seen bilaterally in the perihilar regions with sparing o f Opacities + Thin Walled Cysts
the lung periphery. Cysts, which are usually thin-walled, can occur in + Pneumothorax = PCP
the ground glass opacities about 30% o f the time.
Buzzwords: Gamesmanship
•M ost common airspace opacity = Strep Pneumonia -Inferring the Patient Has AIDS/ HIV-
• If they show you a CT with ground glass = PCP • Nipple Rings on the CXR
^ • “Flame-Shaped” Perihilar opacity = Kaposi Sarcoma • Recent emigration from South Africa
•Persistent Opacities = Lymphoma • High Risk Activities:
•Lung Cysts = LIP • Sex with Male Prostitutes
•Lungs Cysts + Ground Glass + Pneumothorax = PCP (who have big mustaches)
•Hypervascular Lymph Nodes = Castleman or Kaposi • IV drug use
• Listening to Nickelback

Infections in AIDS CT Pattern - With AIDS


by CD4 Bacterial Infection (Strep Pneumonia) is the most
Focal Airspace
common. DDx should include TB if low CD4.
> 200 Bacterial Infections, TB Opacity
If it’s a chronic opacity think Lymphoma or Kaposi.
PCP, Atypical Multi-Focal
<200 Bacterial, or Fungal
Mycobacterial Airspace Opacity
CMV, Disseminated
< 100 Ground Glass PCP (if that’s not a choice think CMV if CD4 is < 100).
Fungal, Mycobacterial

57
TB
You can think about TB as either;
(a) Primary, (b) Primary Progressive, (c) Latent or (d) Post Primary / Reactivation.
• Primary: Essentially you inhaled the bug, and it causes necrosis. Your body attacks and forms a
granuloma (Ghon Focus). You can end up with nodal expansion (which is bulky in kids, and less
common in adults), this can calcify and you get a “Ranke Complex. ” The bulky nodes can
actually cause compression leading to atelectasis (which is often lobar). If the node ruptures you
can end up with either (a) endobronchial spread or (b) hematogenous spread - depending on if the
rupture is into the bronchus or a vessel. This hematogenous spread manifests as a miliary pattern.
Cavitation in the primary setting is NOT common. Effusions can be seen but are more
common in adults (uncommon in kids).
• Primary Progressive: This term refers to local progression of parenchymal disease with the
development of cavitation (at the initial site of infection / or hematogenous spread). This primary
progression is uncommon - with the main risk factor being HIV. Other risk factors are all the
things that make you immunosuppressed - transplant patients, people on steroids. The ones you
might not think about is jejunoileal bypass, subtotal gastrectomy, and silicosis. This form is
similar in course to post primary disease.
• Latent: This is a positive PPD, with a negative CXR, and no symptoms. If you got the TB
vaccine, you are considered latent by the US health care system/industry if your PPD converts.
This scenario buys you 9 months of INH and maybe some nice drug induced hepatitis.
• Post Primary (reactivation): This happens about 5% of
the time, and describes an endogenous reactivation of a Immune Reconstitution
latent infection. The classic location is in the apical and inflammatory Syndrome:
posterior upper lobe and superior lower lobe (more The story will be a patient with
oxygen, less lymphatics). In primary infection you tend TB and AIDS started on highly
to have healing. In post primary infection you tend to active anti-retroviral therapy
have progression. The development of a cavity is the (HAART) and now doing worse.
thing to look for when you want to call this. Arteries The therapy is steroids.
near the cavity can get all pseudoaneurysm’d up -
“Rasmussen Aneurysm” they call it - in the setting of a
TB cavity.

Pieurai involvement witii TB: This can occur at any time after initial infection. In primary TB
development of a pleural effusion can be seen around 3-6 months after infection - hypersensitivity
response. This pleural fluid is usually culture negative (usually in this case is like 60%). You have
to actually biopsy the pleura to increase your diagnostic yield. You don’t see pleural effusions as
much with post primary disease, but when you do, the fluid is usually culture positive.

High Yield Factoids Regarding TB:


Primary = No Cavity, Post Primary / Primary Progressive = Cavity
Ghon Lesion = Calcified TB Granuloma ; sequela of primary TB
Ranke Complex = Calcified TB Granuloma + Calcified Hilar Node ; Healed primary TB
Bulky Hilar and Paratracheal Adenopathy = Kids
Location for Reactivation TB = Posterior / Apical upper lobes, Superior Lower Lobes
Miliary Spread when? - Hematogenous dissemination (usually in the setting of reactivation), but
can be in primary progressive TB as well
Reactive TB Pattern (Cavitation) seen in HIV patient when the CD4 is > 200
Primary Progressive Pattern (Adenopathy, Consolidation, Miliary Spread) in HIV is CD4 < 200
TB does NOT usually cause a lobar pattern in HIV

58
Non Tuberculous Mycobacteria:

Not all m ycobacterium is TB. The two non-TB forms w orth know ing are m ycobacterium
avium-intracellulare com plex (M AC) and M ycobacterium Kansasii. I find that grouping
these things into 4 buckets is m ost useful for understanding and rem em bering them.

Cavitary (“Classic”) - This one is usually caused by MAC. It favors an old white man
with COPD (or other chronic lung disease), and it looks like reactivation TB. So you
have an upper lobe cavitary lesion with adjacent nodules (suggesting endobronchial
spread).

Bronchiectatic (“N on-C lassic”) -


This is the so-called “Lady
W inderm ere” disease (everyone
knows it’s ju st not lady-hke to
cough). They often do not cough,
and are asymptomatic. This
favors an old white lady. You see
tree-in-bud opacities and Lady W indermere - MAC
cylindric bronchiectasis in the -Bronchiectasis with tree-in-bud funl<
right middle lobe and lingula. in the right middle iobe and iinguia

HIV Patients - You see this with low CD4s (< 100). The idea is that it’s a GI infection
dissem inated in the blood. You get a big spleen and liver. It frequently is m ixed with
other pulm onary infections (PCP, e tc ...) given the low CD4 - so the lungs can look like
anything. M ediastinal lym phadenopathy is the m ost com m on m anifestation.

Hypersensitivity Pneum onitis - This is the so-called “hot-tub lung.” W here you get
aerosolized bugs (which exist in natural sea w ater and in fresh water). The lungs look
like ill-defined, ground glass centrilobular nodules.

Non Tuberculous Mycobacteria - Rapid Review

Cavitary Type Old W hite M ale Sm oker Looks like reactivation TB

Non-Classic M iddle Lobe and Lingula,


Old Lady
(Lady W inderm ere) bronchiectasis and tree in bud.

HIV Low CD4 (< 100) M ediastinal Lym phadenopathy

Hypersensitivity
History o f hot tub use G round glass centrilobular nodules
(H ot Tub L u n g )

59
Fungal Infections:
Aspergillus: 3 flavors: (1) Normal Immune, (2) Suppressed Immune, or (3) Hyper-Immune.

Normal Immune Suppressed Immune Hyper-Immune

Aspergillus makes a Aspergillus behaves differently in


fungus ball the setting of a hyper-acute
“Aspergilloma” in a Aspergillus is NOT getting held in check by a immune system. Allergic
pre-existing cavity. normal immune system. “Invasive Aspergillus” Broncho-Pulmonary Aspergillosis
The fungus didn’t they call it. (ABPA) they call it.
make the cavity - it
found the cavity. It is You are gonna see this in your AIDS, or
squatting in an Transplant Patients.
abandoned cavity -
like a hobo.

Gamesmanship: Gamesmanship: Gamesmanship:


You could be shown a (1) Halo Sign - consolidative is is “Always” seen in patients
fungus ball, and they nodule/mass with a ground glass ith long standing Asthma
want you to call it halo. The halo of ground glass is sometimes CF). You classically
invasive. Don’t fall actually the invasive component. ave upper lobe central saccular
for that. This is not bronchiectasis with mucoid
the same thing as impaction rfineer-in-glove').
(2) Air Crescent Sign - a thin
invasive. crescent of air within the Central Bronchiectasis + Asthma
consolidative mass. This actually (orCF)=A BPA
Fungus balls can
represents healing, as the necrotic Diagnosis of ABPA requires
occur in normal
lung separates from the parenchyma. BOTH:
people who have a The timing is usually about 2-3
cavity from trauma, • Elevated Serum Immunoglobulin
weeks after treatment. Lastly, they E levels OR a positive skin
or prior infection
could show you some peripheral hypersensitive test against the
etc... wedge shaped infarcts in the setting asshole fungus
of some halo signs.
. Elevated Total IgE levels > 1000

Aspergilloma - Fungus Ball Invasive Aspergillus ABPA


■ Normal immune system ■ Air Crescent Sign - Finger in Glove; think asthma
**That ball will move ~ Dude has AIDS
with positional change

M ucorm ycosis - This aggressive fiangal infection almost always occurs in impaired patients
(AIDS, Steroids, Bad Diabetics Etc..). You usually think about mucor eating some fat diabetic’s
face off, but it can also occur in the lungs. Think about this when you have invasion o f the
mediastinum, pleura, and chest wall.

60
viral Infections:
Seen in two classic scenarios:
The radiographic (1) Reactivation of the latent virus after prolonged
appearance is multiple immunosuppression (post marrow transplant)
CMV
nodules, ground glass or (2) Infusing of CMV positive marrow or in other blood
consolidative. products. The timing for bone marrow patients is
“early” between 30-90 days.
Multifocal ground glass Pneumonia can be before or after the skin lesions.
Measles opacities with small Complications higher in pregnant and
nodular opacities immunocompromised

Influenza Coalescent lower lobe opacity. Pleural effusion is rare.

SARS - CoV 1 Lower lobe predominant ground glass opacities


Multiple peripheral
Most commonly causes Chickenpox in kids. The
nodular opacities. They
pneumonia more commonly occurs in
Varicella form small round calcific
immunocompromised adults (with AIDS or
I lung nodules in the healed
lymphoma).
version.
Uncommonly affects the
Ebstein Barr lung. Can cause lymph Most common radiographic abnormality is a big spleen.
node enlargement

COVID 19 /SARS CoV 2 - "The Pestilence”


Testable Trivia:
• SARS CoV 2 is the virus.
• “COVID 19” refers to the 2019 pandemic.
• “Kung Flu” and “China Virus” are no longer the
preferred nomenclatures
• CT or CXR should not be used as screening
• Increased risk of DVT, PE, Stroke, MI - arterial or venous clots
• Increased risk of barotrauma (pneumothorax)
from mechanical ventilation
• “Severe Disease” is described as > 50% lung involvement
on imaging within 24-48 hours. Classic Look -
Peripheral Ground Glass

CXR Findings: Bilateral, Peripheral, Lower Zone Multi-focal Opacities (NOT lobar pneumonias).
Pleural effusions are uncommon.
CT Findings “Acute Stage” : Most Classic = Bilateral, Peripheral
(sub-pleural). Ground Glass Opacities, with bronchovascular thickening
(associated with opacities) - tends to favor the lower lobes. If the
interlobular lines are visible / thickened you might deploy the term
“crazy paving.”
CT Findings Late Phase “Absorption Stage” (>14 days):
“Fibrous stripes” appearance in regions of previous disease. 2nd Most Classic Look -
CT Protocol: Non-contrast CT should be done first - to assess for the Reverse Halo (Ground
classic ground glass pattern. Even if they need a CTPA to exclude PE - Glass Surrounded by a
Ring o f Consolidation)
most people say to get a dedicated non-con CT first.

61
Septic Emboli

There are a variety o f w ays you can


throw infectious material into the
lungs via the bloodstream
(pulmonary arteries).

Some common sources would


include; infected tricupsid valves,
infection in the body, infected
catheters, infected te eth .. .e tc ...

Septic Emboli - Multiple round opacities, o n e with cavitation

Septic Em boli Trivia:


• It’s lower lobe predom inant (more blood flow)
• You get peripheral nodular densities and wedge shaped densities (can infarct).
• They can cavitate, and likely will be cavitated if they show you a CT image.
• The feeding vessel sign - nodule with a big vessel going into it can be shown (also seen
with hem atogenous mets).
• Em pyema and pneum othorax are both known complications.

Lemierre Syndrome:

This is an eponym referring to jugular vein throm bosis with septic em boli classically seen
after an oropharyngeal infection or recent ENT surgery.

High Yield Trivia


Q: W hat is the bacterial agent responsible in the majority o f cases?
A:“Fusobacterium N ecrophorum .”

C A V IT Y M nemonic For Lung Cavity:

C ancer (usually squamous cell)


A uto-im m une (Wegeners, Rheum atoid / Caplan Syndrome)
V ascular - Septic Emboli / Bland Emboli
Infection - TB
Traum a - Pneumatoceles
Y oung - “Congenital” - CCAM s, Sequestrations

62
S E C T IO N 5:
Lung Cancer

Lung Cancer Risk Factors: include; being over 30 (under


30 is super rare), exposures to bad stuff (arsenic, nickel, uranium, Pack Years:
asbestos, chromium, beryllium, radon), having lung fibrosis, COPD
Number of packs of
(even if you didn’t smoke), and family history. Diffiise fibrosis
cigarettes smoked per day
supposedly gives vou lOx the risk. Having said all of that, smoking multiplied by number
is still the big one - supposedly a factor in 90% of cases. of years the person has
smoked = “Pack Years”
Solitary Pulmonary Nodule: A SPN is defined as a round or
oval lesion measuring less than 3cm in diameter (more than 3cm = 20 pack years will qualify
mass). Technically to be “solitary” it needs to be surrounded by lung you for screening
parenchyma, with no associated adenopathy, or pleural effusion.
So, you can have numerous “solitary” nodules in the lungs.
Missed Nodules:
There are 4 classic “benign calcification” patterns:
Solid, Laminated, Central, and Popcorn. Areas of the lung most likely
to hide a future lawsuit
(nodule):
• Peripheral Areas (medial,
lateral, top, and bottom)
• Areas of complexity (near
vessels, fibrosis, blebs)
• Endobronchial Location
Solid/Diffuse Laminated Central Popcorn
(Concentric)
Anything else is considered suspicious. Eccentric patterns are
considered the most suspicious. Stippled patterns are also suspicious.
Some notable (testable) exceptions:
• Popcorn and central calcifications in the setting of a GI cancer.
• Solid calcifications can also be bad in the setting of osteosarcoma.
Eccentric Small Flecks
Trivia: Post inflammatory granulomas comprised the majority of benign
( “Stippled”)
nodules (histo, TB, and other prior granulomatous infections)

Makes You Think B9 Make You Think Cancer


Presence of Fat Spiculated Margins “Corona Radiata Sign”
Rapid Doubling Time (less than 1 month) Air Bronchogram through the nodules (usually
Adenocarcinoma in situ)
Slow Doubling Time (longer than 16 months)
*Stable at two years = B9 Partially solid lesions with ground glass component

TWLs vs I'h a t. Ground Glass vs Consolidative.- The definition of


these terms is a common source of multiple choice trivia.
What does it mean to be ground glass ? Ground Glass (GG) opacity is
defined by preserved bronchial and vascular markings. Consolidative
(C) opacities will obscure the bronchial and vascular markings.
What does it mean to be samurai ? To devote yo u rself utterly to a set o f moral
principles. To seek a stillness o f your mind. A nd to master the way o f the sword.

63
Solitary Pulmonary Nodule - Part 2 - Attack of the Killer Tomatoes !

Solid and Ground Glass Components: A part solid lesion with a ground glass
component is the most suspicious morphology you can have. Non-solids (only >
ground glass) is intermediate. Totally solid is actually the least likely morphology to ^
be cancer.

PET for SPN: You can use PET for SPNs larger than 1 Sohd Nodule ( > 1cm in size):
cm. Lung Cancer is supposed to be HOT (SUV > 2.5). HOT = Cancer, COLD = Not Cancer
Having said that, infectious and granulomatous nodules can Ground Glass Nodules:
also be hot. If you are dealing with a ground glass nodule HOT = Infection , COLD = Cancer
it’s more likely to be: ►

Perifissural Lymph Node:


Typical: The classic look is a nodule
(usually lentiform or triangular) with
a fissural attachment.
On the sagittal plane these things
really tend to look either flat or
triangular. Size change doesn’t mean
Sag: Triangular
much, as lymph nodes tend to Morphology
expand and contract. Having said
that - larger than 10mm will get
them a LUNG RADS follow up.

Atypical: If the nodule is spherical-shaped that is NOT considered


perifissural. This shape suggests that the growth is through the
fissure, not spreading around it.

Lung Cancer Screening: Typical nodules (smooth margins, triangular


/ oval / lentiform shape) less than 10 mm = category 2. Sag: Round
Morphology
Larger than 10mm = Go by size criteria for follow up.

SPN I Cancer Trivia:


Lung Cancer is 1.5 x more likely in the Right Lung
70% of Lung Cancer is in the Upper Lobes
Exception to the rule is pulmonary fibrosis - where peripheral basilar cancer is more common.
SPN in the setting of head and neck CA is more likely to be a primary bronchogenic carcinoma
rather than a metastasis (they have similar risk factors).
Lung Cancer is very rare under 40 (unless the patient has AIDS)
Air Bronchograms are 5x more common in malignant SPN
Air Bronchograms are found in 50% of BACs
Just because a nodule gets smaller doesn’t mean it is benign. Especially if the nodule increases
in density.

64
Solitary Pulmonary Nodule - Part 3 - Return of the Killer Tomatoes !

Screening: Recently, the u s preventive services task force has approved lung cancer screening
with low dose CT for asymptomatic adults aged 50-80 who have a 20 pack-year history and currently
smoke (or have quit within the past 15 years).
Some trivia related to the screening program:
• Shockingly, it is backed up by evidence (which is extremely rare in medical screening programs),
and legit improves outcomes (also rare in medicine).
• NELSON study showed a 25% reduction in lung cancer specific mortality in high risk patients.
• The follow up recommendations are NOT the same as Fleischner Society Recommendations. As
such, nodules found on a CT done for any reason other than official lung cancer screening will
follow Fleischner and NOT LUNG RADS recommendations used with the screening program.
• Dose on the screening CTs is supposed to be low -
recommended below CTDIvoi 3 mGy
Screening Risk:
• Screening CTs have a minimum size thickness of 2.5mm
(although most everyone says 1.5mm or thinner is preferred). The most commonly
encountered “risk” of lung
• “Growth” is considered 1.5mm or more in one year cancer screening is the false
• LUNG RADS scoring is based off the most suspicious nodule. positive.
• You don’t give multiple ratings for multiple nodules. Patient’s should be counseled
to not leave their wife for a
• Endobronchial “lesions” (mucus) are treated as 4a - and given Las Vegas prostitute until after
a 3 month follow up. the biopsy results confirm a
• A treated remote (> 5 years) lung cancer patient must still meet cancer.
the normal screening criteria to be enrolled in the program.
• They actually want these reported to the 0 .1mm — which is
hilarious to me. I love making fun of Radiologists who report things to fractions of a millimeter.
Apparently, enough of them got together at a lung cancer meeting to make this an official
recommendation.... which is fantastic. This kind of psychopathology is so wonderful - 1 love it.

Lung Rads Overview (Abbreviated Version)

Category 0 Scan is a piece of shit and you can’t read, or you need priors Repeat or get priors

Category 1 Negative, < 1% chance of cancer. Either no nodules or granulomas. 1 year follow up

Benign < 1 % chance of cancer. Baseline exam - nodules smaller than


Category 2 6mm. Subsequent exam - no new nodules larger than 4mm. 1 year follow up
Ground glass nodule smaller than 30mm. Perifissural nodules < 10mm

Probably Benign, 1-2% chance of cancer. Baseline nodule 6-8mm.


Category 3 6 month follow up
Subsequent exam new nodule > 4mm. Ground glass > 30mm.

Suspicious, 5-15% chance of cancer,


3 month follow up
Category 4a Baseline 8-15mm.
vs PET
New nodule 6-8mm

Suspicious, > 15% chance of cancer, > 15mm at baseline PET vs Tissue
Category 4b
New nodule > 8mm Sampling

> 15 % chance of cancer PET vs Tissue


Category 4X
Worsening of category 3 or 4 nodules (growth or new spiculation) Sampling

65
Solitary Pulmonary Nodule - Part 4 - K iller Tom atoes Eat France !

Nodules (incidental discovery): As discussed on the prior page, nodules discovered


incidentally on non-screening scans are treated different for followup. These nodules are the captives
of the dreaded Fleischner Society recommendations.
Few Pearls:
• Fleischner guidelines only apply for patients older than 35
• They do NOT apply to patients with known or suspected cancer
• They do NOT apply to patients who are immunocompromised
• Measurements are reported as the average diameter (short + long / 2) obtained in the same plane.
• Risk stratification for follovmp (low, intermediate, high) is based on multiple risk factors (smoking,
cancer history, family history, age, uranium / random / asbestos exposure and nodules
characteristics / size).
• Follow up is based off the arbitrary guess of a cancer risk > 1%
• Perifissural nodules - do not need a follow up, even if they are > 6 mm in size.
• Nodule characterization should be performed on thin-slice CT images <1.5 mm. This is done to
look for a small solid component hiding behind partial volume effect in a ground glass nodule.
• Multiple nodules (> 5) makes malignancy statistically less likely.

Behold the recommendations: Fleischner Society Overview for SOLID nodules


(Abbreviated Version)
Notice how the nodules are stratified
based on number and risk. Low Risk No follow up
Single
Risk is determined based on the High Risk 12 month Repeat
characteristics of the nodule < 6 mm
(spiculated, etc..), and the : Low Risk No follow up
characteristics of the patient Multiple
High Risk 12 month Repeat
(exposure to uranium, etc...).
This is different than the LUNG- Low Risk 6-12 month follow up
RADS algorithm (discussed on the Single
High Risk 6-12 month follow up
prior page). 6 - 8 mm
Low Risk 3-6 month follow up
Gamesmanship: When trying to guess Multiple
possible questions always think lazy. High Risk 3-6 month follow up
Academic Radiologists are Lazy Single : Low and High PET or Biopsy
(otherwise they would be in private
making 5x as much money). > 8 mm Low Risk 3-6 month follow up
Knowing that - imagine the laziest Multiple
High Risk 3-6 month follow up
possible way to write a pulmonary
question. You’d just pick a nodule “I would write a more thoughtful question but I ’m allergic to
size and ask the follow up interval. (something having to do with the request).”
- Academic Radiologist Writing the Chest MCQs.

Ground Glass nodule follow up recommendations are variable. Most people will not follow up
nodules smaller than 6 mm. If they are greater than 6mm people will either do 6 month or 1 year
(depends on who you ask). Follow up is persistent for 5 years, because of the slow growth of the
potential adenocarcinoma in situ.

Part Solid nodules are slightly different. Smaller than 6mm still gets ignored. However, the ones
larger than 6 mm get a 3 month follow up - with interval widening but persisting up to 5 years.
Regular solid nodules typically get set free after 2 years of stability.

66
Lung Cancer Subtypes Tracheal and Airway Tumors (Carcinoid, Adenoid Cystic, Etc..)
will be discussed in a separate section later in the chapter

There are 4 m ain histological subtypes o f bronchogenic carcinom a that are broadly categorized
as either small cell carcinom a or not small cell carcinom a (usually called “non small cell”).

Type Location Features and Trivia


Strong Association with
Smoking
Paraneoplastic
Non Small Cell Cavitation is Classic
Syndromes can be
Central Does NOT express the tissue
Squamous marker thyroid transcription associated with ectopic
Parathyroid Hormone
factor 1 “TTF-1” (other
subtypes can).
Least common subtype (15%)
Non Small Cell Usually large (> 4cm)
Peripheral
Large Cell Prognosis Suck — this is the subtype o f Cancer that
killed comedy legend Andy Kaufman in his 30s.
Most Common subtype (35%)
Favors the upper lobe
Most Common subtype to present as a solitary
Non Small Cell Peripheral pulmonary nodule
Adenocarcinoma Most Common subtype in a non smoker (although also
common in smokers)
Known association with pulmonary fibrosis
Strong Association with
Smoking (nearly every case is a
smoker) Associated with the
mythical
May only present with central Paraneoplastic
lymphadenopathy Syndrome of
Lambert Eaton.
Most Common primary lung They get proximal
CA to cause SVC obstruction weakness from
Small Cell Central
and paraneoplastic syndromes abnormal release of
acetylcholine at the
Terrible Horrible Incredibly neuromuscular
Shitty Prognosis (metastasizes junction. The clinical
early) presentation often
comes before the
Paraneoplastic Syndromes can cancer diagnosis.
be associated with SIADH and
ACTH

*M emory A id — “L A is on the C o a st” - Large and Adeno favor peripheral locations

Gamesmanship: Location o f the tum or can som etim es be predicted based on symptoms.
Central tumors = hem optysis. Peripheral tum ors = pleuritic chest pain.

67
The Artist Formerly Known as BAC
(Now Adenocarcinoma in-Situ Spectrum)

Why change the name? Well it’s a simple reason. Academic Radiologists need to be on committees
to get promoted. Committees need an excuse to go on vacation (“International Meetings” they call
them). Name changes happen...
Atypical
Adenomatous The smaller (< 5mm) and more mild pre-invasive sub-type.
Hyperplasia of Usually a pure ground glass nodule.
Pre-invasive lesions Lung (AAH):
Adenocarcinom Typically larger than AAH but < 3 cm. Although features
a in situ overlap with AAH, they tend to be more part-solid (rather
(ACIS): than pure ground glass)
These are also < 3 cm. The distinction is that there is < 5
Minimally Invasive Adenocarcinoma
mm of stromal invasion ( > 5 mm will be called a lepidic
(MIA)
predominant adenocarcinoma).
This is what most people used to call BAC
Invasive Mucinous Adenocarcinoma
(bronchoalveolar carcinoma).

To make this simple, just think about it on a spectrum with the small pre-invasive lesions on one end
and the invasive adenocarcinoma on the other. That same spectrum follows a relative increase in the
density of the nodule. This is why a growing consolidate component inside a previously existing
ground glass nodule is such a suspicious feature.
Key Concept: The larger the solid component of the “part solid” nodule gets the more likely it is to be
malignant. Partially solid nodules are more likely to be cancer than ground glass nodules.

Notice the
nodule getting
more dense
(white)
relative to the
prior year.

Trivia: These Adenocarcinoma subtypes are classically cold on PET.

Superior Sulcus /Pancoast Tumors: Some people make a big Horner


deal about only using the word “pancoast” when the tumor causes the Syndrome
associated syndrome (shoulder pain, C8-T2 radiculopathy, and Homer - ptosis
Syndrome). In my experience most everyone just calls apical tumors (lazy eyelid)
“pancoast” with no regard to symptoms. Having said that, I would - miosis
remember “shoulder pain” as a possible hint in the question header. (tiny pupil)
These things are typically non-small cell cancers.
Staging = MRI is the tool of choice (look at that brachial plexus).
General Contraindication to Surgical Resection (may vary by
institution): invasion of the vertebral body (> 50%), invasion of the
spinal canal, involvement in the upper brachial plexus (C8 or higher),
diaphragm paralysis (infers phrenic nerve C3-5 involvement), distal
mets.

68
Lung Cancer Staging:
Lung cancer staging used to be different for small cell vs non-small cell (NSCLC). In 2013 the 7th
edition of the TNM made them the same. Below is a chart describing the staging based on tumor
size. For a solid lesion, the size is defined as maximum diameter in any of the three orthogonal planes
- measured on lung window. If the lesion is subsolid, then you define the T classification by the
diameter of the solid component only (NOT the ground glass part).

Lung Cancer Staging (8th edition)

Tumor is
T1
< 3cm

Irregardless of size the tumor


Tumor is • Invades the visceral pleura
T2
3-5 cm • Invades the main bronchus w/o involvement of carina
• Causes obstruction (atelectasis or pneumonia) that extends to the hilum

Irregardless o f size the tumor Pancoast (Superior


• Invades the chest wall Sulcus) Tumor that is
Tumor is
T3 • Invades the pericardium limited to
5-7 cm
• Invades the phrenic nerve (diaphragm paralysis) involvement of T1
• Has one or more satellite nodule in the same lung lobe and T2 nerve roots

Irregardless of size the tumor


Pancoast (Superior
• Invades the mediastinal fat or great vessels
Tumor Sulcus) Tumor that
T4 • Invades the diaphragm
> 7cm involves level C8 or
• Involves the carina
higher
• Has 1 or more satellite nodule in another lobe but the same lung

Pulmonary Vein Anatom y Trivia:


A more detailed look at the staging criteria states specifically
the “intrapericardial portions of the superior and inferior
right and left pulmonary veins, is classified as T4.” This
kind of trivial minutia makes great MCQ fodder.
So when is the intrapericadial portion involved ?
The key is to draw a line from the pericardium (arrow) through
the pulmonary vein (dotted black line). Anything deep to that
(checkered in the drawing) is considered T4.
Vocab:
Multiple Lesions: The handling of multiple lung lesions is complicated. Synchronous =
Deciding if lesions are going to be treated as synchronous primaries or a single Two or more
lung cancer with metastatic disease often requires a discussion at tumor board primary
(after imaging, and path results). carcinomas which
coexist at the time
If the lesions are decided to be separate primaries each cancer will be staged of diagnosis.
separately within the TNM system and given an overall stage. If the lesions are
decided to be metastatic their distribution will alter the stage. Metachronous =
A cancer that
develops
consequently
(some time
interval) after the
first primary.
T3 (two in same lobe) T4 (two in different lobes,
but still the same lung) (two in different lungs)
69
lung Cancer Staging Continued - Nodal Disease:
Nodes

Ipsilateral within the lung N1 is a worse prognosis than NO (no nodes) but
N1 up to the hilar nodes. the management is not changed.

In many cases NOT Resectable


N2 Ipsilateral mediastinal or subcarinal nodes Only those with microscopic disease (negative
mediastinoscopy) will benefit from resection

Contralateral mediastinal
or contralateral hilum. NOT Resectable
N3 *pwbably
Or
Scalene or Supraclavicular nodes.

First it is im portant to point out that CT is unreliable for nodal staging. PET-CT is far
superior, regardless o f the size threshold that is chosen. This is why PET is pretty much
always done on lung cancer patients prior to surgical evaluation.

For the purpose o f multiple choice (and real life) the m ost im portant anatomy boundary to
consider is the distinction between level 1 nodes (which are N3) and level 2 nodes (which are
N2).

In some cases, this Level 1


can literally make (N3)
the difference
between resectable
disease or not. The Level 2
border is the lower (N2)
level o f the
clavicles / upper
border o f the
manubrium (above
this is level 1).

Typical Contraindications to Lobectom y I Resection


The big issues (things that m ake lobectom y im possible) are going to be:
• Grow th o f the tum or through a fissure
• Invasion o f the Pulm onary Vasculature Stage 3B implies
• Invasion o f the main bronchus N3 or T4 disease
• Invasion o f both the upper and low er lobe bronchi.
O ther findings that will m ake the tum or N O T resectable:
• N 2 * (if the tum or is > 5cm ) or N3 N odal D isease — corresponding to a Stage 3B cancer
• M ulti-lobar Disease
• M alignant Pleural Effusion

Trivia: Prevascular Lymph nodes can NOT easily be reached by mediastinoscopy.

70
Lung Cancer - Treatment:
Wedge Resection vs Lobectomy: This is on the fringe of what should be considered fair
game, and I’m certain the decision varies by institution and the size and composition (percentage of
brass) of the surgeon’s testicles. In general, if a stage 1A or IB cancer is peripheral and less than 2cm
they can consider a wedge resection. The advantage to doing this over a lobectomy is preserving
pulmonary reserve. If the tumor is larger than 3cm then lobectomy seems to (in general) be a better
option.

Bronchopleural Fistula:

This is an uncommon complication of pneumonectomy,


that has a characteristic look and therefore easy to test.
So normally after a pneumonectomy the space will fill
with fluid. If you see it filling with air then this is the
dead give away. Day 1 Day 2 D ays

If you are a weirdo, you could confirm the diagnosis


Normal - Becomes More Fluid Filled
with a xenon nuclear medicine ventilation study, which
will show xenon in the pneumonectomy space. The
major risk factor is ischemia to the bronchi (disrupted
blood supply from aggressive lymph node dissection,
or using a long bronchial stump).

Compensatory Emphysema
(Postpneumonectomy Syndrome): Day 1 Day 2 Day 3
BP Fistula - Becomes More Air Filled
This is the vocabulary used to describe hyper­
expansion of one lung to compensate for the absence of
the other one.
Radiation Changes
Radiation Changes - The appearance of radiation
pneumonitis is variable and based on the volume of lung Early
Late
involved, how much/long radiation was given, and if within 1-3 months
chemotherapy was administered as well. Dense
Homogenous or consolidation,
Rib Fractures - Ribs within the treatment field are
patchy ground traction
susceptible to degradation and fracture.
glass opacities. bronchiectasis, and
RFA/MWA Pearl - It is normal to see bubbles in the volume loss.
lesion immediately after treatment.

Recurrent Disease - Recurrence rates are relatively high (especially in the first 2 years).
From a practical stand point, I always focus my attention towards the periphery of the radiation bed,
regional nodes, and/or the bronchial stump. A usefiil concept is to focus on morphology - radiation
scarring is usually not round. If you see something with a round morphology - especially if it is
growing over time, that is highly suspicious.

Other findings concerning for recurrence:


• Enhancing solid tissue along the resection line (or bronchial stump) which is enlarging over time.
• New enlarged mediastinal nodes with a short axis greater than 1 cm
• New pleural effusion (that persists on follow ups)

71
- Mimics and Other Cancers -

Pulmonary Hamartoma - This Kaposi Sarcoma: The most common lung tumor
is NOT a cancer, but to the uninitiated is AIDS patients (Lymphoma is number twoV The
can look scary. It is usually described tracheobronchial mucosa and perihilar lung are favored.
as is an Aunt Minnie because it will The buzzword is “flame shaped.”
have macroscopic fat and “popcorn”
calcifications. It is the most common Key Points:
benign lung mass. It’s usually • Most common lung
incidental, but can cause symptoms if tumor in AIDS
it’s endobronchial (rare - like 2%). (requires CD4 < 200)
• Most common hepatic
Technically the neoplasm in AIDS
fat is only seen • Buzzword = Flame
in 60%, but for Shaped Opacities
sure if the exam • Slow Growth, with
shows it, it will asymptomatic patients
have fat. (despite lungs looking
These can be terrible)
hot on PET, • Bloody pleural
they are still effusion is common
Popcorn (50%). '‘Flame Shaped” Hilar Opacities
benign.
Calcifications • Thallium Positive,
Fat Density Gallium Negative

Mets - Metastatic disease to the lungs can be thought of in 3 categories;

Direct Invasion Hematogenous Mets Lymphangitic


The most common manifestation of Carcinomatosis (LC)
The most common
situation is an hematogenous mets to the lung is the The most common cause of
esophageal carcinoma, pulmonary nodule (usually multiple, unilateral LC is actually
lymphoma, or in a random distribution, and bronchogenic carcinoma lung
malignant germ cell favoring the lower lobes which have cancer invading the lymphatics.
tumor. greater blood volume). “Cannonball The most common
Mets” are classically from renal extrathoracic culprits are
This is seen with
cell or choriocarcinoma (testicle). breast, stomach, pancreas, and
cancer of the
mediastinum, pleura, or The nodules tend to be smoother than prostate.
chest wall. More rarely the primary neoplasm. The main The finding is nodular
you are going to have culprits are breast, kidney, thyroid, thickening of the interlobular
mets to the pleura than colon, and head & neck squamous septa and subpleural
invading the lung. cells - which classically cavitate. interstitium. Unlike interstitial
Even more rarely you Feeding Vessel Sign: A prominent fibrosis, this thickening
can have malignant pulmonary vessel heading into a classically does NOT distort
mesothelioma, which nodule- classic for a hematogenous the pulmonary lobule.
can invade the lung. On origin (although nonspecific). On
multiple choice, it multiple choice think
should be obvious. If
you find yourself 1: Hematogenous mets or
squinting, it is probably 2: Septic emboli.
not real.

72
Lymphoma - There are basically 4 flavors o f pulmonary lymphoma; primary, secondary, AIDS
related, or PTLD. Radiographic patterns are variable and can be lymphangitic spread (uncommon),
parahilar airspace opacities, and/or mediastinal adenopathy.

* Primary: This is rare, and usually


non-Hodgkin in subtype. You define it SL‘Condar\ M il. Scrondiirv III
as the lack of extrathoracic involvement
for 3 months. Almost always (80%) we 80-90% of lymphoma 10-20% of lymphoma
are talking about a low grade cases cases
MALToma.
45% have intrathoracic 85% have intrathoracic
• Secondary: Here we are talking disease at presentation disease at presentation
about pulmonary involvement of a 25% have pulmonary 40% have pulmonary
systemic lymphoma. This is much parenchymal disease parenchymal disease
much much more common than primary
lung lymphoma. The thing to see is that Pulmonary involvement Lung involvement
NHL is much more common, but if you frequently occurs in the almost always associated
have HL it is more likely to involve the absence of mediastinal with intrathoracic lymph
lungs. With HL you gets nodes and disease node enlargement
parenchyma, in NHL you might just get
parenchyma.

• PTLD: This is seen after solid organ or stem cell transplant. This usually occurs within a year of
transplant (late presentations > 1 year have a more aggressive course). This is a B-Cell lymphoma,
with a relationship with EB Virus. You can have both nodal and extra nodal disease. The typical look
is well-defined pulmonary nodules / mass, patchy airspace consolidation, halo sign, and interlobular
septal thickening.

• AIDS related pulmonary lymphoma (ARL) - This is the second most common lung tumor in
AIDS patients (Kaposi’s is first). Almost exclusively a high grade NHL. There is a relationship with
EBV. It is seen in patients with a CD4 < 100. The presentation is still variable with multiple
peripheral nodules ranging from 1 cm-5 cm being considered the most common manifestation.
Extranodal locations (CNS, bone marrow, lung, liver, bowel) is common. AIDS patient with lung
nodules, pleural effusion, and lymphadenopathy = Lymphoma.

Nukes Cross Over Blitz

Thallium is a potassium analog. Things with a functional Na/K/ATP pump tend to be alive.
Hence anything this is “ahve” will be thallium positive.

Gallium is an Iron analog. Iron is an


inflammatory marker (acute phase Kaposi Sarcoma Lymphoma
reactant) hence things that are
Thallium 201 Positive Thallium 201 Positive
“smoldering” tend to be gallium negative,
and things that are inflamed - infection, Gallium67 Negative Gallium®'^ Positive
active sarcoid, most cancers - tend to be
Gallium hot.

In general, it is safe to say “Lymphoma is HOT on Gallium.” Where things can get sneaky is
the subtype. Hodgkin is nearly always Gallium Avid. Certain Non-Hodgkin subtypes can be
Gallium cold. As such (and because it’s not 1970) PET is usually used for staging and not
Gallium.

73
S E C T IO N 6:
C o n g e n it a l

Sequestration, CCAMs, and a bunch of other congenital


path is discussed in detail in the Peds chapter. Here are
some other randos.

Poland Syndrome - Unilateral absence of a pectoral


muscle (white arrow). Can cause a unilateral hyper-lucent
chest. Can have limb issues (small weird arms / hands).
Bronchial Atresia - This is a congenital obliteration of a short segment of a lobar, segmental,
or subsegmental bronchus near its origin (most commonly involves the apical-posterior segment of
the left upper lobe). Like many atresias a vascular insult in utero is the probable etiology.
On CT, you will see a hyper lucent region with decreased vascularity. Bronchial atresia is usually
It is somewhat counterintuitive that loss of the bronchus would make asymptomatic, although
the lung trap air (hyper lucent). The explanation is collateral flow they can present with
through the various pores of Kohn and canals of Lambert. You will recurrent pneumonia.
often see a large “nodule” (sometimes with a branching or “finger in Hyperinflated lobe is the
the glove” morphology) nearby in the hilum — which represents most common radiographic
mucoid impaction in the blind ended bronchus. appearance (sometimes with
a hilar nodule visible).
The primary differential would be an endobronchial tumor. If you can
measure (or they show you) a low density focus (< 25 HU) in the CT will show a blind ending
mucus plug then you are safe to call it mucoid impaction. If you can’t bronchus, filled with mucus
and the distal lung will be
make the distinction —> next step = bronchoscopy.
Hyper-Inflated (lucent).

AVM - They can occur sporadically. For the purpose of multiple choice when you see them think
about HHT (Hereditary Hemorrhagic Telangiectasia / Osier Weber Rendu). Pulmonary AVMs are
most commonly found in the lower lobes (more blood flow), and can be a source of right to left shunt
(worry about stroke and brain abscess). The rule of treating once the afferent vessel is 3 mm is
based on some tiny little abstract and not powered at all. Having said that, it’s quoted all the time,
and a frequent source of trivia that is easily tested.
Persistent Left SVC - This is the most common congenital venous anomaly of the chest. It
usually only matters when the medicine guys drop a line in it on the floor and it causes a confusing
post CXR (line is in a left paramedian location). It usually drains into the coronary sinus. In a
minority of cases (like 5%) it will drain into the left atrium, and cause right to left shunt physiology
(very mild though). This is typically shown on an axial CT at the level of the AP window, or with a
pacemaker (or line) going into the right heart from the left.
Swyer-James - This is the classic unilateral lucent lung. It typically occurs after a viral lung
infection in childhood resulting in post infectious obliterative bronchiolitis (from constrictive
bronchiolitis). The size o f the affected lobe is smaller than a normal lobe (it’s not hyper-expanded).

Horseshoe Lung: Rare as fuck.


Defined by fusion of the posterior basilar
In segments of the lower lobes behind the heart.
The most likely testable points are:
(1) What is it ? and (2) Q: What is the association ?
A: Scimitar syndrome

74
S E C T IO N 7:
C ystic L u n g D isease

Pulmonary Birt Hogg


Langerhans Ceil Lymphangiomyomatosis (LAM) Dube
Histiocytosis (LCH) (BHD)
Smoker, Young (20s-30s) | Child bearing aged women & Tuberous Sclerosis

Starts out with : Pathophysiology is that it is estrogen dependent Gamesmanship:


centrilobular nodules I (why it strongly favors women).
with an upper lobe Gamesmanship: Suggesting TS (without telling Association with
predominance. you the patient has TS) could be the step 1 renal findings
“clinical triad” — even though < 50% of TS (bilateral
These nodules eventually
cavitate into cysts which patients have the complete triad. oncocytomas, and
are thin walled to start, chromophobe
1 - Seizures,
RCCs).
and then some become 2 - Mental Retardation,
more thick walled. 3 - Adenoma Sebaceum
Association with a
Later in the disease you SSS: Seizures, Sebaceum, & Stupid bunch of gross skin
primarily see cysts. stuff.
or Showing the kidneys with multiple AMLs
Thicker Bizarre shaped, THIN WaUed
THIN Walled ROUND Cysts
- occurs when 2 or more OVAL “floppy”
cysts merge together. cysts.
Upper / Mid Zone Lower Zone
Uniform or “Diffuse” Distribution
Predominant predominant and
favor the
50% of cases can paramediastinal
spontaneously resolve This is usually progressive despite attempts at regions.
(especially if you stop hormonal therapy (tamoxifen).
smoking).
Spares the Association with chylothorax
costophrenic angles. (which is HIGH YIELD Trivia).

LCH LAM BHD

Spares Costophrenic Angles


Chylothorax
Bilateral
Renal M asses

75
Lymphocytic Interstitial Pneumonitis
(LIP): This is a benign lymphoproliferative
disorder, with infiUration of the lungs. It has an
association with autoimmune diseases (SLE, RA,
Sjogrens). The big one to know is Sjogrens
which is concomitant in 25% of LIP cases. The
other one to know is HIV - which is the LIP in a
younger patient (children, - LIP in H IV positive
adults is rare). There is also an association with
Castlemans. The appearance of LIP varies
depending on the underlying cause. The cystic
lung disease is usually thin walled, “deep within LIP: Sjogrens / Kids with HIV
the lung parenchyma,” and seen predominantly Cysts and Ground Glass are dominant features.
with Sjogrens. The dominant feature described as
ground glass or nodules is seen more in the other
causes and is far beyond the scope of the exam.

When I say LIP... You say Sjogrens & HIV


When I say LIP in a kid... You say HIV

When I say “perivascular cysts ”you say LIP


LIP often has the descriptor of “Perivascular
cysts” - where multiple vessels seen coursing
Pneumocystis Pneumonia (PCP): along the cysts
The most common opportunistic infection in AIDS.
Cystic form of PCP (classic after
Earlier in the chapter, I detailed how to tell if the
aerosolized prophylaxis), has apical These patients
patient has AIDS on a multiple choice test. predominant thin walled cysts.^V ^^ are high risk for
Remember to look for the classic history of
pneumothorax.
listening to Nickelback or having sex with male
prostitutes while wearing a motorcycle outfit.
The typical buzzwords are “ground glass
appearance, predominantly in the hilar and mid
lung zones”.
Pneumatoceles are present in 30% of cases. In
patients receiving aerosolized prophylaxis, a cystic
form is more common, which may have bilateral
thin walled upper lung predominant cysts.
Gallium scan will show diffuse uptake
PCP: Ground Glass - the principal finding,
(Thallium will be negative).
favors the midlung / peri-hilar regions.
When I say AIDS + Ground Glass Lungs....
You say PCP
When I say AIDS + Pneumothorax You say PCP

Vanishing Lung Syndrome: Vanishing Lung Risk Factors:


This is an idiopathic cause of giant bullous emphysema,
resulting from avascular necrosis of the lung parenchyma • Smoking (tobacco)
and hyperinflation. It favors the bilateral upper lobes, and is • Alpha-1 antitrypsin deficiency
defined as bullous disease occupying at least one-third of • Male
a hemithorax. The most common demographic is a young • Smoking Marijuana is also a
man. About 20% of these guys have alpha-1 antitrypsin described risk factor (although it
deficiency. Tension pneumothorax is a described is probably bullshit propaganda
complication. from the pharmaceutical industry)

76
Not all who wander are lost
Not all who have normal CXRs are normal
Emphysema
The textbook definition is "permanent enlargem ent o f the
“COPD”
airspaces distal to the term inal bronchioles accom panied by
destruction o f the alveolar wall without clear fibrosis.” This is a diagnosis made with
spirometry (airflow obstruction
Things to know: (1) the CXR findings and (2) the different
that is not fully reversible).
types. Even in severe cases
(especially those caused by
CXR Findings: Until it’s really really bad, CXR doesn’t have chronic bronchitis) you can
have a pretty normal CXR.
direct signs, but instead has indirect signs. Flattening o f the
hemidiaphragm s is regarded as the m ost reliable sign. The Testable pearl - not everyone
with COPD has bad
AP diameter increases. The retrosternal clear space becom es
emphysema, and bad COPD
larger. There is a paucity of, or pruning o f the blood vessels. can have a normal CXR.

Classic “Flattening” of the Diaphragm


Defined as the highest level of the dome less than 1.5
cm above a line drawn between the costophrenic and
the Cardiophrenic Sulci Enlarged Retrosternal Air Space

Trivia:

• Saber Sheath Trachea - Diffuse coronal


narrowing o f the trachea, sparing the
extrathoracic portion. This is said to be
pathognomonic for COPD.

• Surgery to remove bad lung “volum e Normal


reduction” is sometimes done Saber Sheath

If the M ain PA is larger than the A orta P A :A > 1 :


COPD (PA/A Ratio > 1) patients have a PA (Black Arrow)
Larger than Aorta
worse outcome (pulm onary HTN can be
(White Arrow)
caused by emphysema).

77
Emphysema - The legend Ceminues
Types:
Upper Lobes
Central
• Centrilobular: By far the most
common type. Com m on in -Smoker
asymptomatic elderly patients. It has
an apical to basal gradient - favoring
the upper zones o f each lobe. It
appears as focal lucencies, located
centrally w ithin the secondary
pulm onary lobule, often with a
central dot representing the central
bronchovascular bundle.

This central dot sign (the vessel


within the secondary pulm onary lobule
is a buzzword.

This is the type o f em physem a


dominant in sm okers.

Panlobular: In contradistinction to centrilobular


this one favors the low er lobes. It also has a more
uniform distribution across parts o f the secondary
pulm onary lobule. The association is w ith alpha 1
antitrypsin. A piece o f trivia is the “Ritalin Lung”
from IV Ritalin use can also cause a pan-lobular
appearance (“Ritalin keeps you from ‘trypsn’ o u f ’)-
I f they show this it w ill be in the coronal view on C T
to dem onstrate the lower lobe predominance.
Lower Lobes / Diffuse
Patient’s will present in their 60s and 70s (unless they
-Alpha 1 Anti-trypsin Def
smoke - then they present in their 30s). Sm oking
-IV Ritalin
accelerates the process.

Paraseptal: This one is found adjacent to the pleura


and septal lines with a peripheral distribution w ithin the
secondary pulm onary lobule. The affected lung is almost
always sub-pleural, and dem onstrates small focal lucencies
up to 10 m m in size. This looks hke honeycom bing but is
less than 3 bubbles thick. Sub-pleural
-Smoking vs Idiopathic

78
S E C T IO N 8:
P n eu m o co n io sis

As a general rule, these are inhaled so they tend to be upper lobe predominant. You can have
centrilobular nodules (which makes sense for inhalation), or often perilymphatic nodules - which
makes a little less sense, but is critical to remember * especially with silicosis & CWP.

Asbestos Exposure: The term “Asbestosis” refers to the changes of pulmonary fibrosis -
NOT actual exposure to the disease. The look is very similar to UIP, with the presence of parietal
pleural thickening being the “most important feature” to distinguish between IPF and Asbestosis.
Obviously, the history of working in a ship yard or finding asbestos bodies in a bronchoalveolar
lavage is helpfiil.

Things to know about Asbestos:


• “Asbestosis” = the lung fibrosis associated with
exposure, NOT actual exposure
• Interstitial pattern looks like UIP + parietal pleural
thickening
• There is a 20 year latency between initial exposure and
development of lung cancer or pleural mesothelioma
• There is an association with extrapulmonary cancer
including: Peritoneal mesothelioma, GI cancer. Renal
Cancer, Laryngeal Cancer, and Leukemia
• Benign pleural effusions are the “earliest pleural-based
phenomenon” associated with exposure - still with a lag
time of around 5 years “Holly Leaf”

Benign Asbestosis Related Changes:


Pleural effusion is the earliest and most common. Pleural
plaques may develop around 20-30 years, with calcifications
occurring around 40 years.
These plaques tend to spare the apices and Costophrenic
angles. Described as a “Holly L eaf’ appearance on CXR.

Round Atelectasis: Atelectasis associated with pleural


findings is sometimes called the “asbestos pseudotumor. ”
The appearance of rounded atelectasis is classically
described as a mass like opacity, next to pleural thickening /
scaring, with a swirl or “comet tail” convergence of the
nearby broncho vascular structures.
Gamesmanship: If there isn’t adjacent pleural thickening
- its not round atelectasis... it is a cancer.
Next step: PET/CT and “biopsy the motherfucker.” Round Atelectasis

Malignant Mesothelioma - About 80% of them have had asbestos exposure (NOT dose-
dependent). The lag time is around 30-40 years from exposure. I’ll discuss this more later in the
chapter with our dedicated section on the pleura.

79
Silicosis: This is seen in miners, and
quarry workers. You can have simple
siUcosis, which is going to be multiple
nodular opacities favoring the upper
lobes, with egg shell calcifications o f the
hilar nodes. You also get perilym phatic
nodules. The com phcated type is called
progressive m assive fibrosis (PM F).
This is the form ation o f large m asses in
the upper lobes with radiating strands.
You can see this with both silicosis and
coal workers pneum oconiosis (something
similar also can happen with Talcosis).
These masses can sometimes cavitate -
but you should always raise the suspicion Progressive Massive Fibrosis
o f TB when you see this (especially in the -Large Apical Masses with Radiating Strands
setting o f silicosis).
M R I : Cancer vs PM F
Silicotuberculosis: Sihcosis actually raises your
Cancer = T2 PM F = T2
risk o f TB by about 3 fold. If you see cavitation in the Dark
Bright
setting o f silicosis yo u have to think about TB.

Coal Workers Pneumoconiosis: This is the result o f exposure to “w ashed coal.”


Just like silicosis there are simple and com plicated forms. There is also an increased risk o f
TB (just like silicosis). The simple form was multiple nodular opacities, w ith calcifications
showing a central nodular dot. The small nodule pattern tends to have a perilym phatic
distribution. The com plicated form gives you a progressive m assive fibrosis that is
similar to that seen in silicosis.

Additional Inhalational Diseases - Not Worthy of a Full Discussion


j

G eneralized granulom atous disease


M etal used in aircraft and
Berylliosis with hilar adenopathy and upper lobe
space industries
predom inant reticular opacities.

Pulm onary Edem a Pattern. Recovery


Silo Filler’s Disease N itrogen Dioxide
is typically w ithin 5 weeks.

Filler in tablets, sometimes Hyperdense m icronodules, with


Talcosis injected (along w ith drugs) conglom erate masses (sim ilar to PMF).
in IV drug users. G round glass opacities

80
S E C T IO N 9:
ILDS
D o n ’t Be S cared H o m ie

Everyone seems to be afraid o f interstitial lung diseases. The concept is actually not that
complicated, it’s ju st com plicated relative to the rest o f chest radiology (which overall isn’t
that com phcated). The trick is to ask yourself two m ain questions: (1) Acute or Chronic ? -
as this narrows the differential considerably, and (2) W hat is the prim ary finding ? - as this
will narrow the differential further. Now, since we are training for the artificial scenario o f a
multiple choice test (and not the view box). I ’ll try and keep the focus on superficial trivia,
and associations. Rem em ber w hen you are reading to continue to ask yourself “how can
this material be w ritten into a question?”

Vocab: Like m ost o f radiology, the bulk o f understanding the pathology is know ing the
right words to use (plus, a big vocabulary m akes you sound smart).

• Consolidation == Density that obscures underlying vessels


• Ground Glass Opacity = Density that does NOT obscure underlying vessels
• Secondary Pulm onary Lobule = The basic unit o f pulm onary structure and function.
It is the smallest part o f the lung that is surrounded by cormective tissue. In the middle
runs a term inal bronchial with an accom panying artery. A round the periphery runs the
vein and lymphatics.
Anatomy of the Secondary Pulmonary Lobule

outlining tine septa


Anatomy Questions = Fair Game and Easy to Write for MCQs
Stuff like “what goes around the periphery ?” or “what goes through the middle of the lobule T

81
Nodule Vocabulary (Random, Perilymphatic, Centrilobular)

Random Perilymphatic Centrilobular

Telling them apart can be done by fir s t asking i f they abut the pleura?

If the answer is no they are


centrilobular.
Subpleural and
Perilymphatic
Peribronchovascular
If the answer is yes, then Pleural Nodules
ask do they follow a No Predominance Random
peribronchovascular
pattern,

if the answer is no then No Pleural Nodules Centrilobular


they are random, if the
answer if yes then they are
perilymphatic.

Nodule Pattern Key DDx

•Sarcoid (90%),
Perilymphatic •Lymphangitic Spread of CA
•Silicosis

•Miliary TB
Random •Mets
•Fungal

•Infecdon
Centrilobular •RB-ILD
•Hypersensitivity Pneumonitis (if ground glass)

82
Patterns
Interlobular Septal Thickening:
Reticular abnormality, that outlines the
lobules’ characteristic shape and size (about 2
cm). It’s usually from pulm onary edem a
(usually symmetric a n d smooth), or
lym phangitic spread o f neoplasm {often
asymmetric and nodular). K erley B Lines
Smooth & Nodular &
are the plain film equivalent. Symmetric Asymmetric

Honeycombing: Cystic areas o f lung destruction in a subpleural location. This is a


hallmark o f UIP. Paraseptal em physem a is a mimic, but the distinction is made by how
many rows o f bubbles.

One Row of Bubbles = Two-Three Rows of Bubbles


Paraseptal Emphysema = Honeycombing.

Pathology
Idiopathic Interstitial Pneum onias - These are N O T diseases, but instead lung reactions to
lung injury. They occur in a variety o f patterns and variable degrees o f inflam m ation and
fibrosis. The causes include: idiopathic, collage vascular disease, m edications, and
inhalation.

For practical purposes the answ er is either (a) UIP or (b) N ot UIP. N ot UIP w ill get better
with steroids. UIP will not. UIP has a dismal prognosis (sim ilar to lung cancer). N ot UIP
often does ok. The exam will likely not m ake it this simple, and will instead focus on
buzzwords, patterns, and associations (w hich I w ill now discuss).

83
UIP (Usual Interstitial Pneumonia) - The m ost common ILD. W hen the cause is
idiopathic it is called IPF. On CXR the lung volume is reduced (duh, it’s fibrosis). Reticular
pattern in the posterior costophrenic angle is supposedly the fir s t fin d in g on CXR.

Buzzwords include: Definite UIP Criteria:


Honeycombing (must be present)
Apical to basal gradient (it’s worse in the
• +/- traction bronchiectasis
lower lobes) Reticular abnormalities
Traction bronchiectasis, and honeycombing • Subpleural basal predominant distribution
Honeycombing is found 70% o f the time, ' Absence of inconsistent features: upper lobe
and people expect you to knee jerk UIP when predominance, ground glass > reticulations,
air trapping involving 3 or more lobes
that term is uttered
Histologic Buzzword = Heterogeneous. Definite UIP Pattern =
Don’t Need to Biopsy to Prove Diagnosis
“Histology was heterogeneous” = UIP

It’s important to know that basically any end This vs That: Chronic Hypersensitivity
stage lung disease (be it from sarcoid, RA, Pneumonitis (HP) vs UIP.
Scleroderma, or other collagen vascular disease) Features that favor Chronic HP over UIP;
has a similar look once the disease has ruined the • Air Trapping involving 3 more lobes
lungs. ^Technically honeycombing is uncommon
• Mid-Upper lobe predominant fibrosis
in end stage sarcoid - but the rest o f the lung
looks ja c ked up.
The prognosis is terrible (similar to lung cancer). UIP has an association with smoking.

UIP- Honeycombing, Traction Bronchiectasis, Apical-to-Basal Gradient

Fibrosis and Cancer Go Together Like Tacos and Tuesdays


Pulmonary fibrosis is a risk factor for lung cancer
(especially if these patients are dumb enough to also smoke).

Cancer in the Fibrotic Lung Trivia;


• Favors the lower lobes
• Favors the interface between the fibrotic cvsts and normal lung
• Progressive wall thickening or a developing nodule within a cyst is suspicious for cancer
(enlarging pericystic nodules are dodgy as tuck - hide your kids / hide your wife)
• NELSON Lung Cancer Screening Trial showed cancers associated with cystic lesions were
commonly missed (people thought they were just areas of focal thickening within a bulla).
— Then I said “Come on Judge... let him go, h e ’s doing his best. ”

84
NSIP (Nonspecific Interstitial Pneumonia)

Less Comm on than UIP. Even though the nam e infers that its non-specific, it’s actually a
specific entity. H istologically it is hom ogenous inflam m ation or fibrosis (UIP was
heterogeneous). It is a com m on pattern in collagen vascular disease, and drug reactions.

It comes in 2 flavors (cellular or fibrotic):

■ Ground Glass Alone = Cellular


■ Ground Glass + Reticulation = Cellular or Fibrotic
■ Reticulation + Traction Bronchiectasis = Fibrotic NSIP
■ Honeycombing - uncom m on and usually m inim al in extent

Trivia: N SIP is the m ost common Interstitial Lung D isease in Scleroderma

The disease has a lower lobe, posterior, peripheral predom inance with sparing o f the
immediate subpleural lung seen in up to 50% o f cases. This finding o f im m ediate
subpleural sparing is said to be highly suggestive. G round glass is the NSIP equivalent
o f honeycombing.

UIP NSIP
G radient is less obvious
Apical to Basal Gradient
(but still more in low er lobes)

Heterogeneous Histology H om ogenous H istology

Honeycombing G round Glass

Traction Bronchiectasis M icronodules

85
RB-ILD and DIP:

I’m going to discuss these two together because some people feel they are a spectrum. For
sure they are both sm oking related diseases.

• RB-ILD - Apical Centrilobular ground glass nodules


• DIP - More diffuse GGO, with patchy or subpleural distribution

Some people would just say “fuck it” and stop calling anything DIP. Just call them all RB-ILD and
the bad ones “end stage” — instead of DIP. Until then - think RB-ILD as mild, and DIP as more
severe.

RB-ILD: This tends to be DIP: Desquamative interstitial pneum onia is thought


more upper lobe o f as the end spectrum o f RB-ILD, and generally seen
predom inant (note that in 50 year old heavy smokers. Extras from 1984
DIP tends to be more com edy classic “Revenge o f the N erds” seem to enjoy
lower lobe predominant). going to radiology conferences to voice their
Localized centrilobular disapproval o f the term “D IP” because the pathology is
ground glass nodules (due not desquam ation o f alveolar epithelium but instead
to macrophages filling up represents a filling o f alveolar spaces with
the terminal bronchioles). macrophages. Could these same weirdos be writing the
The pathology tends to questions ? The dark side clouds everything.
involve the entire cross Impossible to see the future is.
section o f lung.
Just think peripheral lower lobe predom inant ground
Respiratory Brochiolitis + glass, with small cystic spaces.
Symptoms = RB-ILD.
You can see
consolidations
- but they are
usually
associated with
cryptogenic
pneumonia.

Localized centrilobular
ground glass nodules -
apical predominant. You can see DIP - Bilateral Fairly Symmetric Basilar
fibrosis - but it is Predominant Ground Glass
rare (like 5-10% ) - 1 w ould not expect that on the exam.

86
Sarcoid: CXR can be used to ‘^Stage” Sarcoid

This is a m ulti-system disease that creates Stage 0= N orm al


“non-caseating granulom as.” The classic Stage 1= H ilar / M ediastinal N odes Only
age is between 20-40. Along those lines, Stage 2= N odes + Parenchym a Disease
if the header to the question describes an Stage 3= Parenchym al Disease
African A m erican female in her 20s-30s Stage 4= End Stage (Fibrosis)
the answer is probably sarcoid. The lungs
are by far the most com mon organ affected “Acute Sarcoid” is sometimes referred to
(90%). as “Lofgren Syndrome.” The name sorta
sounds French to me, making it high yield.
Misc Trivia to know: The classic triad is
• Elevated angiotensin-converting enzyme (ACE) (1) Bilateral hilar lymph node
• Hypercalcemia enlargement,
(2) Arthritis (usually in the ankles), and
(3) Erythema Nodosum

M ediastinal lymph nodes are seen in 60-90% o f patients (classically in a 1-2-3 pattern o f
bilateral hila and right paratracheal). They have perilym phatic nodules, w ith an upper
lobe predom inance. Late changes include, upper lobe fibrosis, and traction bronchiectasis
(honeycom bing is rare). Aspergillomas are com m on in the cavities o f patients with end
stage sarcoid.
• 1-2-3 Sign - bilateral hila and right paratracheal
• Lambda Sign - sam e as 1-2-3, but on Gallium Scan
• CT Galaxy Sign - upper lobe masses (conglom erate o f nodules) with satellite nodules

1-2-3 Sign Panda refers


Lambda Sign
“Garland Triad” to the hot CT Galaxy Sign
Panda Sign lacrimal glands
(eyes),
nasopharynx
(nose) and
parotid glands -
seen on Gallium

“Galaxy Sign” - refers


to the CT appearance of
a mass-like central core
with peripheral nodules
resembling a galaxy.
Like many thoracic
Lambda refers to the Greek signs - it is nonspecific
The nodes classically do not
Symbol that resembles the HOT and you can see
abut the cardiac border (in
Hilar and Right Paratracheal something similar with
Lymphoma they would).
Nodes on Gallium Scan TB.

87
S E C T IO N 10:
TRANSPLANT

Lung transplants are done for end-stage pulmonary disease (fibrosis, COPD, etc..)- The complications
lend themselves easily too MCQs, and are therefore high yield. The best way to think about the
complications is based on time.

Immediate Complications (< 24 hours)


Donor-Recipient Size Mismatch up to 25% is ok. You can have a compressed lung (by the
Mismatch hyperexpanded emphysematous lung). Imaging is usually atelectasis.
Secondary to HLA and ABO antigens. It’s rapid and often fatal.
Hyperacute Rejection
Imaging shows massive homogenous infiltration
Early Complications (24 hours - 1 week)
Peaks at day 4 as a non-cardiogenic edema related to ischemia-
Reperfusion Injury
reperfusion. Typically improves by day 7.
Air Leak / Persistent
Defined as a continuous leak for more than 7 days.
Pneumotliorax
Intermediate Complication (8 days - 2 months)
IGround (Hass opacities and intralobular septal thickening.
Acute Rejection
(No ground glass = no rejection). Improves with steroids.
Bronchial Anastomotic
Leaks occur in the first month, stenosis can develop later (2-4 months).
Complications

Late Complications (2-4 months)


iThe most common opportunistic infection.
CMV Infection
Ground glass, trec-in-bud. Rare before 2 weeks.
Later Complications (> 4 months)
Bronchiolitis Obliterans; Affects 50% at 5 years.
Chronic Rejection
Brochiectasis, bronchial wall thickening, air trapping.
Cryptogenic Organizing Occurs with chronic rejection (but more commonly with acute
Pneumonia rejection). Responds to steroids.
PTLD................................. Typically seen within the first year. EBV in 90%.
Upper Lobe Fibrosis Associated with chronic rejection
Chronic Rejection /Bronchiolitis Obliterans Syndrome: This is the major late
complication, that affects at least half of the transplants at 5 years (most commonly at 6 months).
The term bronchiolitis obliterans is often used interchangeably with chronic rejection. The findings
on CT include bronchiectasis, bronchial wall thickening, air trapping, and interlobular septal
thickening. Just think air trapping on expiration seen at or after 6 months = chronic rejection.
Recurrence of Primary Disease after Transplant: For the purpose of multiple
choice tests know that sarcoidosis is the most common recurrent primary disease (around 35%).
Lots of other things can recur.
Lung Cancer after Transplant: Just remember that the native lung is still diseased, and
can get cancer. The highest rate is with pulmonary fibrosis, and the most common risk factor is
heavy tobacco use.

88
S E C T IO N 1 1:
A lv eo la r

Pulmonary Alveolar Proteinosis (PAP): Condition where surfactant


accumulates in the alveoli and terminal bronchioles. For the purpose of multiple choice, this is an
Aunt Minnie - always shown as crazy paving lung (interlobular septal thickening with ground glass).
This can be primary (90%), or secondary (10%). The secondary causes worth knowing are cancer or
inhalation (silico-proteinosis).

Trivia Worth Knowing:


• They are at increased risk of Nocardia infections, and can have Nocardia brain abscesses
• Smoking is strongly associated with the disease
• When seen in children (presenting before age 1) there is a known association with
alymphoplasia
• Can progress to pulmonary fibrosis (30%)
• Its typically central and symmetric and tends to spare the apices and costo-phrenic angles
• Treatment is bronchoalveolar lavage

Crazy Paving - Interlobular septal thickening and ground glass.

This isn’t always PAP, in fact in real life that it is usually NOT PAP. There is a differential that
includes common things like edema, hemorrhage, BAC, Acute Interstitial Pneumonia, and COVID.

Just know that for the purpose of multiple choice tests, the answer is almost always PAP.

CraZy PaviNg
Interlobular Septal Thickening + Ground Glass.

M em ory A id fo r causes o f crazy pavin g = C.R.A.Z. Y Paving: COP/Cancer (BAC)/Covid; Respiratory distress (acute);
Alveolar proteinosis; Zipper (lipoid pneumonia - think o f being so f a t (lipoid) that you can't “Zip ” up your pants); Y (for Y
are you hemorrhaging? - pulmonary hemorrhage); PCP.

89
Lipoid Pneumonia - There are actually two types; endogenous and exogenous.

* Exogenous: A certain percentage o f elderly people becom e absolutely obsessed with


their bowel movements. If you did Family M edicine addressing this psychopathology
w ould steal a certain am ount o f hours out o f your life per week. Lipoid pneum onia is
seen in old people who like to drink/aspirate m ineral oil (as a laxative). It can also be
seen with the aspiration o f vegetable oil or other animal oils. The look on plain film is an
area o f lung opacification that is chronic or slowly increases with time. The look on CT
is a dead give away and the m ost likely way this will be shown is with low attenuation /
fat density in the consolidation. Having said that this is also in the crazy paving
differential.

* Acute Exogenous L ipoid Pneum onia - This is seen in children who accidentally poison
themselves with hydrocarbons, or idiots trying to perform fire-eating or flame blowing.

* Endogenous - This is actually more com mon than the exogenous type, and results from
post obstructive processes (cancer) causing build up o f lipid laden macrophages.

Lipoid Pneumonia - Fat Density in the Consolidation

Gamesmanship: “Why are you showing me the lung on that window?”

Obviously pulmonary pathology is best shown on a lung window. So anytime the test
writer is showing you a pulmonary pathology on a non-lung window, that should cue
you to think about some different things.

(1); Is the finding in the mediastinum or ribs?


(2); If it’s clearly a lung finding then what window are they using?
• Soft tissue window is classically used to show fat in a lesion
- think hamartoma or lipoid pneumonia.
• Bone window might be used to show a diffuse process such as
- pulmonary microlithiasis.

90
Organizing Pneumonia (cryptogenic / cause not known “C O P ”)

This used to be called BOOP, which was a lot more


fun to say. It’s not an active infection (as the name
would suggest) but instead granulation tissue
deposition within the alveolar spaces secondary to
fibroblast proliferation. Because it is not an active
infection antibiotics won’t help. However, it does
respond well to steroids, and has an excellent
prognosis.

There are lots of different causes; idiopathic, prior


infection, drugs (amiodarone), collagen vascular
disease, fumes, etc... You will see the word
“cryptogenic” used when there is no specific cause
known. Then it is COP instead of OP.

The history of “persistent symptoms following


treatment for pneumonia” could be a clue to think Cryptogenic Organizing
OP. The most common “persistent symptoms” are Pneumonia- Reverse Halo
a cough lasting several months, low grade fever,
SOB, and just generally feeling like shit.

Patchy air space consolidation or GGO (90%), in a peripheral or peri-bronchial distribution


(sometimes a perilobular/arcade-like pattern). Opacities tend to be irregular in shape. Both OP and
Chronic Eosinophilic Pneumonia can present as peripheral consolidations. Findings of fibrosis are
typically absent.

R everse Halo (Atoll) Sign is the classic sign - seen in around 30% o f cases;
Consolidation around a ground glass center.

**Reverse Halo Pattern is also a described typical finding o f acute COVID.

Chronic Eosinophilic Pneumonia:


Can be idiopathic or associated with a known ■
antigen. Peripheral eosinophilia (blood test) is
usually present. An asthma history is found in
about 50% of cases. It looks exactly like COP
(both can present as peripheral consolidations).
When you say COP you should say this one too
(some people think it’s the same disease as COP).

Classic Look on CXR:


• Bilateral
• Non-segmental Consohdation
• Favoring the outer 2/3 of the lungs (peripheral)

CT Findings: Peripheral GGO or consolidation.


Upper lobes tend to be favored.

91
THIS vs THAT: Halo Signs

Halo Reverse (Atoll)

-Nodule with ground glass around it


Central ground glass with rim o f
-Represents hem orrhage / invasion into
consolidation
surrounding tissues

-C O P (Classic)
- Invasive A spergillosis (Classic)
-T B
- O ther Fungus
- Pulm onary Infarct
- Hemorrhagic M ets
- Invasive Fungal and W egeners
- Adenocarcinom a in Situ (BAC)
** these can also be seen with regular Halo
- Wegeners
- COVID

THIS vs T H A T :
Hypersensitivity Pneumonitis:
Chronic Hypersensitivity
This is actually common. It’s caused by inhaled organic antigens. It Pneumonitis (HP) vs UIP.
has acute, subacute, and chronic stages. Most of the time it’s imaged Features that favor Chronic
in the subacute stage. HP over UIP:
• Air Trapping involving 3
•Subacute: Patchy ground glass opacities. Ill-defined Centrilobular or more lobes
ground glass nodules (80%). Often has mosaic perfusion, and air
trapping. • Mid-Upper lobe
predominant fibrosis
•Chronic: Looks like UIP + Air trappins. You are gonna have traction
bronchiectasis and air trapping.

Buzzword is “Headcheese” - which I guess refers to


some gross poverty food where all the left overs that
were going bad in the fridge get mixed together into
' some kind of bizarre casserole you’d expect in a book
from the notorious racist Dr. Seuss.

The technical finding is a juxtaposition of 3 different regions


• Ground Glass (increased attenuation),
• Mosaic Attenuation - From Air Trapping (decreased Headcheese
attenuation - worse of expiration), Ground Glass +
• Normal Lung Air Trapping + Normal Lungs
The explanation for the finding is that the disease causes both
an infiltrative process (ground glass) and also and obstructive process (mosaic attenuation / air-trapping)

92
S E C T IO N 12:
A irw ays

Anatomy The basic anatomy o f the trachea is a bunch o f anterior horseshoes o f cartilage,
with a posterior floppy membrane. This membrane can bow inward on expiratory CT (and this
is normal). The transverse diameter should be no more than 2.5 cm (same as the transverse
diameter o f an adjacent vertebral body). It’s normally 1 - 3 mm thick.

Saber-Sheath Trachea:
Coronal diam eter o f less than two thirds the sagittal diameter.

I say “saber-sheath trachea,” you say COPD.

Trivia: The main bronchi will be normal in size.


The tracheal wall will be normal in thickness.
Saber Sheath
Tracheal Disease Game Plan: Three big questions to ask:
(1) Does it involve the posterior membrane ?
(2) is it focal or diffuse ? and
(3) Are there calcification ?

Relapsing Polychondritis: Spares the posterior membrane. Diffuse


anterior and lateral thickening o f the trachea. No calcifications. Characterized by recurrent
episodes o f cartilage inflammation, and recurrent pneumonia.

Wegener’s: Circumferential thickening, which can be focal or long segment.


No calcifications. Subglottic involvement is common.

Tracheobronchopathia Osteochondroplastica (TBO):

Spares the posterior


membrane.

You have
development o f
cartilaginous and
osseous nodules
within the submucosa
o f the tracheal and
bronchial walls.
TBO - Note the sparing of the posterior membrane (arrow)

Amyloidosis: Irregular focal or short segment thickening, which can involve the posterior
membrane. Calcifications are common.

93
Tracheal / Bronchial Disease Summary
Spares the Posterior Membrane Does NOT Spare the Posterior IVIembrane

Recurrent episodes of
cartilage inflammation
(ears, nose, joints, Often confined to the
Relapsing laryngeal and thyroid trachea and main
Amyloid
Polychondritis cartilage). Recurrent bronchi. Calcifications
pneumonia is the most are common.
common cause of
death.

Post Intubation Focal Subglottic

Development of
Tracheobronchopathia cartilaginous and C-ANCA+, Sub-glottic
Osteochondroplastica osseous nodules. Wegeners trachea is the most
(TBO): Typically occurs in common location.
men older than 50.

Tracheal / Bronchial Tumors


Pulm onary N euroendocrine Tumors
1 AIRWAY MOT liN THE AIRWAY J

Typical Carcinoid A typical Carcinoid Large Cell (LCNEC)


-Low-Grade M alignant- -Interm ediate M alignant- and Sm all Cell (SCLC)
-High Grade M alignant-
< 3cm > 3cm
LCNEC: Peripheral
More Likely to Be M ore Likely to Be
pulm onary mass -
Central (tracheal Peripheral (distal to the
around 3.5 cm
bifurcation) segm ental bronchi).
SCLC: Large central /
More likely to appear to M ay not show a
m ediastinal mass
be within the lumen o f bronchial relationship -
involving the hilum
the airway. M ore likely or be partially
to be calcified (-30% ). endobronchial. Trivia: Bronchial
Age ~ 60s Carcinoid is more likely to
A ge ~ 50s Met inside the eyeball
Association with Sm oking (uveal tract). Where as a
GI carcinoid is most likely
to met to the extra ocular
Typical & Atypical carcinoids show enhancement post contrast.
muscles

94
Tracheal / Bronchial Tumors Continued
Tumors o f the trachea are not com m on in the real world.

Extrem ely rare in the trachea (< 1%).


More typically seen Endobronchial
Carcinoids usually have a central endobronchial location
(although they can rarely be found in the pulm onary parenchym a-
presenting as a nodule or mass in an older patient).
Pulmonary carcinoid tends to be slow grow ing and locally
invasive (only m et to nodes about 10% o f the time).
No surprise they can cause obstructive symptoms. They can also
cause hem optysis because they are highly vascular.
Carcinoid
For the purpose o f multiple choice = No association w ith smoking
(**there maybe an association with the atypical sub-type)
An octreotide scan can be used to localize a carcinoid tumor.
PET can be falsely negative in ~ 25% o f cases (they can be cold
on PETI
Rarely they can cause a carcinoid syndrome w ith flushing e tc ...
The valvular degradation that occurs tends to be on the left side
(mitral and aortic), as opposed to the GI carcinoid syndrome
which targets the right side (tricuspid and pulm onic).

M ost com m on tracheal malignancy


Favors the upper trachea, and prefers the posterior lateral trachea
20x more likely to be in the trachea (relative to carcinoid).
Adenoid Cystic Has a variable look - can be thickening, a mass, or a nodule.
It is NOT associated with smoking.
They are usually in the main or lobar bronchus.
Submucosal location so overlying m ucosa is frequently intact.

Most Com m on tracheal malignancy.


A ssociated w ith smoking,
Squamous Cell
Often multifocal (10% ), favors the lower trachea / proxim al
bronchus

Mets Usually via direct extension (lung, thyroid, esophagus)

M ost com m on benign tum or o f the trachea


Squam ous Cell
W hen it’s a single papillom a think sm oking.
Papillom a
W hen it’s m ultiple papillom as think HPV.

95
Subglottic Stenosis
Post Intubation Stenosis - is the most common cause o f
subglottic tracheal stenosis in an adult.
Classic Look: Focal Subglottic circumferential stenosis,
with an hourglass configuration.

Fuckery: If you see imaging signs that clearly suggest


subglottic stenosis but are told the patient has never been
intubated — then you are looking at the rare case of adult
croup (acute laryngotracheobronchitis) - which can also give
this appearance.

Cystic Fibrosis - The sodium pump doesn’t work and they end up with thick secretions and
poor pulmonary clearance. The real damage is done by recurrent infections.
Things to know:
Bronchiectasis (cylindrical - progresses to varicoid)
It has an apical predominance (lower lobes are less affected)
Hyperinflation
• Pulmonary Arterial Hypertension-
Mucus plugging (finger in glove)

Primary Ciliary Dyskinesia: Those little hairs in your lungs that clear secretions don’t
work. You end up with bilateral lower lobe bronchiectasis (remember that CF is mainly upper lobe).
Other things these kids get is chronic sinusitis (prominent from an early age), and impaired fertility
(sperm can’t swim, girls get ectopics). They have chronic mastoid effusions, and conductive hearing
loss is common (those little ear nerve hair things are fucked up too). An important testable fact is
that only 50% of the primary ciliary dyskinesia patients have Kartagener’s Syndrome.

CF Primary Ciliary Dyskinesia


Abnormal Mucus, Cilia can’t move it Normal Mucus, Cilia don’t work
Normal Sperm, Absent Vas Deferens Abnormal Sperm (they can’t swim). Normal Vas Deferens
Upper lobe bronchiectasis Lower lobe bronchiectasis

Williams Campbell Syndrome - Huge zebra that manifests as congenital cystic


bronchiectasis from a deficiency of cartilage in the 4*-6* order bronchi.

Mounier-Kuhn (Tracheobronchomegaly)
If you ever say to yourself “that’s one big fucking trachea” - this
is the diagnosis. Defined by a massive dilatation of the trachea
(> 3cm). It’s not well understood and probably the result of a
gypsy curse. “Mounier-Kuhn” sounds like a gypsy curse to me.
The recommended therapy in most cases of a gypsy curse is to
keep the paw of a dead rabbit (3 years old) under your pillow from
the day you feel the first symptoms until the third full moon -
although that is probably beyond the scope of the exam. Mounier-Kuhn—
Note the Big Fucking Trachea

96
Small Airways Disease
Bronchiolitis - This is an inflammation of the small airways. It can be infectious (like the viral
patterns you see in kids) or inflammatory like RB-ILD in smokers, or asthma in kids.
A ir Trapping - When you see areas of lung that are more lucent than
others - you are likely dealing with air trapping. Technically, air
trapping can only be called on an expiratory study as hypoperfusion in
the setting of pulmonary arterial hypertension can look similar. Having
said that, for the purpose of multiple choice test taking, 1 want you to
think (1) bronchiolitis obliterans in the setting of a lung transplant, or
(2) small airway disease - asthma / bronchiolitis.

Tree in Bud - This is a nonspecific finding that can make you think
small airway disease. It’s caused by dilation and impaction of the
centrilobular airways. Because the centrilobular airways are centered
5-10 mm from the pleural surface, that’s where they will be. It’s
usually associated with centrilobular nodules.

Follicular Bronchiolitis - This is an Small Airw ay Disease


inflammatory process seen in rheumatoid Infectious
arthritis or Siogrens. It’s not well understood and Bronchiolitis Tree-in-bud
is related to lymphoid hyperplasia. It looks like
centrilobular ground glass nodules with scattered Smokers. Centrilobular
areas of bronchial dilation. RB-ILD ground glass nodules
(upper lobe predominant)
Constrictive Bronchiolitis - This is
another inflammatory process that can be seen in Sub-Acute Inhaling dust / other misc
viral illness, transplant patients, drug reactions, Hypersensitivity garbage. Centrilobular
or inhalation injury. It is also often identified in Pneumonitis Ground glass nodules
patients with DIPNECH. It occurs secondary to
mononuclear cells which form granulation tissue RA and Sjogrens.
Follicular
and plug the airway. You see air trapping on Centrilobular ground glass
Bronchiolitis
expiratory imaging. This is supposedly the cause nodules. Likes lower lobes
of Swyer-James hyperlucent lung.
Viral, Drugs, Transplant,
Constrictive
Inhalation. Air-Trapping.
Bronchiolitis
Aspiration Pneumonia DIPNECH
Stroked-out old people and
Aspiration Patterns
drunks love to aspirate.
(depends on what you aspirated)
The testable trivia is to know
the typical location of Aspiration of Gastric Acid Airspace opacity, if massive can look
aspiration; posterior segment of “Mendelson s Syndrome ” like pulmonary edema
upper lobes and superior
segment of lower lobes if supine Aspiration of water or
“Fleeting Opacity”
when aspirating, bilateral basal neutralized gastric
that resolves in hours
lower lobes in upright contents
aspiration. May favor the right
Gives you a real pneumonia, can get
side, just like an ET tube. Aspiration of Germs
para-pneumonic effusion, empyema,
(often mouth bugs)
The most common complication or even broncho-pleural fistula.
is infection which can manifest
Aspiration of Oil
as an empyema Lipoid Pneumonia. Low density
(which can then get a broncho­ (often m ineral oil)
pleural fistula).
97
S E C T IO N 13:
S ystem ic

Collagen Vascular Disease - Interstitial lung diseases are common in patients with
collagen vascular diseases. The associations are easily tested, so I made you this chart. I tried
to hit the high points o f testable trivia.

Collagen V ascular Disease Pulmonary Manifestations

More pleural effusions and


Lupus Fibrosis is uncommon. Can
pericardial effusions than with
*its never Lupus get a “shrinking lung.”
other connective tissue disease

Reticulations with or without


Looks like UIP and COP. Lower honeycombing, and
Rheumatoid Arthritis
lobes are favored. consolidative opacities which
are organizing pneumonia

NSIP > UIP; lower lobe Look for the dilated fluid
Scleroderma
predominant findings. filled esophagus.

Extensive ground glass


Sjogrens LIP attenuation with scattered thin
walled cysts.

Usually unilateral first, then


Ankylosing Spondylitis Upper lobe fibrobullous disease
progresses to bilateral.

Caplan Syndrome = Rheumatoid Arthritis + Upper Lobe Predominant Lung Nodules.


These nodules can cavitate, and there may also be a pleural effusion.

“Shrinking Lung” - This is a progressive loss o f lung volume in both lungs seen in
patients with Lupus ( “ S” hrinking “ L ”ung for “ SLe”). The etiology is either diaphragm
dysfunction or pleuritic chest pain.
Trivia: Most common manifestation o f SLE in Chest = Pleurifis with/without pleural effusion.

Hepatopulmonary syndrome - This is seen in liver patients with the classic history o f
"'shortness o f breath when sitting up.'" The opposite o f what you think about with a CHF
patient. The reason it happens is that they develop distal vascular dilation in the lung bases
(subpleural telangiectasia), with dilated subpleural vessels that don’t taper and instead extend
to the pleural surface. When the dude sits up, these things engorge and shunt blood - making
him/her short o f breath. A Tc MAA scan will show shunting with tracer in the brain (outside the
lungs). They have to either tell you the patient is cirrhotic, show you a cirrhotic liver, or give
you that classic history if they want you to get this.

98
Wegener Granulom atosis (Granulomatosis with Polyangiitis)

The classic triad is upper tract, lung, and kidneys


This is now called
(although this triad is actually rare). The lungs are “Granulomatosis with
actually the m ost com mon organ involved (95%). Polyangiitis” because Wegener was
There is a highly variable look. a member of the Nazi party.
It is also possible that he worked
The most com mon presentation is also probably the with the deep science unit Hydra
which plagued the world with
most likely to be tested; nodules with cavitation.
schemes of global domination and
The nodules tend to be random in distribution with genocide.
about h alf o f them cavitating. They can also show Seriously, I ’ve heard the guy was a
you ground glass changes which may represent real asshole. Not just a Nazi, but a
hemorrhage. bad tipper, and a habitual line
stepper (deliberately didn’t wash
his hands after he took a shit).

Nasal Septum Perforation Nodules some of which are Cavitated


Pleural Effusions (seen ~ 15% of the time)

Goodpasture Syndrome -

A nother autoimmune pulm onary renal syndrome. It favors young men. It’s a super
nonspecific look with bilateral coalescent airspace opacities that look a lot like edem a (but
are hemorrhage). They resolve quickly (w ithin 2 weeks). If they are having recurrent
bleeding episodes then they can get fibrosis. Pulm onary hem osiderosis can occur from
recurrent episodes o f bleeding as well, with iron deposition m anifesting as small, ill-defined
nodules.

99
S E C T IO N 14:
P l e u r a , C h est W a l l

Plaque:
Pleural Calcifications
If they show you a pleural plaque they probably want you to say asbestos- (other than asbestos)
related disease. • Old Hemothorax
Old Infection
Remember the plaques: • TB
• Don’t show up for like 20-30 years after exposure. • Extraskeletal
• Typically spares the Costophrenic angles. Osteosarcoma

Malignant Mesothelioma: The most common cancer of the pleura. About 80% of them have
had asbestos exposure, and development is NOT dose-dependent. Lag time is ~ 30-40 years from
exposure.

Key Features: Buzzword = Pleural Rind


• Circumferential Pleural Thickening extending to
the medial surface of the pleura (near the heart)
• Pleural Thickness > 1 cm
• Extension into the fissure = highly suggestive

Gamesmanship:
• If the lateral pleural is thick - the most common
cause is a prior trauma. Look for old rib fractures.
• I f the m edial pleural is thick - this is not a com m on
thing. Think m esotheliom a. Look for plaques to
suggest prior asbestosis exposure. Pleural Rind = Grey Arrows
Fissure Extension = White Arrow
Evaluating Direct Invasion & Mets:
• Mesothelioma believes in nothing Lebowski with a known tendency for direct invasion.
• MRI with Contrast = For evaluating local chest wall, diaphragm and pericardial invasion
•4
• PET CT = Good for mets. Useful for evaluating treatment response.

Solitary Fibrous Tumor of the Pleura (SFTP) - This is a solitary (usually) tumor arising
from the visceral pleura. The key is to know that they are NOT associated with asbestos, smoking,
or other environmental pollutants. They can get very large, and be a source of chest pain (although
50% are incidentally found).
Trivia:
- Not associated with asbestos, smoking, or other environmental pollutants
- Even when they are big, they are usually benign
- Doege-Potter syndrome occurs in like 5% of cases. This is an episodichypoglycemia (tumor can
secrete an insulin like growth factor)
- Hypertrophic osteoarthropathy occurs in like 30% of the cases.
Lipoma - This is the most common benign soft tissue tumor of the pleura. Thepatients sometimes
feel the “urge to cough.” They will not cause rib erosion. They “never” turn into a sarcoma. The
differential consideration is extra-pleural fat, but it is usually bilateral and symmetric.

100
Metastases - Here is the high yield trivia on this. As a general rule the subtype of
adenocarcinoma is the most likely to met to the pleura. Lung cancer is the most common primary,
with breast and lymphoma at 2”^ and Remember that a pleural effusion is the most common
manifestation of mets to the pleura.

Pleural Effusion: Some random factoids on pleural effusions that could be potentially testable.
There has to be around 175 cc of fluid to be seen on the frontal view (around 75cc can be seen on the
lateral). Remember that medicine docs group these into transudative and exudative based on protein
concentrations (Lights criteria). You are going to get compressive atelectasis of the adjacent lung.
Subpulmonic Effusion - A pleural effusion can accumulate between the lung base and the diaphragm.
These are more common on the right, with “ski-slopping” or lateralization of the diaphragmatic
peak. A lateral decubitus will sort it out in the real world.

Normal
Comparison
Encysted Pleural Effusion
- It is possible to have pleural fluid
Right Sided: Notice Left Sided: The key collect between the layers of the
the high point of the here is the increased pleura creating a oval / round
diaphragm is shifted space between the appearance mimicking a Cancer.
laterally stomach bubble and
lung base.
Empyema - Basically this is an infected pleural effusion. It can occur with a simple pneumonia
but is seen more in people with AIDS. Usually these are more asymmetric than a normal pleural
effusion. Other features include enhancement of the pleura, obvious septations, or gas.

Empyema Necessitans - This is the fancy


THIS vs THAT:
Latin word for when the empyema eats through the 1
chest wall and into the soft tissues. It’s classically Empyema Pulmonary Abscess
seen with TB (70%), with the second most
common cause being actinomyces. Lentiform Round
Split Pleural Sign Claw Sign
(thickening and (acute angle with pleura)
Diaphragmatic Hernia - These can be separation of the
acquired via trauma, or congenital. The congenital visceral and parietal
ones are most common in the back left pleura)
(Bochdalek), with anterior small and right being Treated with chest NOT treated with chest
less common (Morgagni). The traumatic ones are tube tube (risk of
also more common on the left (liver is a buffer). bronchopleural fistula).

Paralysis - This is a high yield topic because you can use fluoro to help make the diagnosis.
Obviously the dinosaurs that write these tests love to ask about fluoro (since that was the only thing
they did in residency). Diaphragmatic paralysis is actually idiopathic 70% of the time, although
when you see it on multiple choice tests they want you to think about phrenic nerve compression
from a lung cancer. Normally the right diaphragm is higher, so if you see an elevated left
diaphragm this should be a consideration. On a fluoroscopic sniff test you are looking for
paradoxical movement (going up on inspiration - instead of down).

101
SECTIO N 15:

M ediastinal M a sses

THIS vs THAT: Mediastinal Mass vs Big Pulmonary Artery


Strategy 2: Hilum Convergence Sign

PA Dilation: Hilar Mass:


Vessels converge onto the Vessels converge towards
Strategy 1: The vessels should
lateral margin of the the waist of the heart, below
be linear in morphology.
Bulky Hilum the Bulk Hilum
Not “lumpy / bumpy” like a
bulky node or mass.

Anterior:
T h y m u s : The thymus can do a bunch of sneaky things. It can rebound from stress or
chemotherapy and look huge. It can get cysts, cancer, carcinoid, etc...

• Rebound - Discussed in detail in the Peds chapter. After stress or chemotherapy the thing can
blow up 1.5 times the normal size and simulate a mass. Can be hot on PET.
• Thymic Cyst - Can be congenital or acquired. Acquired is classic after thoracotomy,
chemotherapy, or HIV. They can be unilocular or multilocular. T2 bright is gonna seal the deal.
• Thymoma - So this is kind of a spectrum ranging from non-invasive thymoma, to invasive
thymoma, to thymic carcinoma. Calcification makes you think it’s more aggressive. The thymic
carcinomas tend to eat up the mediastinal fat and adjacent structures. The average age is around
50, and they are rare under 20. These guys can “drop met” into the pleural and retroperitoneum, so
you have to image the abdomen.
• Associations: Myasthenia Gravis, Pure Red Cell Aplasia, Hypogammaglobinemia.

• Thymolipoma - 1 only mention this zebra because it has a characteristic look. It’s got a bunch of
fat in it. Think “fatty mass with interspersed soft tissue. ”

G e rm C e ll T u m o r: Almost always Teratoma (75%). Mediastinal Teratoma - This is


the most common extragonadal germ cell tumor. They occur in kids (below age 1) and adults
(20s-30s). They are benign, but carry a small malignant transformation risk. Mature subtypes are
equal in Men and Women, but immature subtypes are exclusively seen in men (which should be easy
to remember). There is an association with mature teratomas and Klinefelter Syndrome. The
imaging features include a cystic appearance (90%), and fat. They can have calcifications
including teeth - which is a dead give away.

102
Pericardial Cyst:
This is uncommon and benign. The classic location is
the right anterior cardiophrenic angle. This classic
location is the most likely question.

Fibrosing Mediastinitis (Sclerosing Mediastinitis):

This is a proliferation of fibrous tissue that occurs within the


mediastinum. There are two described subtypes:

(1) Granulomatous (the one everyone thinks about)


classically caused by histoplasmosis). Other causes include
TB, and Sarcoid.
This will look like a soft tissue mass with calcifications that
infiltrates the normal fat planes of the mediastinum.

(2) Non-Granulomatous, is the more rare subtype that people read about once and then
forget. Some people will call this form “idiopathic” although the subtype is better thought of as a
response to an autoimmune disease (SLE, RA, Behcet, etc...) or a complication to radiation therapy.
There is also a testable association with the headache medication methvsergide. It’s associated with
retroperitoneal fibrosis when idiopathic.
This subtype also looks like a soft tissue mass but tends to be more infiltrative, lack calcifications.
and can enhance post contrast.

Both subtypes have been known to cause superior vena cava syndrome.

Bronchogenic Cyst - These congenital lesions are usually within the mediastinum (most
commonly found in the subcarinal space) or less commonly intraparenchymal. For the purpose of
the exam, they are going to be in the subcarinal region, causing obliteration of the azygoesophageal
line on a CXR, and being waterish density on CT.

Lymphadenopathy - Could be mets, could be infection, could be reactive. It is generally


abnormal to be larger than 2cm in short axis (makes you suspect cancer).

Mediastinal Lipom atosis - Excess unencapsulated fat seen in patients with iatrogenic
steroid use, Cushings, and just plain old obesity.

Thyroid: - see the endocrine chapter. Lymphoma: - see the cancer section o f this chapter

Posterior Mediastinal Masses:


Neurogenic - The most common posterior mediastinal mass is one of neurogenic origin. This
includes schwannomas, neurofibromas, and malignant peripheral nerve sheath tumors.

Bone Marrow - Extramedullary hematopoiesis (EMH) is a response to failure of the bone


marrow to respond to EPO. Classic conditions include CML, Polycythemia vera, myelofibrosis,
sickle cell, and thalassemia.

103
S E C T IO N 16:
P u lm o n a r y A r t e r ie s & V e in s

The two major forms of anomalous anatomy are described as either Partial (PAPVR) or
Total (TAPVR) anomalous pulmonary venous return. In both forms, pulmonary veins drain into the
systemic system - not the left atrium. Taking blood with oxygen (pulmonary vein blood) which should
normally go into the aorta and dumping it back into the right sided of the heart (left to right shunt) has
the potential to overload that system and potentially cause pulmonary hypertension / right heart
failure (depending on the severity of the shunt).

PAPVR: Partial anomalous pulmonary venous return, is defined as one (or more) of the four
pulmonary veins draining into the right atrium (or the SVC). It is often of mild or no physiologic
consequence. However, there are associations which can cause problems (and are highly testable).

Trivia:

• PAPVR is associated with


ASDs (most classic is the
sinus venosus type).
• PAPVR + Pulmonary
Hypoplasia, should make you
think Scimitar Syndrome
(more on this in a bit)
• The most common pattern is
the right superior pulmonary Sinus venosus ASD. Notice the RV
Right sided veins (stars) is dilated (it is bigger than the LV) -
vein draining into the SVC draining into the SVC
(at the junction of the SVC dude has right heart failure
and RA). If reformatted
creatively they
Left sided PAPVR is less can show the
common (on multiple choice). classic “vertical
vein” (arrow) as
When it is shown the classic the left superior
look is the “vertical vein” pulmonary vein
draining a left pulmonary vein empties into the
into the left brachiocephalic brachiocephalic
vein (stars).
Left sided PAPVR (arrow) can mimic
a duplicated SVC at this axial level

Scimitar Sign (the dilated vein): The dilated right inferior


pulmonary vein (PAPVR) draining into the IVC. The appearance is
that of a Turkish sword - primed and ready to cast terror into the
hearts of those who disbelieve.

Scimitar Syndrome - Sword Vein + Tiny Lung


1 - Big Sword Vein (PAPVR) - arrow*
2 - Unilateral Pulmonary Hypoplasia - bracket*

104
TAPVR: Total anomalous pulmonary venous return, is a cyanotic heart disease characterized by all
of the pulmonary venous system draining to the right side of the heart. Really there is nothing I can
say - it’s a total eclipse of the heart. I ’ll discuss it more in the cardiac chapter.

Meandering Pulmonary Vein: This is the lost Dad vein, refusing to ask for directions
because that is admitting failure. I learned at an early age that “a warrior does not ask for directions.”
In the same spirit, this proud samurai vein follows an anomalous course, wandering / “meandering”
around the lungs eventually making its way back to the left atrium. It takes the scenic route, but it
does always return to the left atrium. This is an important detail.

• PAPVR / scimitar veins do NOT drain into the left atrium (they cause left-to-right shunting).

• Meandering Veins DO drain into the left atrium (they do NOT cause left-to-right shunting).

Pulmonary Vein Aneurysm - These are typically referred to as a varix (varix can refer to any
enlarged vessel - but is usually code for vein and is the preferred nomenclature dude).

Classic scenario - Mitral regurgitation causing dilation of the right pulmonary vein - usually inferior.

Classic scenario - Asymmetric pulmonary edema involving the right upper lobe also from mitral
regurgitation. Why upper and not lower ? — The only logical explanation: God did it to confuse
you. Just remember, lower vein grows, and the upper lung shows (it turns white from pulmonary
edema). “He is a grower not a shower” - in this case refers to the inferior pulmonary vein (not a
flaccid penis).

Proximal Interruption of the Pulmonary


Artery: Basically you have congenital absence of the right
(or left) PA with the more distal pulmonary vasculature present.
The side it is on is determined by the side of the arch (it occurs
on the side opposite of the aortic arch). Some people will call
this thing “unilateral absence of the PA”, but the preferred
nomenclature is “interruption” because the anomaly only
involves the proximal pulmonary artery - the distal pulmonary j*-
arterial tree is maintained - supplied by collateral vessels (shown
by the black arrows).

How it could be shown:


• Classically with volume loss of one hemi-thorax (could be on CXR or CT), then a contrast CT
shot through the heart with only one PA. Normally, you might think one PA is just volume
averaging - but once you’ve been shown volume loss on one side your suspicion for this should
be raised.

Trivia:
• It’s seen on the opposite side of the aortic arch (Absent right PA with left-sided aortic arch,
Absent left PA with right-sided aortic arch).
• Associated with PDA
• Interrupted left PA is associated with TOP and Truncus
• They can get recurrent infections due to the lack of arterial blood supply

105
Pulmonary Embolism:
r H isto r ica l
This is a significant cause o f mortality in
S ig n s o f P E on a C X R
hospitalized patients. The gold standard is
catheter angiography, although this is invasive and Westermark
Regional Oligemia
carries risks. As a result tests like the D-Dimer Sign
(which has an almost 100% negative predictive Fleischner
value), and the DVT lower extremity ultrasound Enlarged Pulmonary Artery
Sign
were developed. Now, the CTPA is the primary
tool. Hampton’s Peripheral Wedge Shaped
Hump opacity
Differentiating acute vs chronic PE is usefiil. Pleural Obviously not specific, but
Effusion seen in 30% of PEs.

THIS vs THAT: Massive vs Sub-IVIassive PE: clinical risk stratification

THIS v s THAT:Acute VS Chronic PE


Acute Chronic
Clot is Central Clot (if seen) is more Peripheral. May be “web like”
Venous Dilation Shrunken Veins with collateral vessels
Perivenous soft tissue edema Calcifications within the thrombi and within the venous walls
Pleural Effusion is common Lungs may show a mosaic attenuation pattern
Acute PE can cause sudden Chronic PE is a well described cause of pulmonary
death from arrhythmia or acute hypertension (obstruction in the vascular bed causes an increase
right heart failure. in vascular resistance)
CTPA is the exam of choice. VQ Scan (believe it or not) is probably superior to CT.

• Massive = Hypotension (SB? < 90)


• Sub-Massive = Stable BP, but RV dysfiinction or positive cardiac enzymes (myocardial
necrosis).

Rigilt Heart Strain - findings on CT (ECHO is still the gold standard).

RV Dilated Larger Contrast


Than LV Reflux into the
-Probably the best sign Hepatic Veins
-Leftward bowing of -Less Reliable.
the ventricular septum
is also helpful.

106
Pulmonary Infarct Mimics: A pulmonary infarct is a wedge-shaped opacity that is going to
“melt” (resolve slowly), and sometimes can cavitate. Obviously a cavitary lesion throws up lots of
flags and makes people say TB, or cancer. When it’s an opacity in the lung and the patient doesn’t
have a fever, sometimes people think cancer - plenty of pulmonary infarcts have been biopsied.

Pulmonary Veno-Occlusive Disease: Uncommon variant of primary pulmonary


hypertension, that affects the post capillary pulmonary vasculature. For gamesmanship: PAH +
Normal Wedge, you should think this. The normal wedge pressure differentiates it from other post
capillary causes; such as left atrial myxoma, mitral stenosis, and pulmonary vein stenosis.

Pulmonary Artery Aneurysm/Pseudoaneurysm_- Think about three things for


multiple choice; (1) Iatrogenic from swan ganz catheter *most common (2) Behcets, (3) Chronic
PE. When they want to lead towards swan ganz they may say something like “patient in the ICU.”
The buzzwords for Behcets are: “Turkish descent” and “mouth and genital ulcers.”

• Hughes-Stovin Syndrome: This is a zebra cause of pulmonary artery aneurysm that is similar
(and maybe the same thing) as Behcets. It is characterized by recurrent thrombophlebitis and
pulmonary artery aneurysm formation and rupture.

• Rasmussen Aneurysm: This has a cool name, which instantly makes it high yield for testing.
This is a pulmonary artery pseudoaneurysm secondary to pulmonary TB. It usually involves
the upper lobes in the setting of reactivation TB.

• Tetralogy of Fallot Repair Gone South: So another possible testable scenario is the patch
aneurysm, from the RVOT repair.

Pulmonary Hypertension - Pulmonary arterial pressures over 25 are going to make the
diagnosis. I prefer to use the “outdated” primary and secondary way of thinking about this.

Primary: Idiopathic type is very uncommon, seen in a small group of young women in their 20s.
Secondary: This is by far the majority, and there are a few causes you need to know: Chronic PE ,
Right Heart Failure/ Strain, Lung Parenchymal Problems- (This would include emphysema, and
various causes of fibrosis). COPDers with a pulmonary artery bigger than the aorta (A/PA ratio)
have increased mortality (says the NEJM).

Imaging Signs o f Pulmonary HTN: The numbers people use for what is abnormal are all over the
place - if forced I’d pick 29 mm. A superior strategy is to compare the size of the aorta and
pulmonary artery (a normal PA should not be bigger than the aorta). You can also compare the
segmental artery-to adjacent bronchus (> 1:1 is abnormal). Mural calcifications of central
pulmonary arteries (seen in Eisenmenger phenomenon) have been described. Additional nonspecific
signs include right ventricular dilation / hypertrophy, and centrilobular ground-glass nodules.

Two other modem strategies are based purely on morphology.

Banana and Egg: Carina Crossover:


Visualization of the Right PA crosses the
main pulmonary Carina midline
artery (egg) at the anteriorly. It normally
level of the aortic crosses in a more caudal
arch (banana) location.

107
S E C T IO N 17:
T r a u m a

Diaphragmatic Injury: There is a lot of testable trivia regarding diaphragmatic injury and
therefore it is probably the most high yield subject with regard to trauma:
Things to Know:
• Left side is involved 3 times more than the right (liver is a buffer)
• Most ruptures are “radial”, longer than 10 cm, and occur in the posterior lateral portion
• Collar Sign - This is sometimes called the hour glass sign, is a waist-hke appearance of the
herniated organ through the injured diaphragm
• Dependent Viscera Sign - This is an absence of interposition of the lungs between the chest wall
and upper abdominal organs (liver on right, stomach on left).

Tracheo-Bronchiai Injury: Airway injury is actually pretty uncommon. When it does


occur it’s usually within 2 cm of the carina. Injury close to the carina is going to cause a
pneumomediastinum rather than a pneumothorax - that is a testable fact. When you get a tracheal
laceration, it most commonly occurs at the junction of the cartilaginous and membranous portions of
the trachea.
Most Common Site of Injury: injured site
• Penetrating Trauma = Cervical Trachea
• Blunt Chest Trauma = Distal Trachea (within 2.5 cm of the carina)

Macklin Effect: This is probably the most common cause of pneumomediastinum in trauma
patients (and most people haven’t heard of it). The idea is that you get alveolar rupture from blunt
trauma, and the air dissects along bronchovascular sheaths into the mediastinum.

Boerhaave Syndrome:
You probably remember this from step 1. When you see air in
The physical exam buzzword was these locations (white
“Hammonds Crunch.” Basically you have a outlines) you need to
ruptured esophageal wall from vomiting, think about ruptured
resulting in pneumomediastinum / (1) Alveoli or
mediastinitis. (2) Ruptured Trachea or
Esophagus
Vomiting & Chest Pain: Next Step =
Esophagram with Water Soluble Contrast
History of vomiting should make you think Esophagus

Pneumopericardium:
This is air confined to the pericardial sac.
The most common causes are:
• Mechanical Ventilation (PEEP)
• Thoracic surgery
• Penetrating Trauma
• Infectious pericarditis with
gas-producing organisms
Pneumomediastinum Pneumopericardium
This vs T H A T : The difference between -Air Extends Above -Air does NOT extend
Pneumopericardium and Pneumomediastinum the Great Vessels above the Great Vessels
is highly testable.

108
Flail Chest: This is 3 or more segmental (more than one fracture in a rib) fractures, or more than
5 adjacent rib fractures. The physical exam buzzword is “paradoxical motion with breathing.”
Classic History: “Thrown o ff a balcony after giving a married woman a foot message. ”

Hemothorax: If you see pleural fluid in the setting of trauma, it’s probably blood. The only
way I can see them asking this is a density question; a good density would be 35-70 H.U.

Pneumothorax: This will look different depending on if it is upright or supine.


Look at the Apex - The
Bubble of Air Rises in
the Upright Position

Hard Rule: Thin


pleural line that curves
at the apex. Make sure
its not the scapula
IPRIGHT dummy.
Look for the “deep Look at the Base -In the
Soft Rule: Lung Markings don’t go past the sulcus” as the supine position the highest
pleural line (sometimes they do - if they are in costophrenic angle is part of the pleural space is at
front of or behind the pneumothorax). abnormally deepened by the lung base under the
pleural air inferior surface of the lung.
Tension Pneumothorax: Intrapleural air accumulates through a valve
type mechanism during inspiration/expiration. The the pressure builds and stuff
gets pushed over (squishing the heart) and causing the classic physical exam
findings of distended neck veins and a deviated trachea.
Classic CXR signs include:
(1) Deviation of the Trachea / Mediastinum
(2) Flattening or Inversion of the ipsilateral diaphragm
Treatment: Needle thoracostomy (4G needle - inserted at the 2nd intercostal space midclavicular

Extrapleural Hematoma:

This is a little tricky, and they


could show you a picture of it.
If you have an injury to the
chest wall that damages the
parietal pleura then you get a
hemothorax. If you have an
injury to the chest wall, but
your parietal pleural is still 4'''
-Nj ..
intact, you get an extrapleural
hematoma. The classic
history is “persistent fluid
collection after pleural drain/ ^ A
tube placement.” The
buzzword / sign is displaced Extrapleural Hematoma-
extrapleural fat. — Arrows on the Displaced Fat ■

There is a paper out there that suggests a biconvex appearance is more likely arterial and should be
watched for rapid expansion. This may be practically useful, but is unlikely to be asked. Just
know the classic history, and displaced extrapleural fat sign.

109
Pulmonary Contusion: This is the most common lung injury
from blunt trauma. Basically you are dealing with alveolar
hemorrhage without alveolar disruption. The typical look is non-
segmental ill-defined areas of consolidation with sub pleural sparing.
Contusion should appear within 6 hours, and disappear within 72 hours
(if it lasts longer it’s probably aspiration, pneumonia, or a laceration).

Pulmonary Laceration: So a tear in the lung will end up


looking like a pneumatocele. If they show you one it will probably
have a gas -fluid (blood) level in it. These things can be masked by
surrounding hemorrhage early on. The major difference between
contusion and laceration is that a laceration resolves more slowly and
can even produce a nodule or a mass that persists for months.

Fat Embolization Syndrome: This is seen in the setting of a long bone fracture or
Intramedullary rod placement. You get fat embolized to the lungs, brain, and skin (clinical triad of
rash, altered mental status, and shortness of breath). The timing is 1-2 days after the femur
fracture. The lungs will have a ground glass appearance that makes you think pulmonary edema.
You will not see a filling defect - like a conventional PE. If they don’t die, it gets better in 1-3
weeks.
Barotrauma: Positive pressure ventilation can cause alveolar injury, with air dissecting into
the mediastinum (causing pneumomediastinum and pneumothorax). Patients with acute lung injury
or COPD have a high risk of barotrauma from positive pressure ventilation. Lungs with pulmonary
fibrosis are actually protected because they don’t stretch.

Malpositioned
Chest Tubes:
The most common look
for the “malpositioned”
chest tube is “side holes
outside the pleura.”
These tubes have an
end hole (black arrow)
Good Unacceptable
but also side holes
(100 years dungeon - no
(white arrows).
Those side holes are supposed to be inside the pleura (overlying the black part of the chest on a
CXR). If they aren’t - you’ll get leaks etc... I will talk more about chest tubes in the IR chapter.
The placement of a tube in a fissure is sorta controversially bad (might be ok). Hopefully they
won’t ask that — if they do, simply read the mind of the person who wrote the question to
understand their preexisting bias.
Other more serious com plication: The ED will occasionally ram them into the parenchyma.
At no point during that procedure (which I imagine feels like pushing a straw into an orange)
could they possible think this is normal.... But yet it does happen. Truth is it’s more likely to
occur in the setting of background lung disease or pleural adhesions. You’ll see blood around the
tube. Bronchopleural fistula may occur as a sequela, as will a lawsuit in which the CXR will be
“exhibit A.”

Blunt Cardiac Injury: If you have hemopericardium in the setting of trauma, you can
suggest this and have the ED correlate with cardiac enzymes and EKG findings.

110
S E C T IO N 18:
L in e s a n d D e v ic e s

Central Lines: The m ain w ay to ask questions about central lines is to show them
being m alpositioned and asking you w here they are. A n abrupt bend at the tip o f the
catheter near the cavo-atrial junction should make you think azygos. I f it’s on the left side
o f the heart, it’s either (1) arterial or (b) in a duplicated SVC.

This is a sneaky trick,


related to Central
Lines. They can
show you the pseudo
lesion / hot quadrate
sign (seen with SVC
syndrome), and then
show you a CXR
with a central venous
catheter.

The idea is that central lines are a risk factor for SVC occlusion.

Swan Ganz Catheter:

Used to measure pressures in the left atrium, evaluate cardiac output, and can be used to help
differentiate cardiogenic vs non-cardiogenic (ARDS) forms of pulmonary edema.
The catheter is optimally positioned between the main pulmonary artery and the interlobar arteries-
this can be proximal right or left sided pulmonary artery. As a general rule the line shouldn’t go
more than 2cm beyond the mediastinal shadow. On a CXR, the tip at the edge of the mediastinal
shadow is ideal.

Ideal Position - Catheter at the


Edge of the Mediastinal Shadow Distal Migration (too far out) Proximal Migration (too far in)
(left or right) Can Result in - Can Result in Arrhythmia.
Pulmonary Infarcts

Ill
intra-Aortic Balloon Pump (lABP): This is used in cardiogenic shock to help with
“diastolic augmentation,” - essentially providing some back pressure so the vessels of the great arch
(including the coronaries) enjoy improved perfusion.
For the purpose o f multiple choice tests you
can ask three things:
(1) What is the function? - decrease LV
afterload and increase myocardial
perfusion,
(2) What is the correct location ? the balloon
should be located in the proximal
descending aorta, just below the origin of
the left subclavian artery (balloon
terminates just above the splanchnic
vessels)
(3) Complications ? - dissection during
Id e a l Position - in the region o f the left m ain stem bronchus
insertion, obstruction of the left
subclavian from malpositioning

CHF: CHF is obviously not an ILD. However, it can sorta look like one on Chest X-Ray so I opted
to lump it in here. Congestive heart failure occurs because of cardiac failure, fluid overload, high
resistance in the circulation, or some combination of the three. There are three phases / stages of
CHF, and these lend themselves to testable trivia.
Cephalization of vessels, Big heart.
Stage 1 “ Redistribution” Wedge Pressure 13-18
Big Vascular Pedicle

Kerley Lines, Peribronchial Cuffing,


Stage 2 “ Interstitial Edema” Wedge Pressure 18-25
Less distinct Central Vessels

Airspace “fluffy” opacity,


Stage 3 “Alveolar Edema” Wedge Pressure > 25
Pleural Effusion
*Swan Ganz Pulmonary Wedge Pressures are an indirect measurement o f left atrial pressure. They
can help prove a cardiogenic etiology to pulmonary edema.

Right Heart Failure:


This is less common than left heart failure, which ironically is the most common cause. Left heart
failure causes pulmonary venous HTN which causes pulmonary arterial HTN, which causes right
heart failure. Some other less common causes of right heart failure include chronic PE and right­
sided valve issues (tricuspid regurg). The imaging features of right heart failure include dilation of
the azygos vein, dilation of the right atrium, dilation of the SVC, ascites, big liver, and contrast
reflux into the hepatic veins on CTPA.

Endotracheal Tube (ETT) Positioning - The tip of the ETT should be about 5 cm
from the carina (halfway between the clavicles and the carina). The tip will go down with the chin
tucked, and up with the chin up (“the hose goes, where the nose goes”). Intubation of the right
main stem is the most common goof (because of the more shallow angle) - this can lead to left
lung collapse. You can sometimes purposefully intubate one lung if you have massive pulmonary
hemorrhage (lung biopsy gone bad), to protect the good lung.

112
Cardiac Conduction Device
Locations: Leads are placed in the RA, RV, and LV.
Types:
RA: These are usually placed in the • Pacemakers - thin
right atrial appendage - which is wire(s) only
located superior to the body of the RA.
• Implantable Cardiac
These leads should:
Defibrillators QCDs')
(1) Course inferiorly into the right
- the one with
atrium - then
“shock coils” - i.e.
(2) Curve upward and anteriorly -
the thick bands.
on the lateral
• Mixed
RV: These are usually placed in the right (Pacemaker + ICD)
ventricular apex. These leads should:
• “Cardiac
(1) Have the tip of the lead to the left
resynchronization
of the spine.
therapy device” is
(2) On the lateral view, it should be seen
the vocab word the
pointing anteriorly bi-ventricular
pacemaker (RV + LV
LV: The leads get there via the coronary and usually RA).
sinus to the posterior / lateral cardiac
vein - as shown on this awesome
diagram I drew.
(1) The tip of the lead to the left of
the spine.
(2) On the lateral view, it should be
seen pointing POSTERIORLY

Com plications:

Acute Same stuff you get with central line placement: pneumothorax, hemothorax, etc.

Rib Clavicle The leads are mostly commonly fractured in the region of the clavicle, and first
Crush rib. If you have to look for a broken lead — look there first.

o —
Norm al
Terminal connector pin
Generator
displacement. This is actually
Related common. Displaced
Terminal Pin

Sometimes it is shown on a
The number is 3 mm. If the lead is lateral CXR - the lead looks
M yocardial
3 mm within the epicardial fat you like its poking through the
Perforation should suspect penetration. wall (because it is)

The generator pack gets flipped and twisted in the pocket, leading to lead
Twiddler displacement. This happens because Grandpa just can’t leave the thing alone
Syndrome (gotta put those dementia mittens on him).
Can’t have him dying on u s... we need those social security checks.

113
?ROM£THEUS
L iomhart, m .i>
.

114
115
S E C T I O N 1:
C h a m b e r s

This vs T H A T : Right vs Left Atrium


RIGHT ATRIUM

Anterior
LEFT ATRIUM

Posterior
I
Defined by the IVC

Crista Terminalis The wall of the LA is


smooth Right Atrium
Pectinate muscles arise
from the peak of the Pectinate muscles are
Crista Terminalis and only found on the inner
run straight into the surface of the left atrial
right atrial appendage appendage
RA Appendage has a
LA Appendage has a
triangular appearance Left Atrial
; NARROW Opening
with a WIDE Opening Left Atrium Appendage

Crista Terminalis: Coronary Sinus:


This is a frequently tested The main draining vein of the
normal structure (it’s not a myocardium. It runs in the AV
clot or a tumor). It is a groove on the posterior surface
muscular ridge that runs from of the heart and enters the right
the entrance of the SVC to atrium near the tricuspid valve.
that of the inferior vena cava.

Remember they can put a left


sided pacer lead through the
coronary sinus.

Crista Terminalis
-Not a clot

Eustachian Valve:
Another normal anatomic structure that
can be shown is the IVC valve or
Eustachian valve (arrow). It looks like a
little flap in the IVC as it hooks up to the
atrium. When the tissue of this valve has
a more trabeculated appearance it is called
a Chiari Network.

116
This vs T H A T : Right vs Left Ventricle

RIGHT VENTRICLE LEFT VENTRICLE

Anterior Posterior
Defined by the
Moderator Band

Has a fibrous outflow


Has a muscular
track - continuous with
outflow tract
the mitral value

Extensive Trabeculae
Left Ventricle
Moderator Band:
A consistent morphologic feature of the right ventricle (it -Don’t Call
defines the RVV It acts as part of the right bundle branch it a Clot
electric system.
- Also Don’t
What does it mean to “define ” the R V ?
Call it a
Well, imagine a situation where you have some massively
Comeback
fucked up congenital heart and you are trying to decide
(I been here fo r
which chamber is the actual RV. The solution is to look years, rockin' my
for the moderator band. peers, puttin'
What does it mean to be samurai ? suckers in fear)
To devote yourself utterly to a set o f moral principles. To seek a stillness M oderator Band
o f your mind. And to master the way o f the sword.

The Chamber of Secrets


Remember that most academics disdain Muggle-bom wizards, believing pure-bloods are superior.
This is their motivation for showing cardiac MRI cases. They know community trained muggles
don’t get exposed to this stuff. Don’t allow yourself to be petrified, if cardiac MRI questions are
determining if you pass or not - than in the immortal words of Kurt Osiander. “you fucked up a long
time ago.”

Cardiac MRI always starts with a series of localizer images (non-gated, wide field of view) which
will cover the entire heart. The “standard views” are then selected from this data - typically in
diastole.

The 2 Chamber:
If you want to get fancy this is a “two chamber
long axis single obhque.” This view resembles
a “basilisk fang” and is obtained from a
bisection of the LV, parallel to the IV septum
(by using that localizer scout). Coronary
This displays the LV and LA (2 chambers). Sinus
This is good for a few things (1) Wall motion / Global LV function
(especially motion o f the inferior an d anterior walls) and (2) Papillary
Mitral valve issues - regurg, etc. The anatomy trick would be to Muscle
have you ID the coronary sinus on this view.
= Mitral Valve

117
The 4 Cham ber Long Axis:

If you want to get fancy this is a “four chamber


horizontal long axis.” This view lays out the
ventricular septum like the “Sword of Godric
Gryffmdor” and is obtained from a plane parallel to
the long axis of the LA and the LV (by using that 2
chamber view).

This displays the LV, LA, RV, and RA (4 chambers). = Mitral Valve
^ = Tricuspid Valve
^ = Triforce of Power

Classic Trivia: The Horizontal Long Axis (4 chamber view)


allows assessment of both the mitral and tricuspid valves.

Classic Trivia: This view is not great for looking at many congenital heart problems.
The notable (testable) exception to this is
the AV canal defect which is seen best on this view (arrow).

The LV Short Axis:

This is a “down the barrel of polyjuice


potion” series of views taken
perpendicular to the long axis of the heart.
Papillary
We can use the 4 chamber long axis to Muscles
obtain these views.

Classic Trivia: The short axis view is


obtained perpendicular to the 4 chamber
horizontal long axis

The 3 Chamber:

Some people will call this an "apical


long axis view." It is obtained by using
a short axis view around the mid-base
region near the aortic outflow tract.

This is the “filthy mud blood” view - an impure hybrid between


the 4 and the 2. The testable advantage is the visualization of the
the left ventricular outflow tract (LVQT). - and is ideal for
looking at flow through this area (i.e. aortic regurg).

A way a question could be asked is "what view is best for aortic


regurg? / stenosis?" or “which of the following views” - and make
you pick out the picture of the 3 chamber. Or just straight ask you - what is this view?

118
Chamber Enlargement
There are described patterns of chamber enlargement on CXR. These are testable and in most cases
total bullshit - but they are worth going over (games are best played as games).

Right Atrial Enlargement Right Ventricle Enlargement Left Ventricle Enlargement

Shifting of the Cardiac Apex is Tilted Cardiac Apex is Tilted


Right Heart Border Up and Out Down and Out
(most people say > 3cm) R U sure the Right Vent Goes ? Left Ventricle Enlargement
is such a Let Down

Left Atrial Enlargement - this is the most easily tested and has the most classic signs.

Frontal View Indirect Signs:


• Widening of the Carinal Angle (> 90) Lateral View Indirect Sign:
• Elevation of the Left Main Bronchus • Walking Man Sign

Normal For Walking Man Sign:


Comparison Classic sign describing the
posterior displacement of
the left mainstream
“‘Double Density Sign” - bronchus.
this is a direct sign of
enlargement, which This creates an upside
describes the down “V” shape with the
superimposed second intersection of the right
contour on the right bronchus (looks like a man
heart, from enlargement walking).
of the right side of the
left atrium A woman would never
walk in such a way -
wouldn’t be lady like.

119
Echogenic Focus in Left Ventricle:

Relatively common sonographic


observation seen on pre-natal
ultrasound. It is a calcified
papillary muscle that usually
goes away by the third
trimester. So who gives a shit?
Well they are associated with
an increased incidence of
Downs (13%).

Don’t get it twisted, having one


means nothing other than you
should look for other signs of
Downs (most o f the time it’s
normal). Echogenic Focus in Left Ventricle

Lipomatous Hypertrophy of the Interatrial Septum:

This has a very classic look of a dumbbell (bilobed) appearance of fat density in the atrial septum,
sparing the fossa ovalis. This sparing of the fossa ovalis, creates a dumbbell appearance {when it
doesn’t spare it think lipoma). It’s associated with being fat, old, and going to yard sales to argue over
the price of broken fumiture and used socks.

Trivia: It can cause supraventricular arrhythmia, although usually does nothing.

Trivia: It can be hot on PET because it’s often made of brown fat.

> 2cm

Lipomatous Hypertrophy of the


Normal Lipoma
Interatrial Septum
Common Uncommon

Fat in the atrial septum, thicker than 2cms < [{(Encapsulated)}] >

Spares the fossa ovalis Does NOT spare the fossa ovalis

If multiple = tuberous sclerosis


Is usually PET HOT , T1 bright,
Can be PET HOT
Drops out on Fat-Sat
Rarely associated with arrhythmia Rarely associated with arrhythmia
(usually asymptomatic) (usually asymptomatic)

“I must encapsulate the word encapsulated,” Prometheus thought to himself as he hunted the keyboard for
symbols to assist in his bizarre compulsion.

120
S E C T I O N 2:
CORONARIES

Questions regarding the coronaries will likely come in two flavors:


(1) Normal - mostly anatomy vocab, and (2) ABnormal - pathologies.

Normal: There are three coronary cusps; right, left, and non-coronary (posterior).
The left main comes off the left cusp, the right main comes off the right cusp.

Posterior Descending Artery


(PDA) - 65%-80% With regard to w hat perfuses what,
-(*the other 20% have the P D A the following are high yield factoids:
supplied by the left coronary)
• RCA perfuses SA node 60%
• RCA perfuses AV node 90%

Circumflex
(LCX)
Obtuse
Acute Marginal IWarginals'
Conus “Supply the
*About 1/2 the time this Diagonals
— Acute Angle
is the first branch. “L A D for D ”
M argin”
-It supplies the
ventricle outflow tract.
Septal Branches

V ascular Perfusion Territories - a high yield topic.

Dominance: Coronary Dominance is determined by what vessel gives rise to the posterior
descending artery and posterior left ventricular branches (most are right - 85%).
The safe word “yellow” is used to draw the attention of the dominant artery.

You can be “co-dominant” if the posterior descending artery arises from the right coronary artery
and the posterior left ventricular branches arise from the left circumflex coronary artery.
Although, Christian Gray would never-approve of such a dynamic.

121
NOTNormal: Anomalies of the Origin, Course, and Termination:
Malignant Origin: There are like a million billion different types
and configurations. The main bad guy people talk about is the LCA
from the Right Coronary Sinus, coursing between the Aorta and
Pulmonary Artery (arrows). This anomaly is more eyil than Skeletor
from He-Man , with a high rate of sudden cardiac death.
By the power of Gray skull - they will always fix this thing.
For the purpose of
multiple choice
(real life is more complex) Malignant coronary artery
- Left Coronary from with origin from the opposite
the Right Cusp gets sinus and an interarterial
Fixed course is the 2nd most
common cause of sudden
- Right Coronary from
cardiac death in young
the Left Cusp gets
patients
Fixed if symptomatic
(most common is hypertrophic
cardiomyopathy).

ALCAPA: Anom alous Left Coronary from the Pulmonary Artery. Myocardial Bridging:
There are two types; This is an intramyocardial course of a
(a) Infantile type (they die early) coronary artery (usually the LAD).
- CHF & dilated cardiomyopathy, This is usually asymptomatic -
(b) Adult (still at risk of sudden death). although it may cause symptoms as the
The easiest question to write would be to ask about diameter decreases with systole.
“STEAL SYNDROME” - which describes a reyersed Besides making a good MCQ - it can
(retrograde) flow in the LCA as pressure decreases in the
be an issue for CABG planning. In
pulmonary circulation. Arrow on Left Coronary many cases the bridged segment is
- off Pulmonary Artery (PA) spared from atherosclerosis, with
I. .1 plaque classically located in a segment
Left ■M just proximal to the bridging

Bridged segment (in the muscle)


shown with bracket.

I -
ALCAPA with anomalous Left
Nobody would STEAL this.

Coronary Artery Aneurysm: By definition this is a yessel with a diameter greater than 1.5x
the normal lumen. Most common cause in adults is atherosclerosis. Most common cause in children
is Kawasaki (spontaneously resolyes in 50%). They can occur from lots of other yasculitides as
well. Last important cause is iatrogenic (cardiac cath).

Coronary Fistula: Defined as a connection between a coronary artery and cardiac chamber or
great yessels. It’s usually the RCA, with drainage into the right cardiac chambers. They are
associated / result in coronary aneurysm. I f you see crazy dilation o f the coronaries - think this.

122
Coronary I Cardiac CT
Who is the ideal patient to get a coronary CT? There are few groups of people getting these.
(1) Intermediate risk for MI and/or atypical chest pain patients. A negative coronary CT will help
stop a stress test or cath from occurring. Why do a procedure with risks on someone with GERD?
(2) Suspected aberrant coronary anatomy.
(3) Evaluating stents (larger than 3mm) or CABG patency
(4) Preoperative assessment for transcatheter aortic valve implantation (TAVI/TAVR)
What makes someone intermediate risk for a MI ? Framingham Risk Score 10-20%
Whatfindings suggest increased risk for a major adverse cardiac event (MACE) ? A bunch of
calcium in the vessels - calculated as a “high Agatston score (>160).” Remember this works for
calcified plaque — not so good with non-calcified disease.

What is the ideal heart rate? To reduce motion related artifacts a slow heart rate is preferred. Most
books will tell you under 60 beats per min. Beta blockers are used to lower the heart rate to
achieve this ideal rate.

Are there contraindications to beta blockers? Yup. Patients with severe asthma, heart block,
acute chest pain, or recent snorting of cocaine off the breasts of a prostitute named Chastity -
should not be given a beta blocker.

Are all heart blocks contraindications to beta blockers ? 2nd and 3rd Degree are
contraindications. A 1st degree block is NOT.

What if I can Hgive the beta blocker? Can he still have the scan? Yes, you just can’t use a
prospective gating technique. You’ll have to use retrospective gating.

What is the difference between prospective and retrospective gating?


-P rospective: “Step and Shoot” - R-R interval * data acquisition triggered by R Wave
• Pro; There is reduced radiation b/c the scanner isn’t on the whole time
Con: No fiinctional imaging
Con: More sensitive to heart rate variability (high heart rate or irregular rhythm)
Trivia: Always axial, not helical
-Retrospective: Scans the whole time, then back calculates
Pro: Can do functional imaging
Pro: Less sensitive to heart rate variability
Con: Higher radiation (use of low pitch - increases dose)
Trivia: This is helical (not Sparta)

What component(s) of the cardiac cycle trigger imaging acquisition ?


-P rospective: Data acquisition triggered by R Wave
-R etrospective: Data is acquired throughout the cardiac cycle

What is best for valvular evaluation ?


-R etrospective: This type o f gating is required f o r evaluating the m itral a n d tricuspid valves.

Other than beta blockers, are any other drugs given for coronary CT? Yup. Nitroglycerine is
given to dilate the coronaries (so you can see them better).

Are there contraindications to nitroglycerine ? Yup. Hypotension (SEP < 100), severe aortic
stenosis, hypertrophic obstructive cardiomyopathy, and Phosphodiesterase (Viagra-Sildenafil,
“boner pills”) use.

123
S E C T IO N 3:
Valves

The Mouth of M adness - Using M RI to Identify and Quantify Valve Disease.

Stenotic I Regurgitant VENC


Flow Jets Velocity-encoded cine MR
- Showing a flow void (arrows) on one imaging (VENC), is a set
side of the valve - makes you think parameter of pliase-contrast
valve pathology. imaging, used for quantifying
VENC Correct
the velocity of flowing blood
Solid Black
The testable feature of the VENC
technique is the velocity range.
You must select a
velocity 20-25% higher than the
expected maximum.
If the velocity range is too low
you will get the classic aliasing
- Use orthogonal planes through the (white areas) artifact. You fix VENC set too low
valve to see these flow voids this by increasing the peak Velocity aliasing
- Use a transverse plane through the velocity parameter. (white areas)
valve in questions to characterize its
morphology (example - to look at the
If the velocity range is too high A Both Aortic Valve
the image will be so noisy that
leaflets of the aortic valve). you won’t be able get an accurate Replacement and
Mitral Valve
peak measurement.
Annuloplasty (the
incomplete metal
ring) are MRI Safe.
Valves on the C X R - A classic source o f M ultiple Choice Trivia

I like the two intersecting line method on both


the frontal and lateral chest to answer these Front View: Line drawn from Lateral View: Line drawn
kinds o f questions. the Left Hilar Angle to the from From the carina to the
• Pulmonic is the Most Superior Right Cardio-phrenic Angle. anterior costophrenic angle.
• Tricuspid is the Most Anterior Aortic Above - Mitral Below. Aortic Above - Mitral Below.
• Aortic is in front o f the Mitral on the Lateral
• The mitral valve is larger than the aortic.
(if you see 2 metal rings, the larger is the mitral) Know that a pacemaker wire
going through a valve makes it
The pointy parts of the mechanical the tricuspid valve (lead
valves {Carpentier-Edwards
terminates in the right ventricle).
aortic valve) point out
(towards the direction of blood flow)

124
Looking at Valves: ECHO is almost always the best first choice to look at the valves. We study so
much MRI for board exams we forget that ECHO is actually superior. Having said that MRI trivia is
more likely to show up on this exam - so lets dish out some trivia:

• Transverse images through the aortic valve let you look at the morphology (how many cusps)
• Orthogonal Long Axis Views let you look for flow voids on one side of the valve
• Transverse slices positioned at the valve (or slightly below) allow you quantify regurgitant volume —
measuring during diastole

Aortic Stenosis: This may be congenital (bicuspid) or Acquired (Degenerative or Rheumatic


Heart). Increased afterload can lead to concentric LV hypertrophy. Peak velocity through the valve
can be used to grade the severity. Velocity-encoded cine MR imaging (VENC), which also answers
to the name “velocity mapping” or “phase-contrast imaging”, is an MRI technique for quantifying the
velocity of flowing blood (if anyone would happen to ask). Dilation of ascending aorta is due to jet
phenomenon related to a stenotic valve. Aortic Stenosis comes in three flavors:
(a) valvular, (b) subvalvular, (c) and supravalvular. Valvular is the most common (90%).

• When I say “Supra-valvular Aortic Stenosis” you say Williams Syndrome


• When I say “Bicuspid Aortic Valve and Coarctation” you say Turners Syndrome

Bicuspid Aortic Valve: This is very common, some


sources will say nearly 2% of the general population. As a
result, it becomes the source of significant fuckery with regard
to one particular multiple choice question - “what is the most
common congenital heart disease?” The answer is probably
bicuspid aortic valve, but because it’s often asymptomatic and
not a problem till later in life when it gets stenotic and causes
syncope - 1 think it messes with peoples’ math. How do you
handle this question? Well... if they list bicuspid aortic valve
then you have to pick it. If they don’t list it then the answer is Bicuspid Valve
VSD. (the way people
usually show it)
Trivia to know:
• Aortic Stenosis is the most common complication
• Bicuspid aortic valve (even in absence of stenosis) is an B icuspid Valve
independent risk factor for aortic aneurysm. Severity of (the most
valve dysfunction does not predict aneurysm formation. common
• Association with Cystic Medial Necrosis (CMN) appearance in the
• Association with Turners Syndrome, and Coarctation real world ~
• Association with AD Polycystic kidney disease “type 1 variant”)

Aortic Regurgitation:

Seen with bicuspid aortic valves, bacterial endocarditis, Marfan’s, aortic root dilation from HTN, and
aortic dissection. How rapid the regurgitation onsets determines the hemodynamic impact (acute
onset doesn’t allow for adaptation). Step 1 question was “Austin Flint Murmur.”

Mitral Stenosis:
Ortner’s Syndrome
(Cardio Vocal Hoarseness)
Rheumatic heart disease = most common cause. Could be
Hoarseness caused by
shown as a CXR with left atrial enlargement (double density compression of the left recurrent
sign, splaying of the carina, posterior esophageal laryngeal nerve by an enlarged
displacement, walking man - etc...). left atrium

125
Mitral Regurgitation: The most common acute causes are endocarditis or papillary muscle /
chordal rupture post MI. The chronic causes can be primary (myxomatous degeneration) or
secondary (dilated cardiomyopathy leading to mitral annular dilation). Remember the isolated Right
Upper Lobe pulmonary edema is associated with mitral regurgitation.

Pulmonary Stenosis: Just like in


the Aortic Valve, comes in three flavors: Williams Syndrome Supra-Valvular Stenosis
(a) valvular, (b) subvalvular, (c) and
supravalvular. Valvular is the most N oonan’s S yndrom e Valvular S ten o sis
common, and can lead to ventricular
hypertrophy. Associated with Noonan
K
Syndrome (male version of turners). TOF Sub-Valvular S ten o sis
“Peripheral Pulmonary Stenosis” is
seen with Alagille syndrome (kids with
absent bile ducts). Williams can give Pulmonary Stem»sis Syndromes
you supra-valvular aortic stenosis (and
pulmonic).

Pulmonary Regurgitation: The classic scenario is actually TOF patient who has been repaired.
TOF repair involves patch repair of the VSD and relief of the RV outlet obstruction. To fix the RV
obstruction the pulmonary valve integrity must be disrupted. Eventual failure of the valve (regurgitation)
is the primary complication of this procedure.

Cardiac MRI is used to guide the timing of pulmonary regurg repair. If the valve is repaired before the
RV is severely dilated (150 ml end diastolic volume) the outcomes are good. If the RV reaches a certain
degree of dilation- it typically won’t return to normal and the patient is pretty much fucked.

Tricuspid Regurgitation: Most common


Gamesmanship: — Rheumatic heart
form of tricuspid disease, due to the relatively weak disease most commonly involves the mitral
annulus (compared to the mitral). May occur in the and aortic valves. Anytime there is multivalve
setting of endocarditis (IV drug use), or carcinoid disease, think Rheumatic Fever!
syndrome (serotonin degrades the valve). The most
common cause in adults is pulmonary arterial Step 1 Trivia: Rheumatic heart disease is
hypertension. A testable pearl is that TR causes RV an immune modulated response to Group A-
dilation (NOT RV Hypertrophy). Beta hemolytic strep.

Ebstein Anomaly: Seen in children whose moms used Lithium (most cases are actually
sporadic). The tricuspid valve is hypoplastic and the posterior leaf is displaced apically (downward).
The result is enlarged R A , decreased RV (“atrialized”), and tricuspid regurgitation. They have the
massive “box shaped” heart on CXR.

Tricuspid Atresia: Congenital anomaly that occurs with RV hypoplasia. Almost always has an
ASD or PFO. Recognized association with asplenia. Can have a right arch (although you should
think Truncus and TOF first). As a point of potentially confusing trivia; tricuspid atresia usually has
pulmonary stenosis and therefore will have decreased vascularity. If no PS is present, there will be
increased vascularity.

Carcinoid Syndrome: This can result in valvular disease, but only after the tumor has met’d to
the liver. The serotonin actually degrades heart valves, typically both the tricuspid and pulmonic
valves. Left sided valvular disease is super rare since the lungs degrade the vasoactive substances.
When you see left sided disease you should think of two scenarios:
(1) primary bronchial carcinoid, or (2) right-to-left shunts.

126
S E C T IO N 4:
G reat v e s s e l s

The m ost com mon variant in branching is the


“bovine arch” in which the brachiocephalic
artery and left com m on carotid artery arise from
a common origin.

ovine Arch - Or “Bo-vizzle" per Snoop Dog

The term inology right arch / left arch is


described based on the aortic arch’s relationship
to the trachea.

When 1 say R ight Arch with M irror Branching,


You say congenital heart.

There are 5 types o f right arches, but only two are w orth know ing (Aberrant Left, and M irror
Branching). The trick to tell these two apart is to look fo r the origin o f the left subclavian.

Originating from the Front o f the Arch =


M irror Image

*this is the bad one (lots o f congenital heart)

Originating from the Back o f the Arch =


Aberrant Left Subclavian

127
Right Arch with Mirror Branching: Although these are often asymptomatic they
are strongly associated with congenital heart disease. M ost com m only they are associated
with TOR However, they are m ost closely associated with Truncus. Obviously, this tricky
wording lends itself nicely to a trick question.

* I f there is a mirror image right arch, then 90% w ill have TOF (6% Truncus).
* I f the person has Truncus, then they have a mirror image right arch 33% (TOF 25%).

Right Arch with Aberrant Left Subclavian: The last


branch is the aberrant left subclavian artery. This is a vascular ring
because the ligam entum arteriosum (on the left) com pletes the “ring’
encircling the trachea.

Left Arch Aberrant Right


Subclavian: The most com mon arch
anomaly. Although it is usually
asymptomatic it can som etim es be
associated witli dysphagia lusoria, as the
RSCA passes posterior to the esophagus. The
last branch is the aberrant right subclavian
artery. The origin o f the RSCA may be
dilated = Diverticulum o f Kom m erell.
Kommerell

Double A ortic Arch: The m ost


common vascular ring. As a point o f
trivia, symptom s may begin at birth and
include tracheal com pression and/or
difficulty swallowing. The right arch is
larger and higher, and the left arch is
sm aller and lower. A rches are posterior to
the esophagus and anterior to the trachea
(encircling them both).

128
Subclavian Steal Syndrome/Phenomenon: So there is a “ Syndrom e” and
there is a “Phenom enon.” The distinction between the two m akes for an excellent distractor.

• SS Phenom enon: Stenosis and/or occlusion o f the proxim al subclavian with


retrograde flow in the ipsilateral vertebral artery.

* SS Syndrome: Stenosis and/or occlusion o f the proxim al subclavian artery with


retrograde flow in the ipsilateral vertebral artery AND associated cerebral ischemic
symptoms.

Occlusion proxim al to
the vertebral in the
LVert
subclavian can result in
retrograde “stolen ’’flo w
fro m the vertebral
artery.

Angiogram

A
(A

“Theft”

'Catheter

If the level o f stenosis and/or occlusion is proxim al to the vertebral artery, reversal o f flow
in the vertebral artery can occur, resulting in the theft o f blood from the posterior
circulation. W hen the upper limb is exercised, blood is diverted away from the brain to the
arm. Cerebral symptom s (dizziness, syncope, e tc ...) depend on the integrity o f collateral
intracranial flow (PCOM s).

Subclavian Steal is alm ost always caused by atherosclerosis (98% ), but other very testable
causes include Takayasu A rteritis, Radiation, Preductal Aortic Coarctation, and Blalock-
Taussig Shunt. In an adult they will show atherosclerosis. I f they show a teenager / 20 year
old it’s gonna be Takayasu. Case books love to show this as an angiogram, and I think
that’s the m ost likely w ay the test will show it. They could also show a CTA or M RA
although I ’d say that is less likely. M ore on this in the vascular chapter

A ortic A n eu ry sm a n d V asculitis: Will be discussed in the Vascular Chapter

129
S E C T IO N 5:
CONGENITAL HEART

An extremely high yield and confusing topic which dinosaur Radiologists love to ask questions about
on CXR. Similar to my discussion in the thoracic chapter about examining the liver of a goat to
determine the will of the Gods - this is perhaps even more ridiculous (and therefore high yield).

I will now teach you to add confusion to a bad situation by suggesting a diagnosis on CXR instead of
waiting for ECHO or MRl. I will attempt to provide a methodology for single answers on CXR
cases.

My thoughts on multiple choice questions regarding congenital heart is that they will come in 3
flavors: (A) Aunt Minnie, (B) Differentials with crappy distractors, and (C) Associations / Trivia.

Aunt Minnies I Differentials:

There are a few congenital heart cases that are Aunt Minnies, or easily solvable (most are differential
cases). Bottom line is that if they want a single answer they will have to show you either an Aunt
Minnie or a differential case, with crappy distractors.

With regard to straight-up Aunt Minnies, I think the usual characters that most third year medical
students memorize are fair game.

Scimitar Sword: PAPVR


with Hypoplasia

130
The easily solvable ones will be shown as a right arch with the associations o f T ru n cu s {more
closely associated) and T O F (m ore com m on overall). Or, they will show you the big box
heart and want Ebsteins (which is an A unt M innie). A nother classic trick with regard to the
big box heart is non-cardiac causes o f high output failure (Infantile Hem angioendotheliom a
and Vein o f Galen M alformation). The rem aining cyanotic syndromes basically look the
same, so the questions m ust be either (a) crappy distractors (none o f the others are cyanotic,
e tc ...), or (b) trivia (which is more likely).

Right
(1) What side is the arch on ? TOF or Truncus

Left
Truncus TOF
Types 1-3

Massive
Cyanotic Ebsteins or
(2) Heart Size ------- ►
Pulmonary Atresia without V S D
*They have to tell you this,
Non-Cardiac (w o n ’t be cyanotic)
if they want a single answer
Normal -Infantile Hemangioendothelioma
-Vein o f Galen M alform ation

NOT-Cyanotic (3) Pulmonary Blood Flow


o

// V%
-TAPVR (especially type 3) -TOF
-D-Transposition -Ebsteins
-Truncus (look for R Arch) -Tricuspid Atresia
-“Tingle Ventricle”

:
Cyanotic Not Cyanotic
With regard to identifying bad distractors I think the TOF ASD
easiest w ay is the cyanotic vs not cyanotic disorders. TAPVR VSD
They literally m ust tell you the kid is cyanotic,
Transposition PDA
otherwise there is no way to know.
Truncus PAPVR
Tricuspid Aortic Coarctation
Atresia (adult type - post ductal)

131
There are a few other key differentials that may make it easier to w eed out bad distractors, or
get “which o f the following do N O T” questions.

Survival dependent on
CHF in Newborn Small Heart DDx
admixture - Cvanotics

Adrenal Insufficiency
TAPVR (Infracardiac type “III”) TAPVR (has PFO)
(Addisons)
Congenital Aortic or Mitral
Transposition Cachectic State
Stenosis
Left Sided Hypoplastic Heart TOP (has VSD) Constrictive Pericarditis
Cor Triatriatum Tricuspid Atresia (has VSD)

Infantile (pre-ductal) Coarctation Hypoplastic Left

Trivia and Associations;


VSD: The most common congenital heart disease. There are several types with Membranous (just
below the aortic valve) being the most common (70%). Outlet subtypes (infundibulum) must be
repaired as the right coronary cusp prolapses into the defect. On CXR we are very nonspecific (big
heart, increased vasculature, small aortic knob). They could ask or try and show splaying of the carina
(from big left atrium). About 70% of the small ones close spontaneously.

PDA: The PDA normally closes around 24 hours after birth (functionally), and anatomically around 1
month. A PDA should make you say three things (1) Prematurity, (2) Maternal Rubella, (3) Cyanotic
Heart Disease. CXR is nonspecific (big heart, increased pulmonary vasculature, large aortic arch
“ductus bump”). You can close it or keep it open with meds.

ASD: Several types with the Secundum being the most Syndromic ASD Trivia
common (50-70%). The larger subtype is the Primum,
(results from an endocardial cushion defect), is more likely Downs Syndrome - ASDs in
to be symptomatic. Only Secundums may close without general, but mostly the AV Canal /
treatment (Primum, AV Canal, Sinus Venosus will not). Ostium Primum Spectrum Types
Primums are not amenable to device closure because of
proximity to AV valve tissue. On CXR, if it’s small it will DiGeorge syndrome and Ellis-Van
show nothing, if it’s large it will be super nonspecific (big Creveld syndrome also have AV
heart - typically right sided, increased vasculature, small Canal / Ostium Primum Spectrum
aortic knob). It’s more common in female. Types
• When I say hand/thumb defects + ASD, Holt Oram’s most common cardiac
you say Holt Oram manifestation is the ASD.
• When I say ostium primum ASD (or endocardial cushion), Adults with AV septal defects have
you say Downs a 10% risk of recurrence of heart
disease in their kids
• When I say Sinus Venosus ASD,
you say PAPVR

132
AV Canal: Also referred to as an endocardial cushion defect - is in the spectrum of Ostium
Primum defects. They happen secondary to deficient development of a portion of the atrial septum, a
portion of the inter-ventricular septum, and the AV valves. Strong association with Downs. You can’t
use closure devices on these dudes either. Surgical approach and management is complex and beyond
the scope of this text.

• Trivia: O f all the congenital heart stuff with Downs patients - AV Canal is the most common
• Trivia: Four chamber horizontal long axis view is the best view to see AV Canal Defects

Unroofed Coronary Sinus (Coronary Sinus ASD): This is a rare ASD which
occurs secondary to a fenestrated (as in the cartoon) or totally unroofed coronary sinus. The most
important clinical is that you can get paradoxical emboli and chronic right heart volume overload.

Trivia: Holes in the


roof of the
STRONG coronary sinus
association with a allow for two
persistent left SVC. way flow:

ASD Subtypes:

This is a true defect


1 Ostium Can be associated
Secundum Involves the region of the fossa ovalis with pulmonary ^
most common type Mid septum hypertension ^

Spectrum of the Most primum ASDs


Ostium Primum atrioventricular (AV) occur within the are relatively large
(Term describes a septal defects endocardial cushion and lead to right
spectrum of defects) defect. heart dilation.
the complete form of Optimus Prime was
also known as AV which also includes a associated with large
canal defects or large ventricular anomalies of the
endocardial cushion septal defect and a atrioventricular (AV)
defects common AV valve valves, ventricular Associated I
septal defect (VSD) with Down
Lower Septum syndrome.

Defect in the SVC-right atrial junction


Sinus Or less commonly the IVC - Right Atrial Associated with
Junction
Venosus PAPVR
Upper Septum (near the SVC)
i
Unroofed
Coronary Defect (partial or Allows a left-to-
Associated with a
1Sinus complete) in the
right shunt to occur
via an “unroofed”
persistent left
(coronary sinus Coronary Sinus SVC
ASD) coronary sinus
least common type

133
PAPVR: Partial anom alous pulm onary venous return,
is defined as one (or more) o f the four pulm onary veins
draining into the right atrium. It is often o f m ild or no
physiologic consequence. It is often associated with
ASDs (secondum and sinus venosus types).

• When I say Right Sided PAPVR,


you say Sinus Venosus ASD

o RUL: SV C association with sinus


venosus type ASD

• When I say Right Sided PAPVR + Pulmonary


Sinus Venosus ASD
Hypoplasia, you say Scimitar Syndrome

TAPVR: Total anomalous pulmonary venous return, is a cyanotic heart disease characterized by all
of the pulmonary venous system draining to the right side of the heart - essentially a total eclipse of
the heart.

A large PFO or less commonly ASD is required for survival (this is a high yield and testable point).
I need you (PFO) now tonight. And I need you more than ever. And if you only hold me tight. We’ll
be holding on forever.....

There are 3 types, but only two are likely to be tested (cardiac type II just doesn’t have good testable
features). All 3 types will cause increased pulmonary vasculature, but type 3 is famous for a full on
pulmonary edema look in the newborn (every now and then I fall apart).

Type 1: Supracardiac:
• Most Common Type
• Veins drain above the heart, gives a snowman appearance.

Type 2: Cardiac
• Second Most Common Type

Type 3: Infracardiac
• Veins drain below the diaphragm (hepatic veins or IVC)
• Obstruction on the way through the diaphragm is common
and causes a full on pulmonary edema look
Snow Man: TAPVR
Key Points on TAPVR:
(Supracardiac
• Supracardiac Type = Snowman
• Infracardiac Type = Pulmonary Edema in Newborn Turn Around, Bright Eyes
• Large PFO (or ASD) needed to survive
• Asplenia - 50% of asplenia patients have congenital heart disease. O f those nearly 100% have TAPVR,
(85% have addidonal endocardial cushion defects).

134
Transposition: This is the most com m on cause o f cyanosis during the first 24 hours. It
is seen m ost com m only in infants o f diabetic mothers. The basic idea is that the aorta arises
from the right ventricle and the pulm onary trunk from the left ventricle {ventricularterial
discordance).

Which one is the Right Ventricle ? You have to find the m oderator band (that defines the RV)

Just like TAPVR survival depends on an ASD, VSD, or PDA {most com m only VSD). There
are two flavors: D & L. The D type only has a PDA coimecting the two systems. W here as
the L type is “L ucky” enough to be com patible with Life.

D-Transposition: Classic
radiographic appearance is the Aorta Systemic PA ¥ LV

“egg on a string” . Occurs from


# ♦ ♦ ♦
discordance between the
ventricles and the vessels. The PDA
RV RA Lungs LA
intra-atrial baffle (M ustard or
Senning procedure) is perform ed
to fix them. A orta will be In D-Transposition, the ductus may be the only connection
between the two systems, which would othenwise be
anterior and to the R t o f the PA.
separate (and not compatible with life)

L-Transposition: The L tvpe is Aorta Systemic RA LV


4
“Lucky” enough to be
congenitally corrected. This
occurs from a “double
discordance” w here the atrium RV
LA ♦ Lungs PA
hooks up with the wrong
ventricle and the ventricle hooks In L-Transposition of the great vessels - there is an
up with the w rong vessel. Aorta inversion of the ventricles, leading to a “congenital
will be anterior and to the L t o f correction. ” No PDA is needed.
the PA.

A corrected D-transposition has a very


characteristic appearance, lending itself to an Aunt
M innie-type question.

The PA is draped overtop the Aorta, which occurs


after a surgeon has perform ed the “LeC om pte
M aneuver” — sounds French so m ust be high yield.
Corrected D Transposition
via Jatene Arterial Switch

135
Tetralogy of Fallot (TOF): The most common cyanotic heart disease. Describes 4 major
findings; (1) VSD, (2) RVOT Obstruction - often from valvular obstruction, (3) Overriding Aorta, (4) RV
hypertrophy (develops after birth). The degree of severity in symptoms is related to how bad the RVOT
obstruction is. If it’s mild you might even have a “pink tef ’ that presents in early adulthood. This is
called a pentalogy of Fallot if there is an ASD. Very likely to have a right arch.
Surgically it’s usually fixed with primary repair. The various shunt procedures (Blalock-Taussig being the
most famous) are only done if the kid is inoperable or to bridge until primary repair.
Trivia: The most common complication following surgery is pulmonary regurgitation.

Truncus Arteriosus: Cyanotic anomaly where there is a single trunk supplying both the pulmonary
and systemic circulation, not a separate aorta and pulmonary trunk. It almost always has a VSD, and is
closely associated with a right arch. Associated with CATCH-22 genetics (DiGeorge Syndrome).

THIS vs THAT: Coarctation of the Aortic


Coarctation: (Narrowing the of the Aortic Lumen)
• There are two subtypes
(as shown in the chart) infantile Adult
• Strong Association with Presents with heart Leg Claudication
Turners Syndrome (15-20%). failure within the first BP differences between
• Bicuspid Aortic valve is the most common week of life. arms and legs.
associated defect (80%). Pre-Ductal (Before the Post-Ductal (Distal to
• They have more berry aneurysms. left Subclavian A.) left Subclavian A.)
• Figure 3 sign (appearance of CXR). Aortic Arch = Aortic Arch = Normal
• Rib Notching: most often involves 4‘h - 8* ribs. Hypoplastic Diameter
It does NOT involve the 1®‘ and 2"^ because those
Collateral Formation is
are fed by the costocervical trunk. More Likely

Hypoplastic Left Heart: Left ventricle and aorta are hypoplastic. They present with pulmonary
edema. Must have an ASD or large PFO. They also typically have a large PDA to put blood in their arch.
Strongly associated with aortic coarctation and endocardial fibroelastosis.

Cor Triatriatum Sinistrum:


This is a very rare situation where
you have an abnormal pulmonary
vein draining into the left atrium
(sinistrum meaning left) with an
unnecessary fibromuscular
membrane that causes a sub division
of the left atrium. This creates the
appearance of a tri-atrium heart.
This can be a cause of unexplained
pulmonary hypertension in the peds
setting. Basically it acts like mitral
stenosis, and can cause pulmonary
edema. The outcomes are often bad
(fatal within two years), depending
on surgical intervention and Cor Triatriatum Sinistrum
associated badness.

136
S E C T IO N 6:
- ISCHEM IC HEART

Imaging regarding ischemic heart disease is going to fall into two modalities; cardiac MR, and
Nuclear. Cardiac MRI currently offers the most complete evaluation o f ischemic heart disease.

Myocardial infarction typically is initiated by rupture o f an unstable coronary atherosclerotic


plaque, leading to abrupt arterial occlusion. The wave front o f necrosis always starts
subendocardial and progresses to the subepicardium. The ischemic necrosis will affect not
just the myocardium but also blood vessels. The destruction o f small capillaries will not allow
contrast to the area o f injury. This is termed “microvascular obstruction” and manifests as
islands o f dark signal in an ocean o f delayed enhancement. The presence o f microvascular
obstruction is an independent predictor o f death and adverse LV remodeling.

Testable Vocab:

• Stunned Myocardium: After an Acute Injury (ischemia or reperfusion injury), dysfunction


o f myocardium persists even after restoration o f blood flow (can last days to weeks). A
perfusion study will be normal, but the contractility is crap.

• Hibernating Myocardium: This is a more chronic process, and the result o f severe CAD
causing chronic hypoperfusion. You will have areas o f decreased perfusion and
decreased contractility even when resting. D on’t get it twisted, this is not an infarct. On
an FDG PET, this tissue will take up tracer more intensely than normal myocardium,
and will also demonstrate redistribution o f thallium. This is reversible with
revascularization.

• Scar: This is dead myocardium. It will not squeeze normally,


so you’ll have abnormal wall motion. It’s not a zom bie. ‘‘He's Dead Jim ”
It will NOT come back to life with revascularization.

Stunned Hibernating Infarct /Scar


Wall Motion Abnormal Wall Motion Abnormal Wall Motion Abnormal
Normal Perfusion
Abnormal Fixed Perfusion Abnormal Fixed Perfusion
(Thallium or Sestamibi)
Will NOT Redistribute with
Will Redistribute with Delayed
Delayed Thallium, will NOT take
Thallium and will take up FDG
up FDG
Associated with chronic high
Associated with acute MI Associated with chronic prior MI
grade CAD

Diastolic Dysfunction = Cardiac MRI Probable Contraindications:


Echocardiography ICDs / Pacemakers
Cochlear Implants
Systolic Dysfunction = Intracranial Shrapnel
Cardiac MRI
** Cardiac Stents are usually safe

137
Delayed imaging: It works for two reasons: (1) Increased volume o f contrast material
distribution in acute myocardial infarction (and inflammatory conditions) (2) Scarred
myocardium washes out more slowly. It is done using an inversion recovery technique to
null normal myocardium, followed by a gradient echo. T1 shortening from the Gd looks bright
(“Bright is Dead”).

Why stress imaging is done: Because coronary arteries can auto-regulate, a stenosis
o f 85% can be asymptomatic in a resting state. So demand is increased (by exercise or drugs)
making a 45% stenosis significant. An inotropic stress agent (dobutamine) is used for wall
motion, and a vasodilator (adenosine) is used for perfusion analysis.

Typical Sequence Pattern / Technique - with approximate times in minutes:

10 25 35 40
Misc: mo
o 3 <D
o ■O
• Velocity
Encoding Delay
z o
m o • Coronary S “■
(D IVIRA 3
< (D
. Etc.. 3

IVIRI in Acute iVil: Cardiac MRI can be done in the first 24 hours post MI (if the patient is
stable). Late gadolinium enhancement will reflect size and distribution o f necrosis.
Characteristic pattern is a zone o f enhancement that extends from the subendocardium
toward the epicardium in a vascular distribution. M icrovascular obstruction will present as
islands o f dark signal in the enhanced tissue (as described above), and this represents an acute
and subacute finding . M icrovascular obstruction is N O T seen in chronic disease as these
areas will all turn to scar eventually.

In the acute setting (1 week) injured myocardium will have increased T2 signal, which can be
used to estimate the area at risk (T2 Bright - Enhanced = Salvageable Tissue).

THIS vs THAT: A cute VS Chronic IVil:

• Both have delayed enhancement


How do you diagnose
• If the infarct was transmural and chronic you may have Myocardial Infarction with
thinned myocardium Contrast Enhanced MR?

• Acute will have normal thickness (chronic can too but (1) Delayed Enhancement
shouldn’t for the purposes o f MC tests. follows a vascular
distribution,
• T2 signal irom edema may be increased in the acute setting.
Chronic is T2 Dark (scar) (2) The enhancement
extends from the
• You w on’t see M icrovascular Obstruction in Chronic endocardium to the
epicardium

138
M icrovascular Obstruction: Islands o f dark
tissue in an ocean o f late G d enhancement. These
indicate m icrovascular obliteration in the setting o f an
acute infarct. The Gd is unable to get to these regions
even after the restoration o f epicardial blood flow.
M icrovascular obstruction is a poor prognostic finding,
associated with lack o f functional recovery.
Microvascular Obstruction
Key Point: It’s NOT seen in ciironic infarct.

Trivia: M icrovascular obstruction is best seen on first pass im aging (25 seconds)

Ventricular Aneurysm: This is rare (5%), but


can occur as the result o f MI. The question is always
true vs false:

* True: M outh is w ider than body. M yocardium


is intact. Usually anterior-lateral wall.

* False: M outh is narrow com pared to body.


M yocardium is NOT intact (pericardial
adhesions contain the rupture). Usually
posterior-lateral wall. H igher risk o f rupture.

False Aneurysms are Usually Posterior Lateral


True Aneurysms are Usually Anterior Lateral

Viability - You can grade this based on % o f transm ural Viability Imaging:
thickness involved in the infarct. Segmental imaging (imaging
over multiple heart beats)
• <25%: likely to improve with PCI T1 post contrast (10-15 min
delay) inversion recovery
• 25-50%: may improve
gradient echo
• 50-100%: unlikely to recover function

What is the timing on the bad sequelae of an Mi?

Dressier Syndrome (effusion) 4-6 weeks

Papillary M uscle Rupture 2-7 Days

Ventricular Pseudoaneurysm 3-7 Days

Ventricular Aneurysm M onths - Requires rem odeling and thinning.

M yocardial Rupture W ithin 3 Days (50% o f the time)

139
S E C T IO N 7:
NON-lSCHEMIC HEART

Dilated Cardiomyopathy: Defined as dilatation with an end diastolic diameter greater than
55mm, with a decreased EF. Can be idiopathic, ischemic, or from a whole list of other random crap
(Alcohol, Doxorubicin, Cvclosporine. Chagas, etc...). The ischemic variety may show
subendocardial enhancement. The idiopathic variety will show either no enhancement or linear
mid-myocardial enhancement. There is often an association with mitral regurgitation due to
dilation of the mitral ring.

Restrictive Cardiomyopatliy: Basically anything that causes a decrease in diastolic


function (inability to “fill”)- Can be the result of myocardium replaced by fibrotic tissue (endocardial
fibroelastosis), infiltration of the myocardium (Amyloidosis), or damage by iron (hemochromatosis).
The most common cause is actually amyloid.

Amyloidosis.- Deposits in the myocardium causes abnormal diastolic Restrictive =


function with biatrial enlargement, concentric thickening of the left Myocardial Process
ventricle and reduced systolic function of usually both ventricles. Seen Constrictive =
in 50% of cases of systemic amyloid. Has a terrible prognosis. You can
Pericardial Process
sometimes see late Gd enhancement over the entire subendocardial
circumference. Clues: pt with hx of multiple myeloma, RA, Crohn’s.

Amyloid Classic Scenario: A long TI is needed (like 350 milliseconds, normal would be like 200). TI
will be so long that the blood pool may be darker than the myocardium.

• Buzzword: “Difficult to suppress myocardium”.


• Buzzword: “Abnormal urinary light chains (AL)”

Eosinophilic Cardiomyopathy (Loeffler).- Bilateral Ventricular thrombus is the classic


phrase / buzzword. You will need a long TI to show the thrombus.

Constrictive Pericarditis: Historically this used to Constrictive Peals


be TB or Viral. Now the most common cause is iatrogenic Most common cause in the first
secondary to CABG or viral (seems that “idiopathic
world = idiopathic presumed viral
presumed viral” is often listed as the most common cause in
the developed world. On CT the pericardium is too thick (> Most common cause in shit hole
0.4 cm), and if it’s calcified that is diagnostic. Calcification is places (“developing nafions”) =
usually largest over the AV groove. “Sigmoidization” is seen TB
on SSFP cine imaging: The ventricular septum moves toward
the left ventricle in a wavy pattern during early diastole Classic Finding on CXR or CT =
(“Diastolic Bounce”). This “bounce” will be most Pericardial Calcifications
pronounced during inspiration - indicating ventricular Classic Finding on MRI =
interdependence. Diastolic Septal Bounce on CINE
imaging
THIS vs THAT: Constrictive vs Restrictive
Cardiomyopathy:

• Pericardium is usually thickened in constrictive


• Diastolic septal bounce is seen in constrictive (Sigmoidization of the septum)
• No Abnormal Late Gd Myocardial Enhancement on Constrictive Disease

140
Myocarditis;

Inflammation of the heart can come from lots of causes {often


viral i.e. Coxsackie virus).

The late Gd enhancement follows a non-vascular distribution


preferring the lateral free wall.

The pattern will be epicardial or mid wall


(NOT subendocardial) — as shown on the case

Sarcoidosis: Cardiac involvement is seen in 5% of Sarcoidosis cases, and is associated with an


increased risk of death. Signal in both T2 and early Gd (as well as late Gd) will be increased. Late
Gd pattern may be middle and epicardial in a non-coronary distribution. Focal wall thickening from
edema can mimic hypertrophic cardiomyopathy. It often involves the septum. The RV and
papillaries are RARELY affected.

Tai(otsubo Cardiomyopathy - A takotsubo is a Japanese Octopus trap, which looks like a pot
with a narrow mouth and large round base. The octopus will go into the pot, but then can’t turn around
and get out (sorta like medical school). A condition with Chest pain and EKG changes seen in post
menopausal women after they either break up with their boyfriend , win the lottery, or some other
stressful event has been described with the shape of the ventricle looking like a takotsubo. There is
transient akinesia or dyskinesia of the left ventricular apex without coronary stenosis.
Ballooning of the left ventricular apex is a buzzword. No delayed enhancement.

Cardiac MRI - Late Gadolinium Enhancement (LGE):


The backbone of multiple choice cardiac MRI related trivia.
Both diseased and normal myocardium will take up gadolinium / enhance - but it depends on when you image.
Early (1-3 mins) you will see normal tissue drink up contrast. Late (5-20 mins) contrast washes out of the
normal tissue and is retained by pathology (lots of different pathologies). The patterns that you see is helpful
for making the diagnosis (picking the answer on multiple choice).
The classic branch-point is ischemic vs non ischemic:
• Ischemic: Enhancement starts subendocardially and spreads transmurally toward the epicardial surface - in a
distribution corresponding to a known coronary artery territory
• Non-Ischemic: Enhancement is often located in the mid-wall of the ventricle - patchy or multifocal in
distribution (not corresponding to a known coronary artery territory)

C D ® CD
Subendocardial:
Infarct
Transmural:
infarct
Subendocardial Circumferential:
Amvloidosis *can also b e transmural

C D ® 03 Midwall:
Myocarditis, Idiopathic Dilated CM
Midwall: Epicardial:
Myocarditis, Sarcoidosis Myocarditis, Sarcoidosis
Midwall:
HCM

141
S E C T IO N 8:
GENETICS

Arrhythm ogenic Right Ventricular


Cardiomyopatliy (ARVC):

Characterized by fibrofatty degeneration of the RV


leading to arrhythmia and sudden death. Features
include: dilated RV with reduced function, fibrofatty
replacement of the myocardium, normal LV, and
micro-aneurysms on CINE. People use this major/
minor criteria system that includes a bunch of EKG
changes that no radiologist could possibly understand (if
they are stupid enough to ask just say left bundle
branch block). Watch out for the use of fat sat to
ARVC - Dilated RV, with Fat in the Wall
demonstrate the fat in the RV wall.

Hypertrophic Cardiomyopathy: Abnormal hypertrophy (from disarray of myofibrils) of


the myocardium that compromises diastole. There are multiple types but the one they are going to
show is asymmetric hypertrophy of the intraventricular septum. The condition is a cause of sudden
death. There is a subgroup which is associated with LVOT obstruction (“hypertrophic obstructive
cardiomyopathy”). Venturi forces may pull the anterior leaflet of the mitral valve into the LVOT
(SAM - Systolic Anterior Motion of the Mitral Valve). Patchy midwall delayed enhancement of
the hypertrophied muscle may be seen, as is an independent risk factor for sudden death.

Noncompaction:

Left ventricular noncompaction is


an uncommon congenital
cardiomyopathy that is the result
of loosely packed myocardium.
The left ventricle has a spongy
appearance with increased
trabeculations and deep Systole Diastole
intertrabecular recesses.
Noncompaction - Spongy LV with No Myocardial Thickening

As you might expect, these guys get heart failure at a young age. Diagnosis is based of a ratio of
non compacted end-diastolic myocardium to compacted end-diastolic myocardium of more
than 2.3:1.

Buzzword: Prom inent Ventricular Trabeculation

Muscular Dystrophy: Becker (mild one) and Duchenne (severe one) are X-linked
neuromuscular conditions. They have biventricular replacement of myocardium with connective
tissue and fat (delayed Gd enhancement in the midwall). They often have dilated cardiomyopathy.
Just think kid with dilated heart and midwall enhancement.

142
S E C T I O N 9:
^ T umors

Mets: Mets to the heart and pericardium are much more common than primary cardiac tumors (30x
more common). Also, not surprising, a cancer eating into the side of your heart has a poor prognosis
(Las Vegas hookers, cocaine, and vengeance against your enemies should be considered).

The pericardium is the most common site affected (by far). The most cormnon manifestation is a
pericardial effusion (second most common is a pericardial lymph node). Melanoma may involve the
myocardium.

• Trivia: Most common met to the heart is lung cancer (pericardium and epicardium)
• Trivia: Melanoma has the highest percentage of cases that met to the heart.

Fuckery: Watch the language of the cardiac met question — overall lung is more common (because
lung cancer is fairly common). However, melanoma has a higher percentage of cardiac involvement.
“Creative” (deliberate attempt to trick you) wording on the question should be closely scrutinized.
Mumble to yourself “Godless cocksuckers” as you correctly navigate these attempts to deceive you.

Angiosarcoma: Most common primary malignant tumor of the heart in adults. They like the RA
and tend to involve the pericardium. They often cause right sided failure and/or tamponade. They are
bulky and heterogenous. Buzzword is “sun-ray” appearance which describes enhancement
appearance of the diffuse subtype as it grows along the perivascular spaces associated with the
epicardial vessels.

Left Atrial Myxoma: Most common primary cardiac THIS VS THAT:


tumor in adults (rare in children). They are associated with Tumor vs Thrombus:
MEN syndromes, and Blue Nevi (Carney Complex). They
Cardiac M RI is the w ay to tell.
are most often attaclied to the interatrial septum. They
may be calcified. They may prolapse through the mitral • Tum or will enhance
valve. They will enhance with Gd (important • Throm bus will NOT enhance.
discriminator from a thrombus).

Rhabdomyoma: Most common fetal cardiac tumor. It is a hamartoma. They prefer the left
ventricle. Associated with tuberous sclerosis. Most tumors will regress spontaneously (those NOT
associated with TS are actually less likely to regress).

Fibroma: Second most common cardiac tumor in childhood. They like the IV septum, and are
dark / dark on T1/T2. They enhance very brightly on perfusion and late Gd.

Fibroeiastoma: Most common neoplasm to involve the cardiac valves (80% aortic or mitral).
They are highly mobile on SSFP Cine. Systemic emboli are common (especially if they are on the
left side).

143
Cardiac Tumors I Mimics
ZVI6^3static
Myxoma Fibroelastoma Rhabdomyoma Angiosarcoma Disease Thrombus

The most
Most common Much more Most common
2nd most common Most common common
primary common than
primary cardiac primary cardiac primary intra-cardiac
cardiac tumor Primary “mass”
tumor (adult) tumor (infants) MALIGNANT
(adult) tumors
tumor

Lung cancer
is the most
Adult (30-60) common.
with distal Adult (50-60) -
emboli and ViOWtilly an Melanoma
fainting spells. incidental finding. Infant with goes to the
tuberous heart with
Younger If they are sclerosis the greatest
people are symptomatic its percentage
likely from emboli (but
syndromic (stroke /TIA ) prevalence is
(Carney less than
Complex) lung)

Favors the left


Involves the atrial
Arise from the appendage
cardiac valves -
intra-atrial Favor the (A-Fib),
aortic is most
septum , ventricular Favors the right Favors the
common —
usually myocardium atrium pericardium
usually the aortic Left
growing into side of the aortic Ventricular
the left atrium Apex (post
cusp.
MI)

Pericardial
About 1/4 have Most are small - They tend to be
thickening =
calcification less than 1cm. multiple
invasion

“Ball with Discriminator:


stalk attached Fibroma is T2
to the inter- Discriminator:
dark
atrial septum ” Vegetations tend Discriminator:
Rhabdomyoma is
Dynamic to involve the Large Pericardial ; Thrombus
T2 Bright
imaging will valve free edges. heterogenous nodularity ^won’t
show Fibroelastoma mass and effusion enhance.
*Fibroma is the
mobility / does NOT do Tumors will.
2nd most common
prolapse of the that.
tumor in this age
“ball”.
group

144
S E C T IO N 10:
P erica rd iu m

Pericardial Effusion: The pericardium is composed of two layers (visceral and parietal), with
about 50cc of fluid normally between the layers. An “effusion” is the situation where the fluid volume
exceeds 50cc between the pericardial layers. This can be from lots and lots of causes - renal failure
(uremia) is probably the most common. For the purpose of multiple choice tests you should think about
Lupus, and Dressier Syndrome (inflammatory effusion post MI).

On CXR they could show this 3 ways:


(1) Normal Heart on Comparison,
Now Really Big Heart.
• Classic history of a 30 year old female
with an ugly rash on her face (lupus).
• Classic history of a recent MI (Dressier
syndrome).
(2) Giant Water Bottle Heart,
(3) Lateral CXR with two lucent lines
(epicardial and pericardial fat) and a Pericardial Effusion: “Oreo Cookie Sign”
central opaque line (pericardial fluid) - the
so called “oreo cookie sign.”

Cardiac Tamponade: Pericardial effusion can cause elevated pressure in the pericardium and
result in compromised filling of the cardiac chambers (atria first, then ventricles). This can occur with
as little as lOOcc of fluid, as the rate of accumulation is the key factor (chronic slow filling gives the
pericardium a chance to stretch). The question is likely related to short-axis imaging during deep
inspiration showing flattening or inversion of the intraventricular septum toward the LV, a
consequence of augmented RV filling. Another indirect sign that can be shown on CT is reflux of
contrast into the IVC and azygos system.

Pericardial Cysts: Totally benign incidental finding. Usually seen on the


right cardiophrenic sulcus. They do not communicate with the pericardium.
This would be most easily shown as an ROI measuring water density along
the ri^ht cardiophrenic sulcus.

Congenital /Acquired Absence: Even though you


can have total absence of the pericardium - the most common Leftward
situation is partial absence of the pericardium over the left Shift of the
atrium and adjacent pulmonary artery. When the left Heart
pericardium is absent the heart shifts towards the left. They
could show you a CT or MRI with the heart contacting the left
chest wall, and want you to infer partial absence. Another piece
of trivia is that cardiac herniation and volvulus can occur in Tongue of
patients who undergo extrapleural pneumonectomy (herniation Lung (arrow)
can only occur if the lung has also been removed). Between the
Trivia: Left atrial appendage = most at risk for strangulation Aorta and PA

145
S E C T IO N 1 l:
S u rg eries

Palliative Surgery for the H ypoplastic Left Heart: Surgery for Hypoplasts
is not curative, and is instead designed to extend the life (prolong the suffering) o f the child.
It is done in a 3 stage process, to protect the lungs and avoid right heart overload:

(1) N orw ood or Sano - w ithin days o f birth

(2) Glenn - at 3- 6 months

(3) Fontan at 1 I/2 to 5 years

(1 a) Norwood: The goal o f the surgery is to create an unobstructed outflow tract from
the systemic ventricle. So the tiny native aorta is anastom osed to the pulm onary trunk, and
the arch is augm entented with a graft (or by other methods). The ASD is enlarged to create
non restrictive atrial flow. A Blalock-Taussig Shunt (see below) is used betw een the right
Subclavian and right PA. The ductus is rem oved as well to prevent over shunting to the lungs.
Apparently, when this goes bad it’s usually from issues related to dam age o f the coronary
arteries or over shunting o f blood to the lungs (causing pulm onary edema). As a point o f
trivia, sometimes the thymus is partially rem oved to get access.

Maintain Pulmonary Flow


- Shunt (BT or Sano)

Creation of
Unobstructed
Flow
- Enlarged
Aorta (graft)
- Enlarged
ASD

146
(1 b) Sano: Same as the Norwood, but instead o f using a Blalock-Taussig shunt a conduit is
made connecting the right ventricle to the pulmonary artery. The disadvantage o f the BT Shunt
is that it undergoes a steal phenomenon (diverted to low pressure pulmonary system).

(2a) Classic Glenn: Shunt between the SVC and right


pulmonary artery (end-to-end), with the additional step o f sewing
the proximal end o f the Right PA closed with the goal o f reducing
right ventricular work, by diverting all venous return straight to the
lung (right lung).

(2b) Bi-Directional Glenn: Shunt between the SVC and the


right pulmonary artery (end-to-side). The RPA is left open, letting
blood flow to both lungs. This procedure can be used to address
right sided heart problems in general, and is also step two in the
palliative hypoplastic series. If it’s being used as step two the Glenn
previously placed Blalock-Taussig Shunt or Sano shunt will come Bi-Directional
down as the Glenn will be doing its job o f putting blood in the
lungs.

(3) Fontan Operation: Used for Hypoplastic Hearts. The old


school Fontan consisted o f a classic Glenn (SVC to RPA), closure o f the
ASD, and then placing a shunt between the Right atrium to the Left PA.
The idea is to let blood return from systemic circulation to the lungs by
passive flow (no pump), and turn the right ventricle (the only one the kid
has) into a functional left ventricle. There are numerous complications
including right atriomegaly with resulting arrhythmias, and plastic
bronchitis (they cough up “casts o f the bronchus” that look like plastic).
Blood
Bypasses
R Atrium
other Surgeries;
High Yield Point:
Classic Blalock-Taussig Shunt: Originally
developed for use with TOR Shunt is created G lenn = Vein to Artery
between the Subclavian artery and the pulmonary (SVC to Pulmonary Artery)
artery. It is constructed on the opposite side o f the Blalock-Taussig = Artery to Artery
arch. It’s apparently technically difficult and often (Subclavian Artery to Pulmonary Artery)
distorts the anatomy o f the pulmonary artery.

Modified Blalock-Taussig Shunt: This is a gortex


shunt between the Subclavian artery and pulmonary artery,
and is performed on the SAME SIDE as the arch. It’s easier
to do than the original.

BT- Shunt

147
Pulmonary Artery Banding: Done to reduce pulmonary
artery pressure (goal is 1/3 o f systemic pressure). M ost common
indication is CHF in infancy with anticipated delayed repair. The
single ventricle is the most common lesion requiring banding.

Atrial Switch: Mustard and Senning procedures are used to


correct transposition o f the great arteries by creating a baffle within
the atria in order to switch back the blood flow at the level o f in­
flow. The result is the right ventricle becomes the systemic
ventricle, and the left ventricle pumps to the lungs. This is usually
done in the first year o f life.

Senning: Baffle is created from the right atrial wall and


atrial septal tissue W ITHOUT use o f extrinsic material

Mustard: Involves the resection o f the atrial septum and


creation o f a baffle using pericardium (or synthetic material).

Rastelli Operation: This is the most commonly


used operation for transposition, pulmonary outflow
obstruction, and VSD. The procedure involves the
placement o f a baffle within the right ventricle
diverting flow from the VSD to the aorta (essentially
using the VSD as part o f the LVOT). The pulmonary
valve is oversewn and the conduit is inserted between
the RV and the PA. The primary advantage o f this
procedure is the left ventricle becomes the systemic
ventricle. The primary limitation o f this procedure is
that the child will be committed to multiple additional surgeries
because the conduit wears out and must be replaced.

Jatene Procedure: This is another arterial switch method that


involves transection o f the aorta and pulmonary arteries about the
valve sinuses , including the removal o f the coronaries. The great
arteries are switched and the coronaries are sewn into the new aorta
(formerly the PA). Apparently this (Lecompte Maneuver) is very
technically difficult, but the advantage is there is no conduit to go
bad, and the LV is the systemic ventricle.

Ross Procedure: Performed for Diseased Aortic Valves in Children. Replaces the aortic
valve with the patient’s pulmonary valve and replaces the pulmonary valve with a
cryopreserved pulmonary valve homograft. Follow-up studies have shown interval growth o f
the aortic valve graft in children and infants.

148
Bentall Procedure:

Operation involving composite graft


replacement o f the aortic valve, aortic root and
ascending aorta, with re-implantation o f the
coronary arteries into the graft. This operation
is used to treat combined aortic valve and
ascending aorta disease, including lesions
associated with Marfan syndrome.

Summary - Most Common Surgery / Key Points

Glenn Blalock Taussig Fontan

Artery to Artery It’s complicated with multiple


Vein to Artery
(Subclavian Artery to Pulmonary versions - steps are unlikely to
(SVC to Pulmonary Artery)
Artery) be tested

Primary Purpose: Take


Primary Purpose: Bypass the
systemic blood directly to the Primary Purpose: Increase
right ventricle / direct systemic
pulmonary circulation (it pulmonary blood flow
circulation into the PAs.
bypasses the right heart).

Most Testable Complications:


Most Testable Complications: Most Testable Complications:
-Enlarged Right Atrium
-SVC Syndrome -Stenosis at the shunt’s
causing arrhythmia
-PA Aneurysms pulmonary insertion site
-Plastic Bronchitis

THIS vs THAT: Heart Transplant Types

Orthotopic Heart Transplant: All o f the heart is removed, except the circular
part o f the left atrium (the part with the pulm onary veins). The donor heart is trim m ed
to fit to the left atrium.

Heterotopic Heart Transplant: The recipient heart remains in place, and the
donor heart is added on top. This basically creates a double heart. The advantages o f
this are (1) it gives the native heart a chance to recover , and (2) gives you a backup if
the donor is rejected.

149
fROMETHEUS
Lionhact, m,t>.

150
PROMETHEUS LIONHART, M .D .

PEDIATRICS
4

151
S E C T IO N l:
S k u ll & Scalp

Craniosynostosis
“Craniosynostosis” is a fancy w ord for prem ature fusion o f one or several o f the cranial
sutures. The consequence o f this prem ature fiasion is a w eird looking head and face (with
resulting difficulty getting a date to the prom). Besides looking like a gremlin (or a cone­
headed extraterrestrial forced to live as a typical suburban human), these kids can also have
increased intracranial pressure, visual impairment, and deafness.

There are different nam ed types depending on the suture involved - thus it’s w orth spending
a mom ent reviewing the names and locations o f the normal sutures.

Metopic Bones Fusion Fusion


Involved Direction Order

Coronal
Metopic Front to First
Frontal
(frontal) Back (2-3 months)

Frontal & Lateral to


Coronal Second
Parietal Medial
Sagittal
Parietal & Lateral to
Lambdoid Third
Occipital Medial

Back to
Sagittal Parietal Fourth
Front

Lambdoid

Sutures norm ally have a serrated (saw tooth) contour


With early closure the suture will lose the serrated appearance - becom ing m ore dense
and sharp. Eventually the suture will disappear completely.
For the purpose o f multiple choice, you should think about synostosis (early closure) as
likely syndromic - and focus your m em orizing on this point.
Having said that, sagittal and unilateral coronal synostosis are typically idiopathic

152
Craniosynostosis Continued.. “ P la g io c e p h a ly ”
Potential Source o f Fuckery
Pathology / Sub-type Trivia:
Metopic Synostosis This word basically means “flat.”
“Trigonocephaly”
You will see it used to describe
unilateral coronal synostosis as
‘‘anterior plagiocephaly.”

You will see it used to describe


unilateral lambdoid synostosis as
“posterior plagiocephaly. ”

The problem is that many people


use the word “plagiocephaly” to
describe the specific entity of
“deformational plagiocephaly ” -
which is just benign positional
Sagittal Synostosis molding, not a pathologic early
“Scaphocephaly” Coronal Synostosis closure. On the following page, I’ll
“Brachycephaly” go into more detail on this. Just
know you may be required to read
the question writer’s mind when the
word is used to differentiate
Lambdoid Synostosis between the benign and pathologic
“Turricephaly” (if bilateral) entities.

Eyes are close together (hypotelorism)


Ethmoid sinuses underdeveloped
“Quizzical
Metopic Medial part of the orbit slants up
Trigonocephaly Eye”
(frontal) Single suture synostosis most frequently
appearance
associated with cognitive disorders (growth
restriction of the frontal lobes)
Unilateral subtype is more common.
Unilateral type causes the ipsilateral orbit to
“Harlequin
elevate, and contralateral frontal bone to
Coronal Brachycephaly Eye”
protrude “frontal bossing”
* if unilateral.
Bilateral form is Rare - should make you think
syndromes {Borat’s brother Bilo*).
Least
I u j -j Turricephaly Tall Cranium (oxycephaly, acrocephaly)
Lambdoid „ Common
(if bilateral) See Next Page for Unilateral Discussion
Form

Long, Narrow Head.


Looks like an upside-down boat.
Scaphocephaly Most
Usually the kids have a normal IQ
Sagittal or Common
Usually the kids do NOT have hydrocephalus
Dolichocephalic Form
Associated with Marfans
(both are tall and skinny).
*For the purpose o f this text, Borat’s brother Bile is presumed to have syndromic mental retardation /
developmental delay. Not iatrogenic encephalopathy related to the intracranial placement o f a red haired
woman's tooth - as therapy fo r demon induced epilepsy - which is suggested in some reports.

153
THIS vs THAT: Positional Plagiocephaly vs True Unilateral Lam bdoid Synostosis

POSITIONAL LAMBDOID
PLAGIOCEPHALY CRANIOSYNOSTOSIS
Ipsilateral
Anterior Ear
Movement

Ipsilateral Posterior Ear


Movement (sometimes)

Infants that sleep on the same side every night I f this is bilateral think underlying
develop a flat spot on the preferred dependent Rhombencephalosynapsis
area of the head (occipital flattening).
Onset: Weeks After Birth Onset: Birth

Ipsilateral Ear: Anterior Ipsilateral Ear: Posterior / Inferior


Frontal Bossing: Ipsilateral Frontal Bossing: Contralateral
Most common cause of an abnormal Rare as Fuck
skull shape in infant
Management is conservative
Management is Surgery
(sleep on the other side for a bit)

N EX T STEP: For the purpose o f multiple


T riv ia
choice, there are numerous
Outside o f the jungle (or the year 1987), the random bone buzzwords
diagnosis o f synostosis is going to be made with that are supposed to elicit
CT + 3D. If asked what test to order I would say the reflexive diagnosis
CT with 3D recons. Having said that, they could NF-1 in your brain when
you hear / read them.
show you a skull plain film (from 1987) and ask
you to make the diagnosis on that.
The more common ones include:
If the test writer was feeling particularly cruel • Absence / Dysplasia o f the Greater
and bitter he/she could show the diagnosis Sphenoid Wing,
with ultrasound. In that case, remember that • Tibial Pseudoarthrosis,
a normal open suture will appear as an • Scoliosis, and
uninterrupted hypoechoic fibrous gap • Lateral Thoracic Meningocele.
between hyperechoic cranial bones (Bright -
Dark - Bright, Bone - Suture - Bone). I ’d like to add “bone defect in the region
o f the lambdoid suture ” or the “asterion
Although certain MR gradient sequences can defect” to that list o f reflex generators.
be used, MRI has traditionally been
considered unreliable in identifying sutures It’s rare and poorly described - therefore
individually. potentially high yield.

154
Clover Leaf Skull Syndrome
• Also referred to as Kleeblattschadel for the purpose o f
fucking with you
• Contrary to w hat the nam e m ight imply - this complex
deformity is not associated w ith an increased ability to hit
green lights, reliably find good parking spots, or w in the
lottery. I think that’s because the shape is m ore 3 leaf clover, and not
4 le a f One m ight assume, a head shaped like a 4 le af clover w ould
probably be luckier.
• Instead, this deformity is characterized by enlargem ent o f the head
with a trilobed configuration, resem bling a three-leaved clover.
• Results from premature synostosis o f coronal and lam bdoid sutures
(most commonly), but often the sagittal closes as well.
• Hydrocephalus is a com m on finding. All the sutures are
closed
• Syndromic Associations: Thanatophoric dysplasia, A pert syndrome except the metopic
(severe), Crouzon syndrome (severe) and squamosal

Additional Craniosynostosis Syndromes:

M ost o f the time (85%) prem ature closure is a prim ary (isolated) event, although
it can occur as the result o f a syndrome (15%). The two syndromes w orth having
vague fam iliarity with are A pert’s and C rouzon’s.

Brachycephaly (usually)
A pert’s Fused Fingers (syndactyly) - “ sock hand”
S’ S —typically sym m etrically fused hands and feet
VV

Brachycephaly (usually)
1st A rch structures (m axilla and m andible hypoplasia).
H ydrocephalus (more than A pert’s)
C rouzon’s Chiari I m alform ations 3: ~70% o f cases
Associated w ith patent ductus arteriosus and aortic
coarctation.
Short central long bones (hum erus, femur) - “rhizom elia’'

Crouzon’s ‘C ’s: Coronal sutures fused, Can’t Chew (1st arch structures), Chiari I,
Coarctation, hydroCephalus, Central bones short (rhizomelia). Crazy eyes (exopthalmos).

155
^ JA th is vs THAT: Skull Markings

Convolutional Markings ^pper ,1


Lucken#liadel - “Lacttnar"

Oval, round, and finger shaped


Normal gyral impressions on The same thing as convolutional
markings (the normal gyral defects (craters) within the
the inner table of the skull.
inner surface of the skull. Most
impressions), just a shit ton
prominent in parietal bones.
more of them.
Different than Copper Beaten
You also see them along the in that:
You see them primarily during anterior portions of the skull not
(A) They aren’t gyriform.
normal rapid brain growth (age just the posterior.
(B) They aren’t related to
3-7).
increased ICP.
(C) They are usually present at
Usually mild and favors the
Think about things that cause birth.
posterior skull.
increased intracranial
pressure. Instead they are the result of
If you see them along the more
defective bone matrix.
anterior skull then you should
think about a “copper beaten” Classic examples;
• Craniosynostosis Classic Association:
skull from the increased
• Obstructive Hydrocephalus • Cliiari II malformation /
intracranial pressure.
Neural Tube Defects.

Lytic Skull Lesions:


Lytic skull lesions in kids can come from a couple o f different things (LCH, Infection, Mets,
Epidermoid Cysts, Leptom eningeal Cysts, e tc ...). The two I w ant you to focus on are LCH and
the Leptomeningeal Cyst (which I will discuss later in the chapter).

LCH (Langerhans Cell Histiocytosis) - Too many fucking dendritic cells - with local invasion. It
is a sorta pseudo m ahgnancy thing. N obody really understands it .... For the purpose o f the exam
think about this as a beveled hole in the skull. The skull is the m ost common bone involved with
LCH. It is a pure lytic lesion (no sclerotic border). The beveled look is because it favors the
inner table. It can also produce a sequestrum o f intact bone (“button sequestrum).

Gam esm anship: I f they tell you (or infer) the kid has neuroblastom a
A - think about a met.

156
Parietal Foramina
These paired, mostly round, defects in the
parietal bones represent benign congenital
defects. The underlying cause is a delayed or
incomplete ossification in the underlying
parietal bones.

They can get big and confluent across the


midline. Supposedly, (at least for the big ones
> 5mm) they are associated w ith cortical and
venous anomalies.

Wormian Bones
In technical terms, there are a bunch o f extra squiggles around the lam bdoid sutures.
“Intrasutural Bones” they call them.

These things are usually idiopathic - however, if


you see more than 10 you should start thinking
syndromes.

^ Gam esm anship:

< 1 0 = Idiopathic
> 10 = First think O steogenesis Im perfecta
> 10 + A bsent Clavicle = C leidocranial

There is a m assive differential, but I w ould ju st


rem em ber these “ P O R K -C H O P ”

Pyknodysostosis
O steogenesis Im perfecta
R ickets
K in k y H air Syndrome
(Menke s /F ucked Copper Metabolism)

C leidocranial Dysostosis
H ypothyroidism / H ypophosphatasia
O n e too m any 21st chrom osom es (Downs)
-Extra Squiggles-
Prim ary A cro-osteolysis (Hajdu-Cheney)

157
Dermoid / Epidermoid of tlie Situll
In the context o f the skull, you can
Epidermoid Dermoid
think about these things as occurring
from the congenital misplacem ent o f Only Skin Skin + Other Stuff
Histology (Squamous Like Hair Follicles,
cells from the scalp into the bony
Epithelium) Sweat Glands Etc..
calvarium.
Age of Present between Typically have an
The result is a growing lump o f Onset age 20-40 earlier presentation
tissue (keratin debris, skin glands,
Parietal Region is Tend to be midline.
e tc ...) creating a bone defect with Location Most Common The skin ones tend to
benign appearing sclerotic borders. ( “behind the ears ”) be around the orbits.

There are a few differences between Associated with


Encephaloceles -
the two subtypes that could be especially when
potentially testable (contrasted midline
masterfully in the chart). - More
Heterogeneous,
Although, I suspect a “what is it ? ” CT CSF Density - Calcifications
type question is more likely. As (internal or peripheral)
may be present
such, look through some google
image examples to prepare yourself T1 Variable, T1 Bright,
MRI T2 Bright, T2 Bright,
for that contingency.
NO Enhancement +/- Wall Enhancement

158
th is vs THAT: Scalp Trauma

There are 3 scalp hem atom a subtypes. Because the subtypes are fairly similar, there is a high
likelihood a sadistic multiple choice w riter will attem pt to confuse you on the subtle
differences - so let’s do a quick review.

Subgaleal Caput
Hemorrhage Cephalohematoma Succedaneum

Scalp Skin

Aponeurosis
Periosteum LV_"_'> '

Skull

C ap
Subgaleal llem orrliagc Cxphaldhcinaloinii
Succeda

Subcutaneous
Deep to the Aponeurosis
Under the Periosteum Hemorrhage
Location (between aponeurosis and
(skin o f the bone) (superficial to
periosteum)
the aponeurosis)
Suture NOT limited by Limited bv suture lines NOT limited by
Relationship suture lines (won’t cross sutures) suture lines

Outer border may calcify as a rim


Covers a much larger area
Trivia and leave a deformity - sorta like
than a cephalohematoma
a myositis ossificans.

Usually requires no intervention


Potentiallv life-threatening (resolves within a few weeks)
Requires no
- rapid blood loss.
intervention
Can get super infected (E.Coli).
Complications (resolves within
Often not seen undl 12-72 Abscess would require drainage.
a few days)
hours post delivery. Can cause skull osteomvelitis.

Prolonged
Cause Vacuum Extraction Instrument or Vacuum Extraction
Delivery

159
Skull Fractures
Accidental (and non-accidental) head trauma is supposedly (allegedly, allegedly) the most common
cause of morbidity and mortality in children. As you might imagine, the pediatric skull can fracture
just like the adult skull - with linear and comminuted patterns. For the purpose of multiple choice, I
think we should focus on the fracture patterns that are more unique to the pediatric population:
Diastatic, Depressed, and “Ping-Pong”

• Diastatic Fracture: This is a fracture along / involving the suture. When they intersect it is usually
fairly obvious. It can get tricky when the fracture is confined to the suture itself The most common
victim of this sneaky fracture is usually the Lambdoid, followed by the Resident reading the case on
night float .. .with Attending backup (asleep in bed). How does one know there is traumatic injury to
a suture ? Classically, it will widen. This is most likely to be shown in the axial or coronal plane so
you can appreciate the asymmetry ( > 1 mm asymmetry relative to the other side).

• Depressed Fracture: This is a fracture with inward displacement of the bone. How much inward
displacement do you need to call it “depressed” ? Most people will say “equal or greater to the
thickness of the skull.” Some people will use the word “compound” to describe a depressed
fracture that also has an associated scalp laceration. Those same people may (or may not) add the
word “penetrating ” to describe a compound fracture with an associated dural tear.
Will any o f those people be writing the questions ? The dark side clouds everything. Impossible to
see the future is.

• Ping Pong Fracture: This is actually another subtype of depressed fracture but is unique in that it is
a greenstick or “buckle” type of fracture. Other potentially testable differences include:

• Outcomes: Ping Pong fractures typically have a favorable / benign clinical outcome
(depressed fractures have high morbidity).
• Etiology: Diastatic and depressed fracture types usually require a significant whack on
the head. Where as “ping pong” fractures often occur in the setting of birth trauma
(Mom’s pelvic bones +/- forceps).
• Imaging Appearance: Ping Pong fractures are hard as fuck to see. To show this on a
test you’d have to have CT 3D recons demonstrating a smooth inward deformity. You
could never see that shit on a plain film. I can’t imagine anyone being a big enough
asshole to ask you to do that. Hmmm.... probably.

4 ^ N EX T STEP:
Depressed Fx THIS vs THAT:
Unlike linear fractures (which usually heal without complication), Normal
depressed fractures often require surgery. Some general indications Fi acturc
Suture
for surgery would include:
• Depression of the fragments > 5mm (supposedly fragments more > 3mm < 2mm
than 5mm below the inner table are associated with dural tears),
Wide Center Equal Width
• Epidural bleed
“Darker” “Lighter”
• Superinfection (abscess, osteomyelitis)
Straight
• “Form” (cosmetic correction to avoid looking like a gargoyle), Squiggly
Line, with
Line, with
• “Function” (if the frontal sinus is involved, sometimes they need Angular Curves
to obliterate the thing to avoid mucocele formation). Turns

160
Leptomeningeal Cyst - ‘‘Growing Skull F racture"

A favorite o f board examiners since the Cretaceous Period.


Typical Pathogenesis:
IZ
- Step 1; You fracture your noggin, AND
tear the underlying dura.
- Step 2: Leptomeninges herniate through the tom dura mto
the fracture site.
Step 3: Over time (a few months) CSF
pulsations progressively widen the
fracture site and prohibit normal healing.
Step 4: You know you shouldn’t, but you
just can’t resist the urge to poke your own
brain through the now cavernous cranial Initial
itial ^ 2 Months
onths
defect. Film Later

Step 5: The poking triggers a powerfiil hallucinogenic experience. You have a telepathic
conversation with a room filled with self transforming elf machines.
You are overwhelmed with tremendous curiosity about exactly
what/who they are and what they might be trying to show you.
Step 6: You develop epilepsy from poking your brain too much.
Or was it not enough? - you can’t remember

Sinus Pericranii
A rare disorder that can be shown as a focal skull defect with an associated vascular malformation. The
underlying pathology is a low flow vascular malformation - which is a communication between a dural
venous sinus (usually the superior sagittal) and an extra cranial venous structure via the emissary veins.
It is not classically associated with discoloration of the overlying skin.

Most likely way to show this:

(1) MRI - with some type of vascular sequence


post contrast or MRA/TOF.

(2) CT showing the skull defect - wanting a next step


(ultrasound or MRI to demonstrate the vascular
component). Skull Defect +
Vascular
Malformation
Non-Accidental Irauma [ Abusive Head Trauma)
Although car wrecks and falls account for the majority o f skull fractures in children, there
still remains the timeless truth — some people just can’t take scream ing kids.

For the purpose o f multiple choice, the follow clues should make your spider-sense tingle.

*More on N A T later in the chapter.

162
S E C T IO N 2:
B r a in - S e l e c t T o p ic s

Disclaimer: Brain tumors , cord tumors, and a bunch of other random Peds Neuro pathologies are
discussed in detail within the Neuro chapter found in volume 2. The same is true for congenital
heart, certain GU, GYN, and MSK topics - found within their dedicated chapters. If you find
yourself saying “Hey! What about that thing? This asshole is seriously not going to talk about
that? ” Relax, I split things up to reduce redundancy and cluster things for improved retention.

THIS vs THAT:
Enlarged extra-axial BESSI vs Subdural Hygroma
fluid spaces:
B E SSI - Cortical veins are
Extra-axial fluid spaces are adjacent to the inner table -
considered enlarged if they they are usually seen /► -j S m Positive
secondary to enlargement of Cortical Vein
are greater than 5 mm. Sign = BESSI
the subarachnoid spaces 'Vi,
BESSI is the name people (positive cortical vein sign) \^H2pillllllr (or Cortical
throw around for “benign Atrophy)
enlargement of the Subdural Hygroma - This is
subarachnoid space in a collection of CSF in the ^ -----
subdural space. It can be 1
infancy.” from trauma (or idiopathic). /
You see these more in the / Negative
The etiology is supposed to elderly. The testable trivia in / Cortical Vein
this case is that the cortical Sign =
be immature villa (that’s why Subdural
veins are displaced awav from
you grow out of it). the inner table - they are often Hygroma
not seen secondary to
compression.
B E SSI Trivia:
It’s the most common cause of macrocephaly. Enlarged
Typically presents around month 2 or 3, and has symmetric
a strong male predominance. subarachnoid
Typically resolves after 2 years with no spaces favoring
treatment, the anterior aspect
There is an increased risk of subdural bleed - of the brain
either spontaneous or with a minor trauma. This (spaces along the
posterior aspect of
subdural is usually isolated (all the same blood
the brain are
age), which helps differentiate it from non­
typically normal).
accidental trauma, where the bleeds are often of
different ages. Brain parenchyma is normal and there is either
normal ventricle size or very mild
Trivia - Pre-mature kids getting tortured communicating hydrocephalus. Communicating
on ECMO often get enlarged extra-axial meaning that all 4 ventricles are big.
spaces. This isn’t really the same thing as
BESSI but rather more related to fluid changes / stress.

163
Periventricular Leukomalacia ( Hypoxic-lsGhemic Encephalopathy of the Newborn)
This is the result o f an ischemic / hemorrhagic injury, typically from a hypoxic insult during
birthing. The kids who are at the greatest risk are premature and little (less than 1500 g). The
testable stigmata is cerebral palsy - which supposedly develops in 50%. The pathology favors
the watershed areas (characteristically the white matter dorsal and lateral to the lateral
ventricles).

The milder finding can be very subtle. Here are some tricks:

(1) Use PreTest Probability: The kid is described as premature or low birth weight.

(2) Brighter than the Choroid: The choroid plexus is an excellent internal control. The
normal white matter should always be less bright (less hyperechoic) when compared to the
choroid.

(3) “Blush ’’ and “F laring” : These are two potential distractors that need to be differentiated
from legit grade 1 PVL. “Blush ” describes the physiologic brightness o f the
posterosuperior periventricular white matter - this should be less bright than choroid, and
have a more symmetric look. “F laring” is similar to blush, but a more hedgy term. It’s the
word you use if you aren’t sure if it’s real PVL or just the normal brightness often seen in
premature infants white matter. The distinction is that “fla rin g ” should go away in a
week. Grade 1 PVL persists > 7 days.

The later findings are more obvious with the development o f cavitary periventricular cysts.
The degree o f severity is described by the size and distribution o f these cysts. These things take
a while to develop - some people say up to 4 weeks. So, if they show you a day 1 newborn
with cystic PVL they are leading you to conclude that the vascular insult occurred at least 2
weeks prior to birth (not during birth - which is often the case).

Early: Periventricular White Matter Necrosis Subacute: Cyst Formation


(hyperechoic relative to choroid)

T riv ia
The most severe grade (4), which has subcortical cysts, is actually more common in
full term infants rather than preterms.

164
Germinal Matrix Hemorrhage [ GMH)
I like to think about the germinal matrix as an em bryologic seed that sprouts out various
development cells during brain development. Just like a seed needs w ater to grow, the
germinal matrix is highly vascular. It’s also very friably and susceptible to stress.
Additionally, prem ature brains suck at cerebral blood flow auto-regulation. Mechanism:
Fragile vessels + too much pressure/flow = bleeds

An im portant thing to understand is that the germ inal m atrix is an em bryological entity. So it
only exists in prem ature infants. As the fetus matures the thing regresses and disappears.

By 32 weeks, germinal matrix is only present at the caudothalam ic groove.


By 36 weeks, you basically can’t have it (if no GM, then no GM hem orrhage).

Take home p oin t - No G M H e m o rrh a g e in a full te rm in fan t.

Gamesmanship: Sim ilar looking bleed in a fiill term infant say “choroid plexus
hemorrhage” (not GMH).

The scenario will always call the kid a prem ature infant (probably earlier than 30 weeks). The
earlier they are bom the more com m on it is. Up to 40% occur in the first 5 hours, and m ost
have occurred by day 4 (90%). A good thing to rem em ber is that 90% occur in the first week.

Screening: Head US is used to screen for this pathology. Testable trivia includes:

• Who should be screened? Prem ature Infants (<32 weeks, < 1500 grams). Prem ature Infants
with Lethargy, Seizures, D ecreased H em atocrit or a history o f “he d o n ’t look so good.”

• When do you do the head US? First w eek o f life (rem em ber this is w hen 90% o f them occur).
Some people will tell you - “first w eek and first m onth” (but that varies from institution).
Some people will also say - “every kid gets a head US prior to discharge from the N IC U ” - but
that is m ainly done to detect PVL (not necessarily GMH).

165
Germinal Matrix Hemorrhage ( GMH) - Continued
Choroid Plexus is bright (hyperechoic)
on ultrasound. Blood is also bright
(hyperechoic).
Choroid Plexus
You tell them apart based on their
location. Choroid should not extend
anterior to the junction o f the caudate
and the thalamus (the so called - T ^
caudothalamic groove). Caudothaiamic ■
Groove * J
This is the location o f the germinal
matrix.

If you see bright stuff there - that is


your grade 1 bleed.

This could also be shown with MRI


(T1 bright = bleed), same location.

Grade 1 GM
Hemorrhage
-Blood in the CT Groove

Grading System (1-4) - quick and dirty Secondary Consequences GMH

1 - Blood at the caudothalamic groove Grade 1


i 2 - Blood in the ventricles Subependymal Subependymal cyst
but no dilation. / Hemorrhage
I 3 - Blood in the ventricles with dilation
j 4 - Blood in the brain parenchyma
(trom venous infarct)
Resolution
i
Grade 2/3
Intraventricular
Hemorrhage
Few additional things you should see at least once
prior to the exam. Do yourself a favor and
google images of the following;
Hydrocephalus
i
Grade 4
• Caudothalamic groove in the coronal plane Intraparenchymal
• Grade 2,3,4 Bleeds Hemorrhage
• Subependymal Cysts on US \
• Porencephalic Cysts on US Porencephalic Cyst
• Choroid Plexus Cysts on US (infarcted hole in head)
• Ventricular coarctation on US - this is a mimic

166
S E C T IO N 3:
H ead a n d N eck
- S elect To p ic s -

Disclaimer: All o f the temporal bone pathology you would commonly associate with Peds is also
discussed in detail in the Neuro chapter - found in Volume 2. This is also true fo r the classic orbital
pathologies o f childhood (Retinoblastoma etc...)

Midface Malformations: Babies are noraially cute, but sometimes things go


horribly wrong. Although it is easy to focus on the negatives of being cursed with a horrible monster
face, it is not always bad. There are still many promising career opportunities available; ringing the
bell in a tower, working as an assistant for a mad scientist, and so on. The easiest way to think about
the various types of monster baby faces is to group them by location.

Location 1: Nasal Cavity MS

A quick review of some anatomy will really help with


understanding the two primary malformations in this
region.

This is an axial slice through the nasal cavity, at the level


of the maxillary spines (the pointy things “MS”). The
vomer (“V”) is in the middle.

The two key things to know here are the pyriform aperture
(“PA”) is the area in the front (between the maxillary
spines), and the posterior choanae (“PC”) is the area in the Aperture is the business in the Front
back. Choanae is the party in the Back

l^riform Aperture Stenosis: front end o f the nose

The problem is the medial nasal processes are either fused or they are
too big resulting in either total obstruction or stenosis of the nose. The
classic look is an axial CT though the maxillary spines demonstrating
soft tissue extension across the nostrils and a narrowed / inward bow to
the maxillary spines (aperture < 8 mm).

The big trivia is the association with holoprosencephalv. pituitarv


dvsfunction. and the Central M EGAincisor (seen in 75% o f cases). All
o f these things reflect a midline developmental problem so the
association with hypothalamic-pituitary-adrenal axis dysfunction is
highly testable.

— as my Grandma always said ‘face predicts brain ”.

Next Step - You have to image the brain

167
C h0 3 n 3 l AtfSSiS: back end o f the nose

Results from a membrane that separates the nasal cavity


from its normal communication with the oral cavity. It is
usually unilateral but can be bilateral. The bilateral ones
have a history of “cyclical cyanosis that improves with
crying’’ (they mouth breath when they cry). This is an
emergency and typically requires emergent measures to
preserve the airway. The unilateral classic stories are
“can’t pass NG tube, ” and “respiratory distress while
m-
feeding” (neonates have to breath through their noses).
You will also sometimes hear a history of “a continuous
stream o f snot draining from one or both nostrils, ” or the
word “rhinorrhea. ”

There are two different types: bony (90%), and membranous (10%). The
CHARGE
Coloboma, appearance is a unilateral or bilateral posterior nasal narrowing, with
Heart defect, thickening of the vomer.
Atresia (Choanal)
Retarded growth, Trivia: There are many syndromic associations including CHARGE. Crouzons,
Genitourinary DiGeorge, Treacher Collins, and Fetal Alcohol Syndrome. CHARGE is the one
abnormalities people mention the most.
Ear anomalies

Location 2; Nasofrontal Region

These are embryology problems where the dural membrane in the prenasal space don’t regress. This fuck
up in regression leaves a patent tract which things can protrude or grow into.

Congenital Dermal Sinus with Dermoid / Epidermoid Cysts


Usually when people talk about these things they are referring to the spina bifida style midline
lumbosacral region defects. However, we are going to stay focused on the face. During normal
development there is a dural tract in the prenasal region that is supposed to regress and close. If it
doesn’t you can have dermoid or epidermoid cysts form anywhere along the course of the tract. They
will be midline, and will NOT change in size with crying or jugular vein compression. For
gamesmanship, consider a sinus tract anytime you see a cyst in these locations.

Sinus tracts may or


may not have
associated dermal
or intraosseous
cysts.

Cysts may or may


not have sinus
tracts.
Dural communication will require
communication through the foramen cecum.
168
Nasal Glioma

These things aren’t actually gliom as - they have no neoplastic


features. They are more like “brain in the w rong place.”
Some people call them “nasal cerebral heterotopia,” but those
people are assholes. They m ight grow - but the grow th rate
tends to parallel brain growth rate.

Just like the dermoids I talked about on the prior page, these
things don’t change size if you cry like a baby or com press the
jugular vein.

They don’t enhance. They ju st look like a soft tissue mass in


the nose (o f a baby).

Encephaloceles
As we discussed, if the dural diverticulum that extends
through the foramen cecum in early developm ent doesn’t
involute you end up with a tract. If intracranial contents
herniate into this tract you get yourself an encephalocele.

These things have a bunch o f associations (facial clefts,


callosal issues, interhem ispheric lipomas) — mostly midline
stuff, which fits with the theme.

Intranasal encephaloceles hang dow nward into the


superomedial nasal cavity. For the purpose o f m ultiple choice,
a nasal catheter will be displaced laterally by the
encephalocele (medial displacem ent with a dacryocystocele).

They CAN change size with crying (like a baby) or com pression o f the jugular vein.

M any times in this text “Biopsy the M otherfucker” will be my recom m endation as the next step.
This is NOT one o f those times. D o n ’t biopsy th is .... M other fuckers. The concern is CSF
leaking and meningitis.

169
Location 3; Nasolacrim al Apparatus

Dacryocystocele

Dacryocystoceles are the result of an obstructed nasolacrimal duct (the duct that drains your eye into your
nose). This duct is the reason you make pathetic snotty nose sounds when your high school boyfriend
dumps you for that slutty girl you hate. In babies poorly understood congenital obstruction of this duct
(and resulting dacryocystocele formation) is actually the second most common cause of neonatal nasal
obstruction after choanal atresia.

These things often show up in the ER after they get infected and acquire the rank of “dacryocystitis.”

Craniofacial Syndromes
There are 3 big ones that people talk about. Crouzans, Aperts, and Treacher Collins Syndrome
(Mandibulo- facial Dysostosis). I’ve got a chart on Crouzans & Aperts earlier in the chapter (pg 41), so
I’ll just touch on Treacher Collins.

Treacher Collins Syndrome ( Mandibulo- facial Dysostosis 1

These are the kids with the tiny jaws that are
super dangerous to intubate. They have small
absent zygomatic arches, narrow but over
projected maxilla, and mandible hypoplasia with
a retruded weak / unmanly chin.

Skinny Protruded Maxilla


Tiny Retruded Chin (destined to drink soy milk) Normal M andible
Looks like a Horseshoe T.C. Mandible
Tiny Absent Zygomas / Malar Bones Has a Concave Curve of
the Horizontal Ramus
Useless Trivia: Problem is with the 1st and 2nd Branchial Arches (nothing else does that)

170
E ctop ic Thyroid:

Thyroid topics will be covered again in the endocrine chapter. I do want to mention one or two now for
completeness. To understand ectopic thyroid trivia you need to remember that the thyroid starts
(embryology wise) at the back of the tongue. It then descends downward to a location that would be
considered normal. The “pyramidal lobe” actually represents a persistence of the inferior portion of the
thyroglossal duct - that is why this thing is so variable in appearance. Sometimes this process gets all
fucked up and the thyroid either stays at the back of the tongue (lingual thyroid) or ends up half way
down the neck or even in the chest (ectopic thyroid).

Trivia to know:
• Most “developed” countries test for low thyroid at birth (Guthrie Test). That will trigger a workup for
either ectopic tissue or enzyme deficiencies.
• Nukes (1-123 or Tc-MIBI) is superior to ultrasound for diagnosing ectopic tissue. This is by far the
most likely way to show this on a multiple choice exam. I guess CT would be #2 - remember thyroid
tissue is dense because of the iodine.
• Ultrasound does have a preoperative role in any MIDLINE neck mass - with the point of ultrasound
being to confirm that you have a normal thyroid in a normal place. If you resect a midline mass
(which turns out to be the kids only thyroid tissue) you can expect an expensive well rehearsed didactic
lecture on pediatric neck pathology from an “Expert Witness” sporting a $500 haircut.
• Lingual thyroid (back of the tongue) is the most common location of ectopic thyroid tissue.

Thyroglossal Duct Cyst:

As we discussed previously, thyroid related pathology can occur


anywhere between the foramen cecum (the base of the tongue)
and the thyroid gland. In this situation we are talking about the
duct (which is the embryological thyroid interstate highway to the
neck) failing to involute fully. What you get is a left over cyst -
hence the name. The classic locations are (I) at the base of the
tongue, and (2) midline anterior to the hyoid. Now textbooks will
make a big deal about these things becoming slightly lateral below
the hyoid. Do NOT get hung up on that. For the purpose of multiple
choice remember these guys are midline. Midline is the buzzword.

Things to know:
• Classic Buzzword / Scenario = Midiine Cyst in the Neck of a Kid.
• Next step once you find one = confirm normal thyroid location and/or
look for ectopic tissue (Ultrasound +/-Tc-MIBI, or I-123).
• They are cystic (it’s not called a “Duct Solid”)
• Enhancing nodule within the cyst = CANCER (usually papillary)
• They can get infected - look for rim enhancement + fatty stranding
• Rx is Sistrunk procedure (yes, that’s actually the name)
- remove cyst, tract + hyoid, decreases recurrence

Dermoid Cyst

It is true that dermoids almost always occur below the clavicles, but
when they do happen in the neck they have a pretty classic look;
midline sublingual / submandibular space with a “sac of marbles”
appearance. The marbles are lobules of fat within fluid.

171
Branchial Cleft Cyst (BCC)

Another cystic em bryologic remnant. There are a bunch o f types (and subtypes... and sub­
subtypes) and you can lose your fucking mind trying to rem em ber all o f them - don’t do that.
Just rem em ber that by far the most com m on is a 2"^ Branchial Cleft Cyst (95%). The angle
of the m andible is a classic location. They can get infected, but are often asymptomatic.
Extension o f the cyst between the ICA and EGA (notch sign) ju st above the carotid
bifurcation is pathognomonic.

How can this be asked on multiple choice:

• What is it ? - M ost likely on CT or MRI. Ultrasound w ould be tough, unless they clearly
labeled the area “lateral neck” or oriented you in some other way.

• Location Fuckery. They could (and this w ould be super mean) ask you the relationship o f a
type 2 based on other neck anatomy. So - posterior and lateral to the subm andibular gland
or lateral to the carotid space, or anterior to the sternocleidom astoid. How I w ould handle
that? Just rem em ber it’s going to be lateral to everything. Lateral is the buzzword.

• Mimic - They could try and trick you into calling a necrotic level 2 lymph node a BCC.
Thyroid cancer {history o f radiation exposure) and nasopharyngeal cancer {history o f HPV)
can occur in “early adulthood.” If you have a “new ” BCC in an 18 year old - it’s probably a
necrotic node. N ext Step = Find the cancer +/- biopsy the m other fucker.

I say LATERAL
cyst in the neck, Type II - BCC
you say branchial
cleft cyst

I say M IDLINE
cyst in the neck,
4
'i:.
you say
thyroglossal duct
cyst

172
Jugular Vein Pathology
There are two jugular vein issues that occur in kids that I should probably mention.

Septic Thrombophlebitis - i.e. clotted jugular vein. You see this classically in the setting of a
recent pharyngeal infection (or recent ENT surgery).

How can this be asked on multiple choice:

• What is it ? - Showing the clotted vein with the appropriate clinical history.

• “Lemierre’s Syndrome ” - Seeing if you know that it has a fancy syndrome name.

• Next Step? Looking in the lungs for septic emboli. This could also be done in the reverse. Show
you the septic emboli, give you a history of ENT procedure (or recent infection), and have you ask
for the US of the neck veins.

• USMLE Step 1 Association Trivia = Fusobacterium necrophorum is the bacteria that causes the
septic emboli. As this bacteria sounds like a Marvel Comic villain the likelihood of it being asked
increases by at least 5x.

Phlebectasia - Idiopathic dilated jugular vein.

How can this be asked on multiple choice:

• What is it ? - Showing the dilated vein.

• Trivia - It is NOT related to a stenosis. There are no other


signs of venous congestion.

• Trivia - It gets worse with the Valsalva maneuver - “neck


mass that enlarges with valsalva. ” Phlebectasia - Dilated Jugular Vein

MlscVeno/Lymphatics
Venous, Arterial and Lymphatic Malformations are complex with overlapping features and
numerous classification schemes. I ’ll cover them again in the vascular chapter - this is just a brief
peds neck tangent. Both lymphatic and venous malformations can both look like a large trans-
spatial multi-cystic mass in the neck. They can both have fluid levels.

If you must try and tell them apart - you could try this:
• Venous Malformations will have enhancement o f the cystic spaces.
• Lymphatic Malformations will have enhancement o f the septa.
• Phleboliths — suggests venous.

In many cases they coexists together .... so ... y eah ... hopefiilly the person writing the question
understands that.

173
Hemangioma of Infancy

These things are actually the most com mon congenital lesions in the head and neck. Just like
any hem angiom a they contain vascular spaces with varying sizes and shapes. M ost people
consider them a “tum or” more than a vascular malformation.

Things to know:
• H ow they look = Super T2 bright, with a bunch o f
flow voids. Diffusely vascular on doppler. Vocabulary Trivia -
• Phases = Typically they show up around 6 months o f T H IS vs THAT:
age, grow for a bit, then plateau, then involute (6-10 Infantile Hemangiomas = NOT
years). Usually they require no treatment. present at birth. Show up around
• Indications fo r Treatment = Large size / Rapid growth 6 months of age. Nearly always
with mass effect on the airway or adjacent vascular involute.
structures. Fucking with the kids eye m ovem ent or Congenital Hemangiomas =
eyelid opening. Present at birth. May (RICH) or
• Treatment = Typically m edical = Beta blocker may not (NICH) involute.
(propranolol)
• Associations = PHACES Syndrome (discussed later
in the chapter) - think this if it is intra-cranial or m uhiple.

Cystic Hygroma (Lymphangioma)

This is another cystic lesion o f the neck, which is most likely to be shown as an OB
ultrasound (but can occur in the Peds setting as well). The classic look / location is a cystic
mass hanging o ff the back o f the neck on OB US (or in the posterior triangle if CT/MRI).

Trivia:

• Associations: T u rn ers (most


common association). D owns
(second most com mon association).
A ortic C o arctatio n (most com mon
CV abnormality), Fetal Hydrops
(bad bad bad outcomes).

• Septations = Worse Outcome

• T2 Bright (like a hemangioma).


Does NOT enhance (hemangiomas C ystic H y g ro m a (L ym p h an g io m a)
typically do).
This Turners Baby can’t . ... Turn its head.

174
Fibromatosis Coli (<<Congenital Torticollis”)

This is a benign “mass” of the sternocleidomastoid


in neonates who present with torticollis (chin points
towards the opposite side - or you could say they
look away from the lesion). It’s really just a benign
inflammation that makes the muscle look crazy big.

Ultrasound can look scary, until you realize it’s just


the enlarged SCM. Ultrasound is still the best
imaging test. Sometimes it looks like there are two
of them, but that’s because the SCM has two heads.
It goes away on its own, sometimes they do passive
physical therapy or try and botox them.
- This Dragon has Two Heads -
Most common cause of a neck “mass” in infancy
Classic scenario is a 4 week old with a palpable neck mass and torticollis toward the affected side
Best imaging test = US.
Things that make you think it’s not FC: mass is outside the SCM, or internal calcifications - in
which case you should think to yourself... nice try Mother fuckers- that’s a neuroblastoma.

Gamesmanship:

So... there can be significant fuckery with the “direction” things curve or people look depending
on how the question is ask. What do I mean ?

If you made the mistake of just memorizing the word “towards” in association with fibromatosis
coli you might get tricked if the options were: A - Patient looks towards the involved side.
B - Patient looks toward the uninvolved side. You’d run into the same problem with the word
“away.”

Now, that might seem obvious once I spell it out like that but I’m pretty sure at least a few of you
were making a flashcard that had only the word “towards” on it. You have to assume the test
writer has the worst intentions for you. Don’t provide them with any opportunity to trick you.

Rhabdomyosarcoma
Although technically rare as fiick, this is the most common mass in the masticator space of a kid (~ 70
% occur before age 12). Having said that if you see it in the head/neck region it is almost always in the
orbit. In fact, its the most common extra-occular orbital malignancy in children (dermoid is most
common benign orbital mass in child). The most classic scenario would be an 8 year old with painless
proptosis and no signs of infection.

What do sarcomas look like?

I’ll talk about this more in the MSK chapter, but in general I’ll just say they look mean as cat shit
(enhancing, solid, areas of necrosis, etc..).

175
SE C T IO N 4:
A irw a y

Croup

This is the most com mon cause o f acute upper airway obstruction in young children. The
peak incidence is between 6 months and 3 years (average 1 year). They have a barky
“croupy” cough. It’s viral. The thing to reahze is that the lateral and frontal neck x-ray is
done not to diagnosis croup, but to exclude something else. Having said that, the so-called
“steeple sign ” - with loss o f the normal lateral convexities o f the subglottic trachea is your
buzzword, and if it’s shown, that will be the finding. Questions are still more likely to center
around facts (age and etiology). The culprit is often parainfluenza virus.

Epiglottitis

In contrast to the self-limited croup, this one can kill you. It’s m ediated by H. Influenza and
the classic age is 3.5 years old (there is a recent increase in teenagers - so don’t be fooled by
that age). The lateral x-ray will show marked swelling o f the epiglottis {thumb sign). A fake
out is the “om ega epiglottis” which is caused by oblique imaging. You can look for thickening
o f the aryepiglottic folds to distinguish.

Trivia: Death by asphyxiation is from the aryepiglotic folds (not the epiglottis)

Exudative Tracheitis (Bacterial Tracheitis)

This is an uncom m on but serious (possibly deadly) situation that is found in slightly older
kids. It’s caused by an exudative infection o f the trachea (sorta like diptheria). It’s usually
from Staph A. and affects kids between 6-10. The buzzw ord is linear soft tissue fillin g defect
within the airway.

C ro u p E p ig lo ttitis E x u d a tiv e T r a c h e itis


6 months - 3 years Classic = 3.5 years, but now
6-10 years
(peak I year) seen w ith teenagers too
Linear soft tissue filling
Steeple Sign: loss o f the normal defect (a m em brane) seen
Thumb Sign: marked
shoulders (lateral convexities) o f within the airway.
enlargement o f epiglottis
the subglottic trachea Irregular tracheal walls
(plaques)

Viral
H-Flu Staph. A
{Most Common - parainfluenza)

176
Retropharyngeal Cellulitis and A b sce ss

Discussed in detail in the Neuro chapter o f Volume 2. I’ll just say quickly that you do see this
most commonly in young kids (age 6 months -12 months). If they don’t show it on CT, they
could show it with a lateral x-ray demonstrating massive retropharyngeal soft tissue
thickening. For the real world, you can get pseudothickening when the neck is not truly lateral.
To tell the difference between positioning and the real thing, a repeat with an extended neck is
the next step.

Subglottic Hemangioma
PHACES
Hemangiomas are the m ost com mon soft tissue mass in the
P- Posterior fossa
trachea, and they are m ost com m only located in the subglottic
(Dandy Walker)
region. In croup there is symmetric narrowing with loss o f
H - H em angiom as
shoulders on both sides (Steeple Sign). In contradistinction,
A- A rterial anom alies
subglottic hem angiomas have loss o f ju st one o f the sides.
C - C oarctation o f aorta,
cardiac defects
Trivia
E- Eye abnormalities
• Tends to favor the left side
S- Subglottic
• 50% are associated with cutaneous hem angiomas
hem angiom as/ Sternal
• 7% have the PHACES syndrome
C left/ Supraum bilical
raphe
More on hem angiomas later in the chapter.

Laryngeal Cleft

This is a zebra. The classic scenario is contrast appearing in the tracheal without laryngeal penetration
(aspiration). They could also show you a “thin tract of contrast extending to the larynx or trachea.” This
entity is a communicating defect in the posterior wall of the larynx and the esophagus or anterior hypo
pharynx. There are a bunch of different cleft classifications - 1 can’t imagine that shit is appropriate for
the exam. I would just know:

• This is a thing - maybe google a Fluoro swallow picture of it


• These things have other complex malformations associated with them (usually GI)
• It is very tricky to call it with certainty on a swallow exam - definite diagnosis is always made with
direct visualization / scope (so a next step type question would recommend endoscopy to confirm).

Tracheoesophageal Fistula - This is discussed later in the chapter (GI section)

Papilloma - If you see a lobulated grape looking thing in the airway - think Papilloma, especially if
the lungs are full of nodules (solid and cavitated). When I say Papilloma, You say HPV - typically from
perinatal (birth canal) transmission. These things are usually multiple (papillomatosis) and therefore have
multiple areas of airspace disease (atelectasis ect.). Some potential gamesmanship — because these thing
are typically multiple you will have more areas of air trapping then you would compared with an
aspirated crayon (or green bean), or even a solitary endobronchial lesion like a carcinoid. Multiple areas
of air trapping - think Papillomatosis over carcinoid or a foreign body. Having said that the nodules are a
more common finding... lots of them.

177
- Gamesmanship -
Frontal and Lateral Neck Radiographs

For the frontal, there are


two main things to think
about.

(1) Croup and


(2) Subglottic
Hemangioma

You can tell them apart by


the shouldering.

If you can’t tell.... try and


let the history bias you.
Cough? Fever? Think
Croup.

For the lateral, there are 4 main things to think about:

Epiglottitis Retropharyngeal Tonsils (adenoids) Exudative


Abscess Tracheitis
Looks like a thumb Not seen till about 3-6
Too wide months, and not big Linear Filling Defect
If the ordering ( > 6mm at C2, till around 1-2 years.
suspects the or > 22mm at C6) It’s usually staph
diagnosis, do NOT Too big when they
bring this kid to x- Next Step = CT encroach the airway
ray. Have them do don’t forget to look in
a portable. the mediastinum for
“Danger Zone ”
extension.

178
S E C T IO N 5:
C h e s t

Before we proceed with the trivia I need to make sure you know / can do two things for me.

1) Know how to tell if a neonatal chest is hyper-inflated or not. Don’t get hung up on this low vs
normal - that’s a bunch of bologna. Just think (a) Hyper-inflated, or (b) NOT Hyper-inflated.
The easiest way to do this is to just count ribs.
More than 6 Anterior, or 8 Posterior as they
intersect the diaphragm is too much. As a
quick review, remember that the anterior ribs
(grey) are the ones with a more sloping course
as they move medially, where as the posterior
ribs (black) have a horizontal course.

Other helpful signs suggesting hyperinflation:


• Flattening of the diaphragms
• Ribs take on a more horizontal appearance
• Some increased lucency under the heart

2) Know what “Granular” looks like. Know what “Streaky” or “Ropy” looks like. My good
friends at Amazon are not capable of printing a clear picture of these so I want you to stop reading
and
A. Go to google images
B. SesLTch “Granular neonatal chest x-ray." Look at a bunch of examples. Maybe even
download a few of them for review.
C. Search “Streaky Perihilar neonatal chest x-ray.” Look at a bunch of examples. Maybe even
download a few of them for review.
D. Search “Ropy neonatal chest x-ray." Look at a bunch of examples. Maybe even download a
few of them for review.

Random Pearl; We are going to talk about the presence of a pleural effusion as a discriminator. One
pearl is to look fo r an accentuated (thick) minor fissure on the right. If you see that shit, kid probably
has an effusion. Confirm by staring with fierce intensity at the lung bases to look for obliteration of
the costophrenic sulcus.

Alphabet - M N o P
As we proceed forward with the
trivia, pay close attention to lung High Volumes (1) Meconium
volumes, and the words “granular”, + Perihilar Streaky (2) Aspiration
“streaky”, and “ropy.” (3) Non GB Neonatal

You will find that you can divide Not High (low or
the big 5 in half by doing something (4) S S D
normal) Volumes
like this: (5) Group B Pneumonia
+ Granular

179
- Life in the NiCU ( just as glamorous as it sounds ) -

Meconium Aspiration
This typically occurs secondary to stress (hypoxia), and is more common in term or post-mature babies
(the question stem could say "post term ” delivery). The pathophysiology is all secondary to chemical
aspiration.
Trivia:
• The buzzword “ropy appearance” of asymmetric lung densities
• Hyperinflation with alternative areas of atelectasis
• Pneumothorax in 20-40% of cases

How can it have hyperinflation?? Aren’t the lungs full of sticky shit
(literally) ??? The poop in the lungs act like miniature ball-valves (“floaters” I
call them), causing air trapping - hence the increased lung volumes.

Reality vs Multiple Choice: “Meconium Staining” on the amniotic flu id is common (like ISVo o f all
births), but development o f “aspiration syndrome” is rare with only 5% o f those 15% actually have
aspiration symptoms. Having said that, if the question header bothers to include “Green colored
amniotic flu id ” or “Meconium staining” in the question header they are giving you a major hint.
D on’t overthink a hint like this. I f they ask “What color was George Washington s white horse ? ” the
answer is NOT brown.

Transient Tachypnea of the Newborn (TTN)


The classic clinical scenario is a history of c-section (vagina squeezes the fluid out of lungs normally).
Other classic scenario histories include “diabetic mother” and/or “maternal sedation.” Findings are
going to start at 6 hours, peak at one day, and be done by 3 days. You are going to see coarse
interstitial marking and fluid in the fissures.
Trivia:
• Classic histories: C-Section, Maternal Sedation, Maternal Diabetes
• Onset: Peaks at day 1, Resolved by Day 3
• Lung Volumes - Normal to Increased

Surfactant-Deficient Disease (SDD)


This is also called hyaline membrane disease, or RDS. It’s a disease of pre-mature kids. The idea is
that they are born without surfactant (the stuff that makes your lungs stretchy and keeps alveolar
surfaces open). It’s serious business and is the most common cause of death in premature newborns.
You get low lung volumes and bilateral granular opacities (just like B-hemolytic pneumonia). But,
unlike B-hemolytic pneumonia you do NOT get pleural effusions. As a piece of useful clinical
knowledge, a normal plain film at 6 hours excludes SDD.

Surfactant Replacement Therapy


They can spray this crap in the kid’s lungs, and it makes a huge difference (decreased death rate etc...).
Lung volumes get better, and granular opacities will clear centrally after treatment. The post treatment
look of bleb-like lucencies can mimic PIE.
Trivia:
•Increased Risk of Pulmonary Hemorrhage
•Increased Risk of PDA

180
Neonatal Pneumonia (Beta-Hemolytic Strep - or “GBS”)
This is the most common type of pneumonia in newborns. It’s acquired during exit of the dirty birth
canal. Premature infants are at greater risk relative to term infants. It has some different looks when
compared to other pneumonias (why I discuss it separately).
Trivia:
•It often has low lung volumes (other pneumonias have high)
•Granular Opacities is a buzzword (for this and SDD)
•Often (25%) has pleural effusion (SDD will not)
•LESS likely to have pleural effusion compared to the non Beta hemolytic version (25% vs 75%)

Neonatal Pneumonia (not Beta-Hemolytic Strep - “Non GB” or “Non GBS”)


Lots of causes. Typical look is patchy, asymmetric perihilar densities, effusions, and hyperinflation.
Will look similar to surfactant deficient disease but will be full term. Effiisions are also much more
likely (they are rare in SDD).

Persistent Pulmonary HTN


Also called “persistent fetal circulation”. Normally, the high pulmonary pressures seen in utero (that
cause blood to shunt around the lungs) decrease as soon as the baby takes his/her first breath. Dr.
Goljan (Step 1 wizard) calls this a “miracle,” and used this basic physiology to deny evolution. When
high pressures persist in the lungs it can be primary (the work of Satan), or secondary from hypoxia
(meconium aspiration, pneumonia, etc...). The CXR is going to show the cause of the pulmonary
HTN (pneumonia), rather than the HTN itself

Pulmonary interstitial Emphysema (PIE) Chronic Lung Disease - CLD I


(Bronchopulmonary Dysplasia - BPD)
When you have surfactant deficiency and they put you
on a ventilator (which pulverizes your lungs with This is the kid bom premature (with resulting
PEEP), you can end up with air escaping the alveoli surfactant deficiency), who ends up being
and ending up in the interstitium and lymphatics. On tortured in ventilator purgatory. His/her tiny
CXR it looks like linear lucencies (buzzword). It’s a little lungs take a ferocious ass whipping from
warning sign for impending Pneumothorax. Most positive pressure ventilation and oxygen
cases of PIE occur in the first week of time toxicity — “barotrauma” they call it. This
(bronchopulmonary dysplasia - which looks similar - beating essentially turns the lungs into scar,
occurs in patients older than 2-3 weeks). Surfactant inhibiting their ability to grow correctly. That
therapy can also mimic PIE. The treatment is to is why people call this “a disease of lung
switch ventilation methods and/or place them PIE side growth impairment.”
down.
Rarely (zebra rare) PIE can progress to a large cystic • Classic Vignette: Prolonged ventilation in a
mass - which can cause mediastinal shift. tiny (<1000 grams), premature kid (<32
weeks)
Trivia:
• Classic Look: Alternating regions of
• Consequence of ventilation
• Fibrosis (coarse reticular opacities), and
• Usually occurs in the first week of life
Hyper-Aeration (cystic lucencies).
• Warning Sign for Impending Pneumothorax
• Treatment is to put the bad side down • Buzzword: “Band like opacities”
• Buzzword = Linear Lucencies

This T H A T : PIE vs CLD


Both have cystic lucencies. The difference (classic peds) is TIME.
PIE = First week of life CLD = After 3 to 4 weeks' postnatal age

181
d v . Solving Cases Using Buzzwords
When I say “Post Term Baby, ” How this could work:
You Say M econium Aspiration
Blah Blah Blah , Post Term Delivery o f a
W hen I say "C-Section, ” beautiful baby girl. What is the diagnosis?
You say Transient Tachypnea (A) RDS
(B) Transient Tachypnea
When I say “M aternal Sedation ’ (C) Meconium Aspiration
You say Transient Tachypnea

When I say “Premature ”


You say RD S

Solving Cases Using Lung Volumes:


High (flat diaphragms):
Low:
• Meconium Aspiration
•Surfactant Deficiency (no pleural effusion)
• Transient Tachypnea
•Beta-Hemolytic Pneumonia (gets pleural effusions)
• Non BH Neonatal Pneumonia

How this could work:

Blah Blah Blah , This Picture o f a beautiful baby


girl. What is the diagnosis?

(A) RDS
(B) Transient Tachypnea
(C) Meconium Aspiration

182
-Congenital Chest-

Pulmonary Hypoplasia: “Tiny Lung’

This is the situation where the lung(s) look tiny or incompletely developed. It can occur from primary
reasons, but the secondary causes seem to lend themselves more readily to multiple choice questions.

You can think about secondary causes as either:

(A) Things inside the chest pushing on the lungs during development (most commonly a congenital
diaphragmatic hernia / bowel in the chest, but sometimes from a neuroblastoma, giant congenital
heart, or sequestration)

(B) Things outside the chest pushing on the lungs during development Potter Sequence
(classically a fucked up skeleton / ribs - as you might see in a short
rib dwarf like Jeune syndrome “asphyxiating thoracic dystrophy,” but No Kidneys
sometimes a large intra-abdominal mass compressing the thorax)

(C) Things outside the chest fucking with normal lung development No Pee
(classically low fluid “oligohydramnios” ). Remember, the
developing baby has to breath in the amniotic fluid for normal lung
No Amniotic Fluid
development. Anything that drops fluid (premature rupture of
membranes. Potter sequence, renal problems) will mess up the lungs.
Hypoplastic Lungs
OB Ultrasound Buzzwords:
’Reduced Thoracic Circumference (TC)
’Fetal Lung: Head Ratio < 1

Congenital Diaphragmatic Hernia (CDHs):

Most commonly they are Bochdalek type. B is in the Back - they are typically posterior and to the
left. The appearance on CXR is usually pretty obvious.
LARGE Congenital
Trivia: Diaphragmatic Hernia
•Usually in the Back , and on the left (Bochdalek)
•If it’s on the right - there is an association with GBS Pneumonia Buncha Bowel in the
•Mortality Rate is related to the degree of pulmonary Hypoplasia Chest Smashing a Lung
•Most have Congenital Heart Disease
•Essentially all are malrotated Pulmonary Hypoplasia of
the Smashed Lung
Gamesmanship
•One trick is to show the NG tube curving into the chest.

Bronchogenic Cysts:

Typically an incidental finding. They are generally solitary and unilocular. They typically do NOT
communicate with the airway, so if they have gas in them you should worry about infection.

183
Bronchopulmonary Sequestration

The easiest way to think about these things is “extra lung that is NOT connected to the airways or
vasculature normally.” Some people will call them “accessory lung tissue.” Other people will call
them “bronchopulmonary foregut malformations” or BPFMs, but those people tend to be weirdos. The
kind of people who send out Christmas cards with the dog dressed as Santa.

Sequestrations Have:
- NO communication with the airway
- NO communication with the pulmonary arteries
- They USUALLY have an Aortic Feeder Vessel

These are grouped into intralobar and extralobar with the distinction being which has a pleural
covering (which you can NOT see on imaging). The venous drainage is different (intra to
pulmonary veins, extra to systemic veins). You can NOT tell the difference radiographically.
The practical difference is age and presentation.

Intralobar Extralobar

More Common (75%) Less Common (25%)

Presents in Adolescence Presents in Infancy

Respiratory Distress or Cyanosis


Recurrent Infections
(often related to associated congenital
(bacteria migrates in from pores o f Kohn)
anomalies)

Associated Congenital Anomalies


- ‘Extra’f or ‘Extra’things-
Rare to Have Associated (CCAMs, congenital diaphragmatic hernia,
Congenital Anomalies vertebral anomalies, congenital heart disease,
pulmonary hypoplasia)

Most Common (60%) in the Left Lower Lobe, 90% in the Left Lower Lobe
Posterior Segment. 40% in the RLL 10% Below the Diaphragm
Uncommon in the Upper Lobes. “subdiaphragmatic”

Has its own pleural cover


you can’t see that shit on CT or MR
No Pleural Cover
you can’t see that shit on CT or MR
*The pleural covering is protective against
infection, which is why they rarely get infected

Pulmonary Venous Drainage Systemic Venous Drainage

Gamesmanship: I say recurrent pneumonia in same area, you say intralobar sequestration.

184
Congenital Lobar Emphysema (CLE)

The idea behind this one is that you have bronchial pathology (maybe atresia depending on what
you read), that leads to a ball-valve anomaly and progressive air trapping. On CXR, it looks
like a lucent, hyper-expanded lobe.
Trivia:
• It’s not actually emphysema - just air trapping secondary to bronchial anomaly
• It prefers the left UPPER lobe (40%)
• Treatment is lobectomy

Gamesmanship: The classic way this is shown in case conference or case books is with a series
o f CXRs. The first one has an opacity in the lung (the affected lung clears fluid slower than
normal lung). The next x-ray will show the opacity resolved. The following x-ray will show it
getting more and more lucent. Until it’s actually pushing the heart over.

Note the mass


effect on the heart

Congenital Cystic Adenomatoid Malformation (CCAM)


- (Other Aliases Include - C P A M )

As the name suggests it’s a malformation o f adenomatoid stuff


that replaces normal lung. M ost o f the time it only involves one
lobe. There is no lobar preference (unlike CLE which favors the
left upper lobe). There are cystic and solid types (type 1 cystic
is the most common -70% ). There is a crop o f knuckle heads
who want to call these things CPAMs, which I ’m sure is
evidence based and will really make an impact in the way these
things are treated. As such, you should be ready for them to be
called CPAMs on the exam , don’t let that trip you up. CCAMs Schematic of a the most
communicate with the airway, and therefore at least components common “look” for a CCAM /
o f them can fill with air. M ost o f these things (like 90%>) will CPAM. They tend to have
large cysts, surrounded by
spontaneously decrease in size in the third trimester. The
smaller cysts.
treatment (at least in the USA) is to cut these things out, because
o f the iddy bitty theoretical risk o f malignant transformation
(pleuropulmonary blastoma, rhabdomyosarcoma).

Q: What i f you see a systemic arterial feed er (one coming o ff the aorta) going to the CC AM ?
A: Then it’s not a CCAM, it’s a Sequestration. — mumble to yourself “nice try assholes”

185
Left Upper Lobe:

Think Congenital Lobar


Emphysema (CLE) first

But, remember CCAI\/I has no


lobar prevalence, so it can be
anywhere

Left Lower Lobe:

Think Sequestration First


Congenital Diaphragmatic
Hernia (CDHs) favors this
side too

Case 1. Newborn with congenital heart disease

A. Intralobar Sequestration
B. Extralobar Sequestration
C. Congenital Lobar Em physema

Case 2. 10 year old with recurrent pneumonia

A. Intralobar Sequestration
B. Extralobar Sequestration
C. Congenital Lobar Em physema

**Intralobar is seen older kids,


**Extralobar is seen in infants with co-morbids
** CLE is in the upper lobe

186
-Primary Lung Tumors-
THIS vs THAT: Prim ary Lung Tumor vs Congenital M alformation: Normal 2nd
As a general rule, developmental / congenital lung lesions will be present on the Trimester US
mid-second trimester US. For the purpose of multiple choice, if the history tells +
you that the 2nd Trimester US was normal and they show you a mass - think Infant with
Lung Tumor
Primary Tumor. If instead they tell you (or show you) a lung mass on the second
trimester US - think Congenital Malformation (CPAM - etc..). There are Primary
exceptions to this rule - I’m not going to get into it and confuse the issue. This is a Lung Tumor
“general rule” for gamesmanship.

Pleuropulmonary Blastoma (PPB) - This is the most


common primary lung malignancy in children. They can look a PPB
lot like CCAMs and even have different types (cystic, mixed, Big Fucking Mass (B.F.M.)
solid) - but should be absent on the second trimester US. These in the chest of a 1-2 year old
things are usually right sided, pleural based, and without chest
Shouldn’t have an eaten-up
wall invasion or calcifications. No rib invasion (helps
rib (Askin tumors often do)
distinguish it from the Askin /E w ing Sarcoma o f the Chest Wall -
if they won't tell you the age), No calcification. The more solid 10% of the time they have a
types can have mets to the brain and bones. The cystic type seem multilocular cystic nephroma.
to occur more in kids less than a year old, and be more benign.

Inflammatory Myofibroblastic Tumor (IMT) - This is the most cormnon primary lung
mass in children. It is benign (PPBs are malignant). These are solid masses, typically lobulated and
often calcified. They have a lower lobe predominance. The tend to look like fluid on MR (T2 bright)
because they are composed of “myxoid” stroma. It can look like a sequestration (given the arterial
supply) or the solid types of PPB.

-Catheters/Lines-
Umbilical Venous Catheter (UVC) - AUVC passes from the
umbilical vein to the left portal vein to the ductus venosus to a hepatic vein
to the IVC. You don’t want the thing to lodge in the portal vein because
you can infarct the liver (or cause portal hypertension).
Things to know about UVCs:
• The ideal spot is at the IVC - Right Atrium junction.
• Clot forming in a portal vein branch can cause lobar atrophv
• Development of a "Cystic Liver Mass" (Hematoma) can suggest UVC
erosion into the liver.
Umbilical Artery Catheter (UAC) - AUAC passes from the
umbilicus, down to the umbilical artery, into an iliac artery than to the
aorta. Positioning counts, as the major risk factor is renal arterial
thrombosis. You want to avoid the renal arteries by going high (T8-T10),
or low (L3-L5)
Things to know about UACs:
• It goes down first
• It should be placed either high (T8-T10) or low (L3-L5)
• Should stay left of midline on AXR (aorta on the left)
• Omphalocele is a contraindication

187
ECHO - Extracorporeal Memdrane Oxygenation
Neonatologists prim arily use this device to torture sick babies - hopefully into revealing the
various government secrets they have stolen, or the location o f their organization’s
underground lair. “Enhanced interrogation” or “tem porizing m easure o f last resort,” they call
it - to get around the Geneva Conventions.

Oh you want to pretend yo u c a n ’t talk? Get the EC M O catheters!

Alternatively, it can be used as a last resort in neonatal sepsis, severe SSD, and meconium
aspiration. Actually, in cases o f m econium aspiration ECM O actually does work
(sometimes).

Types:

There are two main types; (1) Veno-Arterial “V-A”,


and (2) Veno-Venous “V-V”
V enoArterial Veno Venous
“V-A” “V_V”
In both cases deoxygenated blood is removed from the
right atrium and pumped into a box (artificial lung) to Catheter
get infused with oxygen. The difference is in how it is Position; Catheter
returned. RA + Aorta (near Position;
the origin of RA+RA
the innominate “Dual Lumen”
In V-A, blood goes back to the Aorta (you can see artery)
why this w ould help rest the left ventricle). The
Heart + Lung
catheter is usually placed at the origin o f the Lung Support
Support
innominate or “overlying the arch.”
Carotid artery
and Rigiit
In V-V blood goes right back into the right atrium. In
Jugular Vein are
this situation even if the lungs were totally clogged “Sacrificed” to
with shit (meconium aspiration) and no oxygen tile Volcano God
exchange was happening it w ouldn’t matter because in the hopes of a
plentiful harvest.
the blood that is being pum ped (RA -> RV ->
Pulmonary Artery) already has oxygen in it.

188
ECHO - Extracorporeal Membrane Oxygenation - Condnued
Things to Know:

• Lung W hite Out = N orm al. M echanism is variable depending on who you asked. The way
I understand it is that the airway pressure suddenly drops off causing atelectasis, plus you
have a change in the circulation pattern that now mim ics the fetal physiology (mom =
artificial lung). Resulting oxygen tension changes lead to an edem a like pattern. A multiple
choice trick could be to try and make you say it is worsening airspace disease, or reflects the
severity o f the lung injury. D on’t fall for that. It’s an expected finding.

• Consequences o f V-A. I mentioned on the prior page that they typically ligate the carotid
when they place the Arterial catheter. No surprise that they will be at increased risk for
neurologic ischem ic com plications as a result.

• Crucial Com plication = H em orrhage. The com bination o f anticoagulation (necessary in


ECM O) and being sick as stink puts these kids at super high risk for head bleeds. This is
why they will get screened for germ inal matrix hem orrhage prior to being placed on ECM O
and then routinely screened with head ultrasounds (expertly read by the second year resident
on call).

Catheter Position Gam esm anship:

I think there are two likely ways a multiple choice question could be structured related to
ECM O catheter position. The first would be to show you a series of daily radiographs with the
latest one dem onstrating m igration of one or both catheters. Thats the easy way to do it.

The sneaky way would be to show you the


catheter with the lucent distal end and the dot
marker on the tip. This would be particularly
evil as there are tons o f different catheter
brands and looks, but if I was going to try and
trick you, this would be the way that I w ould do
it.

In this case, the venous catheter looks falsely


high if judged by the lucent tip, but that round
metallic m arker (near the black arrow) shows
the tip in the RA.

The arterial catheter “overlies the region o f the


aortic arch,” which is normal.

189
-Special Situations in Peds Ciiests-

Viral - In all ages this is way more common than bacterial infection. Peribronchial edema is the
buzzword for the CXR finding. “Dirty” or “Busy” Hilum. You also end up with debris and mucus in
the airway which causes two things (1) hyperinflation and (2) subsegmental atelectasis. Respiratory
Syncytial Virus (RSV) - This will cause the typical non-specific viral pattern as well. However,
there is the classic testable predilection to cause a segmental or lobar atelectasis — particularly in the
right upper lobe.

Round Pneumonia - Kids get round pneumonia. They love to show


this, and try to trick you into thinking it’s a mass. Younger than 8 you are
thinking round pneumonia, round pneumonia, round pneumonia - with S.
Pneumonia being the culprit. The PhD trivia is that these occur because you
don’t have good collateral ventilation pathways. Round pneumonia is
usually solitary, and likes the posterior lower lobes. Take home message: No
CT to exclude cancer, just get a follow up x-ray.

Neonatal atypical peripheral atelectasis (NAPA) - It is best


to think about this as a cousin or uncle of Round Pneumonia (they are in the
same family). It is essentially the same thing except it is peripheral. The
classic look is a round, pleural based “mass” in the apex of the lung.
Similarly to the “round pneumonia” this is a transient finding and will
resolve as the primary process improves.

Lipoid Pneumonia - Classic history is a parent giving their newborn a teaspoonful of olive oil
daily to cultivate “a spirit o f bravado and manliness. ” Although this seems like a pretty solid plan, and
I can’t fault their intentions - it’s more likely to result in chronic fat aspiration. Hot Sauce is probably a
better option. Most people will tell you that bronchoalveolar lavage is considered the diagnostic
method of choice. CXR is nonspecific - it is just airspace opacities. CT is much more likely to be the
modality used on the exam. The classic finding is low attenuation (-30 to -100 HU) within the
consolidated areas reflecting fat content.

Bronchial Foreign Body:

The key concept is that it causes air trapping.


The lung may look more lucent (from air
trapping) on the affected side. You put the
affected side down and it will remain lucent
(from air trapping). Another random piece of
trivia is that under fluoro the mediastinum will
shift AWAY from the affected side on
expiration.

Key Point: A normal inspiratory CXR is meaningless. Don’t forget that the crayon / green bean is going
to be radiolucent. You need expiratory films to elicit air trapping. Normally, the bottom lung is gonna
turn white (move less air). If there is air trapping the bottom lung will stay black.

190
Swyer Janies - This is the classic unilateral lucent lung. It typically occurs after a viral lung
infection in childhood resulting in post infectious obliterative bronchiolitis. The size of the affected
lobe is smaller than a normal lobe (it’s not hyper-expanded).

Papillomatosis - Perinatal HPV can cause these soft tissue masses within the airway and lungs.
It’s also seen in adults who smoke. "Multiple lung nodules which demonstrate cavitation ” is the
classic scenario. Some testable trivia includes the 2% risk of squamous cell cancer, and that
manipulation can lead to dissemination. The appearance of cysts and nodules can look like LCH
(discussed more in the thoracic chapter), although the trachea is also involved.

Sickle Cell /Acute Chest - Kids with sickle cell can get “Acute chest.” Acute chest actually
occurs more in kids than adults (usually between age 2-4). This is the leading cause of death in sickle
cell patients. Some people think the pathology is as such: you infarct a rib -> that hurts a lot, so you
don’t breathe deep -> atelectasis and infection. Others think you get pulmonary microvascular
occlusion and infarction. Regardless, if you see opacities in the CXR of a kid with sickle cell, you
should think of this.

Gamesmanship (how do you know it is sickle cell?)


• Kid with Big Heart
• Kid with bone infarcts (look at the humeral heads)
• Kid with H shaped vertebra (look on lateral)
• Cholecysectomy Clips (look at right upper abdomen)

CF Related Trickery
Cystic Fibrosis- So the sodium pump doesn’t work and
they end up with thick secretions and poor pulmonary Fatty Replaced Pancreas on CT
clearance. The real damage is done by recurrent infections.
Abdominal Films with
Things to know: Constipation
• Bronchiectasis
(begins cylindrical and progresses to varicoid) Biliary Cirrhosis (from blockage
• It has an apical predominance of intrahepatic bile ducts), and
(lower lobes are less involved) resulting portal HTN
• Hyperinflation
• They get Pulmonary Arterial Hypertension
• Mucus plugging (finger in glove sign)
• Men are infertile (vas deferens is missing)

THIS vs THAT:
Primary Ciliary Dyskinesia - The motile part of
the cilia doesn’t work. They can’t clear their lungs and get Primary
recurrent infections. These guys have lots of bronchiectasis Cystic
Ciliary
just like CF. BUT, this time it’s lower lobe predominant Fibrosis
(CF was upper lobe). Dyskinesia

Upper Lobe Lower Lobe


Things to know: Predominant - Predominant -
• Bronchiectasis (lower lobes) Brochiectasis Bronchiectasis
• 50% will have Kartageners (situs inversus).
So, 50% will not Infertile - Men are Infertile - Men’s
• Men are infertile (sperm tails don’t work) Missing the Vas Sperm Don’t Swim
• Women are sub-fertile (cilia needed to push eggs around) Deferens For Shit

191
S E C T I O N 6:
MEDIASTINAL M ASSES

Anterior: That Asshole is Guilty!

Normal Thymus: This is the most common mediastinal Things that make you think the
“mass.” It’s terribly embarrassing to call a normal thymus a thymus is a cancer!
mass, but it can actually be tricky sometimes. It can be pretty • Abnormal Size fo r patients Age
big in kids less than 5 (especially in infants). Triangular (really big in a 15 year old)
shape o f the thymus is sometimes called the “sail sign.” Not • Heterogenous appearance
to be confused with the other 20 sail signs in various parts o f • Calcification
the body, or the spinnaker sail sign, which is when • Compression o f airway or
pneumomediastinum lifts up the thymus. vascular structure

Thymic Rebound: In times o f acute stress (pneumonia, radiation, chemotherapy, bums), the
thymus will shrink. In the recovery phase it will rebound back to normal, and sometimes larger
than before. During this rebound it can be PET avid.

Lymphoma: This is the most common abnormal mediastinal mass in children (older
children and teenagers). Lymphoma vs Thymus can be tricky. Thymus is more in kids under 10,
Lymphoma is seen more in kids over 10. When you get around age 10, you need to look for
cervical lymph nodes to make you think lymphoma. If you see calcification, and the lesion has
NOT been treated you may be dealing with a teratoma. Calcification is uncommon in an untreated
lymphoma.

Complications: Compression o f SVC, Compression o f Pulmonary Veins, Pericardial Effusion,


Airway Compression.

ALL / Leukemia - can appear very similar to Lymphoma (soft tissue mass in the anterior
mediastinum). In this scenario, most people will tell you that Lymphoma can NOT be
differentiated from Leukemia on imaging
alone.
I say, Extra Gonadal Germ Cell Tumor,
Germ Cell Tumor (GCT): On You Say Klinefelter’s Syndrome (47XXY)
imaging, this is a large anterior mediastinal
mass arising from or at least next to the Klinefelter patients have the worst syndrome
thymus. It comes in three main flavors, each ever. They have small penises, they get male
o f which has a few pieces o f trivia worth breast cancer, and as if things couldn’t possibly
knowing: get worse... they get germ cell tumors in their
chest. In fact, they are at 300x the risk of
1- Teratoma - Mostly Cystic, with fat and
getting a GCT. Pineal gland Germ Cells have
calcium
also been reported in Klinefelter patients, giving
2 - Seminoma - Bulky, solid and lobulated. them vertical gaze palsy. In that case, they can’t
“Straddles the midline” even look up to the sky and say “Why God* ?!
Why Me!? Why Klinefelter's!?”
3 - NSGCT - Big and Ugly - Hemorrhage
and Necrosis. Can get crazy and invade the **God, Allah, Mother Earth, Celestial Deity NOS
lung.

192
Middle
Lymphadenopathy - M iddle mediastinal lym phadenopathy is m ost often from
granulomatous disease (TB or Fungal), or from lymphoma.

M e d ia s tin a i A d e n o p a tiiy T riv ia :

Cause: EBV (-90%), CMV (-10%)


Classic Scenario: Adolescent with hilar adenopathy, splenomegaly,
Mononucleosis and fatigue.
Trivia: Rash after Antibiotics (Amoxicillin)
Trivia: “No sports for 3 weeks” - to avoid splenic injury

Distribution of nodes in usually unilateral, right hilar, right paratracheal


Primary TB
Lower and Middle lobe consolidation is common

Midwest and Southeast United States


Histoplasmosis Most have normal CXRs
Can have hilar adenopathy
Southwest United States
Coccidioidomycosis Usually looks like consolidation and nodules
Can have hilar adenopathy

Often cited as the “most common” anterior mediastinal mass in a kid.


Lymphoma
Hilar involvement (usually bilateral) is more common with Hodgkin

Uncommon in children — it usually presents in the early-mid 20s.


Sarcoid
Case reports of early onset Sarcoid exist.

Duplication Cysts - These fall into three categories (a) bronchogenic, (b) enteric,
(c) neuroenteric. The neuroenterics are traditionally posterior mediastinal.

• Bronchogenic - w ater attenuation - close to the trachea or bronchus. Trivia:

• Tend to be middle m ediastinal (70%), the rest are in the hilum


• Typically filled with m ucus or fluid

• Enteric I Esopiiageai - w ater attenuation close to the esophagus (low er in the


mediastinum). Trivia:

• Abutm ent o f the esophagus is the key finding.


• Can com m unicate w ith the lum en o f the esophagus - and have air/fluid levels.
• Usually on the right, involving the distal esophagus
• Can be m iddle or posterior mediastinal in location
• Second most com m on GI lumen duplication cyst (distal ileum #1)

193
Posterior
Neuroblastoma - This is the most common posterior mediastinal mass in a child under 2. This
is discussed in complete detail in the GU PEDs section. I’ll just mention that compared to abdominal
neuroblastoma, thoracic neuroblastoma has a better outcome. It may involve the ribs and vertebral
bodies (on X-rays look for rib splaying and erosions). Also, remember that Wilms usually mets (more
than neuroblastoma) to the lungs, so if it’s in the lungs don’t forget about Wilms. More on this later in
the chapter.

You can have other Neuroblastic tumors in this region as well. If Neuroblastoma is the most
undifferentiated and aggressive the spectrum looks something like this:

Most Mature Intermediate Undifferentiated


Ganglioneuroma Ganglioneuroblastoma Neuroblastoma

t When Compared with Neuroblastoma, Ganglioneuromas are:


• Less Aggressive
• More Circumscribed (less invasive)
• Less Likely to have Calcifications (although they still can)
• Found in Older Children (think 2nd decade) - vs.
neuroblastoma (think 1st decade)

These tumors can NOT be differentiated with imaging alone


(but that doesn’t mean someone can’t write a multiple choice

Ewing Sarcom a - This is discussed in com plete detail in the M SK PEDs section.

Askin Tumor (Primitive Neuroectoderm al tumor of the chest wall):


This is now considered part of the Ewing Sarcoma spectrum, and is sometimes called an Ewing
sarcoma of the chest wall. They tend to displace adjacent structures rather than invade early on (when
they get big they can invade). They look heterogenous, and the solid parts will enhance.

Neuroenteric Cyst - By convention these are associated with vertebral anomalies (think
scoliosis, hemivertebrae, butterfly vertebrae, split cord, etc..) - think cyst protruding out of an unsealed
canal / defect. The cyst does NOT communicate with CSF, is well demarcated, and is water density.
Favor the lower cervical and thoracic regions.

Extram edullary Hem atopoiesis - This occurs in patients with myeloproliferative


disorders or bone marrow infiltration (including sickle cell). Usually, this manifests as a big liver and
big spleen. However, in a minority of cases you can get soft tissue density around the spine
(paraspinal masses - usually below T6), which are bilateral, smooth, and sharply delineated.

194
strategy - The Anterior Mediastinal Mass
Lymphoma - In a kid ju st assume it’s Hodgkins (which m eans it’s gonna involve the
thymus). Why assum e H odgkins ? Hodgkins is 4x more com m on than NHL. Hodgkins
involves the thymus 90% o f the time.

Q: H ow the hell do yo u tell a big ass norm al thymus in a little baby vs a lymphoma?
A: M y main move is to go age. Under 10 = Thym us, Over 10 = Lymphoma.

Thym ic Rebound - If the test w riter is headed in this direction they M UST either (a)
bias you with a history saying stuff like “got off chem o” or “got off corticosteroids” or
(b) show you a series o f axial CTs with the thing growing and m aintaining normal
morphology. I think “a” is much more likely.

The Funk: In general ju st think m orphology / density:

• Soft Tissue - Kinda H om ogenous = Think Lym phom a or H yperplasia

• Fat = Germ Ceil Tumor {Why God!? Why K linefelters!?)

• Water = Congenital Stuff - Think Lym phangiom as

Strategy - Th e Posterior Strategy - Th e B.F.M.


Mediastinal Mass “Big Fucking M ass”

First Rule o f Peds M ultiple Choice Test


Taking - TIM ING! AGE!

Under 10 - Think m alignant,


Think neuroblastom a.

2nd D ecade - Think benign.

If it’s a round mass- Think about


Ganglioneurom as & Neurofibrom as
I f you see a B.F.M. in the chest o f a kid,
If it’s cystic (and there is scoliosis) think
you basically have tw o choices:
Neuroenteric Cyst

If they show you coarse bone (1) Askin Tumor (PNET / Ewings) -
trabeculation - with an adjacent mass (or *AGE 10+, look for an eaten up rib.
a history o f anem ia) - Think (2) Pleuropulm onary Blastom a
Extram edullary Hem atopoiesis *AGE is typically less than 2.

195
S E C T IO N 7:
Lum in al GI

Esophageal Atresia I
TE fistula: This can occur
in multiple subtypes, with the
classic ways of showing it
being a frontal CXR with an
NG tube stopped in the upper
neck, or a fluoro study (shown
lateral) with a blind ending sac
or communication with the Esophageal Atresia H-Type Atresia
N-Type Fistula (85%)
tracheal tree. NO Fistula (10%) (1%)
Delayed Diagnosis
There are 5 main subtypes, only 3 (shown above) are worth knowing (being familiar with) for the
purpose of the exam.

Things to Know About Esophageal Atresia / TE Fistula:


•Diagnosis is made with a Fluoro swallow exam
•Most Important Thing To Know are the VACTERL associations (more on this later)
• The most common subtype is the N-Type (blind ended esophagus, with distal esophagus hooked up
to trachea
• Excessive Air in the Stomach = H type (can also be with N type)
• No Air in the Stomach = Esophageal Atresia
• The presence of a right arch (4%) must be described prior to surgery (changes the approach).

Gamesmanship: Fake out for TE fistula is simple aspiration. Look for the presence or absence of
laryngeal penetration to tell them apart (if shown a dynamic Fluoro swallow exam).

VACTERL: This is extremely high yield. VACTERL is a way of remembering that certain
associations are seen more commonly when together (when you see one, look for the others).

They occur with different frequency: VACTERL association is diagnosed when 3


or more of the defined anomalies affect a
V- Vertebral Anomalies (37%) patient.
A- Anal (imperforate anus) (63%)
C- Cardiac (77%) Therefore, keep investigating when 1-2 of
TE - Tracheoesophageal Fistula these anomalies are found.
or Esophageal Atresia (40% )
R -R e n a l (72%) The heart and kidneys are the most
L - Limb (radial ray) - 58% commonly affected organs in this

Trivia: If both limbs are involved, then both kidneys tend to be involved. If one limb is involved, then
one kidney tends to be involved.

Stricture: Around 30% of kids with a repaired esophageal atresia will end up with a focal
anastomotic stricture. Strictures can also be seen with caustic ingestion (dishwashing soap) - but those
tend to be long segment. Reflux (if chronic and severe) is another possible cause.

196
Esophageal Foreign Bodies: Kids love to stick things in their mouths (noses and ears).
This can cause a lot o f problems including direct compression o f the airway, perforation, or even
fistula to the trachea. Stuff stuck in the esophagus needs to be removed.

TH IS vs THAT: Trachea vs Esophagus:

The esophagus is a dirty sock, it flexes to accommodate that big piece o f steak you didn’t even
bother to chew. The trachea is rigid, like that math teacher I had in high school (who hated
m usic... and colors), but unlike the math teacher it has a flexible membrane in the back.

The point o f me Esophagus


mentioning this is to
help you problem solve
a “where is the coin ”
type question. The
esophagus will
Frontal View
accommodate the coin
Coronal Coin =
so it can be turned in
any direction. The
trachea is rigid and will
force the coin to rotate
into the posterior
membrane — so it will
Trachea
be skinny in the AP Frontal View
direction. Turned Coin = Trachea

Additional trivia relates to swallow ed batteries, magnets, and pennies - chart on the follow ing
page.

197
Ingested Metallic Foreign Bodies

One nnagnet is ok. Two or more magnets is a


problem. Tlie reason is that tiiey can attract Surgical Consult
Magnets each other across intestinal walls leading to NOT an MRI (dumbass)
obstruction, necrosis, perforation, and a law suit.

Less of a problem relative to other types of Serial plain film exams.


batteries, but can cause serious problems if you Remove if they stay in
AA or AAA Batteries need them for the DVR control because Gam e of the stomach for more
Thrones season 7 is on (spoiler; it sucked). than 2 days.

Stuck in the esophagus


= big problem, gotta
They look like coins, except they get them out within 2
have two rings. The literature is
hours.
not clear, but it appears that
Disc Batteries
modern batteries rarely leak
Stuck in the stomach =
(leaking is bad - caustic
problem, gotta get
chemicals, heavy metals etc..).
them out within 4 days.

o
Remove if:
Copper Pennies are relatively
safe. Retention in the
Coins
(Including Pennies esophagus for more
M ake sure it is not a disc battery
minted prior to 1982) than 24 hours or
(coins have one order ring, disc
Stomach for more than
battery has two).
28 days.

Those minted after 1982 contain mostly Zinc


which when combined with stomach acid can
cause gastric ulcerations, and if absorbed in
great enough quantity can cause zinc toxicosis
(which is mainly pancreatic dysfunction /
pancreatitis). The ulcers are the more likely thing
Pennies to happen, so just remember that.
Remove from Stomach
-minted after 1982
So how the hell can you tell the date of a penny
that is swallowed? Either (a) the question stem
will have to say something like - “2 year old child
playing with father’s collection of 1984 pennies”,
or the more likely (b) showing you the penny with
characteristic radiolucent holes - from erosion.

Usually Lead Paint Dust -


Remove immediately
or any object with lead paint.
from stomach. Distal
Lead
passage must be
Bad because the gastric acid leads to
confirmed.
immediate absorption.

Esophagus = Remove Immediately


Stomach = Remove Immediately
Sharp Object NOS
Post Pylorus = Follow vs Surgery — if it does perforate the small bowel it
will be at the 1C valve.

198
Vascular Impressions
This is a very high yield topic for the purpose o f multiple choice exams. Like m y Grandma
always said, you never get a second chance to make a first vascular impression.

1 1 1 ]\ 1
1 1 1
. ■
1 1 1
.........

/
1 )\^ 11
*......... * ■

1

1—1 m 1 1
Innominate Artery
Right Arch with Aberrant
Left, or Left Arch with
P ulm on ary Sling Double Aortic Arch Compression Aberrant Right

Pulmonary Sling:
• The only variant that goes between the esophagus and the trachea.
• Classic question is that this is associated with tracheal stenosis (which isactually prim ary
and not secondary to compression).
• High association with other cardiopulm onary and systemic anomalies: hypoplastic right lung,
horseshoe lung, TE-fistula, im perforate anus, and com plete tracheal rings.
• Treatment is controversial but typically involves surgical repositioning o f the artery

Double A ortic Arch:


• M ost Comm on SYM PTOM ATIC vascular ring anom aly

Left Arch with Aberrant


Right Subclavian Artery
• M ost Comm on Aortic Arch Anomaly
— not necessarily symptomatic.
• “Dysphagia L usoria” - fancy Latin speak (therefore
high yield) for trouble swallowing in the setting o f this
variant anatomy
• “Diverticulum o f K om m erell” pouch like aneurysm al
dilatation o f the proxim al portion o f an aberrant right
subclavian artery

199
-Bowel Obstruction ( In the neonate)-
Bowel obstruction in the neonate can be thought of as either High Low
high or low. Here are causes you should keep in your mind
when you think the question stem is leading towards Midgut Volvulus / Hirschsprung
obstruction. Malrotation Disease
Meconium Plug
Why might you think the question is leading you toward Duodenal Atresia
Syndrome
obstruction? Anytime you are dealing with a neonate, and
the history mentions “vomiting,” “belly pain,” or “liasn’t Duodenal Web Ileal Atresia
passed a stool yet.”
Annular Pancreas Meconium Ileus
The following sections will walk through an algorithm,
starting with plain films for diagnosis (and sometimes Anal Atresia /
Jejunal Atresia
Colonic Atresia
management).

Su b S e c tio n 1: “B u b b le s ”

People who do peds radiology are obsessed with “bubbles” on baby grams. The idea is to develop a
pattern-based approach to bowel obstruction in the newborn.

My preferred “bubble method” favors 8 possible patterns. This is a method originally developed by
the brilliant (and devilishly handsome) Charles Maxfield at Duke.

m.
Single Bubble Double Bubble Triple Bubble

Single Bubble Double Bubble Diffusely Dilated Diffusely Mildly


-plus distal gas -plus distal gas Dilated

200
Sub Section 1: “Bubbles” - Continued

Single Bubble
= Gastric (antral or pyloric) atresia. Duodenal Atresia Trivia:
Double Bubble = Duodenal Atresia • 30% have Downs
(highly specific). Some authors will say • 40% have polyhydramnios and
that UGI is not necessary because of how are premature
highly specific this is. The degree of • The“single atresia” - cannulation
distention will be more pronounced than error
with midgut volvulus (which is a more • On multiple choice test the
acute process). Thought to be secondary “double bubble” can be shown on
to failure to canalize during development 3rd trimester OB ultrasound, plain
(often an isolated atresia) film, or on MRI.

a Triple Bubble = Jejunal Atresia. W hen you call jejunal atresia,


you often prom pt search for additional atresias (colonic). Just rem em ber
that jejunal atresia is secondary to a vascular insult during development.
• “M ultiple Atresia ” - vascular error.

Single Bubble with Distal G as = Can “Bilious ^


mean nothing (lotta air swallowing). If the clinical Vomiting” fiH v Upper
GI
history is bilious vom iting , this is om inous and can mm
be midgut volvulus (surgical emergency). N ext test
would be em ergent U pper GI.

Double Bubble with Distal G as = Seeing


distal gas excludes duodenal atresia. The DDx is a
duodenal web, duodenal stenosis, or m idgut
volvulus. N ext step w ould be upper GI.

Multiple Diffusely Dilated Loops =


Contrast
Suggestive o f a low obstruction (ileum or colon). Enema
Upper
N ext step is contrast enema. I f the contrast enem a is GI
normal you need to follow w ith upper GI (to if
normal
exclude an atypical look for m idgut volvulus).

Contrast
Mildly Dilated, Scattered Loops =
Enema
‘^Sick Belly” - Can be seen w ith proxim al or
Upper
distal obstruction. Will need U pper GI and contrast
GI
enema.

201
Sub Section 2: Upper Gl Patterns

Upper GI on kids is fair game in multiple choice tests, and real life. Often the answ er o f this
test can equal a trip to the OR for kids, so it’s no trivial endeavor.

N orm al M alrotation Corkscrew Duodenum Complete Duodenum


Obstruction

Malrotation - Normally, the developmental rotation of the gut


places the ligament of Trietz to the left of the spine (at the level of
the duodenal bulb). If mother nature fucks up and this doesn’t
happen, you end up with the duodenum to the right of the midline
(spine). These patients are at increased risk for midgut volvulus
and internal hernias. If you see the appearance of malrotation and
the clinical history is bilious vomiting, then you must suspect
midgut volvulus.

Trivia regarding Malrotation


• Associated with Heterotaxy Syndromes. Associated with
Omphaloceles.
• Classically shown as the SMA to the right of the SMV
(on US or CT).
• False Positive on UGI - Distal Bowel Obstruction
displacing the duodenum (because of ligamentous laxity). Classic Malrotation:
• Small bowel follow through shows all
the bowel on one side
• The duodenum does not cross the midline

G am esm anship: in an in fa n t-

I say “Non-Bilious Vomiting” — You Say Hypertrophic Pyloric Stenosis

Next Step ? ■ lj|||||pr Ultrasound

I say “Bilious Vomiting” — You Say Mid Gut Volvulus {till proven otherwise)

Next Step? Upper GI

202
Ladd’s Band
C orkscrew Duodenum - This is diagnostic of
midgut volvulus (surgical emergency). The appearance
is an Aunt Minnie.
Ladd’s Bands - In older children (or even adults)
obstruction in the setting o f m alrotation will present as
intermittent episodes o f spontaneous duodenal
obstruction. The cause is not m idgut volvulus (a
surgical emergency) but rather kinking from L add’s
Bands.
So what the hell is a “L a d d ’s B a n d ” ? We are talking Malrotati
about a fibrous stalk o f peritoneal tissues that fixes the
cecum to the abdom inal wall, and can obstruct the
duodenum.

Com plete Duodenal O bstruction - Strongly associated with m idgut volvulus. If

Ladd’s Procedure -

Procedure to prevent m idgut


volvulus. Traditionally, the
Ladd’s Bands are divided, and
the appendix is taken out. The
small bowel ends up on the
right, and the large bowel ends
up on the left. They are fixed
in place by adhesions (just by
opening the abdomen).

It is still possible to develop


volvulus post L add’s (but it’s Ladds Procedure:
• Divide the Adhesive Ladds Bands
rare - 2-5%).
• Widen the Mesentery to a safe distance
• Take out the appendix (bill extra for that)

Preduodenal Portal Vein


you were thinking duodenal atresia, look
for distal air (any will do) to exclude that An anatomic variant where the portal vein sits
thought. Plus, as discussed above, you anterior the 2nd part of the duodenum.
want to see a dilated duodenum (double Associated with duodenal obstruction
bubble) for duodenal atresia. 50"/o of the time. It isn’t the cause, it just
happens to be present with other things (Ladds
Partial Duodenal Obstruction:
Bands, Annular Pancreas, etc..).

203
If the kid is vom iting this m ight be from extrinsic
narrowing (Ladd band, annular pancreas), or intrinsic
(duodenal web, duodenal stenosis). You can’t tell.

Hypertrophic Pyloric Stenosis:


Thickening of the gastric pyloric musculature, which results
in progressive obstruction. Step 1 buzzword is “non-bilious
vomiting.” Here is the most likely multiple choice trick; this
does NOT occur at birth or after 3 months.

Specific age range of 2-12 weeks (peak at 3-6 weeks).

Criteria is 4mm and 14mm (4mm single wall, 14mm length).

The primary differential is pylorospasm (which will relax


during exam). The most common pitfall during the exam is
gastric over distention, which can lead to displacement of the
antrum and pylorus - leading to false negative.

False positive can result from off axis measurement.

The phenomenon of “paradoxical aciduria ” has been


described, and is a common buzzword.

Classic History: Infant with projectile vomiting


Caterpillar Sign: Multiple areas of peristalsis in the stomach.
Next Step = Ultrasound
Ultrasound Criteria: 4 mm (wall) and 14 mm (length)
Treatment: Tell the kid to stop being a pussy — if that
doesn’t work - surgery (pyloromyotomy) Caterpillar Sign:
Stomach with Areas of Peristalsis

Gastric Volvulus- This comes in two flavors; organoaxial and mesenteroaxial.

•Organoaxial - The greater curvature flips over the lesser curvature (rotation along the long axis).
This is seen in old ladies with paraesophageal hernias { ‘O ’fo r ‘O lder’).

•Mesenteroaxial - over the mesentery (rotation along short axis). The antrum flips near
the GE junction. Can cause ischemia and needs to be fixed. Additionally this type causes
obstruction. This type is more common in kids.

Clinically, Borchardt Triad:


1. Inability to pass NGT
2. Severe epigastric pain
3. Retching without vomiting
Duodenal Web: This is best thought o f as “almost duodenal atresia. ” The reason I say
that is, just like duodenal atresia, this occurs from a failure to canalize, but instead o f a total

204
failure o f canalization (like duodenal
atresia) this bowel is only partially
canalized, leaving behind a potentially
obstructive web.

Trivia to know:
- Because the web is distal to
am pulla o f Vater - you get bile-
Stomach
stained emesis
■ Associated with malrotation and
Downs syndrome
■ The “wind sock” deform ity is seen
■^
more in older kids - w here the w eb­
like diaphragm has gotten stretched.

W eb

Annular Pancreas: Essentially


an em bryologic screw up (failure o f
ventral bud to rotate with the
A Greater \
duodenum), that results in encasem ent Curvature Greater
Curvature
o f the duodenum.

In Kids = Think Duodenal Obstruction

In Adults = Think Pancreatitis

How it could be shown:

• On CT: Look for pancreatic tissue


(same enhancem ent as the nearby
normal pancreas) encircling the
descending duodenum.

• On Fluoro: Look for an extrinsic


narrow ing o f the duodenum. Obviously
this is non-specific (typical barium -
voodoo), use the location and clinical
j Pancreas Surrounding the Duodenum
history to bias yourself.

205
Sub Section 3: ‘‘Low Obstruction” in a Neonate

Just like the upper GI and “bubble” plain film in sections 1 & 2, the low er obstruction can
be approached with a pattern-based method. You basically have 4 choices; Normal,
Short M icrocolon, Long M icrocolon, and a Caliber Change from micro to normal.

Normal:
• This is what normal looks like:

Short M icrocoion -
• Think about Colonic Atresia

Long M icrocoion - This can be seen with meconium ileus or distal ileal atresia.

•Meconium iieus - ONLY in patients with CF.


The pathology is the result o f thick sticky meconium
causing obstruction o f the distal ileum. Contrast will
reach ileal loops, and dem onstrate multiple filling
defects (meconium). This can be treated with an enem a
-enema is both diagnostic a n d therapeutic.

■Distal ileal Atresia - This is the resuh o f


intrauterine vascular insult. C ontrast will NOT reach
ileal loops. This needs surgery.

206
Caliber Change - This can be seen with small left colon syndrome or Hirschsprungs

•Small Left Colon (Meconium Plug)


Syndrome - This is a transient functional colonic
obstruction, that is self limited and relieved by
contrast enema.

*M ost Testable Fact:


It is NOT associated with CF.

*2nd M ost Testable Fact:


It is seen in infants o f diabetic m others or
if m om received m agnesium sulfate for eclam psia

•Hirschsprung Disease - Failure o f the V


ganglion cells to migrate and innervate the distal
colon. Affected portions o f the colon are small
in caliber, whereas the norm ally innervated
colon appears dilated.

Trivia:
•It’s 4:1 more com mon in boys.
•10% association w ith Downs.
•Diagnosis is made by rectal biopsy.

How it can be Shown:


-Enema - Rectum sm aller than the Sigm oid
“R ecto-sigm oid ratio < 1”
-Enem a - R ectum with “sawtooth pattern”
Represents bowel spasm

Presentation:
(1) N ew born who fails to have BM > 48
hours (or classically > 72 hours)
(2) “Forceful passage o f m econium after
rectal exam ”
(3) One m onth old w ho shows up “sick as
stink” w ith N EC bowel

•Total Colonic Aganglionosis - This is a super rare variant o f H irschsprungs, and


can m im ic m icrocolon. The piece o f com m only asked trivia is that it can also involve the
term inal ileum.

207
Meconium Peritonitis:

This is a potential complication o f bowel atresia


or meconium ileus. It has a very characteristic
look. It’s a calcified mass in the mid
abdomen , traditionally shown on plain film. It
is the result o f a sterile peritoneal reaction to an
in-utero bowel perforation.

Usually, the perforation seals off prior to birth


and there is no leak.

imperforate or E ctop ic Anus:

This can range from simple m embranous anal atresia to an arrest o f the colon as it descends
through the puborectalis sling. The thing to know is fistula to genitourinary tract. Imperforate
anus is also associated with a tethered cord (probably need a screening ultrasound).

• / say “Baby with no asshole ” , yo u say “VACTERL ”


• 1 say “Baby with no a ssh o le” , yo u say “Screening US fo r tethered c o rd ”

Opinions are like Assholes


Everyone has One

Unless you have an Imperforate Anus


Then you probably have VACTERL and a Tethered Cord
and probably don’t have well formulated opinions... cuz you are a baby
A baby with no asshole

208
-Obstruction in an Oider Ciiiid-
“M y belly hurts ” questions in an older child.

This scenario should make you think o f 6 main things - Ciassic DDx “AIIV1” -
- the classic AA-II-MM or “AIM” differential - with Appendicitis, Adhesions
appropriate credit given to the brilliant and under
inguinal Hernia, intussusception
appreciated Aldrich Killian (A.I.M.s founder).
I iViidgut Volvulus, iVieckels

Appendicitis - In children older than 4 this is the most common cause for bowel obstruction. If they
show this in the PEDs section it’s most likely to be on ultrasound. In that case you can expect a blind-
ending tube, non-compressible, and bigger than 6 mm.

Inguinai Hernia: This is covered in more depth in the GI chapter. Big points are that indirect liernias
are more common in kids, they are lateral to the inferior epigastric, and incarceration is the most common
complication. Umbilical hernias are common in kids, but rarely incarcerate.
Trivia to know: This is the most common cause of obstruction in boy 1 month - 1 year.

Intussusception - The age range is 3 months - 3 years, before or after that you should think of lead
points (90% between 3 months and 3 years don’t have lead points). The normal mechanism is forward
peristalsis resulting in invagination of proximal bowel (the intussusceptum) into lumen of the distal bowel
(the intussuscipiens). They have to be bigger than 2.5 cm to matter (in most cases- these are enterocolic),
those that are less than 2.0 cm are usually small bowel-small bowel and may reduce spontaneously within
minutes. Just like an appendix, in the peds section, I would anticipate this shown on ultrasound as either
the target sign or pseudo-kidney.
There are 3 main ways to ask questions about this: (1) what is it ?-
these should be straight forward as targets or pseudo kidneys, (2) lead
points - stuff like HSP (vasculitis), Meckle diverticulum, enteric
duplication cysts, and (3) reduction trivia.

Reducing Intussusception Trivia:


• Contraindications: Free Air (check plain film). Peritonitis (based on
exam)
• Recurrence: Usually within 72 hours
• Success Rates - 80-90% with air (Henoch-Schonlein purpura has a
reduced success rate) Not Every Target is an Ileocolic.
• Risk of Perforation - 0.5% You want this this to be > 2.5 cm
• Air causes less peritonitis (spillage of fecal material) than barium
• Pressure should NOT exceed 120 mmHg
• Needle decompression would be the next step if they perforate and get tension pneumoperitoneum

IVIeckeis Diverticuium: This is a congenital diverticulum of the distal ileum. A piece of total trivia is
that it is a persistent piece of the omphalomesenteric duct. Step 1 style, “rule of 2s” occurs in 2% of the
population, has 2 types of heterotopic mucosa (gastric and pancreatic), located 2 feet from the IC valve, it’s
usually 2 inches long (and 2 cm in diameter), and usually has symptoms before the child is 2. If it has
gastric mucosa (the ones that bleed typically do) it will take up Tc-Pertechnetate just like the stomach
(hence the Meckel’s scan).

High Yield Trivia (Regarding Complications)


• Can get diverticulitis in the Meckels (mimic appendicitis)
• GI Bleed from Gastric Mucosa (causes 30% of symptomatic cases)
• Can be a lead point for intussusception (seen with inverted diverticulum)
• Can Cause Obstruction

209
-Luminal Gl - Special Topics-
Gastroschisis - Extra-abominal evisceration of neonatal bowel (sometimes stomach and liver)
through a paraumbilical wall defect.
Trivia to know:
•It does NOT have a surrounding membrane (omphalocele does)
•It’s always on the RIGHT side.
•Associated anomalies are rare (unlike omphalocele).
•Maternal Serum AFP will be elevated (higher than that of omphalocele)
•Outcome is usually good
•For some reason they get bad reflux after repair.
•Associated with intestinal atresias.
Omphalocele - Congenital midline defect, with herniation of gut at the base of the umbilical cord
Trivia to know:
• DOES have a surrounding membrane (gastroschisis does not)
Pentalogy of Cantrell
• Associated anomalies are common (unhke gastroschisis)
• Trisomy 18 is the most common associated Omphalocoele
chromosomal anomaly Ectopia Cordis
• Other associations: Cardiac (50%), Other GI, CNS, GU, (abnormal location of heart)
Diaphragmatic Defect
Turners, Klinefelters, Beckwith-Wiedemann,
Pericardial Defect
Pentalogy of Cantrell or Stemal Cleft
• Outcomes are not that good, because of associated syndromes. Cardiovascular malformations
• Umbilical Cord Cvsts (Allantoic Cvsts) are associated.

This vs That — Gastroschisis vs Omphalocele

Gastroschisis Omphalocele
Herniated bowel loop through the Hemiated bowel loop through the
ventral body wall ventral body wall

NOT surrounded by a membrane Surrounded by a membrane (peritoneum)

Umbilicus is Normal - positioned to the left of the Umbilicus contains hemiated bow el, and therefore is
defect (defect is on the right) NOT normal

Defect is on the right Defect is midline

High Association with Cardiac (50%) defects and


Associated anomalies are rare Chromosomal Abnormalities

Multiple Syndromic Associations including Turners,


Associated with intestinal atresias iGinefelters, Beckwith-Wiedemaim, and the
Legendary Pentalogy o f Cantrell

Cause: Probably environment - which explains an Cause: Probably Genetic - which explains the
association with bowel atresia associations with the various syndromes

Duodenal Hematoma - Classic injury from bicycle handlebars (or child abuse). You can also
see this as a complication from endoscopy. D3 is the most common location. Look for an ovoid mass
in the lumen/bowel wall or paraduodenal tissues. You could be shown retroperitoneal gas as a way to
suggest perforation.

210
Enteric Duplication Cysts - These are developmental anomalies (failure to canalize). They
don’t have to communicate with the GI lumen but can. They are most commonly in the ileal region
(40%). They have been known to cause in utero bowel obstruction / perforation.

Strategy: A common way to show this is a cyst in the abdomen (on ultrasound). If you have a random
cyst in the abdomen you need to ask yourself - “does this have gut signature? ”

• Cyst with Gut Signature = Enteric Duplication Cyst


• Cyst without Gut Signature = Omental Cyst
• WTF is “Gut Signature ?” - It’s alternating bands of hyper and hypo echoic signal - supposedly
representing different layers of bowel.

Trivia to know: 30% of the time they are associated with vertebral anomalies.

Distal Intestinal Obstruction Syndrome - This is a cause of bowel obstruction in an older


kid (20 year old) with cystic fibrosis. This is sometimes called the “meconium ileus equivalent, ”
because you end up with a distal obstruction (as the name implies) secondary to dried up thick stool. It
more commonly involves the ileum / right colon. Kids who get this, are the ones who aren’t compliant
with their pancreatic enzymes.

Mesenteric Adenitis - Self-limiting, usually viral inflammatory condition of mesenteric lymph


nodes. It is a classic clinical mimic of appendicitis. The finding is a cluster of large right lower
quadrant Ivmph nodes.

Necrotising Enterocolitis (NEC)

This is bad news. The general thinking is that you have an immature bowel mucosa (from being
premature or having a heart problem), and you get translocated bugs through this immature bowel. It’s
best thought of as a combination of ischemic and infective pathology.

Who gets it?


• Premature Kids (90% within the first 10 days of life)
• Low Birth Weight Kids ( < 1500 grams)
• Cardiac Patients (sometimes occult) - they can be full term
• Kids who had perinatal asphyxia
• Hirschsprung Kids that go home and come back - they present around month 1.

What does it look like?


• Pneumatosis - most definitive finding; Look for Portal Venous Gas Next
• Focal Dilated Bowel (especially in the right lower quadrant) - the terminal ileum / right colon is the
region most affected by NEC
• Featureless small bow el, with separation (suggesting edema).
• Unchanging bowel gas pattern - this would be a dirty trick - showing several plain films from
progressing days, with the bowel gas pattern remaining the same.

Pneumatosis vs Poop - The age old question.


• First question - has the kid been feed? No food = No poop.
• Second question - is it staying still? Poop will move, Pneumatosis willstay still.

Useless Trivia:
• Use of maternal breast milk is the only parameter associated with decreased incidence of NEC.

211
S E C T IO N 8:
S olid O rgan GI

Pirates Of the Pancreas-


CF - The pancreas is nearly always (90%) with CF patients.
Inspissated secretions cause proximal duct obstruction leading to the
two main changes in CF: (1) Fibrosis (decreased T1 and T2 signal) and
the more common one (2) fatty replacement (increased Tl).

Patients with CF diagnosed as adults tend to have more pancreas


problems than those diagnosed as children. Those with residual
pancreatic exocrine function can have bouts of recurrent acute
C om plete F a tty R eplacem ent
pancreatitis. Small (l-3mm) pancreatic cysts are common.

High Yield Trivia:


• Complete fatty replacement is the most common imaging finding in adult CF
• Enlarged with fatty replacement = lipomatous pseudohypertrophy of the pancreas.
• Fibrosing Colonopathy: Thick walled right colon as a complication of enzyme replacement therapy.

Shwachman*Diamond Syndrome - The 2nd most common cause of pancreatic


insufficiency in kids (CF #1). Basically, it’s a kid with diarrhea, short stature, and
U Ix eczema. Will also cause lipomatous pseudohypertrophy of the pancreas.

Dorsal Pancreatic Agenesis - You only have a ventral bud (the dorsal bud forgets
to form). Since the dorsal buds makes the tail, the appearance is that of a pancreas without a
tail. All you need to know is that (1) this sets you up for diabetes (most o f your beta cells are
in the tail), and (2) it’s associated with polysplenia.

Pancreatitis - The most common cause of pancreatitis in peds is trauma (seat belt).
NAT: Another critical point to make is that non-accidental trauma can present as pancreatitis.
Mk If the kid isn’t old enough to ride a bike (handle bar injury) or didn’t have a car wreck (seat
" ^ belt injury) you need to think NAT.

Tumors of the Pediatric Pancreas: Even at a large pediatric hospital its uncommon to see more
than 1-2 of these a year. Obviously, they are still fair game for multiple choice. This is what I would
know:
Solid and Papillary Epithelial Neoplasm (SPEN) - The most common pediatric solid
tumor. It’s found in female adolescents (usually asian, or black). The outcomes are pretty good after
surgical resection. If you get shown a case in the peds setting this is
probably it. I’ll mention this thing again in the Adult GI chapter. Peds Pancreatic Mass
Age 1 = Pancreatoblastoma
Age 6 = Adenocarcinoma
Otherwise, I would try and use age as a discriminator
Age 15 = SPEN
(if they are nice enough to give it to you).

212
-liver Masses-
Tumors: For Peds liver tum ors I like to use an age-based system to figure it out. M ass in the
liver, first think - w hat is the age? Then use the narrow DDx to figure it out.

Age0-3: W ith kids that are newborns you should think about 3 tumors:

Infantile H epatic Hemangioma:


Often < 1. Associated with high output CHF, this is
classically shown as a large heart on CX R plus a mass in
the liver. The aorta above the hepatic branches o f the
celiac is often enlarged relative to the aorta below the
celiac because o f differential flow. Skin hem angiom as
are present in 50%. Endothelial growth factor is
elevated. These can be associated with Kasabach-M erritt
Syndrome (the platelet eater).

H ow do they do? - A ctually well. They tend to spontaneously involute w ithout therapy over
months-years - as they progressively calcify.

Hepatoblastoma:
M ost com mon prim ary liver tum or o f childhood (< 5).
The big thing to know is that it’s associated with a bunch
o f syndromes - m ainly hem i-hypertrophy, Wilms,
Beckwith-Weidemarm crowd. Prem aturity is a risk fa c to r
This is usually a well circum scribed solitary right sided
mass, that may extend into the portal veins, hepatic veins,
and IVC. Calcifications are present 50% o f the time. AFP
is elevated. Another piece o f trivia is the hepatoblastom a
may cause a precocious puberty from m aking bHCG.

I w ould know 3 things: (1) Associated with W ilms, (2) AFP, (3) Precocious Puberty

M esenchym al Hamartoma:
This is the predom inately cystic mass (or m ultiple cysts), sometimes
called a “developm ental anomaly.” Because it’s a “developm ental
anom aly” it shouldn’t surprise you that the AFP is negative.
Calcifications are U N C O M M O N . W hat is com m on is a large portal
vein branch feeding the tumor.

213
A>g. >5:

HCC: This is actually the second m ost com mon liver cancer in k ids. You’ll see them in kids
with cirrhosis (biliary atresia, Fanconi syndrome, glycogen storage disease). AFP will be
elevated.

Fibrolam ellar Subtype: This is typically seen in younger patients (<35) without
cirrhosis and a normal AFP. The buzzword is central scar. The scar is sim ilar to the one seen
in FNH with a few differences. This scar does NOT enhance, and is T2 dark (the F N H scar is
T2 bright). As a point o f trivia, this tum or is Gallium avid. This tum or calcifies more often
than conventional HCC.

Undifferentiated Embryonal Sarcom a: This is the pissed o ff cousin o f the


mesenchymal ham artoma. It’s also cystic, but the mass is much more aggressive. It will be a
hypodense mass with septations and fibrous pseudocapsule. This mass has been know n to
rupture.

AnyAge:

Mets: Think about Wilms tum or or N euroblastom a

Now, there are several other entities that can occur in the liver o f yo u n g children / teenagers
including; H epatic Adenoma, Hemangiomas, Focal Nodular Hyperplasia, a n d Angio
Sarcoma. The bulk o f these are discussed in greater detail in the adult G I chapter

Fetal Hepatomegaly
I’m gonna cover the 2 scenarios I think are probably the most testable.

Scenario 1: Mom is a “free spirit.” She was also on that daytime TV show where they
don’t know who the Dad is ... the paternity test show. Dad could be one o f these 17 lucky
guys... and the test comes back and it is none o f them. It was some other asshole that she can’t
even remember. She was on that show. The prenatal US (after 20 weeks) shows a big liver and
big placenta. This is classic congenital syphilis. The liver is the earliest organ involved and
the last to resolve after treatment.

Scenario 2: Mom is a not a free spirit and she wasn’t on that show embarrassing her family,
b u t.... She is 43. The kid has a big liver on 3rd trimester US. This is classic Transient
Abnormal Myelopoiesis (TAM). “TAM” is a preleukemic syndrome seen only in Downs
(trisomy 21). Anytime you are given advanced maternal age (35+) in the question - you should
be thinking Downs. Fetal Hepatomegaly + Downs = TAM. Most o f the time (80%) it will get
better on its own. The other 20% become myeloid leukemia o f Down syndrome - hence the
“preleukemic.”

214
-Congenital Biliary / liver-
Choledochal cysts are congenital dilations o f the bile ducts -classified into 5 types by
some dude nam ed Todani. The high yield trivia is type 1 is focal dilation o f the CBD and is by
far the most com m on. Type 2 and 3 are super rare. Type 2 is basically a diverticulum o f the
bile duct. Type 3 is a “choledochocele.” Type 4 is both intra and extra hepatic. Type 5 is
Caroli’s, and is intrahepatic only. I ’ll hit this again in the GI chapter.

Caroli’s is an AR disease associated with polycystic kidney disease and m edullary sponge
kidney. The hallmark is intrahepatic duct dilation, that is large and saccular. Buzzword is
“central dot sign” which corresponds to the portal vein surrounded by dilated bile ducts.

AR Polycystic Kidney Disease: This will be discussed in greater detail in the renal
section, but kids with A R polycystic kidney disease will have cysts in the kidneys, and
variable degrees o f fibrosis in the liver. The degree o f fibrosis is actually the opposite o f
cystic formation in the kidneys (bad kidneys ok liver, ok kidneys bad liver).

Hereditary Hem orriiagic Telangiectasia (Osler-Weber-Rendu):


Autosomal dominant disorder characterized by m ultiple AVMs in the liver and lungs. It leads
to cirrhosis, and a massively dilated hepatic artery. The lung AVM s set yo u up fo r brain
abscess.

Biliary Atresia: If you have prolonged new born jaundice (> 2 weeks) you should think
about two things (1) neonatal hepatitis, and (2) B ihary Atresia. It’s critical to get this
diagnosis right because they need corrective surgery (Kasai Procedure) prior to 3 months.
Patients with biliary atresia really only have atresia o f the ducts outside the liver (absence o f
extrahepatic ducts), in fact they have proliferation o f the intrahepatic ducts. They will
develop cirrhosis w ithout treatm ent and not do well.

Trivia to Know about Biliary Atresia:


• Associations with Polysplenia, and Trisom y 18
• G allbladder m ay be absent (normal gallbladder —supports neonatal hepatitis)
• Triangle Cord Sign - triangular echogenic structure by the portal vein - possibly rem nant o f
the CBD.
• Hepatobiliary Scintigraphy with 99m Tc-IDA is the test o f choice to distinguish (discussed
in the Nukes Chapter).
• Alagille Syndrome: This is a total zebra. A ll you need to know is hereditary cholestasis,
from paucity o f intrahepatic bile ducts, and peripheral pulm onary stenosis. The purpose o f a
liver biopsy in biliary atresia is to exclude this diagnosis.

Gallstones: If you see a peds patient w ith gallstones think sickle cell.

215
-Spleen-
Sickle Cell - These kids bodies have spleen better days - as the spleen will typically
enlarge progressively and then eventually auto-infarct and shrink (during the first decade). If
the spleen remains enlarged it can run into problem s - mainly acute splenic sequestration
crisis.

I want you to think about sickle cell spleen as either too big or too small (for the purpose o f
multiple choice). First I ’ll ex-spleen the too big problems:

Splenic Sequestration - This is the second m ost com mon cause o f death in SC patients
younger than 10. We are talking about the situation in which the spleen becom es a greedy
little pig and tries to hog all the blood for itse lf

Gamesmanship: History o f abdom inal pain or vital signs suggesting low volum e (high HR,
low BP) with a big spleen. Rem em ber m ost kids with sickle cell will have sm aller spleens
(auto infarct) so a big spleen should be your clue.

O ther problem s you can run into if your spleen stays big are abscess form ation and large
infarcts. These large infarcts are not the same pathophysiology as the “auto-infarct” you
typically think o f with sickle cell. These are the big wedge shaped infarcts (hypo-perfusion on
CT). As a point o f trivia, infarcted splenic tissue should look hypoechoic on US, w ith linear
"bright bands ” — google that if you haven’t seen it before.

N ow the too small problem:

Auto Infarcted Spleen - This is different than the massive infarct in that it is typically the
combined effort o f numerous tiny, unnoticeable, and repetitive micro occlusions leading to
progressive atrophy. Supposedly this doesn’t hurt (large infarcts do). This tends to occur
early and is usually “complete" by age 8. The typical look is going to be a tiny (possibly
calcified) spleen. W hen I say tiny - we are talking like 1cm. In the im aginary w orld o f
multiple choice you m ight not even see the fucking thing.

''Where is the fu c kin g spleen ? ” = Auto Infarct = Sickle Cell

Gamesmanship: I f you don't see the spleen but you do see a gallbladder full o f stones in a kid
less than 15 - you should think Sickle Cell.

216
Polysplenia and Asplenia - H eterotaxia syndrom es are clutch for multiple choice
tests. The m ajor game played on w ritten tests is “left side V5 right side. ”

So what the hell does that mean? Let m e break this dow n like a cardboard box that you intend
to put in your recycling ... because this gets pretty fucking com plicated. I like to start in the
lungs. The right side has two fissures (m ajor and minor). The left side has ju st one fissure.
So if I show you a CXR with two fissures on each side, (a left sided m inor fissure), then the
patient has two right sides. Thus the term “bilateral right sidedness.”

What else is a right sided Heterotaxia Syndromes


structure? The liver. So,
these patients w on’t have a Right Sided Left Sided
spleen (the spleen is a left Two Fissures in Left Lung One Fissure in Right Lung
sided structure).
A splenia Polysplenia
The opposite is true. Since Increased Cardiac
the spleen is on the left, a Less Cardiac M alformations
M alforations
“bilateral left sided” patient Azygos Continuation
will have polysplenia. R eversed A orta/IVC
o f the IVC

Aorta/IVC: This relationship is a little m ore confiising w hen you try to reason it out. The
way I keep it straight is by rem em bering that the IVC is usually on the right. If you are
“bilateral left” then you d o n ’t have a regular IVC — hence the azygos continuation. Then I
just rem em ber that the other one (flipped IV C/A orta) is the other one.

Normal Reversed (Aorta/ ICV) Azygos Continuation


IVC (on the right) -Asplenia -Polysplenia
Aorta (on the left)

Right Isomerism Left Isomerism

217
Heterotaxia Syndrom es I Situs - Cont...

So on the prior page I used terms like “bilateral right sided” and “bilateral left sided” to help
make the concept more digestible. I didn’t make those words up. You will read that some
places but I think to be ready for the full gauntlet o f heterotaxia related fuckery you should
also be ready for words that start with “situs ” and end with "isomerism. ”

Situs Vocab: There are 3 vocab words that start w ith the w ord “Situs.”

• Situs Solitus - Instead o f ju st saying norm al, you can be an asshole and say “Situs Solitus. ”
• Situs Inversus Totalis - Total m irror image transposition o f the abdom inal and thoracic
stuff
• Situs Am biguus (Ambiguous) - This is a tricky w ay o f saying Heterotaxy. o f which you can
have left or right “isom erism .”

“ISOm©risiTl” - 1 guess some asshole really liked organic chem istry.... This is a fancy
way o f saying bilateral right or bilateral left - as explained on the prior page.

Minor Situs
Situs Solitus
Fissure Inversus
(Normal)
Gastric
Gastric Bubble
Bubble on
on Left
Right
Larger part of
Larger part of
Liver on Riglit
Liver on Left
Minor fissure on
Minor fissure
Right
on Left
Inverted
S p le e n Bronchial
'GB' Pattern
Situs Solitus Situs Inversus Associated
“How Pretentious Assholes “Mirror Image” with Primary
Ciliary
Say Norm al”
Dyskinesia

Minor Minor
Fissure, Fissure V.H. with T
Left
Isomerism
Absent Minor
Fissures
Interrupted
IVC
Polysplenia
Biliary
Atresia
Absent Spleen ( 10 %)

Visceral Heterotaxy with Visceral Heterotaxy with


Thoracic Right Isomerism Thoracic Left Isomerism

218
- Top 4 Things for PedsGI-
Biliary Atresia:
• Congenital liver fibrosis cholangiopathy
neonatal jaundice (after 1 week of life)
• US: Bright band of tissue (triangular cord
sign) near branching of common bile duct;
small or absent gallbladder (fasting ~ 3
hours)
) Absent GB • Scintigraphic: No tracer excretion into bowel
by 24 hours
• Biopsy to exclude = Zebra Alagille syndrome
• Treated with Kasai procedure

IVIal rotation:
• Duodenum to the right of the midline
• Increased risk for mid gut volvulus, and
internal hernia
• “Bilious vomiting”
• SMA to the right of the SMV

SMV - 2 o’clock position relative to SMA

Heterotaxia Syndromes
Right Sided Left Sided
Two Fissures in Left One Fissure in Right
Asplenia or Lung Lung
Polysplenia Asplenia Polysplenia
Increased Cardiac Less Cardiac
M alforations M alformations
Azygos Continuation
Reversed Aorta/IVC
o f the IVC

V- V ertebral Anom alies (37%)


A- Anal (im perforate anus) (63%)
C - C ardiac (77%)
TE - Tracheoesophageal Fistula
V A C T ER L or Esophageal A tresia ( 40%>)
R - R e n a l (72%)
L - Limb (radial ray) - 58%

219
S E C T IO N 9:
CONGENITAL GU

Renal Agenesis - This comes in one of two flavors; (1) Both Kidneys Absent - this one is gonna
be Potter sequence related, (2) One Kidney Absent - this one is gonna have reproductive associations.

Most likely way to test this: Show unilateral agenesis on prenatal US - as an absent renal artery (in
view of the aorta) or oligohydramnios - with followup questions on associations. Or make it super
obvious with a CT / MRI and ask association questions.

Unilateral Absence Association


• 70% of women with unilateral renal agenesis have associated genital anomalies (usually
unicornuate uterus, or a rudimentary horn).
• 20% of men are missing the epididymis, and vas deferens on the same side they are missing the
kidney. PLUS they have a seminal vesicle cyst on that side.

Potter Sequence: Insult (maybe ACE inhibitors) = kidneys don’t form, if kidneys don’t form you can’t
make piss, if you can’t make piss you can’t develop lungs (pulmonary hypoplasia).

Lying Down Adrenal or "Pancake Adrenal” Sign - describes the elongated appearance of the adrenal
not normally molded by the adjacent kidney. It can be used to differentiate surgical absent vs
congenitally absent.

Horseshoe Kidney - This is the most common fusion anomaly. The kidney gets hung up on the
IMA. Questions are most likely to revolve around the comphcations / risks:

• Complications from Position - Easy to get smashed against vertebral body - kid shouldn’t play
football or wrestle.
• Complications from Drainage Problems: Stones, Infection, and Increased risk of Cancer (from
chronic inflammation) - big ones are Wilms, TCC, and the Zebra Renal Carcinoid.
• Association Syndrome Trivia - Turner’s Syndrome is the classic testable association.

Crossed Fused Renal Ectopia - One kidney


comes across the midline and fuses with the other.
Each kidney has its own orthotopic ureteral orifice to
drain through. It’s critically important to the patient to
know that “the Ectopic Kidney is Inferior. ” The left
kidney more commonly crosses over to the right
fusing to the normal right kidney lower pole.
Complications include stones, infection, and
hydronephrosis (50%).
The classic way to show this is two axial CTs. The
first at the level of the kidneys hinting that one kidney
may be absent. The second through the bladder (on a
delayed phase) showing two opacified ureters.

Trivia: More common in Males


Trivia: Associated with VACTERL

220
Congenital UPJ Obstruction

This is the most common congenital anomaly of the GU tract in neonates. About 20% of the time,
these are bilateral. Most (80%) of these are thought to be caused by intrinsic defects in the circular
muscle bundle of the renal pelvis. Treatment is a pyeloplasty. A Radiologist can actually add value by
looking for vessels crossing the UPJ prior to pyeloplasty, as this changes the management.

Q: 1970 called and they want to know how to tell the difference between a prominent extrarenalpelvis
vs a congenital UPJ obstruction.
A: “Whitaker Test”, which is a urodynamics study combined with an antegradepyelogram.

Classic History: Teenager with flank pain after drinking “lots of fluids.”
Classic Trivia: These do NOT have dilated ureters (NO HYDROURETER).

Autosomal Recessive Polycystic Kidney


Disease (ARPKD) - These guys get HTN and renal
failure. The liver involvement is different than the adult
form (ADPKD). Instead of cysts they have abnormal
bile ducts and fibrosis. Thus congenital hepatic fibrosis
is ALWAYS present in ARPKD. The ratio of liver and
kidney disease is inverse. The worse the liver is the
better the kidneys do. The better the liver is the worse
the kidneys are. Death is often from portal hypertension.

On ultrasound the kidneys are smoothly enlarged and


ARPKD; Big Bright, with Lost
diffusely echogenic, with a loss of corticomedullary
Corticomedullary Differentiation.
differentiation. In utero you sometimes will not see
urine in the bladder. Cysts tend to be tubular and spare
the cortex.

Neonatal Renal Vein Thrombosis - This is an associated condition of maternal diabetes,


sepsis, and dehydration. It is typically unilateral (usually left). The theory is that it starts peripherally
and progresses toward the hilum. When acute, will cause renal enlargement. When chronic, will
result in renal atrophy.

Neonatal Renal Artery Thrombosis - This occurs secondary to umbilical artery


catheters. Unlike renal vein thrombosis it does NOT present with renal enlargement but instead severe
hvpertension.

Prune Belly (Eagle Barrett Syndrome)


This is a malformation triad which occurs in males. Classically
shown on a babygram with a kid shaped like a pear (big wide
belly).

Triad:
• Deficiency o f abdominal musculature
• Hydroureteronephrosis
• Cryptorchidism
(bladder distention interferes with descent o f testes)

221
-Congenital Ureter and Urethra-
Congenital (primary) iVIEGAureter-This is a “w astebasket” term for an enlarged
ureter which is intrinsic to the ureter (NOT the resuh o f a distal obstruction). Causes include
(1) distal adynamic segment (analogous to achalasia, or colonic Hirschsprungs), (2) reflux at
the UVJ, (3) it ju st wants to be big (totally idiopathic). The distal adynamic type “obstructing
primary m egaureter,” can have some hydro, but generally speaking an absence o f dilation o f
the collecting system helps distinguish this from an actual obstruction.

Retrocaval Ureter (circumcaval)- This is actually a


problem with the developm ent o f the IVC, which grows in a
manner that pins the ureter. M ost o f the time it’s asymptomatic,
but can cause partial obstruction and recurrent UTI. IVP will
show a “reverse J” or “fishhook” appearance o f the ureter.

Retrocaval Ureter

Duplicated System - The main thing to know


Upper Pole
about duplicated systems is the so-called “W eigert-
(obstructed /
M eyer R u le” w here the upper pole inserts inferior and dilated) Inserts
medially. The upper pole is prone to ureterocele Inferior and
Medial
formation and obstruction. The lower pole is prone to
reflux. Kidneys with duplicated systems tend to be
larger than normal kidneys. In girls, a duplicated
Lower Pole
system can lead to incontinence (ureter may insert inserts Superior
below the sphincter - sometimes into the vagina). and Lateral

• Upper Pole Obstructs


• Lower Pole Refluxes

Ureterocele - A cystic dilation o f the intravesicular


ureter, secondary to obstruction at the ureteral orifice. IVP
(or US) will show the “cobra h e a d ” sign, with contrast
surrounded by a lucent rim, protruding from the contrast
filled bladder. This is associated with a duplicated system
(specifically the upper pole).
Cobra Head

Ectop ic Ureter - The ureter inserts distal to the external sphincter in the vestibule. M ore
common in females and associated with incontinence (not associated with incontinence in
men). Ureteroceles are best dem onstrated during the early filling phase o f the VCUG.

222
Posterior Urethral Valves:

This is a fold in the posterior urethra that leads to


outflow obstruction and eventual renal failure (if it’s
not fixed). It is the m ost com mon cause o f urethral
obstruction in male infants.

Trivia: The fold is a Wolfian Duct tissue rem nant

Now, this can be shown a variety o f ways;

Classic VCUG: The key finding on VCUG is an


abrupt caliber change between the dilated posterior
urethra and norm al caliber anterior urethra. PUV on VCUG

Fetal M Rl: The M RI w ould have to show hydro in


the kidney and a “key-hole” bladder appearance.

Pre-Natal U ltrasound Classic Triad:


• Hydronephrosis
• Bladder Dilation
• Oligiohydramnios

“Peri-renal fluid collection” is a buzzword, and it’s


t
the result o f fom iceal rupture. O bviously that is
non-specific and can be seen w ith any obstructive pathology.

Strategy for ^^Next Step ” types o f Questions : VCUG to evaluate


anatomy. This is the gold
standard.
Persistent
Adjunctive M AG 3 to
evaluate function and
Hydronephrosis on Repeat
drainage (obstructive vs
Routine Prenatal Ultrasound
not obstructive)
Screening Once Born

\ [
M R Urography offers
Goes Away function and structure - but
* most cases requires sedation in little
kids

Non-Obstructive Causes of Obstructive Causes of


Hydro in Baby Boys Hydro in Baby Boys
-Vesicoureteral Reflux (VUR) -PU V
-Primary Megaureter -U PJ Obstruction
-Prune Belly = Zebra -Ureteral Ectopia

223
This is sometimes described as a
“short submucosal distal ureteral segment"
Vesicoureteral Reflux (VUR) -
Normally, the ureter enters the bladder at an
oblique angle so that a “valve” is developed.
If the angle o f insertion is abnormal
(horizontal) reflux can develop. This can
occur in the asymptomatic child, but is seen
in 50% o f children with UTIs. The
recom mendations for when the boy/girl with ..A-
a UTI should get a VCUG to evaluate for
VUR is in flux (not likely to be tested). M ost
o f the time VUR resolves by age 5-6. Normal Reflux
-Intravesicular -Intravesicular
ureter is oblique ureter is horizontal
Trivia: Hydronephrosis is the most common
cause o f a palpable renal “mass" in childhood.

There is a grading system fo r VUR which goes 1-5.


One is reflux halfw ay up the ureter,
• Two is reflux into a non-dilated collecting system, (calyces still pointy),
Three you have dilation o f the collecting system, and calyces get blunted
Four the system gets m ildly tortuous,
Five the system is very tortuous.

A sneaky trick w ould be to show the echogenic m ound near the UVJ, that results from
injection o f “deflux”, which is a treatm ent urologist try. Essentially, they make a bubble with
this proprietary com pound in the soft tissues near the UVJ and it creates a valve (sorta).
Anyway, they show it in a lot o f case books and textbooks so ju st like a m idget using a urinal ■
rem em ber to stay on your toes.

Additional Pearl: Chronic reflux can lead to scarring. This s c a rrin g can resu lt in
hypertensio n and/or chronic renal failure.

Additional Pearl: I f the reflux appears to be associated w ith a “hutch” diverticulum - people
will use the vocabulary “Secondary” V UR rather than Prim ary VUR. The treatm ent in this
case will be surgical. Ureteroceles, Posterior Valves, N eurogenic Bladder - are all causes o f
Secondary VUR.

“Hutch” Diverticula
• Occur at or adjacent (usually ju st above) the UVJ.
• Caused by congenital m uscular defect
• Difficult to see on US — better seen with VCUG.
• They are “dynam ic” and best seen on the voiding (m icturition) phase
• If associated with V UR will often be surgically resected

224
-Congenital Bladder-
The Urachus: The umbilical attachment to the bladder (started out being called the allantois, then
called the Urachus). This usually atrophies into the umbilical ligament. Persistent canalization can
occur along a spectrum (patent, sinus, diverticulum, cyst).

• Trivia: Most common complication of


urachal remnant = infection

• Trivia: Urachal anomalies are twice as common in boys


(relative to girls)

• Trivia: The most important piece of trivia is that when


these guys get cancer, it’s adenocarcinoma (90% of
cases). To hint at this multiple choice test writers will
often use the phrase “midline bladder structure”
Urachal Cancer
Urachal Spectrum:

Normal Obliterated Patent Urachus Vesicourachal Urachal Cyst


Urachus (only the Diverticulum Urachal Sinus
ligament remains)

Bladder Exstrophy - This is a herniation of the urinary


bladder through a hole in the anterior infra-umbilical
abdominal wall.
What do you need to know?
• Increased incidence of Malignancy in the extruded bladder
• It’s Adenocarcinoma - just like a Urachal Remnant gets
• Aunt Minnie “Manta Ray Sign” - with
unfused pubic bones. This looks like Bladder Exstrophy
a monster wide pubic symphysis on an
AP pelvic radiograph. Academics name “signs” after animals to get
them published. All part of an elaborate
scene to make associate professor...

Cloacal Malformation - GU and GI both drain into a common opening (like a bird).
This only happens in females.

Neurogenic Bladder - 1 will discuss this more in the adult Urinary chapter, but for kids I want you
to think about spinal dysraphism (tethered cord, sacral agenesis, and all the other flicked up spine stuff).

225
S E C T IO N 10:
SOLID O rgan GU

W hen it comes to solid renal masses - an age-based strategy is the ticket.


Why an age-based strategy? If they give you the age o f the patient, you can use this chart to
eliminate distractors which don’t fit. If you are lucky, you w on’t even have to look at the
picture.

Neonate Around Age 4 Teenager

N ephroblastomatosi s Wilms (95% o f cases before age 10) RCC

Mesoblastic Nephroma Wilms Variants Lymphoma

Lymphoma

Multi locular Cystic Nephroma

Tumor / Mass Rapid R eview Trivia

Mesoblastic
“Solid Tumor o f Infancy” (you can be bom with it)
Nephroma

“Nephrogenic Rests” - left over embryologic crap that didn’t go away


Might turn into Wilms (bilateral wilms especially)
Nephroblastomatosis
“Next Step” - f'u ultrasound till 7-8 years old
Variable appearance

90% + Renal Tumors (95% o f cases before age 10)


“Solid Tumor o f Childhood” - Never born with it
Wilms
Grows like a solid ball (will invade rather than incase)
Met to the lung (most common)

Clear Cell - Wilms Met to Bone

Brain Tumors
Rhabdoid - Wilms
It fucks you up, it takes the money (it believes in nothing Lcbowski)

Micheal Jackson Tumor (Young Boys, Middle Age Women)


Multi-Cystic Big cysts that don’t communicate
Nephroma Septal Enhancement
Can’t Tell it is not Cystic Wilms (next step = resection)

“Solid Tumor o f Adolescent” - most common in 2nd decade


Renal Cell Translocation Subtype = Prior chemotherapy
Carcinoma (RCC) Medullary Subtype = Sickle Cell Trait,
Clear Cell Subtype = VHL (these RCCs are usually bilateral)

Non-Hodgkin
Renal Lymphoma
Multifocal

226
-SolidAge0-3

Nephroblastom atosis - These are


persistent nephrogenic rests beyond 36
weeks. It’s sorta normal (found in 1% o f
infants). But, it can be a precursor to Wilms
so you follow it. W hen W ilms is bilateral,
99% o f the time it had nephroblastom atosis
first. It goes away on its ow n (normally).
It should N O T have necrosis - this makes
you think Wilms. It has a variable
appearance, and is often described as
“hom ogeneous.” A lthough more commonly Nephroblastomatosis
a focal homogeneous ball, the way it’s -Hypodense Rind -
always shown in case conferences and case
books is as a hypodense rind.

Ultrasound screening q 3 months till age 7-8 is the usual routine - to make sure it doesn’t go
Wilms on you.

M esoblastic Nephroma - ""Solid renal tum or o f infancy." This is a fetal ham artoma,
and generally benign. It is the most com mon neonatal renal tum or (80% diagnosed in the first
m onth on life). Often involves the renal sinus. Antenatal ultrasound may have shown
polyhydramnios.

Pearl: If it really looks like a W ilms, but they are ju st too young (< 1 year) then call it
mesoblastic nephroma.

-CysticAge0-3

M ulticystic Dysplastic Kidney - You have m ultiple tiny cysts forming in utereo.
W hat you need to know is (1) that there is “no functioning renal tissue,” (2) contralateral renal
tract abnormalities occur like 50% o f the time (m ost com m only UPJ obstruction).

M CD K vs B a d Hydro?
• In hydronephrosis, the cystic spaces are seen to com municate.
• In difficult cases renal scinfigraphy can be useful. M CD K w ill show no excretory function.

Pearl: M CDK has M ACR Oscopic cysts that do NOT com m unicate

227
Wilms
“Solid renal tum or o f childhood.” This is by far the m ost com m on solid renal tum or o f
childhood. This is N OT seen in a newborn. Repeat, you can NOT be bom w ith this tumor.
The average age is around 3, and nearly all (95%) are found before age 10. It typically
spreads via direct invasion.

Associated Syndromes:

Overgrowth I Say Beckwith-


• Beckwith-Wiedemann - M acroglossia (m ost com mon Wiedemann

finding), Omphalocele, Hemihypertrophy, Cardiac, Big Organs. You Say,


• Sotos - M acrocephaly, Retarded (CNS stuff), Ugly Face •Wilms,
•Omphalocele,
•Hepatoblastoma
Non-Overgrowth
• W AGR - Wilms, Aniridia, Genital, Growth Retardation
• Drash - Wilms, Pseudohermaphroditism, Progressive Glomerulonephritis

Wilms Nevers
Wilms in a 1 year old ?
• N E V E R Biopsy suspected Wilms Think about associated
(you can seed the tract and up the stage) syndromes. Wilms loves to
pal around with:
• Wilms N E V E R occurs before 2 months o f age
• Hemihypertrophy,
(Neuroblastom a can)
• Hypospadias,
• Cryptorchidism

Bilateral Wilms

About 5-10% will have bilateral disease { “Synchronous Bilateral W ilms”)

Wilms Variants (look ju s t like Wilms)


• Clear Cell - likes to go to bones (lytic)
• Rhabdoid - “Terrible Prognosis” - A ssociated with aggressive Rhabdoid brain tumors

228
-CysticAroundAgek ^

Multilocular Cystic Nephroma


“Non-communicating, fluid-filled locules,
surrounded by thick fibrous capsule.” By
definition these things are characterized by the
absence of a solid component or necrosis.

Buzzword is
“protrudes into the renal pelvis.’

There is a classic bimodal occurrence


(4 year old boys, and 40 year old women).
Multilocular Cystic Nephroma
/ like to think o f this as the Michael Jackson lesion
“Protruding into the renal pelvis
- it loves young boys and middle-aged women

Turbo Nerds Are Spoiling M y Fun

One of my favorite jokes has been this “Michael Jackson lesion” to help remember the
age distribution. Unfortunately - a bunch of Academic Nerds got together and decided
that the pediatric cystic nephromas are their own thing now, and the adult ones are
their own thing - with the words "adult cystic nephromas” and "pediatric cystic
t nephroma” as the preferred nomenclature. Assholes.... ruined a great joke.
Gamesmanship: Board exams usually lag a few years behind these kinds of changes. So if you get
a question asking about the age distribution, you may want to still go with the bimodal occurrence -
especially if they don’t say “adult" or “peds” - just make sure there isn’t another more correct
answer. This is where reading all the choices is critically important. Mind reading is also helpful.
Don’t forget to read the mind of the person who wrote the question - so you can understand his or
her bias.

-SolidsinTeenager ^ I’ll discuss RCCs more in the GU Chapter

RCC - This is the second most common renal “Translocation Carcinoma RCC”
malignancy of childhood (Wilms #1) and is the most
Most common RCC sub-type in Kids
common ages 15-19. RCC in kids is different than
Denser than cortex on non-con CT
RCC in adults (a little bit). It is still an enhancing
Solid & Enhancing + Expansile Growth
solid mass - BUT the #1 subtype in kids is
Historv of prior cvtotoxic chemotherapy
“translocation carcinoma.” Clear Cell RCC (#1 in
is classic.
adults is actually rare in kids). If the kid has clear cell
thev should be screened for VHL (Von Hippel Lindau).

RCC vs Wilms: RCC tends to be smaller, occur in an older cohort,


and is more likely to calcify.

Lymphoma - This is never primary. We are talking about


hematogenous spread or direct invasion of another lymphoma. The
most common look is multiple bilateral round expansile masses. _

229
- Bladder Masses / Cancers -
Work up for bladder lesions / masses in kids should start w ith an Ultrasound. A potentially
testable point is that the bladder m ust be full during the exam ination - otherwise you can miss
lesions in the folds o f a thick collapsed bladder wall.

There is a differential for bladder masses in kids, and like basically everywhere else in the body a
very long list o f possible culprits. B u t.... for the purpose o f m ultiple choice, I ’m going to focus
on one entity as it is by far the m ost common.

Rhabdom yosarcom a -

This is by far the m ost com mon m alignant tum or o f the


bladder in kids. It is a cancer o f “m uscle” so you can find
these almost anywhere in the body but ~ 20% o f them occur
in either the prostate or bladder.

There are a bunch o f subtypes with the m ost testable being


the “botryoid.” This is the one that looks like a bunch o f
grapes (supposedly).

Otherwise - it usually ju st looks like a big nodular ugly Schematic Diagram of an


Ultrasound of the Bladder
horrible nightmare inducing mass in the bladder o f a kid showing multiple round masses
with a history o f urinary obstruction. grouped together. Maybe they
look like a bunch of grapes... I
Trivia: guess it depends on which grocery
store you shop at.
The most common location is actually the head/neck
- specifically the orbit and nasopharynx.
A pproxim ately 20% are in the bladder or prostate
A pproxim ately 20%> have m etastatic disease at presentation / diagnosis
There is a bim odal peak (2-4, then 15-17).

You can also have these things in the “paratesticular” region.

Paratesticular Rhabdomyosarcoma:

By far the m ost com mon extra-testicular mass in young m en and the only one really worth
mentioning. If you see a mass in the scrotum that is not for sure in the testicle this is it (unless
the history is kick to the balls from a spiteful young lady -then you are dealing w ith a big fucking
hematoma). If it’s truly a mass - this is the answer.

230
S E C T IO N 11:
A drenal

Neuroblastoma - First thing - make sure it isn’t a Renal Mass - these are frequently
contrasted with Wilms. I guaran-fucking-tee some asshole tries to trick you into calling a
Wilms a Neuroblastoma or vice-versa. Spend time mastering the ability to tell them apart.

Neuroblastomas are the most common extra-cranial solid childhood malignancy. They
typically occur in very young kids (you can be bom with this). 95% o f cases occur before age
10. They occur in the abdomen more than the thorax (adrenal 35%, retroperitoneum 30%,
posterior mediastinum 20%, neck 5%).

Staging: Things that up the stage include crossing STAGE 4 S - HIGH YIELD
the midline, and contralateral positive nodes. These
things make it Stage 3. • Less than 1 year old
• Distal Mets are Confined to Skin,
Better Prognosis Seen with - Liver, and Bone Marrow
— Diagnosis in Age < 1, • Excellent Prognosis.
— Thoracic Primary, Stage 4S.
**A common distractor is to say 4S
Associations: goes to cortical bone. This is false!
• NF-1, Hirschsprungs, DiGeorge, It’s the marrow.
Beckwith Wiedemann
• Most are sporadic

Random Trivia:
• Opsomyoclonus (dancing eyes, dancing feet) - paraneoplastic syndrome associated with
neuroblastoma.
• “Raccoon Eyes” is a common way for orbital neuroblastoma mets to present
• MIBG is superior to Conventional Bone Scan for Neuroblastoma Bone Mets
• Neuroblastoma bone mets are on the “lucent metaphyseal band DDx”
• Sclerotic Bone mets are UNCOMMON
• Urine Catecholamines are always (95%) elevated

Neuroblastoma Wilms

Age: Usually around age 4


Age: usually less than 2 (can occur in utero)
(never before 2 m onths)

Calcifies 90% Calcifies Rarely (<10%>)

Encases Vessels (doesn’t invade) Invades Vessels (doesn’t encase)

Poorly M arginated Well Circum scribed

D oesn’t usually m et to bones (unless clear cell


Mets to Bones
W ilms variant). Prefers lung.

231
Neonatal Adrenal Hemorrhage THIS vs THAT: Hemorrhage VS Neuroblastoma:
- This can occur in the setting of
birth trauma or stress. Ultrasound can usually tell the difference (adrenal
hemorrhage is anechoic and avascular, neuroblastoma is
Trivia: Neonatal adrenal hemorrhage echogenic and hyper-vascular).
is associated with scrotal hemorrhage
MRI could also be done to problem solve if necessary
Trivia: Adrenal hemorrhage is the (Adrenal Hemorrhage low T2 , Neuroblastoma high T2).
most common cause of an adrenal
“mass” in a neonate
Trivia: 70% of adrenal hemorrhage cases occur on the right (due to the compression of the right
adrenal gland between the liver and right kidney)
Trivia: If the hemorrhage occurs on the left - think about renal vein thrombus (left adrenal vein drains
into the renal vein — where as the right one dumps into the IVC). **Question writers are lazy.
Anatomy trivia like this makes for easy questions.
Trivia: Renal vein thrombosis (and overall adrenal hemorrhage) is more common in the infants of
diabetic mothers.

Gamesmanship - “Next Step” Adrenal mass of a neonate.

Neonates that are sick enough to be in the hospital hemorrhage their fucking adrenals all the time. An
adrenal hemorrhage can look just like a mass on ultrasound. Yes, technically it should be anechoic and
avascular - but maybe your tech sucks, or maybe you don’t get shown a picture they just tell you it’s a
mass. The question writer is most likely going to try and trick you into worrying about a
neuroblastoma (which is also going to be a mass in the adrenal).

Next Step? Sticking a needle in it, sedating the kid for MRI, or exposing him/her to the radiation of
CT, PET, or MIBG are all going to go against the “image gently” propaganda being pushed at
academic institutions. Plus it’s unnecessary. As is true with most things in radiology, they either get
better or they don’t. Hemorrhage is going to resolve. The cancer is not.

So the first step is going to be followup ultrasound imaging.

With follow up imaging you should see something like this (depending on the interval);
- Time 0: Adrenal hemorrhage is a well-defined ~ inhomogeneous isoechoic mass, no doppler flow
- Time 1; Liquefaction starts - mass becomes more complex, strands and bands - classically starts to
get a hypo echoic / cystic center area.
- Time 2: Size continues to decrease
- Time 3: Calcifications may occur (peripheral or curvilinear) -as early as 1-2 weeks
- Time 4: Adrenal gland has a normal sizeand shape — calcifications persist.

Gamesmanship - History of “adrenal insufficiency” ?


Does that help you in the setting o f a newborn adrenal mass?

Nope. Most cases are actually caused by a 21 alpha hydroxylase deficiency (congenital adrenal
hyperplasia). Those tend to look different than hemorrhage or a mass- they are more “cerebriform.”
The problem is you can acquire adrenal insufficiency from neoplastic destruction (neuroblastoma) or
regular good old fashioned hemorrhage.

More on Adrenals in the Endocrine Chapter.

232
S E C T IO N 12:
R epro d uctive

Hydrom etrocolpos -

Essentially the vagina w on’t drain the uterus. This condition


is characterized on imaging by an expanded fluid-filled
vaginal cavity with associated distention o f the uterus.

You can see it presenting in infancy as a mass, or as a


teenager with delayed menarche.

Causes include imperforate hvmen (most common'), vaginal


stenosis, lower vaginal atresia, and cervical stenosis.

For multiple choice trivia think about this as a “midline pelvic


mass” , which can cause hydronephrosis (mass effect from Bladder
distended uterus).

Trivia: Associated with Uterus Didelphys Blood-


(which often -7 5 % has a transverse vaginal septum) Filled
Vagina

Ovary- A complete discussion o f ovarian masses is found in


the GYN chapter. I ’ll briefly cover some PEDs specific
ovarian issues. Peds Ovarian Torsion
Torsion: In an adult, ovarian torsion is almost always due It doesn’t need a mass - but it
to a mass. In a child, torsion can occur with a normal ovary, is usually big (3x the size of
the contralateral / normal
secondary to excessive mobility o f the ovary. As described
one.)
in the GYN chapter you are going to see an enlarged
(swollen) ovary, with peripheral follicles, with or without If there is a mass - it is
arterial flow. W hat is “enlarged” ? Unlike an adult you probably a mature cystic
can’t really use a fixed number to call the ovary enlarged teratoma or large follicular
(ovarian volumes in the peds setting are notoriously cyst. It is almost never a
cancer.
variable).
The solution is to compare the ovary in question to the
contralateral size. Suspect torsion if the ovarv is at least 3
times the size o f the opposite “normal” ovary.
Fluid-Debris Levels within the displaced follicles is Normal xorsion - Big (3x the size)
another described adolescent ovarian torsion finding. with fluid-debris levels in
displaced follicles

Masses: About two-thirds o f ovarian neoplasms are benign dermoids/teratomas (discussed in


detail in the GYN chapter). The other one third are cancer — usually germ cells (75%). Mural
nodules and thick septations should clue you in that these m ight be cancer. Peritoneal implants,
ascites, and lymphadenopathy, are all bad signs and would over-ride any benign characteristics
of the mass.

233
Random Scrotum Trivia
Hydrocele: Collection o f serous fluid and is the most common cause o f painless scrotal
swelling. Congenital hydroceles result from a patent processus vaginalis that permits entry o f
peritoneal fluid into the scrotal sac.

Complicated Hydrocele (one with septations): This is either a hematocele vs pyocele. The
distinction is clinical.

Varicocele: Most o f these are idiopathic and found in adolescents and young adults. They are
more frequent on the left. They are uncommon on the right, and if isolated (not bilateral) should
stir suspicion for abdominal pathology (nutcracker syndrome, RCC, retroperitoneal fibrosis).

‘‘Next Step ” - Isolated right-sided varicocele = Abdomen CT

Exclude extrinsic mass, renal vein thrombus, or


portal hypertension causing a splenorenal shunt

HSP: This vasculitis is the most common cause o f idiopathic scrotal edema (more on this in the
vascular chapter).

Acute i*aiu in or around the Scrotum


The top three considerations in a child with acute scrotal pain are (1) torsion of the testicular
appendage, (2) testicular torsion, and (3) epididymo-orchitis.

Epididymitis - The epididymal head is the most common part involved. Increased size and
hyperemia are your ultrasound findings. This occurs in two peaks: under 2 and over 6. You can
have infection o f the epididymis alone or infection o f the epididymis and testicle (isolated orchitis
is rare).

Orchitis - Nearly always occurs as a progressed epididymitis. When isolated the answer is
mumps.

Torsion of the Testicular Appendages - This is the most common cause o f acute
scrotal pain in age 7-14. The testicular appendage is some vestigial remnant o f a mesonephric
duct. Typical history is a sudden onset o f pain, with a Blue Dot Sign on physical exam (looks like
a blue dot). Enlargement o f the testicular appendage to greater than 5 mm is considered by some
as the best indicator o f torsion

Torsion of the Testicle - Results from the testis and spermatic cord twisting within the
serosal space leading to ischemia. The testable trivia is that it is caused by a failure o f the tunica
vaginalis and testis to connect or a “Bell Clapper Deformity”. This deformity is usually bilateral,
so if you twist one they will often orchiopexy the other one. If it was 1950 you’d call in your
nuclear medicine tech for scintigraphy. Now you just get a Doppler ultrasound. Findings will be
absent or asymmetrically decreased flow, asymmetric enlargement, and slightly decreased
echogenicity o f the involved ball.

234
Testicular & Extra-Tesdcular Masses
Testicular Masses can be thought of as intratesticular or
extratesticular. With regard to intratesticular masses, Rhabdomyosarcoma
ultrasound can show you that there is indeed a mass but History: Painless, afebrile,
there are no imaging features that really help you tell which scrotal swelling - age 5.
one is which.
Look: Heterogenous vascular
If the mass is extratesticular, the most likely diagnosis is extra-testicular mass
an embryonal rhabdomyosarcoma from the spermatic
cord or epididymis. ----- ^

Testicular Mircolithiasis - This appears as multiple


small echogenic foci within the testes. Testicular microlithiasis
is usually an incidental finding in scrotal US examinations
performed for unrelated reasons. It might have a relationship
with Germ Cell Tumors (controversiaP. Follow-up in 6 months,
then yearly is probably the recommendation (maybe - it’s very
controversial, and therefore unlikely to be asked).

Testicular Cancer: The two Germ Cell Tumors seen in the first decade of life are
The histologic breakdown: the Yolk Sac Tumor, & the Teratoma.

Germ Cell (90%) Yolk Sac Tumor: Teratoma:


• Heterogeneous • Pure testicular teratomas are only
Seminoma (40%)
Testicular Mass in seen in young kids < 2 (mixed ones
^ - seen more in
< 2 year old tend to be seen around age 25).
the 4th decade • Unlike ovarian teratoma, these guys
• AFP is usually
^ Non- Seminoma (60%) often have aggressive biological
elevated. behavior.
-Teratoma
-Yolk Sac
-Mixed Germ Cell Choriocarcinoma:
• Seen more in the 2nd decade. Bleeding Mets
(Choriocarcinoma)
• Aggressive, highly vascular tumor b-HCG is usually
• High Mortality elevated

Non Germ Cell (10%)


Sertoli Cell Tumors:
• Usually bilateral A subtype is associated with
• “Bumed-out” tumors Peutz-Jeghers syndrome. If
-► Sertoli they show you the Peutz-
(dense echogenic foci that
represent calcified scars). Jegher lips and bilateral
-► Leydig scrotal masses, this is the
answer.

Testicular Lymphoma - THIS vs THAT:


• Immunosuppressed patients are at increased risk for developing Testicular
extranodal/ testicular lymphoma (lymphoma can “hide” in the Calcifications
testes because of the blood testes barrier).
• US: normal homogeneous echogenic tissue is replaced focally or Tiny (micro) = Seminoma
diffusely with hypoechoic vascular lymphomatous tissue.
• Buzzword = multiple hypoechoic masses of the testicle. BIG = Germ Cell Tumor

235
Not Really Reproductive But It Is Pelvic
Sacrococcygeal Teratom a:

This is the most common tumor o f the fetus or infant. These soUd and/or cystic masses are
typically large and found either on prenatal imaging or birth. Their largeness is a problem and
can cause mass effect on the GI system, hip dislocation, and even nerve compression leading to
incontinence.

They are usually benign (80%), although those presenting


in older infants tend to have a higher malignant potential.

The location o f the mass is either:


• external to the pelvis (47%),
• internal to the pelvis (9%), or
• dumbelled both inside and outside (34%).
There is another classification that discusses involvement
o f the abdomen.
The easiest way to remember it is like this:
-Type 1 - Totally extra pelvic
-Type 2 - Barely pelvic, but not abdominal
-Type 3 - Some abdominal
-Tvpe 4 - Totally inside abdomen
** this one has the highest rate o f malignancy.

Trivia: They have to cut the coccyx off during resection.


Incomplete resection o f the coccyx is associated with a
high recurrence rate.

236
S E C T IO N 13:
MSK

Fracture: In general, little kids bend they don’t break.


You end up with lots o f buckles and greensticks. For
problem solving you can get a repeat in 7-10 days as
periosteal reaction is expected in 7-10 days. Kids tend to
heal completely, often with no sign o f prior fracture.

Involvement o f the Physis: The major concern is growth


arrest, probably best asked by showing a physeal bar
(“early” bony bridge crossing the growth plate). You can Physeal Bar
get bars from prior infection, but a history o f trauma is
gonna be the more classic way to ask it.

Salter-Harris Classification lends


itself well to multiple choice: D iaphysis *
M etaphysis
Type 1 : S-Slipped
P h y sis

Complete physeal fracture, with or without


displacement.
Normal
Type 2: A —Above (or “Away from the Joint”)

Fracture involves the metaphysis. This is the


most common type (75%).

Type 3: L - Lower
(3 is the backwards “E ”for Epiphysis)

Fracture involves the epiphysis. These guys have


a chance o f growth arrest, and will often require
surgery to maintain alignment

Type 4: T - Through

Fracture involves the metaphysis and epiphysis.


These guys don’t do as well, often end up with
growth arrest, or focal fusion. They require
anatomic reduction and often surgery.

Type 5: R - Ruined

Compression o f the growth plate. It occurs from axial loading injuries, and has a very poor
prognosis. These are easy to miss, and often found when looking back at comparisons (hopefully
ones your partner read). The buzzword is “bony bridge across physis”.

237
Toddler’s Fracture: Oblique fracture o f the m idshaft o f the tibia seen in a child just
starting to w alk (new stress on bone). If it’s a spiral type you probably should query non­
accidental trauma. The typical age is 9 months - 3 years.

Stress Fracture in Children: This is an injury which occurs after repetitive trauma,
usually after new activity (walking). The m ost com mon site o f fracture is the tibia - proximal
posterior cortex. The tibial fracture is the so-called “toddler fracture” described above. Other
classic stress fractures include the calcaneal fracture - seen after the child has had a cast
removed and returns to norm al activity.

- The Elbow -
M y God... thesepeds elbows.

Every first year resident knows that elevation o f the fat pad (sail sign) should make you think
joint effusion and possible occult fracture. D on’t forget that sometimes you can see a thin
anterior pad, but you should never see the posterior pad (posterior is positive). I like to bias
m yself with statistics when I ’m hunting for the peds elbow fracture. The most common
fracture is going to be a supracondylar fracture (>60% ), followed by lateral condyle (20%),
and medial epicondyle (10%).

Radiocapitellar Line: This is a line through the center o f the radius, which should intersect the
middle o f the capitellum on every view (regardless o f position). If the radius is dislocated it
will NOT pass through the center o f the capitellum

Anterior H um eral Line: This time you


need a true lateral. A line along the
anterior surface o f humerus, should pass
through the middle third o f the
capitellum. With a supracondylar
fracture (the m ost com m on peds elbow Anterior Humeral Line
fracture) you’ll see this line pass through
Radiocapitellar Line
the anterior third.

f
I
I

238
1st 2nd

Ossification Centers are a D\


source of tricicery.

Remember they occur in a set order


(CRITOE),
I
• Capitellum (Age 1),
• Radius (Age 3),
• Internal (medial epicondyle Age 5), 3rd 4th & 5th
• Trochlea (Age 7),
• 01ecranon(Age 9), and
• External (lateral epicondyle Age 11).

-Elbow Tricks: Zig Zag Search Pattern Hunting For the Next Center

Lateral Condyle Fx: This is the second most common distal humerus fracture in kids. The
thing to know is a fracture that passes through the capitello-trochlear groove is unstable. Ortho makes
a big fucking deal about looking for displacement of these things - stressing the need for internal
rotation oblique views to exclude displacement. Displacement = Surgery
Most fractures are related to hyper-extension from
Common Elbow Uncommon Elbow
falls on out-stretched hands. If the question stem tells
you about a fall onto the elbow point or a hyper­ Fractures Fractures
flexion fracture think ulnar nerve injury: lost Lateral Condylar Lateral Epicondyle
sensation in the pinky & 1/2 of the ring fingers and/or
Flexer Carpi Ulnaris or Flexor Digitorum Profiindus - Medial Epicondyle Medial Condyle
denervation.

Trochlea - can have multiple ossification centers, so it can have a fragmented appearance.

Medial Epicondyle Avulsion (Little League EWiovi) - The medial epicondyle is the last
growth plate to fuse - a sneaky trick is to try and get you to call a normal ossification center a fracture.
There are two additional tricks with this one. (1) Because it’s an extra-articular structure, its
avulsions will not necessarily result in a joint effusion. (2) It can get interposed between the
articular surface of the humerus and olecranon. Avulsed fragments can get stuck in the joint, even
when there is no dislocation — but when there is a dislocation, also look for the fragment on post
reduction films.
Anytime you see a dislocation - ask yourself The importance o f I T (crIToe) -
• Is the patient 5 years old ? And if so • You should never see the trochlea and not
• Where is the medial epicondyle ? see the internal (medial epicondyle), if you
do it’s probably a displaced fragment
Nursemaids Elbow: When a child’s arm is pulled on, the radial head may sublux into the
annular ligament. X-rays typically don’t help, unless you supinate the arm during lateral position
(which often relocates the arm).

Get a Dedicated Elbow X-Rav Series


Classic Next Step Elbow Case: (To look for that radial head dislocation).
This is the classic Monteggia fracture-
dislocation pattern - common in kids
(rare in adults).
Obvious Ulnar Shaft Fracture +
Obvious Ulnar Shaft Fracture on forearm x-ray Subtle Radial Head Dislocation
239
llliac Crest
- Avulsion Iniuries: Abdominal
IVluscles
Kids tendons tend to be stronger than their
bones, so avulsion injuries are more ^ ASIS
common (when com pared to aduUs). Sartorius

The pelvis is the classic location to test


) Tensor
Fascia Lata
this. AMS
Rectus Femoris

^ Greater
Symphysis Trochanter
ADDuctor Gluteal
Group Muscles
Ischial
Tuberosity
sity I
Hamstrings Lesser Trochanter
llliopsoas

Patellar Sleeve Avulsion Fracture:


This an acute avulsion of the inferior patellar pole. The classic
look is a fragment of bone at the inferior patella with associated
soft tissue swelling. You typically see this in a 13 year old boy
- with a history of “sudden onset pain after jumping.”

This vs T h a t:
Patellar Sleeve Avulsion vs Sinding-Larsen-Johansson
- Patellar Sleeve Avulsion is acute
- SLJ is chronic

- Chronic Fatigue injurios: Sinding


Larsen
Sinding-Larsen-Johansson - This is a chronic Johansson
traction injury at the insertion o f the patellar tendon
on the patella. It’s seen in active adolescents between
age 10-14. Kids with cerebral palsy are prone to it.

Osgood-Schlatter - This is due to repeated micro


trauma to the patellar tendon on its insertion at the tibial
tuberosity. It’s bilateral 25% o f the time, and more Schlatter
common in boys.

240
- Periosteal Reaction in tiie Newborn -
Congenital Rubella: Bony changes are seen in 50% o f cases, with the classic buzzword
being “celery stalk” appearance, from generaHzed lucency o f the metaphysis. This is usually
seen in the first few weeks o f life.

Syphilis: Bony changes are seen in 95% o f cases.


Bony changes do NOT occur until 6-8 weeks o f life
(Rubella changes are earlier). Metaphyseal lucent
bands and periosteal reaction along long bones can be
seen. The classic buzzword is “W im berger Sign” or
destruction of the medial portion o f the proximal
metaphysis of the tibia. ___________

Wimberger Sign

Caffey Disease - Have you ever seen that giant multiple volume set o f peds radiology
books? Yeah, same guy. This thing is a self limiting disorder o f soft tissue swelling, periosteal
reaction, and irritability seen within Ihe first 6 months o f life. The classic picture is the
really hot mandible on bone scan. The mandible is the most common location (clavicle, and
ulna are the other classic sites). It’s rare as hell, and probably not even real. There have been
more sightings o f Chupacabra in the last 50 years.

Prostaglandin Therapy - Prostaglandin E l and E2 (often used to keep a PDA open) can
cause a periosteal reaction. The classic trick is to show a chest x-ray with sternotomy wires (or
other hints o f congenital heart), and then periosteal reaction in the arm bones.

Neuroblastoma Mets - This is really the only childhood m alignancy that occurs in
newborns and mets to bones.

Physiologic Growth: So this is often called “Physiologic Periostitis o f the N ew born” ,


which is totally false and wrong. It does NOT happen in newborns. You see this around 3
months o f age, and it should resolve by six months. Proxim al involvem ent (femur) comes
before distal involvem ent (tibia). It always involves the diaphysis.
It is N O T physiologic periostitis if:
* You see it before 1 month
* You see it in the tibia before the fem u r
* It does not involve the diaphysis.

Abuse — Some people abuse drugs, some ju st can't stand screaming kids, some suffer both
shortcomings. More on this later.

241
- Other “Aggressive Processes” In Kids -
Langerhans Cell H istiocytosis (LCH) - Also know n as EG (eosinophilic
granuloma). It’s twice as com mon in boys. Skeletal manifestations are highly variable, but
lets just talk about the classic ones:
* Skull - M ost com m on site. Has “beveled edge” from uneven destruction o f the inner
and outer tables. If you see a round lucent lesion in the skull o f a child think this (and
neuroblastom a mets).
* Ribs - M ultiple lucent lesions, with an expanded appearance

* Spine - Vertebra plana

LCH - Beveled Edge Skull Lesion LCH - Vertebra Plana

*77/ touch on this more in the M SK chapter.

Ewing Sarcom a and O steosarcom a are also covered in depth in the M SK chapter

242
Osteomyelitis
It usually occurs in babies (30% o f cases less than 2 years old). It’s usually hem atogenous
(adults it directly spreads - typically from a diabetic ulcer).

There are some changes that occur over time, which are potentially testable.

Newborns - They have open growth plates and perforating vessels which travel from the
metaphysis to the epiphysis. Infection typically starts in the metaphysis (it has the most blood
supply because it is growing the fastest), and then can spread via these perforators to the
epiphysis.

Kids - Later in childhood, the perforators regress and the avascular epiphyseal plate stops
infection from crossing over. This creates a “septic tank” scenario, w here infection tends to
smolder. In fact, 75% o f cases involve the m etaphyses o f long bones (fem ur m ost common).

Adults - W hen the growth plates fuse, the barrier o f an avascular plate is no longer present,
and infection can again cross over to the epiphysis to cause mayhem.

Vessels
Cross
Physeal Vessels Growth
Plate Obliterated, Plate
Growth Plate Closed
Still Open

Infant Child (2-8) Adult

Trivia:
- Hematogenous spread m ore com m on in kids (direct spread in adult)
- M etaphysis m ost com m on location, w ith target changes as explained above
- Bony changes don’t occur on x-ray for around 10 days.
- It’s serious business and can rapidly destroy the cartilage if it spreads into the joint

**Discussed again in the M SK Chapter

243
- Skeletal Dysplasia -
There is a bunch o f vocabulary to learn regarding dysplasias:

English Fancy Doctor Speak


Short Fingers Brachydactyly

Too M any Fingers Polydactyly

Two or More Fused Fingers


Syndactyly
( " S o c k H a n d " - I c a ll it)
Contractures o f Fingers Cam ptodactyly

Radially Angulated Fingers (Usually 5'*>) Clinodactyly

Long, Spider-Like Fingers Arachnodactyly

Limb is Absent Amelia

Limb is m ostly Absent M eromelia

Hands / Feet (distal limbs) are Short Acromelic

Forearm or Lower Leg are short (middle limbs) M esomelic

Femur or Humerus (proxim al limbs) are short Rhizomelic

Short All Over M icrom elic

How I rem em ber the lengths:


• M eso is in the M iddle,
• Aero sounds like A crom egaly (they get big hands), and
•The other one is the other one (Rhizo).

There are tons o f skeletal dysplasias and extensive know ledge o f them is w ay way w ay way
beyond the scope o f the exam. Instead I ’m going to m ention 3 dwarfs, and a few other
miscellaneous conditions.

244
-Dwarfs-
(or “Dwarves” If you are practicing radiology in middle earth):

Achondroplasia - This is the m ost com mon skeletal dysplasia, and is the mostly likely
to be seen at the mall (or on television). It results from a fibroblast growth factor receptor
problem (most dwarfisms do). It is a rhizom elic (short femur, short humerus) d w arf They
often have weird big heads, trident hands (3''^ and 4‘h fingers are long), narrowing o f the
interpedicular distance, and the tom bstone pelvis. A dvanced paternal age is a risk factor.
They make good actors, excellent rodeo clowns, and various parts o f their bodies (if cooked
properly) have m agical powers.

Thanatophoric - This is the most com m on lethal dwarfism . They have rhizomelic
shortening (humerus, femur). The femurs are sometimes called telephone receivers. They
have short ribs and a long thorax, and small iliac bones. The vertebral bones are flat
(platyspondyly), and the skull can be cloverleaf shaped.

Asphyxiating T h o ra cic Dystrophy (Jeune) - This is usually fatal as well. The


big finding is the “Bell shaped thorax” with short ribs. 15% will have too m any fingers
(polydactyly). If they live, they have kidney problem s (chronic nephritis). You can
differentiate a dead Thanatophoric dwarf, from a dead Jeune dw arf by looking at their
vertebral bodies. The Jeune bodies are norm al (the thanatophorics are flat).

Additional Random Dwarf Trivia:


•Ellis-Van Crevaid is the d w a rf with m uhiple fingers.
■Pseudoachondroplasia is this w eird thing not present at birth, a n d spares the skull.
■Pyknodysostosis osteopetrosis, in a dwarf, with a wide angledjaw , & Acro-osteolysis.

The Dwarf Blitz - 6 Things I Would Remem ber About Dwarfs

1. The Vocab: Rhizo (humerus, femur) vs Aero (hands, feet) vs M eso (forearm, tib/fib)
2. M ost dwarfs are R hizom elic - if forced to choose, always guess this
3. The pedicles are supposed to w iden slightly as you descend the spinal column,
Achondroplasia has the opposite - they narrow. If you see a live dwarf, w ith short
femurs / hum erus, and narrowing o f the pedicles then this is the answer.
(technically thantophorics can get this too - but i t ’s more classic fo r achondroplasia)
4. Thanatophoric is your main dead d w a rf U sually the standout feature is the
telephone receiver fem ur (and a crazy cloverleaf head)
5. Jeune is another dead dw arf - but the short ribs really stand out,
6. N obody tosses a D w a rf
/« situations where the distance cannot be jum ped and the dw arf must be tossed, don’t tell the Elf.

245
- Misc Conditions -
Bifid Rib - This is the most common cause of an anterior wall “mass.” If there is just one (usually
the 4th rib) - then it is just a variant. If there are bunches think Gorlin Syndrome.

Gorlin Syndrome - Bifid Ribs, Calcifications of the Falx, basal cell cancers, odontogenic
fe n keratocysts (lytic jaw lesions).

Osteogenesis Imperfecta - They have a collagen defect and make brittle bones. Depending
on the severity it can be totally lethal or more mild. It’s classically shown with a totally lucent skull,
or multiple fractures with hyperplastic callus. Another classic trick is to show the legs with the
fibula longer than the tibia. They have wormian bones, and often flat or beaked vertebral bodies.
Other trivia is the blue sclera, hearing impairment (otosclerosis), and that they tend to suck at
football.

Osteopetrosis - They have a defect in the way osteoclasts work, so you end up with
disorganized bone that is sclerotic and weak (prone to fracture). There are a bunch of different types,
with variable severity. The infantile type is lethal because it takes out your bone marrow. With less
severe forms, you can have abnormal diminished osteoclastic activity that varies during skeletal
growth, and results in alternating bands of sclerosis parallel to the growth plate. Most likely the way
this will be shown is the “bone-in-bone” appearance in the vertebral body or carpals. Picture frame
vertebrae is another buzzword. Alternatively, they can show you a diffusely sclerotic skeleton, with
diffuse loss of the corticomedullary junction in the long tubular bones.

Pyknodysostosis - Osteopetrosis + Wormian Bones + Acro-Osteolysis. They also


n have “wide (or obtuse) angled mandible”, which apparentiy is a buzzword.
\\ VV

Klippel Fell - You get congenital fusion of the cervical spine (sorta like JRA).
The cervical vertebral bodies will be tall and skinny. There is often a sprengel deformity
I? (high riding scapula). Another common piece of trivia is to show the omovertebral bone
- which is just some big stupid looking vertebral body.

Hunters I Hurlers I Morquio - All three of these are mucopolysaccharidoses. Findings


include oval shaped vertebral bodies with anterior beak. The beak is actually mid in Morquio, and
inferior in Hurlers. Clavicles and ribs are often thick (narrow more medially) - like a canoe-
paddle. The pelvis shape is described as the opposite of achondroplasia - the iliac wings are tall and
flaired. The hand x-ray is the most commonly shown in case books and gives you wide metacarpal
bones with proximal tapering.

Few More Trivia Points on Morquio:


•They are dwarfs
•The most common cause of death is cervical myelopathy at C2
•The bony changes actually progress during the first few years of life

246
Neurofibromatosis - Just briefly remember that type 1
can cause anterior tibial bowing, and pseudoarthrosis at the
distal fibula.

This is an Aunt Minnie.

They often have scoliosis. Just think of the elephant man.

Gauchers - This is the most common lysosomal storage


disease. It gives you a big spleen, and big liver among a few
bone signs.

*AVN o f the Femoral Heads


*H-Shaped Vertebra
*Bone Infarcts (lots o f them)
'Erlenmeyer Flask Shaped Femurs

Caudal Regression Syndrome - This is a spectrum


that includes sacral and/or coccyx agenesis. You see it with
NF1
VACTERL and Currarino Triads Syndromes.

Scoliosis - Lateral curvature of the spine, which is usually idiopathic in girls. It can also be from
vertebral segmentation problems. NF can cause it as well (that’s a piece of trivia).

Radial Dysplasia- Absence or hypoplasia of the radius (usually with a missing thumb) is a
differential case (VACTERL, Holt-Oram, Fanconi Anemia, Throbocytopenia Absent Radius). As a
point of trivia TAR kids will have a thumb.

Hand Foot Syndrome - The classic history is hand or


foot pain / swelling in an infant with sickle cell. This is a
dactylitis, and felt to be related to ischemia. It will resolve
on its own, after a few weeks. Radiographs can show a
periostitis two weeks after the pain goes awav.

Blounts (tibia vara). Varus angulation occurring at the


medial aspect of the proximal tibia (varus bowing occurs at
the metaphysis not the knee). This is often bilateral, and
NOT often seen before age 2 (two sides, not before two).
Later in the disease progression the medial metaphysis will
be depressed and an osseous outgrowth classically develops.
You can see it in two different age groups; (a) early - which
is around age 2 and (b) late - which is around age 12.

• Two Sides - Not Before Two


• Two Different Ages (2-3, 12)

Blount’S -Tibia VARA

247
-Fe e t-
Congenital fo o t is a com plicated and confusing topic, about which I w ill avoid great detail
because it is w ell beyond the scope o f the exam. I am going to at least try a n d drop som e
knowledge which could be called upon in the darkest o f hours to w ork these problem s out.

Step 1: Vocabulary. Just know ing the lingo is very helpful for getting the diagnosis on
multiple choice foot questions.

Talipes = Congenital,
Pes = Foot or Acquired
Equines = “Plantar Flexed Ankle”, Heel Cord is often tight, and the heel won't touch the floor
Calcaneus = Opposite o f Equines. The Calcaneus is actually angled up
Varus = Forefoot in
Valgus = Forefoot out
Cavus = High Arch
Planus = Opposite o f Cavus - “bizarro cavus ” - FLAT FOOT
Supination - Inward rotation - “Sole offoot in ” - holding soup with the bottom o f your foot
Pronation - Outward rotation - “Sole o f foot out ”

Step 2: Hindfoot Valgus vs Varus

Normal
T

.A
Hindfoot Valgus Hindfoot Varus
T

F irs t look at the norm al T h in k abo u t this as the talus T h is is the opposite
acu te a n g le the talus sliding nose dow n off the situation, in w hich you
and c a lc a n e u s m a k e on c alc an e u s. T h is m a k e the h a v e a narrow ing of the
a lateral view. a n g le wider. an g le b etw e en the talus
and c alc an e u s.
If the talus slides off you lose
your longitudinal arch - which N otice th e tw o b o n es lay
essen tially c h a ra c te rize s n early parallel - like two
hindfoot valgus. “clubs” laying on top of
ea c h other.
Also, note that the nose
d ow n (n e a rly vertical)
a p p e a ra n c e of the talus .
“Too M any Toes”

248
step 3 - Know the two main disorders. The flat foot (valgus), and the club foot (varus).

(1) Flat Foot (Pes Planus)- This can be congenital or acquired. The peds section will cover
congenital and the adult MSK section will cover acquired. The congenital types can be grouped into
flexible or rigid (the flexible types are more common in kids). The distinction can be made with
plantar flexion views (flexible improves with stress). The ridged subtypes can be further subdivided
into (la) tarsal coalition and Ob') vertical talus. In any case you have a hindfoot valgus.

(la) Tarsal Coalition - There are two main types (i) talus to the calcaneus, and (ii) calcaneus to
the navicular. They are pretty equal in incidence, and about 50% of the time are bilateral. You can
have bony or fibrous/cartilaginous subtypes (fibrous/cartilaginous types are more common).

(i) Talocalcaneal Coalition: (ii) Calcaneonavicular Coalition:

• Occurs at the middle facet. • Occurs at the anterior facet.


• “Continuous C-sign ” • Has the “anteater sign ”
produced from an Where the elongated
“absent middle fa c e t” anterior process o f the
on the lateral view. calcaneus resembles the
blood thirsty nose of a
• Talar beak (spur on
ravenous ant eater.
the anterior talus -
This is best seen on an
white arrow) - seen in
oblique view.
25% of cases.

(lb) Vertical Talus (equinus hindfoot valgus) - This is


sometimes called the “rocker-bottom foot” because the talus is in
extreme plantar flexion with dorsal dislocation of the Navicular -
resulting in a locked talus in plantar flexion. As a point of trivia
this is often associated with myelomeningocele.

(2) Club Foot (Talipes Equino Varus)- Translation - Congenital Plantar Flexed Ankle
Forefoot. This is sorta why I lead with the vocab, all the congenital feet can be figured out based on
the translated language. This thing is more common in boys, and bilateral about half the time. The
toes are pointed down (equines), and the talocalcaneal angle is acute (varus).
M e d ia l D eviatio n
Key features:
• Flindfoot varus
(decreased talocalcaneal angle)
• Medial deviation and inversion
of the forefoot
• Elevated Plantar Arch

Trivia: The most common surgical complication is


over correction resulting in a “rocker bottom” flat Club Foot
foot deformity.

Pes Planes T-C


Hindfoot Valgus Coalition

C-N
Hindfoot Varus Talipes Equino
Varus (Club Foot) Vertical Talus
249
- Hip Dysplasia -

Developmental Dysplasia of the Hip (DDH) - This is seen m ore com m only in
females, children bom breech, and oligohydramnios. The physical exam buzzwords are
asymmetric skin or gluteal folds, leg length discrepancy, palpable clunk, or delayed
ambulation. It’s bilateral about 1/3 o f the time. Ultrasound is done to evaluate (after physical
exam), and is excellent until the bones ossify (then you need x-rays). A com mon trick is to be
careful making a m easurem ent in the first w eek o f life - the laxity im m ediately after birth
(related to maternal estrogen) can screw up the measurements.

A Angles.

On ultrasound the alpha angle, should be


more than 60 degrees. Anything less
than that and your cup is not deep enough
to hold your ball. The plain film
equivalent in the acetabular angle, which
is the com plim entary angle (and
therefore should be less than 30).

Getting them confused? Rem em ber that


the “Alpha Angle is the Alpha M ale ” -
and therefore the bigger o f the two angles.
**But don’t forget that DDH is more
common in wom en (not alpha males).

The acetabular angle should decrease


from 30 degrees at birth to 22 degrees at
age 1. DDH is the classic cause o f an
increased angle, but neurom uscular
disorder can also increase it.

A cet

The position o f the femoral epiphysis


H
(or where it will be) should be below /
H ilgenreiner’s line “H ”, and medial to
Perkin’s Line “ P ” . Shenton’s Line “S”
/ T '
should be continuous. Ideal
Location
F em o ral
Epiphysis

250
Proximal Focal Femoral Deficiency:

This is a congenital zebra, which ranges from absent proximal


femur to hypoplastic proximal femur.

The classic history is “weirdfucked up looking tiny


Frankenstein leg. ”
You get a varus deformity.

This is described as a mimic of DDH,


but DDH will have normal femur leg length (not a fucked up looking tiny Frankenstein leg).

Slipped Capital Femoral


Epiphysis (SCFE)
NORMAL S C FE

This is a type 1 Salter Harris, through


the proximal femoral physis. What
makes this unique is that unlike most
SH Is, this guy has a bad prognosis if X
not fixed.

The classic history is a fat African


American adolescent (age 12-15) with
hip pain.

It’s bilateral in 1/3 of cases (both hips


don’t usually present at same time). “Klein’s Line” - D raw n along th e e d g e of the
fe m u r and should n o rm ally in tersect with lateral
The frog leg view is the money sup erio r fem o ra l epiphysis. T h is line is used to
- this is always the answer on e v a lu a te for S C F E . W h e n the line d o e s n ’t cross
next step questions. th e lateral epiph ysis think S C F E .

***Testable Trivia - 1 IS more


sen sitive for this m e a s u re m e n t

Legg-Calve-Perthes
T H IS vs THAT:
This is AVN of the proximal femoral epiphysis. i

It’s seen more in boys than girls (4:1), and favors white Perthes SCFE
people around age 5-8. These kids tend to be smaller
than average for their age.
O ften Small Overweight
This is bilateral about 10% of the time (less than
W hite Kids Black Kids
SCFE).
The subchondral lucency (crescent sign) is best seen on a Age 5-8 Age 12-15
frog leg. Other early signs include an asymmetric small
ossified femoral epiphysis. MRI has more sensitivity.
The flat collapsed femoral head makes it obvious. Sterile
Bilateral 10% B ilateral 30%
joint effusions (transient synovitis) can be associated.

251
Septic Arthritis- This is serious business ,
and considered the most urgent cause o f painful
hip in a child. W ide joint space (lateral
displacement o f femoral head), should prom pt an
ultrasound, and that should prom pt a joint tap. If
you have low suspicion and don’t w ant to tap the
hip. You could pull on the leg under fluoro and try
and get gas in the joint. This air arthrogram sign
supposedly excludes a joint effusion (and Fluid Distending the Joint Capsule
therefore a septic joint) - depending on who you over the Femoral Neck (arrow)

ask.

Transient Synovitis - This is a sterile


(reactive) hip effusion that occurs in the setting o f
a systemic illness (usually viral URI or GI). As
the name suggests this is “transient” and goes
away in a few days. This is actually very
common. Some sources will say its the most
common hip disorder in growing children (peak
age is around 5). The ED will be in a full panic Hip Joint Effusions Are Seen Here at the Neck
and want you to tap it at 3 am.

TH IS vs THAT: Transient Synovitis vs Septic Arthritis

Telling these apart is actually im portant for real life (not getting sued) since a septic hip will
fucking destroy the kid’s cartilage (usually if it’s m issed for m ore than 4 days).

Ortho (and in very rare situations a “sm art” ED doc) will use a clinical param eter
“the K ocher C riteria” to tell them apart.

If 3/4 are positive = Septic


Criteria is 4 parts:
If CRP is negative and the kid can
• Fever
bear w eight it’s N OT Septic
• Inability to walk
CRP is the strongest independent
. Elevated ESR (or CRP)
risk factor for septic arthritis
. W B C > 12K

Typical workup is going to be:


1. X-Ray Hip series (AP, Lateral, and Frog Leg) which is usually negative in real life but
will probably show medial jo in t w idening on the exam.
2. U ltrasound which will show an effusion.
3. Then a clinical decision based on K ocher Criteria (>2) and “Gut Instinct” to Aspirate
4. M RI w ould only be used if/when hip aspiration can’t/hasn’t been perform ed

252
-Metabolic-
Rickets - N ot enough vitam in D. Affects the most
rapidly growing bones (mostly knees and wrists).
Buzzwords “fraying, cupping, and irregularity along
the physeal margin. ” They are at increased risk for
SCFE. “Rachitic rosary” appearance from expansion o f
the anterior rib ends at the costochondral junctions. As a
pearl, rickets is never seen in a newborn (M om ’s
vitamin D is still doing its thing).

Hypophosphatasia - This looks like Rickets in a


newborn. They will have frayed m etaphyses and bowed
long bones. The underlying pathology is deficient
serum alkaline phosphatase. There is variability in
severity with lethal perinatal / natal forms, and m ore m ild “Cupping” & Fraying
adult forms.

Scurvy - Not enough vitam in C. This is rare as hell outside o f a pirate ship in the 1400s.
For the purpose o f trivia (which multiple choice tests love) the following stuff is high yield:

• Does NOT occur before 6 months o f age (m aternal stores buffer)


o Bleeding Disorders Comm on
o Subperiosteal hem orrhage (lifts up the periosteum )
• Hemarthrosis
• “Scorbutic rosary” appearance from expansion o f the costochondral junctions (very
similar to rickets).

Lead Poisoning - This is m ost com monly seen in kids less than two who eat paint chips.
The classic finding is a wide sclerotic m etaphyseal line (lead line), in an area o f rapid
growth (knee). It will not spare the fibula (as a norm al variant line might).

Lucent Metaphyseal Bands


- This is a classic peds DDx. - LINE.

• Leukem ia
• Infection (TORCH)
• N euroblastom a Mets
• Endocrine (rickets, Scurvy)

253
- Non-Accidental Trauma I NAT) -
“Som e People Just C a n ’t Take Scream ing Kids. ”

Any suspicious fracture should prom pt a skeletal survey (“baby gram ” does N O T count).
Suspicious fractures w ould include highly specific fractures (m etaphyseal com er fracture,
posterior rib fractures) or fractures that don’t make sense - toddler fracture in a non­
ambulatory child.

• Posterior M edial Rib Fracture: In a child under the age o f


3, this is pretty reliable. Supposedly this type o f fracture can
only be made from squeezing a child.

• M etaphyseal Corner Fractures: W hen this is present in a


non-am bulatory patient (infant) it is HIGHLY specific. The
only exception is obstetric trauma. A fter age 1, this becom es
less specific.

• Skull Fracture: The general idea is anything other than a parietal bone fracture (which is
supposedly seen more with an actual accident) is concerning.

• Solid Organ and Lum en Injury - D on’t forget about this as a presentation for NAT.
Duodenal hem atom a and pancreatitis (from traum a) in an infant - should get you to say
NAT. Just think “belly trauma in a kid that is too yo u n g to fa ll on the handle bars o f their
bike

254
S E C T IO N 14:
PEDIATRIC S pin e

-Spinal Cord on Ultrasound -


Normal Ultrasound A ppearance /Anatom y Review:

Conus

Central Canal — Echogenic


Vertebral Bodies - Shadowing

Central Canal

Central Canal - Easily identified as an


echogenic line. A lthough that is sort o f counter
intuitive.

Why w ould a structure with flu id in it be


echogenic? The reason is that you are actually
seeing a “central echo com plex” - w hich is the
interface between the anterior m edian fissure and
the myelinated ventral white m atter commissure.

Rem ember that interfaces betw een things with Ventral


Commissure
large differences in im pedance cause a lot o f
Anterior
reflections (thus an echogenic line). Median Fissure

Trivia: Technically in a new born the central canal is not even fluid filled (it’s packed with
glial fibrils), but that level o f trivia is beyond the scope o f the exam (probably).

255
Low lying cord I Tethered cord: Because the canal grows faster than the cord, a
fixed attachm ent (“tethering”) results in cord stretching and subsequent ischemia. This can be
primary (isolated), or secondary (associated with m yelom eningocele, filum term inale lipoma,
or trauma). The secondary types are more likely shown on MR (to showcase the associated
mass - flu id collection), the prim ary types are more likely shown on US - as a straight
counting game.

Imaging Features: Low conus (b elow L 2), and thickened filum term inale (> 2mm).

A common piece o f trivia used as a distractor is that meningom yelocele is associated with
Chiari m alformations, lipom yelom eningocele is NOT.

High Yield Trivia

• Anal A tresia = High Risk For O ccult Cord Problems (including tethering) - should get
screened
• Low lying / tethered cords are closely linked with Spina Bifida (tufts o f hair)
• Low Dimples (below the gluteal crease) Do NOT need screening,
o These never extend intra-spinally. They m ight later becom e a pilonidal sinuses
- but aren’t ever gonna have shit to do with the cord.
• High Dimples (above the gluteal crease) DO need screening.

For Screening Purposes:


Low dimples (below the butt crease) don’t get screened, basically everything else does.
Most Likely Question Style: “Which of the following does NOT get screened ?”
(Answer = low dimple).

256
-MiscVariant Topics-

Terminal Ventricle (ventriculus


terminalis): This is a developmental variant.
Normally, a large portion o f the distal cord involutes in a
late stage o f spinal cord embryology. Sometimes this
process is not uniform and you get stuck with a stupid
looking cyst at the end o f your cord. These things are
usually small (around 4 mm), and cause no symptoms.
Sometimes they can get very big (like this exam ple) and
cause some neurologic symptoms.
Terminal Ventricle

Pars Interarticularis Defects (Spondylolysis):


This is considered a fatigue or stress fracture, probably developing in
childhood. It is a classic cause o f back pain in an adolescent athlete.
Although they are usually not symptom atic (only 25% are). The
process represents a hole / break in the coimecting bone betw een the
superior and inferior articular facets. If there is forw ard “slippage”
you can deploy the word spondylolisthesis.

Almost always (90% +) you see this at L5 (2nd m ost com m on at L4).
They tend to have more spondylolisthesis and associated degenerative
change at L4-L5 than L5-S1. They can be seen on the oblique plain
film as a “collar on the scottie dog.” The collar on the “scotty dog”
appearance on an oblique plain film is probably the m ost com mon
way they show this in case books and conferences. On the AP view
this can be a cause o f a sclerotic pedicle (the contralateral pedicle -
from wiggle stress). On CT it is usually more obvious with the break
clearly demonstrated.

**Pars Defects with anterolisthesis will have neuroforam inal stenosis,


Scotty Dog = Collar
with spinal canal w idening (w hen severe will have spinal canal
(represent the pars
stenosis as well). If the process is purely a degenerative defect).
spondylolisthesis (not much slippage), the resulting facet arthropathy
will favor the canal w ith less severe effects on the neuro foramina. Anatomy trivia: the
eye is the pedicle.

257
Spinal Dysraphism;
You can group these as open or closed (closed with and w ithout a mass). Open means neural
tissue exposed through a defect in bone and skin (spina bifida aperta). Closed means the
defect is covered by skin (spina bifida occulta).

Open Spinal Dysraphisms: This is the result o f a failure o f the closure o f the prim ary
neural tube, with obvious exposure o f the neural placode through a midline defect o f the skin.
You have a dorsal defect in the posterior elements. The cord is going to be tethered. There
is an association with diastem atom yelia and Chiari II m alformations. Early surgery is the
treatment / standard o f care.

Myelocele: This is the more rare type w here the neural


placode is flush with the skin.

M yelomeningocele: This is the more com mon type


(98%) where the neural placode protrudes above the
skin. These are more com mon with Chiari II
malformations.

Closed Spinal Dysraphisms with Subcutaneous M ass

Meningocele: This is herniation o f a CSF filled sac through a


defect in the posterior elements (spina bifida). It is most typical
in the lum bar or sacral regions. Although they can occur in the
cervical spine. They may be anterior (usually pre-sacral). An
important point is that neural tissue is NOT present in the
sac.

Lipom yelocele / Lipom yelomeningocele: These are lipomas with a dural defect
(form er with the neural placode-lipom a interface inside the canal & latter with it
outside the canal). On exam you are going to have a subcutaneous fatty
mass above the gluteal crease. These are 100% associated with
tethered cord (m yelom eningocele m ay or may not).

Terminal M yelocystocele - This is a herniation o f a terminal


syrinx into a posterior m eningocele via a posterior spinal defect.

258
Closed Spinal Dysraphism s without Subcutaneous M ass

Intradural lipomas - M ost com m on in the thoracic spine along the dorsal aspect.
They don’t need to be (but can be) associated with posterior elem ent defects.

Fibrolipoma o f the filu m terminale - This is often an incidental finding “fatty filum” .
There will be a linear T1 bright structure in the filum terminale. The filum is not
going to be unusually thickened and the conus will be norm ally located.

Tight filu m terminale - This is a thickened filum term inale (> 2 mm ), with a low lying
conus (below the inferior endplate o f L2). You may have an associated terminal lipoma.
The “tethered cord syndrom e” is based on the clinical findings o f low back pain and leg
pain plus urinary bladder dysfunction. This is the result o f stretching the cord with
growth o f the canal.

D erm al Sinus - This is an epithelial lined tract that extends from the skin to deep soft
tissues (sometimes the spinal canal, som etim es a dermoid or lipoma). These are T1 low
signal (relative to the background high signal from fat).

Diastematomyelia - This describes a sagittal split in the spinal cord. They almost always
occur between T 9 -S 1, with normal cord both above and below the split. You can have two thecal
sacs (or just one), and each hem i-cord has its own central canal and dorsal/ventral homs.
Classification systems are based on the presence / absence o f an osseous or fibrous spur and
duplication or non-duplication o f the thecal sac.

Caudal Regression: This is a spectrum o f defects in the caudal region that ranges from
partial agenesis o f the coccyx to lum bosacral agenesis. The associations to know are VACTERL
and Currarino triad. Think about this with m aternal diabetes. “Blunted sharp” high terminating
cord is classic, with a “shield sign” from
the opposed iliac bones (no sacrum).
Currarino Triad:
Fuckery - Note than the M eningocele o f
Currarino is Anterior. It’s not posterior. Anterior Sacral M eningocele,
A potential deploym ent o f fuckery is to \m
• •
put “Posterior” Sacral M eningocele as a • # Anorectal malformation,
distractor for Currarino Triad.
Sacrococcygeal osseous defect
Anyone who w ould do that is an Asshole
(scim itar sacrum).
(a structure which also happens to be
posterior)

259
fROMCTHEUS
Liomhart, m.d.

260
PROMETHEUS LIONHART, M.D.

g a s t r o in t e s t in a i

Between the time when the oceans drank Atlantis, and the rise of the sons of
Aryas, there was an age undreamed of. And unto this, barium, was used to
differentiate various luminal Gl pathologies.
Let me tell you of the days of high adventure!

261
S E C T IO N 1:
L u m in a l

- Esophagus -
Anatomy:
A- Ring
A R m g : The m uscular ring above the
vestibule. Dynamic on swallow.

B R in e: The mucosal ring Below the


vestibule. N ot Dynamic. This is a thin H\a^a'
Hert"a
constriction at the GE junction. Symptomatic
dysphagia can occur if it narrows (historically
defined at <13m m in diameter). If it’s
narrowed (and sym ptom atic) you call it a
Schatzki. Just Say “Shatz-‘B ’-R in g ”

Z L ine: Represents the squam ocolum nar


junction (boundary between esophageal and gastric epithelium). This doesn’t necessarily
correspond with the B-ring. This is an endoscopy finding, and is only rarely seen as a thin
serrated line.

M ucosa should have thin, parallel uniform folds.

Anatomic Trivia

H ypo Cricopharyngeus
P harynx • T h e “true upper esoph ageal sphincter.”
*Location of • This m uscle represents the border
Zen ker Div. between the pharynx and cervical
esophagus.
• T h e typical level is around C 5 -C 6
C ric o ph a ryn g e u s
Swallowing
C ervical
• W h e n you sw allow the larynx does two
E so p hag u s things: (1) it e lev ate s and (2) it m oves
*Location of
anteriorly
K illian-Jam ieson Div.

262
Special Topic - Which Hand Does the Barium Go In ?

According to historical records, Left and R ight Posterior O bhque positions were com monly
used when perform ing a thoracic barium swallow exam. So which hand should the barium
cup go in ??? The trick is to consider the potential for the barium filled cup to obscure the
field o f view as the patient is drinking. I f the cup is placed in the more anterior hand this
could happen... so don’t do that.

Too Long D idn’t Read: LPO = Left Hand RPO = R ight Hand

Left Posterior Oblique Right Posterior Oblique

Mild Fold Thickening.


Reflux Esophagitis: Severe Fold Thickening
Strictures _
and/or Fundiplication
A com mon cause o f fold Barretts (afterfailed H2 blockers
and PPIs)
thickening, which if left
Cancer -
unchecked can cause Esophagectomy
Death -
some serious problems.
That Guy You Hate Marries
In anguish your soul cannot enter Your Now Widowed Wife
Behold the potential “the light” - you decide the best (She is way happier with
option is to haunt your widowed him. She constantly talks
spectrum o f unchecked wife and her new asshole husband shit about you and how you
aggression -> Beetlejuice style. never picked up your dirty
You are featured on the Sci-Fi channel’s
hit paranormal realty TV show “Ghost
Hunters” after manifesting briefly as a
focused ectoplasmic phantasm or a class 3
t towels or helped out with
the dishes.)

full roaming vapor.

263
Barretts: This is a precursor to Feline Esophagus:
adenocarcinom a - that develops secondary to
Often described
chronic reflux. The way this will be shown is
as an Aunt
a high stricture with an associated hiatal M innie
hernia.
You have
(2 ^ B uzzw ord R eticular M ucosal transient, fine
transverse folds
Pattern.
which course
mid and low er
esophagus.
It can be
normal, but has
a high
association
with reflux
esophagitis.....
correlate
clinically.
*Folds are Transient
(they go away with swallowing)

B arrett’s - High Suicluro lliulul Hernia *Folds are O N LY in lower 2/5

Cancer: On barium you want to see the words “irregular


contour”, and “abrupt (shouldered) edges. ” I like to use
stereotypes to remember the subtypes and associations:

Squamous: This is a black guy who drinks and smokes, and


once tried to kill him self with an alkaloid ingestion (drank
lye). The stricture/ulcer/mass is in the mid esophagus.

Adeno: This is a white guy, who is stressed out all the time.
He has chronic reflux (history o f PPI use). He had a scope
years ago that showed Barretts, and he did nothing because
he was too busy stressing out about stuff. The stricture/ulcer/
mass is in the lower esophagus.

Critical Stage: T3 (Adventitia) vs T4 (invasion into adjacent structures) - obviously you need
CT to do this. The earlier stages are distinguished by endoscopy - so not your problem and
unlikely to be tested. T3 vs T4 is in the radiologists world - and therefore the most likely to be
tested.

264
Herniation of the stomach comes in several
flavors with the most common being the sliding
type 1 (95%), and the rolling para-esophageal
type 2. The distinction between the two is
based on the position of the GE junction.
Small type Is are often asymptomatic but do P vM\
have an association with reflux if the function of Sliding (Type 1) GE Para-E (Type 2) GE
the GE sphincter is impaired. The Para-E type 2 Junction ABOVE the Junction BELOW the
has a reportedly higher rate of incarceration. diaphragm — arrow, diaphragm — arrow.

F u n d o p lic a tio n B litz


What is this Fundoplication ? The gastric fundus is
wrapped around the lower end of the esophagus and
stitched in place, reinforcing the lower esophageal
sphincter. The term “Nissen” - refers to a 360 degree
wrap. Loose ( < 360) wraps can also be done, and
have French sounding names like “toupet,” - these are
less high yield.

Early Complication: The early problem with these is


esophageal obstruction (or narrowing). This occurs
from either post op edema, or a wrap made too tight.
You see this peak around week 2. Barium will show
total or near total obstruction o f the esophagus.

Failure: There are two main indications for the procedure (1) hiatal hernia, and (2) reflux. So
failure is defined by recurrence of either of these. The most common reason for recurrent reflux is
telescoping of the GE junction through the wrap - or a “slipped Nissen.”

• Most common reason fo r recurrent reflux ?= slipped Nissen

• Most common reason fo r slipped Nissen ? = short esophagus

• WTF is a “short esophagus ” ? The exact


definition is elusive (and depends on the
phase of the moon). For the purpose of
multiple choice test I’d go with “Hiatal
Hernia that is fixed/non- reducible, and
greater than 5cm ”

• How can you tell if the wrap has slipped ?


Fundoplication wrap should have length
of narrowed esophagus < 2cm (anything
greater suggests a slipped wrap)

• Remember recurrent hernia (GE junction Down


above the diaphragm) is not normal.

What is the treatmentfor a “short esophagus” ?


Collis gastroplasty (lengthening + fundoplication).
Trivia: You cannot vomit after a fundoplication (no matter how bad your wife’s cooking is)

265
Candidiasis: G lycogenic

A hint in the question stem might be “HIV Patient” or “Transplant Acanthosis:


Patient” (someone who is immunocompromised). They could
also tell you the patient has “achalasia” or “scleroderma” - because It is essentially an
motility disorders are also at increased epithelial collection
risk. o f glycogen - but is
known best as a
The most common finding is discrete mimic of
plaque-like lesions. Additional findings candidiasis.
include: nodularity, granularity, and fold
thickening as a resuU o f mucosal Multiple elevated
inflammation and benign nodules in an
edema. asymptomatic
elderly patient.
When it is most severe,
it looks more shaggy
with an irregular
luminal surface.

Ulcers:
CMV and HIV
HERPES: Large Flat
Ovoid Ulcer
Small and multiple with
a halo of edema {CMV & HIV
look the same)
{Herpes has a Halo)
\ CROHN’S: Esophageal involvement is rare (only in severe disease),
i Buzzword is “Aphthous Ulcers” - (discrete ulcers surrounded by mounds of edema)

Esophageal (enteric)
Duplication Cysts:
Lateral View
Esophagram
If they show one of these it will be on CT
could show
(what? GI path not on barium?).
non-specific
extrinsic
Seriously, they would have to show this on
mass effect
CT. It is gonna be in the posterior
mediastinum, and have an RQI showing Next Step - CT
water densitv. This is the only way you can Water density cyst in the posterior
show this. mediastinum abutting the esophagus

Possible Ways This Could Be Tested:

• What is it / What does it look like 7 - Water density cyst in the posterior mediastinum

• Most common location ? - The ileum (esophagus is #2).

• How can they present ? Either as an incidental in an adult, or if they are big enough - as an
infant with dysphagia / breathing problems.

266
Esophageal Diverticulum

Zenker Diverticulum: Killian-Jamieson Pulsion Traction Diverticulum:


Pulsion Diverticulum in the Diverticulum:
back (Z is in the back o f the Mid esophageal, and
alphabet). This one is anterior and lateral. often triangular in shape.

The question they always ask It protrudes through an area of These occur from
is: site of weakness = Killian weakness below the attachment of the scarring (think
Dehiscence or triangle. cricopharyngeus muscle and lateral to granulomatous disease or
the ligaments that help suspend the
TB).
Another sneaky point of trivia esophagus on the cricoid cartilage.
is that the diverticulum arise
from the hypopharynx (not This one is in the cervical
the cervical esophagus). esophagus.

Epiphrenic Diverticula: THIS vs THAT:

Located just above the diaphragm Traction Pulsion


(usually on the RIGHT). Triangular Round
** The para-esophageal hernia is
usually on the LEFT. Will W ill NOT
Em pty Empty
They are considered pulsion types (contains no
(associated with motor muscle the
abnormality). 1walls)

Esophageal Pseudodiverticulosis:

This is an Aunt Minnie. What you have are dilated submucosal


glands that cause multiple small out pouchings. Usually due to
chronic reflux esophagitis.

There is controversy about the whole Candida situation. Per the


Mayo GI book, Candida is often cultured but is not the causative
factor.

267
Papilloma: Eosinophilic Esophagitis:
Classically a young man with a long history of
The most common benign mucosal lesion of dysphagia (also atopia and peripheral
the esophagus. It’s basically just hyperplastic
eosinophilia). Barium shows concentric rings
squamous epithelium.
(distinct look). They fail treatment on PPIs, but
get better with steroids.
Esophageal Web:
Buzzword = “Ringed Esophagus”
Most commonly located at the cervical
esophagus (near the cricopharyngeus). This
thing is basically a ring (you never see
posterior only web) caused by a thin mucosal
membrane.

It’s a risk factor for esophageal and


hypopharyngeal carcinoma

Plummer -Vinson Syndrome: iron def


anemia, dysphagia, thyroid issues, “spoon-
sliapcd luiiK”
Eosinophilic Esophagitis - Rings

Esophageal Spasm:

Most of the time the exam is


normal, but for the purpose
of multiple choice I ’d expect
“corkscrew” tertiary
Normal Submucosal contractions favoring the
venous plexus (higher Web - circumferential distal esophagus.
up and shallow) (anterior and posterior)
in lower cervical region The term nutcracker
esophagus requires
Esophageal Pseudodiverticulosis: manometric findings
(>180mmHg).
Tons of little tiny out-
pouchings which aren’t
Dysphagia Lusoria - THIS IS HIGH
really diverticula but
instead dilated excretory YIELD. Syndrome refers to problem s
mucosal ducts. There is a
swallowing secondary to com pression from
described association with
esophageal strictures (90%), an aberrant right subclavian artery (most
reflux, and candidiasis. patients with aberrant rights don’t have
The appearance is fairly symptoms).
classic and is better
demonstrated with a single ** Refer to p a g e 199 fo r the chart on
phase of contrast (instead of
the fancy double contrast). esophageal vascular impressions.
Seriously, yo u need to know that crap.

268
The Dilated Esophagus
If you get shown a big dilated esophagus (full cheerios, mashed potatoes, and McDonald’s
french fries... or “freedom fries” as I call them), you will need to think about 3 things.

(1) Achalasia: A motor disorder where the distal 2/3 of the esophagus (smooth muscle part)
doesn’t have normal peristalsis ( “absent primary peristalsis ”), and the lower esophageal
sphincter won’t relax.

The esophagus will be dilated above a smooth stricture at


the GE junction (Bird’s Beak).

“Vigorous Achalasia ” - An early / less severe form which


classically has repetitive simultaneous non-propulsive
contractions. It’s more common in women, but the
secondary cancer occurs more in men.

Things to know:

Failure of the lower esophageal sphincter to relax


Increased risk of Candida

“Chagas Disease ” - More common in the jungle. The


esophagus get paralyzed by some parasite transmitted by
a fly. The appearance is identical to Achalasia - and some
people even lump them together (others reserve the term
“achalasia” for idiopathic types only). You’ll simply need
to read the mind of the test question writer to know what
camp he/she falls into.

(2) “Pseudoachalasia” (secondary achalasia) has the appearance of achalasia, but is secondary
to a cancer at the GE junction. The difference is that real Achalasia will eventually relax, the
pseudo won’t. They have to show you the mass - or hint at it, or straight up tell you that the GE
junction didn’t relax. Alternatively they could be sneaky and just list a bunch of cancer risk
factors (smoking, drinking, chronic reflux) in the question stem. Next step would probably be
CT (or full on upper GI).

(3) Scleroderma: Involves the esophagus 80%


of the time. Again the lower 2/3 of the
esophagus stops working normally. The LES
is incompetent and you end up with chronic
reflux, which can cause scarring, Barretts, and
even cancer (Adeno). They will show you
lung changes (most commonly NSIP), and the
barium esophagus (or a small bowel series %»
showing closely spaced valvulae conniventes -
hide bound).
N S IP - Ground G lass with sub-pleural sparring

269
Varices:
Linear often serpentine, filling defects causing a
scalloped contour.

The differential diagnosis for varices includes


varicoid carcinoma (this is why you need them
distended on the study).

T H IS v^THAT:

Uphill Varices Downhill Varices

Caused by SVC obstruction


Caused by Portal Hypertension
(catheter related, or tum or related)

Confined to Bottom H alf o f Esophagus | Confined to Top H alf o f Esophagus

SVC
Obstruction
Portal Esophageal Downhill
Hypertension Veins Varices
Top Half of

Uphill Bottom
Varices Half of
Esophagus

270
- Esophagus - High Yield Trivia -
Path Trivia
Esophagitis Fold Thickening. May have smooth stricture at GE junction if severe
Buzzword; Reticulated Mucosal Pattern
Barretts
Classically shown as Hiatal Hernia + High Stricture
Medication Induced
Ulcers; Usually at the level of the arch or distal esophagus
Esophagitis
Crohns Esophagitis Ulcers ; can be confluent in severe disease
Discrete plaque like lesions that are seen as linear or irregular filling
defects that tend to be longitudinally oriented, separated by normal
mucosa
Candida
Buzzword: Shaggy - when severe

Not always from AIDS, can also be from motility disorders such as
achalasia and scleroderma
Glycogenic Acanthosis Looks like Candida, but in an asymptomatic old person
Herpes Ulcers s Multiple small, with Edema Halo (herpes has halo)
CM V/AIDS Ulcers Large Flat ulcers

Crohn’s Ulcers Esophageal involvement is rare (only in severe disease).

Buzzword is “Aphthous Ulcers” - discrete ulcers surrounded by


Crohn’s Esophagitis
mounds of edema
Buzzword: Bird Beak, - smooth stricture at GE junction

Achalasia Path is failure of LES to relax (but it will slowly relax)

Increased risk of Squamous Cell CA, and Candida


Pseudoachalasia Cancer at the GE junction. Fixed Obstruction, will not relax
Involves the Esophagus 80% of the time

Scleroderma i Looks a little like Achalasia (they will show you lung changes- NSIP)

1 Sequelae of reflux: stricture, Barrets, cancer


Long Stricture DDx: NG tube in too long, Radiation, Caustic Ingestion (classic lye)
Dilated submucosal glands, usually due to chronic reflux esophagitis.
Pseudodiverticulosis
Esophageal stricture is seen in 90% of cases
1Zenker in the back (above cricopharyngeus)
Zenker Diverticula
From the hypopharynx

Lateral (below cricopharyngeus)


Killian-Jamieson
i From the cervical esophagus.

271
- stomach -

Malignant Benign

Width > Depth Depth > W idth

Located within Lumen Project beyond the expected lumen

Nodular, Irregular Edges Sharp Contour

Folds adjacent to ulcer =Folds radiate to ulcer

Aunt Minnie: Carmen M eniscus Sign Aunt Minnie: H am pton’s Line

Can be anywhere Mostly on Lesser Curvature

-GastricTumors-
GIST (Gastrointestinal Stromal Tumor) - These can be benign or malignant

This is the most common mesenchymal tumor of the GI tract (70% in stomach, duodenum is second
most common — colon is actually the least common). Think old person (it’s rare before age 40).

Some tricks to know: Absence o f perigastric


lymphadenopathy favors a
• Lymph node enlargement is NOT a classic feature - benign GIST over a
malignant GIST or
• Malignant ones tend to be big angry mother fuckers Gastric Adenocarcinoma
(>5 cm with ulceration - and possible perforation).

• If they do met - it is typically to the liver.


Carneys Triad
• Having said that, malignancy is rare with these. “ C arn ey ’s E at G arbage”
They typically don’t met anywhere,
which is why lymph node enlargement is uncommon.
C hondrom a (pulmonary)
The association with C arney’s triad Extra Adrenal Pheo
G IST
The association with NF-1

272
- Gastric Tumors 2 - The Final Chapter
Gastric “Cancer" is either Lymphoma (<5%) or Adenocarcinoma (95%).... Rarely a malignant GIST.

Gastric Adenocarcinoma - usually a disease of an old person (median age 70 - rare before 40).
H. Pylori is the most tested risk factor.

Trivia to know: Ulcer Trivia:


Gastric Ulcers - 5% chance of being cancer.
• Can obstruct - if it involves the antrum
Gastric Ulcers occurs from “altered mucosal
• Metastatic spread to the ovary is referred to
resistance”, and favor the bulb
as a K rukenberg Tumor.
Multiple Gastric Ulcer - think chronic aspirin therapy
• Gastroenterostomy performed for gastric
ulcer disease (old school - prior to PPIs) Duodenal Ulcers - Are never cancer
have a 2x - 6x- increased risk for (on multiple choice)
development of carcinoma within the gastric Duodenal Ulcers - 2-3x more common
remnant - “stump cancer” they call it. than Gastric Ulcers.
• Other random risk factors: Duodenal Ulcers occur from “increased peptic acid”
Pernicious anemia, Menetrier’s disease Duodenal Ulcers- usually solitary,
• Step 1 trivia question: swollen left If multiple think ZE.
supraclavicular node = Virchow Node.
The Look: Gastric Adenocarcinoma looks very different (usually) than a GIST.
Gastric Adenocarcinoma
(arrow) is usually a large,
ulcerated, heterogenous
mass.
The “mass” is often
asymmetric wall
thickening - focal and
nodular - more than
12mm.
GIST (arrow) is usually
smoothly marginated and Benign Gastritis would be
exophytic more diffuse stratified gastric
wall thickening

Gastric Lymphoma, can be primary (MALT), or


secondary (systemic lymphoma). The stomach is the
most common extranodal site for non-Hodgkin
lymphoma.
Just like adenocarcinoma - H. Pylori is a risk factor
Even when extensive, it rarely causes gastric outlet
obstruction. It was classically described as “crossing
the pylorus ” , although since gastric carcinoma is like
lOx more common, it is actually more likely to do that.
Has multiple looks and can be big, little, ulcerative, Lymphoma - Crossing the Pylorus
polypoid, or look like target lesions. It can also look like
Linitis Plastica (discussed on the following page). Classic Look =
• Diffiise wall thickening (>1 cm)
Trivia: It can rupture with treatment (chemo).
• Without gastric outlet obstruction

273
- Gastric Tumors 3 - A New Beginning

Gastric C an cer is “More Likely” Than Lymphoma to...

•More Likely to Cause Gastric Outlet Obstruction

•More Likely to be in the distal stomach

•More Likely to extend beyond the serosa and obliterate adjacent fat plains

•More Likely to be a focal mass (95% of primary gastric tumors are adenocarcinoma)

Mets to the Stomach: This is actually very rare.

Melanoma is probably the m ost common culprit, w hen it does occur.


This obviously has a variable appearance but multiple button type
soft tissue nodules is probably the m ost classic look.

Breast (most classic) and Lung are other possibilities -and these are
known for producing a particular look o f diffuse infiltration and a
contracted desmoplastic deformity resem bling a stiff leather bottle.
This is the so called Linitis Plastica appearance - which can also be
a look for lym phoma as above.

Lobular carcinoma is the m ost com mon breast cancer histological


type which presents with gastric metastases

- Misc. Gastric Conditions -


Chronic Aspirin Therapy: “Multiple gastric ulcers” is the buzzword. Obviously this is non­
specific, but some sources say it occurs in 80% of patient’s with chronic aspirin use. As a point of
trivia, aspirin does NOT cause duodenal ulcers.

If you see multiple duodenal ulcers (most duodenal ulcers are solitary) you should think Zollinger-
Ellison.

Areae Gastricae: This is a normal fine reticular pattern seen on double contrast. A favorite
piece of trivia to ask is when does this “enlarge ” ? The answer is that it enlarges in elderly and
patient’s with H. Pylori. Also it can focally enlarge next to an ulcer. It becomes obliterated by cancer
or atrophic gastritis.

274
- Misc. Gastric Conditions - Continued
Menetrier’s Disease: Rare and has a French sounding name, so it’s almost guaranteed to be on
the test. It’s an idiopathic gastropathy, with regular rugal thickening that classically involves the
fundus and spares the antrum. Bimodal age distribution (childhood form thought to be CMV related).
They end up with low albumin, from loss into gastric lumen.

Essential Trivia: Involves the fundus, and spares the antrum.

Ram’s Horn Deformity (Pseudo Billroth 1)


Tapering of the antrum causes the stomach to look
like a Ram’s Horn. This is a differential case, and
can be seen with Scarring via peptic ulcers,
Granulomatous Disease (Crohns, Sarcoid, TB,
Syphilis), or Scirrhous Carcinoma.
Normal
Essential Trivia: The stomach is the
most common GI tract location for sarcoid. Rams Horn - tubular, conical
shape of the distal stomach

Gastric Volvulus
Organoaxial
Two Flavors:

Organoaxial - the greater curvature flips over the lesser curvature. This is
seen in old ladies with paraesophageal hernias. It’s way more common.

Gastric Antrum BELOW the GE Junction

Mesenteroaxial - over the mesentery. Can cause ischemia and


needs to be fixed. Additionally this type causes obstruction. This type is
more common in kids.

Gastric Antrum ABOVE the GE Junction


Mesenteroaxial

Gastric Diverticulum:

The way they always ask this is by trying to get you to call it an
adrenal mass (it’s most commonly in the posterior fundus).

Gamesmanship: Find the normal adrenal.

Gastric Varices: This gets mentioned in the pancreas section, but I just want to hammer home
that test writers love to ask splenic vein thrombus causing isolated gastric varices. Some sneaky
ways they can ask this is by saying “pancreatic cancer” or “Pancreatitis” causes gastric varices.
Which is true.... because they are associated with splenic vein thrombus. So, just watch out for that.

275
- Upper Gl Surgeries -
Gastric Band: Done for patients who have spent years steadily gorging on the
"neglected food groups” such as the whipped cream group, the congealed group,
and the chocolate group. This diet, combined with prolonged ass in a horizontal
position, has left them with no alternative but to undergo a surgical procedure.

The application of an inflatable silicone band around the upper part of the stomach
creates a restrictive pouch. This limits the amount of french fries and ice­
cream that can be eaten at one time. The goal of the procedure is to
inspire the patient to “stop eating” when they feel full - an unnatural
concept to certain populations (mostly southern West Virginia).
-Stomal Stenosis is the most common complication - (band too tight).
Classic history is vomiting. Barium exam will show a dilated proximal
pouch with left over Wendy’s cheeseburger floating in the barium.
-Gastric Band Erosion and gastric leak can occur from pressure
related ischemia - classic look is a CT with the band within the lumen
of the stomach.
- Band Slippage (this is the most likely tested). The band should
be positioned around 2 o’clock (or have a phi angle between 4-58 ... it’s The long axis o f the band should
easier to just remember 5-50). If this angle is off (more flat) it infers the form an acute angle with the long
axis o f the upper spine
band has slipped - which can lead to obstruction.

Billroth 1: Billroth 2:
Pylorus is removed and the Partial gastrectomy, but Bilio-pancreatic
proximal stomach is sewed this time the stomach is (afferent) limb
directly to the duodenum. attached to the jejunum.
Done for Gastric CA, Done for Gastric CA, or
Pyloric Dysfiinction, or Ulcers (distal stomach).
Ulcers. Risks:
Less Post Op Gastritis
• Dumping syndrome
(relative to Billroth 2)
• Afferent loop Gastro-Jejunal
More Early Post Op syndrome (discussed (efferent) or
Complications on the next page) “feeding limb”
(relative to Billroth 2) • Increased risk of gastric CA
10-20 year after surgery
Roux-en-Y: Anastomosis 1: GastroJejunal
Stomach is divided to make a “pouch.” This gastric pouch is
attached to the jejunum. The excluded stomach attaches to the
Bilio-pancreatic
duodenum as per normal. The jejunum is attached to the other
(afferent) limb
jejunum to form the bottom of the Y.
Also done for patients who have spent years steadily gorging on
the "neglected food groups” such as the deep fried group, the
soda group, and the candy group. The procedure can also be Gastro-Jejunal
performed for gastric cancer as an alternative to Billroth if the (efferent) or
primary lesion has directly invaded the duodenum or head of the Anastomosis 2:
Roux”limb
pancreas. Jejuno Jejunal
Supposedly these have less reflux, and less risk of recurrent gastric CA.
They are at increased risk for leaks, gallstones, fistulas, and they have all those awful internal hernias.
276
- Upper Gl Surgeries - Complications -
Afferent Loop Syndrome: A potential complication post billroth 2 (Roux-en-Y and also
Whipple / partial pancreaticoduodenectomy). The idea is that something extrinsic (adhesions, internal
hernia, neoplasm) or intrinsic (scarring from radiation, edema from a marginal ulcer) obstructs the
upstream / afferent limb causing secretions, bile, and pancreas juice to build up. The presentation is
typically belly pain often with bilious vomiting (depending on the level of obstruction). The imaging
is going to show a fluid filled “U-shaped” loop of bowel adjacent to the pancreas. If you are lucky
they will show the common bile duct entering the loop. If you are really lucky you will see the bile
ducts dilated. The pressure from all this back up dilates the gallbladder, and can cause pancreatitis.

Normal Billroth 2 Afferent Limb Syndrome

Obstructed
B ile/
Pancreatic
Juice

Gastro-Jejunal
(efferent) or
Dilated
“feeding limb”
Afferent
Limb

Bilio-pancreatic
Adhesion
(afferent) limb
(or other cause of obstruction)

Dumping Syndrome: Insert Taco Bell Joke or a joke My M other-in-Laws Cooking:


about your wife (or mother in laws cooking). This is a
group of symptoms; diarrhea, nausea, feeling light­ My wife's mother is such a prude -
headed / tired after a meal, - caused by rapid gastric hates when I use profanity at the
emptying (seen classically with Billroth 2 and early in the dinner table. Really upsets her.
post op period after Roux-en Y). She gets all dramatic. Like if I
said the word “shit" she hates that.
This type of dumping is related to rapid transit of
Well... I'm sorry, but that’s what
undigested food from the stomach. Tc Gastric Emptying
her cooking tastes like.
study is an option to diagnose this. The therapy is
typically conversion of Billroth to Roux-en-Y (and
avoiding delicious carbs).

Cancer: With regard to these old peptic ulcer surgeries (Billroths), there is a 3-6 times increased
risk of getting adenocarcinoma in the gastric remnant (like 15 years after the surgery).

Bile Reflux Gastritis: Fold thickening and filling defects seen in the stomach after Billroth I or
II are likely the result of bile acid reflux.

Jejunogastric Intussusception: This is a rare complication in which the jejunum herniates


back into the stomach (usually the efferent limb) and can cause gastric obstruction. High mortality is
present with the acute form.

277
- Upper Gl Surgeries - Part 2 - The Wrath of Complications -
Leak: These usually occur early (within 10 days post op) and When is Barium ok ?
tend to occur at the anastomosis of the stomach and the small
Depends on who you ask - some
bowel (gastrojejunal anastomosis). The best way to look for these
people will say never on post op
is a water soluble oral contrast exam in Fluoro 1-2 days post op - studies. Most people will say it
either supine or supine left posterior oblique (75% of leaks will is ok after you use water soluble
drain to the left). Also get a scout image first - to avoid fuckery, contrast and get a negative. The
then administer the water soluble contrast (not barium dumbass!). use of Barium can increase
Any contrast outside the lumen of the stomach or bowel is positive sensitivity for subtle leaks. The
for a leak (duh). Sometime contrast pools in a way that makes this risk is barium peritonitis (which
a bit tricky — remember to look for folds. The stomach has folds. is bad) so if you do leak some
The small bowel has folds. The spaces outside the stomach and barium, you just don’t want it to
small bowel where contrast might leak... don’t have folds. Look be very much.
for folds. Also look for contrast in drains — that is a positive.

Trivia: There is a paper that says a heart rate > 120 is the single most reliable sign of perforation.

Gastro-Gastric Fistula: C-G Fistula


Oral contrast
Normal (white arrow)
This is seen in Roux-en-Y patients Roux-en-Y going into the
with the classic history of weight excluded stomach
gain years after the surgery (the
fistula allows them to double up on
the bacon cheeseburgers without
any discomfort).
Gets to Eat
The anastomotic breakdown is Normally Cheeseburgers
Excluded From But Gets Full
usually a chronic process, and
Cheeseburgers Quick.
often is not painful. topped with
onion rings.
Marginal Ulcer:

Ulcers at or near the anastomosis of


the stomach and the jejunum
(gastrojejunal) are referred to as
“marginal ulcers.”

The most common location is actually


just distal to the “G-J” anastomosis.

These things occur because the small


bowel is not used to getting exposed to
stomach acid - it don’t like it. Discrete ulcer (arrow) in jejunal Roux limb
near the gastrojejunal anastomosis
They are typically solitary and
variable in size. Notice I illustrated the typical gastric and
jejunal fold patterns.

Gamesmanship: Marginal ulcers are classically solitary. If there are multiple giant (2.5 cm or larger)
ulcers in this same region - you could think more about chronic jejunal ischemia

278
- Upper Gl Surgeries - Part 3 - The Search for Small Bowel Obstruction -
Small bowel obstruction (SBO) is a potential problem for really anyone who has had abdominal
surgery and Roux-en-Y is certainly not immune with the potential for adhesions, internal hernia,
anterior body wall hernias, and strictures. Before I get into the nuts and bolts o f these patterns I want
to discuss some basic principals o f obstruction.

Promethean Dialogue on Bowel Obstruction -


A discourse on Sim ple M echanical Obstruction v^' C losed Loop Obstruction

The problem can be thought o f like this:


Bowel segment
Spaghetti is blocked by a is obstructed at
mechanical force a single point
(adhesions, hernia, tumor, “Simple Mechanical’
That bowl of a small live animal possibly
spaghetti I ate can’t covered in psychoactive
make it from my substances, etc... Bowel segment
stomach to my is obstructed at
“Obstruction”
butthole. two or more
Spaghetti isn’t blocked by adjacent points 1
It is stuck... anything - the bowels just “Closed Loop”
aren’t moving normally
“Ileus”
It is important to understand that there are two main types o f true mechanical obstruction;
(I) simple and (2) closed loop. The causes are different. The outcomes are different. The treatments
are different. They are different - and you need to be able to distinguish between them (in real life,
and on multiple choice).

Simple M echanical: This is a scenario where you have a bowel loop that is obstructed at one point.
Let us take a look at the typical pathophysiology.

1) Your deviant sexual 2) The accumulation of gas 3) The distention is then


impulses have overwhelmed all upstream from the obstacle increased by retention of fluids
logic, and you decide to stick a (gerbil) acts as the primary resulting from both decreased
gerbil up your ass. initial cause o f intestinal absorption and exaggerated
distention. intestinal secretion.
This creates a single lead point
for simple mechanical This is why (a) vou see air fluid levels in simple mechanical
obstruction. obstruction, and (b) the therapy centers around decompression of
these additional secretions through the use of NG tubes.

279
Now, let us contrast simple mechanical obstruction with the more serious problem of a closed loop
obstruction. Closed loops have higher rates of ischemia and are typically treated surgically.

Closed Loop: Closed loop obstructions are defined by the presence of two (or more) points of
obstruction. Let us take a look at the typical pathophysiology

Note the 3 In this case


transition we are only
points dealing
with 2
transition
points
Small bowel
twists and
creates a
closed loop In more severe cases (those that twist up
mega tight) you may not see the
There are often two components:
upstream “supralesional” component - it
(A) the closed loop - which will distend quickly from hasn’t had time to develop before the
secretion and venous stasis. This won’t have gas patient presents to the CT scanner.
(unless its the colon).
(B) the supralesional component - which will distend These cases tend to have faster
slower than the closed (incarcerated) component. progression in ischemic symptoms.
This section mav have air-fluid levels
Radial Layout:
3 Beak Sign: If the twisted segment is long, you
may see vessels and loops converge
Classic morphology
towards a central point.
contains 3 “beaks”
or transition points. The first and second
“beaks” occur
proximately and are
adjacent - “double
dilation”

The third “beak” is Loops converge towards point of


more distal and has obstrucdon - pneumatosis (from
decompressed bowel ischemic necrosis) is commonly seen.
(star) downstream.

In addition to the presence of two


(or more) transition zones there are
some other helpful morphologic
features including the shape of the
loop. Depending on the length and
orientation of the loop involved it
is common to see either a;
C or U Shaped
Fluid Filled Distended Loop
These are best seen on the coronal -
as shown. Coronal view is your C Shaped U Shaped
friend for closed loop obstructions Fluid Filled Distended Loop Fluid Filled Distended Loop
280
Now let us pivot back to the discussion o f obstruction in the setting o f Roux-en-Y surgery.

There are three prim ary types o f SBO described after Roux-en-Y based on their location.

Obstruction of the Alimentary Obstruction of the


(Roux Limb) Biliopancreatic Limb Obstruction Below the J-J

This is a closed loop and high


risk for ischemia / perforation.

It is closed loop because the


stomach is sewed shut
proximally (so both ends are
closed).

Dilated: Gastric pouch and Dilated: Stomach and : Depending on severity -


Jejunal Roux limb Duodenum (these are normally everything may be dilated
excluded and decompressed) i above this level.

Decompressed: Stomach and Decompressed: Gastric pouch


duodenum and Jejunal Roux limb

Best seen on Barium Exam Best seen on CT Best seen on CT

Adhesions are the m ost com m on cause o f SBO in Roux-en-Y — assum ing it was an open
surgical procedure. Laparoscopic procedures are different. Laparoscopic procedures create
less adhesions and allow for more mobility. M obility is bad. M obility lets things tw ist and
slide through defects (closed loop obstructions).

Another factor that prom otes closed loop obstruction is the degree o f w eight loss.
More weight loss = less protective, space occupying m esenteric fat stops shit from twisting.

281
- Upper Gl Surgeries - Part 4 - The Voyage Home ( for internal hernias) -
There be whales here. A lso... internal hernias.

Ante-Colic vs Retro-Colic — this terminology describes the passage o f the Roux-Limb as either
in front o f (Ante) or behind (Retro) the transverse colon. If the surgeon chose to perform the
Retro-Colic subtype he/she would need to create a small defect in the transverse mesocolon to
pull the Roux limb up before connecting it to the excluded stomach. This defect will be a
potential source o f internal hernia.
Transverse
Mesocolon
Defect

There are 3 potential sites.

Defect in the M esenteric D efect at the Behind the Roux Limb


Transverse M esocolon Enteroenterostom y M esentery (Petersen)

R = “Roux” limb R = “Roux” limb

This is the hole the Roux-Loop This is the hole in the Beware of any hernia with a
normally passes through mesentery near the “J -J ” French or Latin name.
(if it’s done in the retrocolic anastomosis. Petersen sounds French to me.
position).
These can occur with antecolic
If it is done antecolic - then you or retrocolic forms.
don’t need to worry about this

282
Small Bowel

Introduction, the Lost Art, and Historical Perspective:

In the m odem age a legitimate working know ledge o f the barium fluoroscopic exam ination is
exceedingly rare. Ever since the late Cretaceous period, when the O ort cloud o f comets rained
down upon the dinosaurs o f the Yucatan Peninsula, legitim ate expertise in the field has been
essentially extinct. N ow all that rem ains o f this ancient practice is a collection o f pretenders
who spend their days in the swamps o f fuckery deploying the phrase “O f course I know what
I ’m looking a t ” as subterfuge.

Understanding that this barbaric ancient practice still represents “fair gam e” on the m odem
boards, I set out on a joum ey to re-discover the lost knowledge. This journey took me across
the globe, into the most prim itive com ers o f the w orld - as these were the only locations the
modality is still com m only practiced. I was searching for any ancient relic that I could use to
constm ct a high yield summary. M y jo u m ey was a treacherous one, necessitating significant
risk to gain favor with the natives. I barely survived several bouts o f m alaria and one (five)
bouts o f gonorrhea in m y quest to discover the ancient methods o f fluoroscopic interpretation.
Eventually, as is often the case in life - my persistence was rewarded. I was confldent I had
finally unearthed a legitim ate m ethod o f interpretation that I could use to com plete the section
on bowel fold pattems.

The m ethod I discovered is as ancient as the practice o f barium driven fluoroscopic imaging.
Originally described in the scrolls o f Xuthal (a city-state o f ancient Valusia) during the
Hyborian Age. The know ledge contained in the scrolls details the chronicle o f a low b o m son
o f a village blacksm ith w hom after the destm ction o f the A quilonian fortress o f Venarium,
was stm ck by w anderlust and began a joum ey. A ccording to his chronicle - this 20 year
joum ey saw him encounter skulking monsters, evil wizards, tavern wenches, and beautiful
princesses.

It was during his travels he leam ed m any things, including the ancient 3 step m ethod o f the
small bowel follow through - the details o f w hich I will now discuss.

283
STEP 1 - Evaluate the Folds

Thin (<3mm) \\ \ \ \ \ 1 1 / / •Mechanical Obstruction


Straight Folds \
\ \\ \\ ////// *
• Scleroderma
with Dilation

^«1 1 I f • Ischemia
Thick Straight Segmental
Folds >3mm Distribution
\\\ W / /m/m/ “
• Adjacent -Inflammation

Thick Straight Diffuse


i £ • Low Protein
\\ \ •Venous Congestion
Folds >3m m Distribution
W . Cirrhosis

- . 1 1 , •Crohns
Thick Folds Segmental
with Nodularity Distribution

A\\%\l ifi^
VI 11f / / 9 •
Whipples
Lymphoid Hyperplasia
Thick Folds
with Nodularity
Diffuse
Distribution
W V
^
(nodules 2-4mm, uniform sizes)
• Lymphoma
• Mets
(nodules > 4mm, variable sizes)
• Intestinal Lymphangiectasia

STEP 2a - Evaluate for Loop Separation with or without Tethering

/V * Ascites,
* Wall Thickening
Loop W ithout
(Crohns, Lymphoma),
Seperation Tethering
^n~ * Adenopathy
) / • M esenteric Tumors

Tethering looks like

Loop With
— ^ I ^ someone is pinching and
/■ pulling the loops towards
Seperation Tethering the displacing mass.

* Carcinoid

A pearl is that an extrinsic processes will spare the mucosa, intrinsic process will alter the mucosa.

284
STEP 2b - If Nodules are Present Evaluate the Distribution and Secondary
Findings To Help Narrow the Differential.

Diffuse micronodules in * W hipples (Tropheryma whipplei)


“Sand Like Nodules”
the jejunum.
Pseudo-W hipples (MAC Infection)

Uniform
Lymphoid Hyperplasia
2-4mm Nodules

Nodules o f Larger or
Cancer - think Mets (Melanoma)
Varying Sizes

Raised islands of mucosa separated by linear


streaks running perpendicular to the lumen of
the bowel.
These streaks represent ulceration.
“Cobblestoning”
This findings (especially when combined with
areas of stricture, and loop separation from fat
proliferation) should make you think
Crohns

STEP 3: Trademark Features:


After comparing this 3 main features - many multiple choice distractors can be eliminated. If necessary
specific features of each disease can be compared - with that discussion to follow.

Bowel is featxireless,
atrophic, and has fold
Ribbon Bowel thickening (ribbon-like).

Graft vs Host

285
STEP 3: Trademark Features - Continued:

Narrow separation
of normal folds
with mild bowel
dilation.
Hidebound Bowel

Scleroderm a

D ilated jejunal
loop with
complete loss o f
Moulage sign
jejunal folds -
(tube of wax)
opacified like a
“tube o f w ax”

Celiac

Jejunum loses folds


to look more like the
norm al Ileum, Ileum
gains folds (in the
Fold Reversal right lower
quadrant) to look
more like normal
Jejunum

Celiac

Thread Like Defect in


A Fucking Worm the Barium Column

As a general rule, avoid


drinking water that pigs
shit in. Ascaris Suum
(asshole demon worm)

286
- Selected Small Bowel Path -
The Target Sign: Clover Leaf Sign:

•Single Target: GIST, Primary This is an Aunt Minnie for Healed Peptic Ulcer of the
Adenocarcinoma, Lymphoma, Duodenal Bulb.
Ectopic Pancreatic Rest, Met
(Melanoma).

•Multiple Target: Lymphoma,


Met (Melanoma)

Whipples: Rare infection (Tropheryma Whipplei). Just like a stripper - it prefers white men in their
50s. The bug infiltrates the lamina propria with large macrophages infected by intracellular whipple
bacilli leading to marked swelling of intestinal villi and thickened irregular mucosal folds primarily in
duodenum and proximal jejunum. The buzzword is “sand like nodules” referring to diffuse
micronodules in the jejunum. Jejunal mucosal folds are thickened. This is another cause of low
density (near fat) enlarged lymph nodes.

Pseudo-Whipples: Also an infection - but this time MAI (instead of T Whipplei). This is seen in
AIDS patients with CD4<100. The imaging findings of nodules in the jejunum and retroperitoneal
nodes are similar to Whipples (hence the name). The distinction between the two is not done with
imaging but instead via an acid fast stain (MAC is positive).

Intestinal Lymphangiectasia: Lymphangiectasia results from obstrucfion to the flow of


lymph from the small intestine into the mesentery. This results in dilation of the intestinal and serosal
lymphatic channels. This can be primary from lymphatic hypoplasia, or secondary from obstruction of
the thoracic duct (or any place in between).

Graft vs Host: Buzzword = Ribbon bowel. It occurs in padents after bone


marrow transplant. It’s less common with modem anti-rejection drugs. Skin,
Liver, and GI tract get hit. Small bowel is usually the most severely affected.
Bowel is featureless, atrophic, and has fold thickening (ribbon-like).

SMA Syndrome: This is a compression of the portion


of the duodenum (D3) by the SMA (it pinches the duodenum
in the midline). Stomach and Proximal Duodenum (DI,D2)
will be dilated.

It is seen in patients who have recently lost a lot of weight


(loss of the normal mesenteric fat under the SMA).

287
Celiac Sprue: Small bowel malabsorption of gluten.

High yield points:

• Can cause malabsorption of iron, and lead to iron deficiency anemia.


• Associated with Idiopathic Pulmonary Hemosiderosis (Lane Hamilton Syndrome)
• Increased Risk of bowel wall lymphoma
• Gold standard is biopsy {surprisingly not barium)
• Dermatitis Herpetiformis - some skin thing (remember that from step 1)
• Tissue transglutaminase antibody (tTG), IgA class (blood screening - step 1 trivia)

Findings (CT / Barium)

• Fold Reversal is the Buzzword (Jejunum like Ileum, Ileum like Jejunum)

• Moulage Sign - dilated bowel with effaced folds (tube with wax poured in it)

• Cavitary Lymph Nodes (low density)

• Splenic Atrophy

Meckel’s Diverticulum I Diverticulitis: This is a congenital true diverticulum of the distal


ileum. A piece of total trivia is that it is a persistent piece of the omphalomesenteric duct. Step 1 style,
“rule of 2s” occurs in 2% of the population, has 2 types of heterotopic mucosa (gastric and pancreatic),
located 2 feet from the IC valve, it’s usually 2 inches long (and 2 cm in diameter), and usually has
symptoms before the child is 2. If it has gastric mucosa (the ones that bleed typically do) it will take up
Tc-Pertechnetate just like the stomach (hence the Meckel’s scan).

High Yield Meckel’s Trivia (Regarding Complications)

• Can get diverticulitis in the Meckels (mimic appendix)


GI Bleed from Gastric Mucosa (causes 30% of symptomatic cases)
Can be a lead point for intussusception (seen with inverted diverticulum)
Can cause Obstruction

Duodenal Inflammatory Disease: You can have fold thickening of the duodenum from
adjacent inflammatory processes of the pancreas or gallbladder. You can also have thickening and fistula
formation with Crohn’s (usually when the colon is the primary site). Primary duodenal Crohns can
happen, but is super rare. Chronic dialysis patients may get severely thickened duodenal folds which
can mimic the appearance of pancreatitis on barium.

Jejunal Diverticulosis: Less common than colonic diverticulosis, but does occur. They occur
along the mesenteric border. Important association is bacterial overgrowth and malabsorption. They
could show this with CT, but more likely will show it with barium (if they show it at all).

Gallstone Ileus: Not a true ileus, instead a mechanical obstruction secondary to the passage of a
gallstone in the lumen of the bowel. Gallstones access the bowel by eroding through the duodenum
(usually). As you can imagine, only elderly or weak patients (those unworthy of serving in the spartan
infantry) are susceptible to this erosion.

Classic multiple choice trivia includes the “Riglers Triad" ofpneumobilia, obstruction, and an
ectopic location o f a gallstone. The classic trick is to try and get you to say that free air - the
A “Riglers Sign" - is part of the Riglers Triad (it isn’t) — mumble to yourself “nice try assholes.”

288
- Trauma to the Bowel and the Bowel in Trauma -
Bowel Trauma:
Direct Signs
Penetrating Trauma (bullets, knives, light saber, the claws of a Spilled Oral Contrast
frantic gerbil desperately attempting to escape the rectum, etc...)
Active Mesenteric Bleed
can yield both direct and indirect signs of injury. It is important
to remember that the absence of direct signs does not exclude
Indirect Signs
injury to the bowel, and the presence of indirect signs does not
confirm it (although it does raise suspicion in the correct clinical Fat Stranding
context). This can be tested by asking you what is a direct or Fluid Layering Along the Bowel
indirect sign, or by showing you a picture containing the findings.

Gamesmanship: Watch the wording on the oral contrast. Yes... oral contrast is helpful when evaluating
possible bowel injury or post surgical leak. However, the type of contrast is important. Barium is always
Bad for any situation where it could possibly end up outside the lumen of the bowel ( “barium gone bad” -
is discussed later in the chapter). So, water soluble contrast is what you want to see. Use your
imagination in how this wording could be used to try and trick you.

Bowel in Trauma — “Shock Bowel” Shock Complex Features Include:


“Hypovolemic Shock Complex”
• Thickened Enhancing Bowel
This is typically seen with severe hypotension , although Loops (small bowel involved
anything that gives you low volume - (cardiac arrest, septic more than larger bowel)
shock, bacterial endocarditis, diabetic ketoacidosis, etc.. so on
• On Non-Contrast, bowel loops
and so forth) can also cause it.
may appear denser than the psoas

. Collapsed IVC
Diffuse Intense
• HYPO-enhancement of solid
Bowel Mucosal
Enhancement

Flat IVC (arrow) ifm


organs (liver and spleen)

• Bilateral delayed nephrograms


(persistent nephrograms)

• HYPER-enhancement of the
adrenals

This vs That: Bowel Trauma vs Shock Bowel


Bowel Trauma Shock Bowel

Focal Diffuse

Wall Thickening with Wall Thickening with


High Attenuation Blood in the Submucosa Near W ater Attenuation Edema

Mucosa enhances normally (or less than normal) M ucosa demonstrates intense enhancement

Secondary signs of injury (free air, leaked contrast, Other signs of shock
mesenteric hematoma, e tc ... e tc ... so on and so forth). (bright adrenals, flat IVC, etc...)

289
-Small Bowel Cancer-
Adenocarcinoma: Most common in the proximal small bowel (usually duodenum). Increased
incidence with celiac disease and regional enteritis. Focal circumferential bowel wall thickening in
proximal small bowel is characteristic on CT. The duodenal web does NOT increase the risk.

Gamesmanship: Adenocarcinoma is more likely to obstruct relative to lymphoma.

Lymphoma: It’s usually the non-Hodgkin


flavor. Patients with celiac, Crohns, AIDS, and
SLE are higher risk. It can look like anything
(infiltrative, polypoid, multiple nodules etc....).
Key piece of trivia is they usually do NOT
obstruct, even with massive circumferential
involvement. The Hodgkin subtype is more likely
to cause a desmoplastic reaction.

Trivia: Small bowel lymphoma favors the ileum.

Gamesmanship: Obstruction is rare (lymphoma is


a "soft" tumor).
Lymphoma - CH’cumferential Involvement

Carcinoid: This has an Aunt Minnie look with


a mass + desmoplastic stranding. “Starburst”
appearance of the mesenteric mass with
calcifications. This tumor most commonly occurs
in young adults. The primary tumor is often not
seen. That calcified crap you are seeing is the
desmoplastic reaction. Liver mets are often hyper
vascular. Step 1 style, you don’t get carcinoid
syndrome (flushing, diarrhea) until you met to the
liver. The most common primary location is the
distal ileum (older literature says appendix). The
appendix, has the best prognosis of all GI primary
sites. Systemic serotonin degrades the heart valves
(right sided), and classically causes tricuspid
regurgitation - more on this in the cardiac chapter.
MIBG or Octreotide scans can assist with diagnosis
Carcinoid - Sunburst Desmoplastic Rx
and staging - more on this in the nuclear medicine
chapter

Mets: This is usually melanoma (which hits the small bowel in 50% of fatal cases). You can also
get hematogenous seeding of the small bowel with breast, lung, and Kaposi sarcoma. Melanoma will
classically have multiple targets.

290
-Hernias-
Spigelian Hernia:

Also referred to as a
“lateral ventral hernia”
for the purpose o f fucking
with you.

The question is probably the Spigelian Hernia


location along the Semilunar at the lateral border o f the rectus
line ("“S ’’fo r “S ”) - which
is diagrammed here as a Above Arcuate Line
vertical dotted line Aponeuroses of the
(white arrows) at the lateral lateral body wall split
border of the rectus muscles. anterior and posterior
to the rectus.
They can occur anywhere along this line and still carry Below Arcuate Line
the name “Spigelian” - but most commonly they are at Aponeuroses of the
or slightly above the arcuate line (horizontal dotted line lateral body wall all
with the black arrows). move anterior to the
rectus.

Lumbar Hernia:

There are two types: superior and inferior - through their respective anatomic triangles.

Superior Lumbar (Grynfeltt-Lesshaft) Inferior Lumbar (Petit)

More common than inferior Less common than superior

Causes are congenital or acquired Causes are congenital


(post-surgery / trauma). or acquired (post-surgery
/ trauma).

Littre Hernia: Hernia with a M eckel D iverticulum in it.

Amyand Hernia: H ernia with the A ppendix in it.

291
Richter Hernia:

Contains only one wall o f bowel and


therefore does not obstruct.

These are actually at higher risk for strangulation.

Only one wall herniates

Obturator Hernia:
T
An old lady hernia. Often seen in patients
with increased intra-abdominal pressure
(Ascites, COPD - chronic cougher,
Pregnancy).
Bowel herniating between the
Usually asymptomatic - but can strangulate. obturator and the pectineus muscles

Can also create a characteristic paresthesia along the inner thigh down to the knee (from
com pression o f the obturator nerve). This has a nam e “H owship-Rom berg Sign ” - which sorta
sounds French to me, so it’s probably testable.

Femoral vs Direct Inguinal vs Indirect Inguinal Hernia:

Femoral Direct Inguinal Indirect Inguinal

Seen in old ladies (F > M ) Less common (than Indirect) More common (than Direct)

Inferior to the inferior


epigastric Medial and anterior to inferior Lateral and superior to inferior
Medial to the Common epigastric artery epigastric artery
Femoral Vein
Defect in Failure of
Hesselbach’s Triangle processus vaginalis to close

Below the Pubic Tubercle Above the Pubic Tubercle Above the Pubic Tubercle

Compression of the femoral


vein (which can look concave). No Compression of the
compresses the 1
This compression often causes
inguinal canal contents of the inguinal canal.
distal collateral venous
engorgement \ — lateral crescent sian

Narrow hernia neck is common NOT covered by internal Covered by internal spermatic
— obstruction is also common. spermatic fascia fascia

292
Internal Hernia: These can be sneaky. The most common manifestation is closed loop
obstruction (often with strangulation). There are 9 different subtypes, o f which I refiise to cover.
I will touch on the most common, and the general concept.

General Concept: This is a herniation o f viscera through the peritoneum or mesentery. The
herniation takes place through a known anatomic foramina or recess, one that has been created
post operatively or through the use o f magical practices banned by the Brotherhood o f Sorcerers
(goetia, necromancy, mutations on intelligent races e tc ..).

Paraduodenal hernias are the most com m on type o f internal hernias. They occur through
French sounded congenital defects (Lanzert and Waldeyer). The left one is way more common.

Gamesmanship: Classic history o f abdom inal pain after eating which im proves w hen the patient
massages their abdom en (gets better after a belly rub).

Left Sided (75%):


Occurs through a defect “The fossa o f L anzert” (L for Left)
in the descending colon m esentery / LUQ.
Classic Look: a sac-like cluster o f dilated small bowel loops
(arrow) in the left anterior pararenal space, behind the IMV
and the ascending left colic artery.

Right Sided (25%):


Occurs through a defect “The fossa o f W aldeyer” in the
ascending colon m esentery / RLQ. These are associated
with malrotation. The classic setting for right-sided
PDHs is non-rotated small bowel, with norm ally rotated
large bowel.

Classic Look: sac o f abnormal dilated small bow el loops


(black arrow) on the right, below the part o f the
duodenum, and behind the SM A (white arrow) which
gets displaced anteriorly.

There are a bunch o f other locations internal hernias can occur (e.g., lesser sac /foram en o f
winslow, pericecal, sigmoid m esocolon, small bow el mesentery, post-gastric bypass surgery etc)
- w hich you can google if y o u ’re a nerd but in general they all follow the same pattern - a sac-
like cluster o f dilated bowel loops w ith tw isted m esenteric vessels in an abnorm al location in
a patient with symptom s o f bowel obstruction.

293
-Ileus-
What is this “ileus ” ? Ileus is best thought about as paralyzed or “sleepy” bowel. You have this thing
called a migrating myoelectric complex, which is basically the electrical system of the bowel. It keeps
the poop train running. This thing works constantly to clear the stomach and small bowel even when you
aren’t eating. It flushes the residual food, secretions, dead cells, all that crap into the colon. The problem
occurs when the myoelectric process finds an excuse to fake outrage. It does this because it lacks any true
purpose in life and needs to virtue signal to obtain validation.
Numerous etiologies including abdominal inflammation, infection, Jordan Peterson, chemical /
pharmacological causes, trauma etc... etc.. so on and so forth — all these things can make the electrical
system “cancel” bowel peristalsis. The classic offender is surgery. Post operative ileus is very common
- and will be the most common clinical setting you will see this. Ileus comes in two flavors:

(1) Generalized; The large and small bowel are extensively air filled but not dilated. Some people will
say “ileus is when the large and small bowel look the same". This is in contradistinction to obstruction
which tends to be asymmetric - dilated small bowel, paucity of large bowel gas.

THIS vs THAT Obstruction: Ileus:

:'/•

..:

Supine: Dilated loops Upright: Many dilated Supine: Disorderly Upright: Fewer and/or
arranged in air-fluid levels in both loops scattered smaller (less dilated)
"stepladder" fashion. limbs of a given loop, throughout the air-fluid levels
Orderly. A bag of at different heights abdomen. A bag of scattered throughout
sausages. (candy canes). popcorn. the abdomen.

(2) Reactive (Focal) Ileus — Remember the migrating myoelectric complex is


easily offended by abdominal infections or inflammation. If the process is
local - the “canceling” of bowel movement will also be local. So - classic
examples would be an appendicitis (RLQ), pancreatitis (central abdomen),
diverticulitis (LLQ) - for the location of a “sentinel loop”.

A “sentinel loop” is a segment of bowel that it dilated —in response to some


adjacent infection or inflammatory process. In this case (black arrow) the
dilated loop is in the central abdomen - so you’d think pancreatitis.

Gasless Abdomen: This is a scenario where there is basically no bowel gas. It is non-specific and can be
the result of fluid filled bowel in the setting of diarrhea or the result of fluid filled bowel in the setting of
obstruction. Clinical context (pre-test probability) is the way to differentiate.

294
- Large Bowel / Rectum -

THIS THAT: Crohns Disease vs Ulcerative Colitis:

Crohns Disease: Typically seen in a young adult (15-30), but has a second smaller
peak later 60-70. Discontinuous involvem ent o f the entire GI tract (mouth -> asshole).
Stomach, usually involves antrum (R am ’s H orn Deformity). Duodenal involvem ent is rare,
and NEVER occurs w ithout antral involvement. Small bowel is involved 80% o f the time,
with the term inal ileum almost always involved (M arked Narrowing = String Sign). After
surgery the “neo-term inal ileum ” will frequently be involved. The colon involvem ent is
usually right sided, and often spares the rectum / sigmoid. Complications include fistulae,
abscess, gallstones, fatty liver, and sacroiliitis.

Crohns Buzzwords

Squaring o f the folds A n early m anifestation from obstructive lym phedema

Skip lesions D iscontinuous involvem ent o f the bowel

Separation o f the loops caused by infiltration o f the mesentery,


Proud loops
increase in m esenteric fat and enlarged lym ph nodes

Irregular appearance to bow el wall caused by longitudinal and


C obblestoning
transverse ulcers separated by areas o f edem a

Pseudopolyps Islands o f hyperplastic m ucosa

Filiform Post-inflam m atory polyps - long and worm-like

Found on anti-m esenteric side. From bulging area o f normal


Pseudodiverticula
wall opposite side o f scarring from disease

M arked narrow ing o f term inal ileum from a com bination o f


String-sign
edema, spasm, and fibrosis

Trivia: Inflammatory Bowel Diseases are Associated with an Increased Risk of Melanoma

295
Ulcerative Colitis:

Just like Crohns, it typically occurs in a “young adult” (age 15-40), with a second peak at 60-70.
Favors the male gender. It involves the rectum 95% of the time, and has retrograde progression.
Terminal ileum is involved 5-10% of the time via backwash ileitis (wide open appearance). It is
continuous and does not “skip” like Crohns. It is associated with Colon Cancer, Primary Sclerosing
Cholangitis, and Arthritis (similar to Ankylosing Spondyhtis).

On Barium, it is said that the colon is ahaustral, with a diffuse granular appearing mucosa.
“Lead Pipe" is the buzzword (shortened from fibrosis).

Here is a key clinical point: UC has an increased risk of cancer (probably higher than Crohns), and it
doesn’t classically have enlarged lymph nodes (like Crohns does), so if you see a big lymph node in
an UC patient (especially one with long standing disease), you have to think that it might be cancer.

More Common In :
Crohns
Crohns vs UC -String Sign at IC Valve

More m -
Common IN

Gallstones Crohns
Primary
Ulcerative
Sclerosing
Colits
Cholangitis
Hepatic
Crohns
Abscess
Ulcerative Coliits
Pancreatitis Crohns -Haustral Loss, Lead Pipe Appearance

THIS V5 THAT:
Crohns UC

Slightly less common in the USA SHghtly m ore com m on in the USA
Discontinuous “Skips” Continuous
Terminal Ileum - String Sign Rectum
Ileocecal Valve “ Stenosed” Ileocecal Valve “O pen”
M esenteric Fat Increased “c reep in g fa t” Perirectal fat Increased
Lymph nodes are usually enlarged Lymph nodes are N O T usually enlarged
Makes Fistulae D oesn’t Usually Make Fistulae

296
MISG Large Bowel Pathology
Toxic Megacolon: Ulcerative colitis, and to a lesser degree Crohns, is the primary cause.
C-Diff can also cause it. Gaseous dilation distends the transverse colon (on upright films), and the
right and left colon on supine films. Lack of haustra and pseudopolyps are also seen. Some people
say the presence of normal hausta excludes the diagnosis. Don’t do a barium enema because of the
risk of perforation. Another piece of trivia is that peritonitis can occur without perforation.

Behcets: Ulcers of the penis and mouth. Can also affect GI tract (and looks like Crohns) - most
commonly affects the ileocecal region. It is also a cause of pulm onary artery aneurysms (test
writers like to ask that).

Diverticulosis /Diverticulitis: Some trivia worth knowing is that diverticulosis actually


bleeds more than diverticulitis. Right-sided is less common (but is seen in young Asians). Fistula
formation is actually most common with diverticulitis, and can occur to anything around it (another
piece of bowel, the bladder, etc..).

Epiploic Appendagitis I Omental Infarct:

Epiploic appendages along the serosal surface of the colon


can torse, most commonly on the left. There is not
typically concentric bowel wall thickening (unlike
diverticulifis).

Omental infarction is typically a larger mass with a more


oval shape and central low density. It is more common on
the right (R O I- right omental infarct). Both entities are
self-limiting.

Appendicitis: The classic pathways are: obstruction (fecalith or reactive lymphoid tissue) ->
mucinous fluid builds up increasing pressure -> venous supply is compressed -> necrosis starts ->
wall breaks down -> bacteria get into wall -> inflammation causes vague pain (umbilicus) ->
inflamed appendix gets larger and touches parietal peritoneum (pain shifts to RLQ).

It occurs in an adolescent or young adult (or any age). The measurement of 6mm was originally
described with data from ultrasound compression, but people still generally use it for CT as well.
Secondary signs of inflammation are probably more reliable for CT.

Gamesmanship: In pregnancy MRI without contrast is the test of choice

Appendix Mucocele - Mucinous cystadcnomas are the


most common mucinous tumor of the appendix. They produce
mucin and can really dilate up and get big. They look similar to
cystadenocarcinomas and can perforate leading to
pseudomyxoma peritonei. On ultrasound the presence of an
“onion sign” - layering within a cystic mass - is a suggestive
feature of a mucocele.

297
e
Colonic Volvulus: Comes in several flavors:

• Sigmoid: Most common adult form. Seen in the nursing home patient (chronic
constipation is a predisposing factor). This is the “Grandma” volvulus. Buzzword is ■I
coffee bean sign (or inverted 3 sign). Another less common buzzword is Frimann
Dahl’s sign - which refers to 3 dense lines (arrows) converging towards the site of
obstruction (star). Points to the RUQ. Recurrence rate after decompression = 50%.

• Cecal: Seen in a younger person (20-40). Associated with people with a “long
mesentery.” More often points to the LUQ. Much less common than sigmoid.

• Cecal Bascule: Anterior folding of the cecum, without twisting. Lotta surgical
text books dispute this thing even being real (they think it’s a focal ileus). The
finding is supposedly dilation of the cecum in an ectopic position in the middle
abdomen, without a mesenteric twist. As the terminal ileum is not involved, don’t

Cecal Volvulus: (less common)


Old Grandma - who needs to poop Younger Person - with prior surgery, a mass, or a 3rd
Extends Towards RUQ Trimester Baby destined to wear the jeweled crown of
Haustra is Lost Aquilonia upon a troubled brow
Ascending and Transverse Extends Towards LUQ
Colon might be Dilated Haustra is maintained
Small Bowel is dilated

Colonic Pseudo-Obstruction (Colonic Ileus, Ogilvie Syndrome): Usually seen after serious
medical conditions and in nursing home patients. It can persist for years, or progress to bowel necrosis
and perforation. The classic look is marked diffuse dilation of the large bowel, without a discrete
transition point.

Diversion Colitis /Pouchitis: Bacterial overgrowth in a blind loop which gets no poop (surgery
that creates a blind loop without poop). Classic with pre-existing Inflammatory Bowl Disease.

Colitis Cystica: This cystic dilation of the mucous glands comes in two flavors:

(1) Superficial: The superficial kind consists of cysts that are small in the entire colon. It’s associated
with vitamin deficiencies and tropical sprue. Can also be seen in terminal leukemia, uremia, thyroid
toxicosis, and mercury poisoning.

(2) Deep /Profunda: These cysts may be large and are seen in the pelvic colon and rectum.

Rectal Cavernous Hemangioma: Obviously very rare. Just know it’s associated
^ ^1 % with a few syndromes; Klippel-Trenaunay-Weber, and Blue Rubber Bleb. They might show
i\. iV you a ton of phleboliths down there.

Gossypiboma: This isn’t really a GI pathology but it’s an abscess mimic. It’s a retained cotton
product or surgical sponge and it can elicit an inflammatory response.

298
- Infections -
Entamoeba Histolytica:

Parasite that causes bloody diarrhea. Can cause liver abscess, spleen abscess, or
even brain abscess. Within the colon it is one of the causes of toxic megacolon.
They are typically “flask-shaped ulcers” on endoscopy. With regard to barium, the
buzzword is '"coned cecum” referring to a change in the normal bulbous
appearance of the cecum, to that of a cone. Involves the cecum and ascending
colon (usually) and unlike many other GI infections, spares the term inal ileum. “Conecn
Cecum"
Colonic TB:

Typically involves the terminal ileum, and is Coned


another cause of the “coned cecum” appearance. Shaped
Cecum is pulled up (via scarring) out of the Cecum
RLQ. Causes both ulcers and areas of
narrowing. Narrow
TI
Two other signs:
Dilated [
(1) Fleischner sign -
Ileum
enlarged gaping IC valve, and narrow TI -
gives the impression of an inverted umbrella.

(2) Stierlin sign - sign of acute on chronic disease. Swollen lips of the IC valve allow for rapid
emptying of the contrast from the cecum but persistent barium in the TI

Colonic CMV: Infections that like


infections that
the Duodenum
like the Terminal
Seen in patients who are immunosuppressed. (and proximal small
Causes deep ulcerations - which can lead to Ileum
bowel)
perforation.
G iardia TB
Step 1 question: Cowdry Type A intranuclear
inclusion bodies (whatever the fuck that means). Strongyloides Yersinia

C-Diff:

Classically seen after antibiotic therapy, the toxin leads to a


super high WBC count. CT findings of the ''accordion sign”
with enhancing edematous mucosa or contrast trapped inside
mucosal folds is always described in review books and is fair
game for multiple choice. The barium findings include thumb
printing, ulceration, and irregularity. Of course it can cause
toxic megacolon as mentioned on the prior page.

Neutropenic Colitis (Typhlitis):

Infection limited to the cecum occurring in severe neutropenia.

299
- Colon Cancer -
Adenocarcinoma: Common cause of cancer
death (#2 overall). Cancers on the right tend to
bleed (present with bloody stools, anemia). Cancers
on the left tend to obstruct. Apple core is a
buzzword.
Gamesmanship: Large bowel intussusception in
adult = Malignancy

Gamesmanship: Colon likes to Met to the Liver.


Liver Mets will classically be T 1 Dark, T2 Mildly
Bright (“evil grey”), heterogenous non-progressive
in enhancement, sometimes target.

Squamous Cell C arcin om a- Apple Core Lesion - Cancer


Occasionally arises in the anus (think HPV).

-Rectal Cancer-
Nodes in Perirectal Fat (>5mm = ABnormal)
Trivia:
- Nearly always (98%) adenocarcinoma
- If the path says Squamous - the cause was HPV Muscularis P.
(use your imagination on how it got there). X
- Lower rectal cancer (0-5 cm from the anorectal angle), Subserosa/Serosa
has the highest recurrence rate.
- MRI is used to stage - and you really only need T2
weighted imaging - contrast doesn’t matter
- Stage T3 - called when tumor breaks out o f the rectum
and into the perirectal fat. This is the critical stage that
changes management (they w ill get chem o/rads prior to
surgery).

High Rectal Cancer Mesorectal Fascia


- Treated with Low
Anterior Resection (LAR
- These patients v Surgical Treatment is
maintain fecal classically determined by the
continence post position of the tumor relative
to the butthole.
Low Rectal Cancer
- Treated with Abdomino- Most people will use the
Perineal Resection (APR) number “5 cm” above the
- These poor bastards will anorectal angle to delineate
end up with colostomies “high” vs “low.”

Both surgical approaches are


performed with a total
mesorectal excision (tumor,
nodes, and blood supply).

300
- Colon Misc Massos -
Lipomas: The second most common tumor in the colon.

Adenoma - The most common benign tum or o f the colon and rectum. The villous adenoma
has the largest risk fo r malignancy.

McKittrick-Wheelock Syndrome:

This villous
adenoma classically
causes a mucous
diarrhea leading to
severe fluid and
electrolyte
depletion.

The clinical
scenario would be
something like “80
year old lady with
diarrhea,
hyponatremia,
hypokalemia,
hypochloremia...
and this” and they
show you a mass in
the rectum / bowel.

Anatomy Trivia:

At the level of S3 the


sigmoid colon sheds its RETRO-Peritoneal
oppressive mesentery Structures
and becomes the • Duodenum
rectum. (2nd and 3rd part)
• Pancreas
15 cms later (around (except tail)
the tip of the coccyx) • Colon
the rectum becomes the (ascending &
anus. The anus (prison descending)
wallet) is an excellent ' Rectum
place to hide things. (proximal 1/3)

Coccyx For the purpose of For the purpose of


multiple choice the muhiple choice the distal
proximal 1/3 of the 1/3 of the rectum is
rectum is entirely extraperitoneal.
"Get your fingers outta my ass-wallet"
retroperitoneal.
301
S E C T IO N 2:
P E R IT O N E A L C A V IT Y

The peritoneal cavity is the space between the various coverings (parietal and visceral peritoneum) in the
abdomen. It can be both a conduit for disease and a source of anatomic trivia.
Anatomic Trivia:
There are 2 primary “sacs” - greater and lesser. No surprise, the “greater”
is the big one. The “lesser” is basically the spot behind the stomach. It is
important to know that the “lesser sac” can be called the “omental bursa” -
for the purpose of flicking with you. The conduit between the greater and
lesser sac is referred to as the epiploic foramen (o f Winslow) - white arrow.
The “greater sac” can be divided again into two parts above (supracolic) and below (infracolic) the
transverse mesocolon. The supracolic and infracolic compartments are connected by the right and left
paracolic gutters along the lateral aspects of the ascending / descending colon. The right one is wider than
the left (the left one owns a sports car to compensate).
Male vs Female: The female peritoneum communicates with the extraperitoneal pelvis via the fallopian
tubes. This is why you’d never squirt barium into the vagina/uterus during a HSG (it would end up
causing a peritonitis and a lawsuit). The male peritoneum does not communicate.
Disease Conduit:
Disease may spread through the abdomen and pelvis by (1) the bloodstream,
(2) Lymphatic extension / (3) Direct invasion - with occurs from spread along the peritoneal ligaments
and mesenteries. Testable examples:
• Gastrohepatic Ligament - allows spread from the stomach, esophagus, and liver
• Gastrosplenic Ligament - allows spread from the stomach to the splenic hilum
• Duodenocolic Ligament - allows spread from the right colon to nodes around the duodenum / pancreas
(4) Intraperitoneal seeding - this occurs via the Pelvic Abscesses tend
natural flow of fluid. The natural intraperitoneal fluid to collect in the areas
mostly flows upward from the pelvis to the upper pulled by gravity in
abdomen via the right (more than left) paracolic the supine position
gutters. Anywhere the fluid tends to spend more time (sick people tend to
/ get caught up (shown by the stars) will be spend more time supine).
predisposed for serosal-based metastases in the
setting of peritoneal carcinomatosis. Additionally, it is said that pus and ascites
tends to flow away from the liver (the opposite
of normal flow). As a result the dependent
areas: right posterior subphrenic recess
(RPSA), anterior subheptic space (ASH),
posterior sub hepatic space / hepatorenal recess
/ Morrison (M) and pelvic cavity tend to be
involved.
ASH
Right ___
Paracolic ^
RPSA
Sigmoid
Lower Mesocolon
recess of
mesentery
Pouch of
Douglas

302
Pseudomyxoma Peritonei: This is a gelatinous ascites that results from either
(a) ruptured mucocele (usually appendix), or intraperitoneal spread of a mucinous neoplasm (ovary, colon,
appendix, and pancreas). It’s usually the appendix (least common is the pancreas).
The buzzword is “scalloped appearance of the liver.” Recurrent bowel obstructions are common.
Peritoneal Carcinomatosis: The main thing to know regarding peritoneal implants is that the
natural flow of ascites dictates the location of implants. This is why the retrovesical space is the most
common spot, since it’s the most dependent part of the peritoneal cavity.
Omental Seeding/Caking: The omental surface can get implanted by cancer and become thick.
The catch-phrase is “posterior displacement of the bowel from the anterior abdominal wall.”

Primary Peritoneal Mesothelioma: People think about mesothehoma involving the


pleura (and it does 75% of the time), but the other 25% of the time it involves the peritoneal
surface. The thing to know is that it occurs 30-40 years after the initial asbestosis exposure.

Cystic Peritoneal Mesothelioma: This is the even more rare benign mesothelioma, that is NOT
associated with prior asbestos exposure. It usually involves a women of child-bearing age (30s).
Barium Gone Bad: Rare but serious complications - two main flavors: (1) Peritonitis & (2) Intravasation.
Barium Peritonitis: This is why you use water soluble contrast anytime you are worried about leak. The
pathology is an attack of the peritoneal barium by the leukocytes which creates a monster inflammatory
reaction (often with massive ascites and sometimes hypovolemia and resulting shock). If no “real doctor” is
available, you should give IV fluids to reduce the risk of hypovolemic shock. The long term sequela of barium
peritonitis is the development of granulomas and adhesions (causing obstructions and an eventual lawsuit).
Barium Intravasation: This is super rare, but can happen. If barium ends up in the systemic circulation it
kills via pulmonary embolism about 50% of the time. Risk is increased in patients with inflammatory bowel or
diverticulitis (altered mucosa).
Mesenteric Mass: Bowel Association -► Duplication Cyst

Unilocular Surgical Site Lymphocele

Cystic Hx of Pancreatitis -► Pseudocyst


Multi-locular Lymphangioma

Many Lesions ► Mets, Lymphoma, Mesothelioma


Solid ^ Fat Containing ► Liposarcoma
Single Lesion Don’t y f Smooth _ GIST, Solitary Fibrous Tumor
> Have Margms
Fa. ^ Infiltrative ^
Ileum Location +
Arterial — Carcinoid
Enhancement
Margins
Desmoid,
Distorting Margins Delayed
Sclerosing
Hyper-Enhancement
Mesenteritis
Mesenteric Lymphoma: This is usually non-Hodgkin lymphoma, which supposedly involves the
mesentery 50% of the time. The buzzword is “sandwich sign.” The typical appearance is a lobulated
confluent soft tissue mass encasing the mesenteric vessels “sandwiching them.”
Misty Mesentery: The imaging appearance of increased mesenteric fat
density centered around the jejunal root, often with a few small nodes. It is
usually mesenteric panniculitis (but can also be lymphoma). Retroperitoneal
nodes (big ones) of the bulky soft tissue “sandwich” would make you think
lymphoma. Most people will get a 6 month follow up to establish stability.

303
S E C T IO N 3:
LIVER / B i l i a r y

Bare Area: The liver is covered by visceral peritoneum except at the porta hepatis, bare area, and
the gallbladder fossa. An injury to the “bare area” can result in a retroperitoneal bleed.

Couinaud System: Functional division of the liver into multiple segments. “Functional” is the
key word. Each segment will have its own biliary drainage, inflow, and outflow.
Falciform / L.

Hepatic Veins

Falciform
Above the g
Portal Vein

*• R. HV

A
Below the Extrapolated
Portal Vein Hepatic Vein
Locations
Portal Vein

Couinaud System Testable Trivia:


• Right Hepatic Vein Divides 7/8, and 6/5
• M iddle Hepatic Vein Divides 4a/8, 4b/5
• Left Hepatic Vein / Fissure for the Ligam entum Teres (falciform) divides 4a/2, 4b/3
• The Portal Vein Divides the Liver into U pper and Low er Segments
• The Caudate Lobe (Segm ent 1) is unique in that it drains directly to the IVC.

“C antlie’s Line” divides the liver into a


fu n ctio n a l left a n d right hepatic lobes.
This line runs from the IVC to the m iddle o f the
gallbladder fossa.

304
Caudate Lobe: The caudate lobe (segm ent 1) has a direct connection to the IVC through
it’s own hepatic veins, which do not com m unicate with the prim ary hepatic veins.
Additionally, the caudate is supplied by branches o f both the right and left portal veins - which
matters because the caudate may be spared or hypertrophied as the result o f various
pathologies such as Budd Chiari, e tc ... (as discussed later in the chapter).

Additional Anatomic Trivia:


Trivia: Along the same lines o f anatomy explaining pathology, the intra-hepatic course o f the
right portal vein is longer than the left, w hich is w hy it is m ore susceptible to fibrosis (this is
why the right liver shrinks, and the left liver grows in cirrhotic morphology).

Trivia: M ost com mon vascular variant = Replaced right hepatic (origin from the SMA)

Trivia: M ost com mon biliary variant = Right posterior segmental into the left hepatic duct.

Normal MRI Signal Characteristics: I like to think o f the spleen as a bag o f water/
blood (T2 bright, T1 dark). The pancreas is the “brightest T1 structure in the body” because it
has enzymes. The liver also has enzym es and is sim ilar to the pancreas (T1 Brighter, T2
darker), ju st not as bright as the pancreas

Fetal Circulation: The fetal circulation anatom y is high yield anatomic trivia.

^ Liver

Placenta Umbilical Vein ^ IVC


^ J f
^ Ductus Venosus
(remnant o f
ligamentum venosum)

305
Classic Ultrasound Anatomy:
There are 4 high yield looks, that are classically tested with regard to ultrasound anatomy. In years
past, these were said to have been shown by oral boards examiners (likely the same dinosaurs
writing the exam).

* .• / siyiA
Trivia:
Pancreas ... -'-i- LRV
should be
more
echogenic
than liver A

S = Splenic Vein, SM A = SMA, LRV = Left Renal Vein,


A= Aorta, VB = Vertebral Body, IVC = IVC

RHA - Right Hepatic Artery, CBD = Com m on Bile Duct, PV = Portal Vein

M ickey M ouse Sign: (1) Bile Duct, (2)


Hepatic Artery, (3) Portal Vein
Fat in the Falciform Ligam ent /
Ligam entum Teres

306
Promethean Dialogue on the Liver -
A discourse on the liver, cirrhosis, p o rta l HTN, a n d the developm ent o f H C C

My idea is that by leading with a discussion o f norm al physiology, and the changes that
occur with diffuse liver injury, that a lot o f the processes and changes that occur with
cirrhosis will make more sense (and be easier to remember). If you are in a rush to cram for
the test just skip this discussion and move on to the charts. I f you have m ore time, I think
understanding the physiology is worthwhile.

Hepatocyte injury can occur from a variety o f causes including viruses, alcohol, toxins
{alfatoxins i.e. peanut fungus), and nonalcoholic fatty liver disease. These injuries result in
increased liver cell turnover, to which the body reacts by forming regenerative nodules. The
formation o f regenerative nodules is an attempt by the liver not ju st to replace the dam aged
hepatocytes but also to com pensate for lost liver function. In addition to activation o f
hepatocytes, stellate cells living in the space o f Disse becom e active and proliferate
changing into a m yofibroblast -lik e cell that produces collagen. This collagen deposition
causes fibrosis.

The developm ent o f fibrosis first puts the squeeze on the


right portal vein (which usually has a longer intrahepatic Another consequence of the
course). This causes atrophy o f segments 6 and 7, and Longer Right Portal Vein Course
is that Hepatic Abscess (often
com pensatory hypertrophy o f the caudate, segments 2 from ascending hematogenous
and 3. Because o f these changes, some people will try sources) nearly always (75%+)
and use a caudate / right lobe ratio {C/RL >0.75 is 99% involves the right hepatic lobe.
specific) to call cirrhosis.

Morphologic Changes of Cirrhosis

Left Medial /
Right Anterior
Atrophy
^ Left Segment
Hypertrophy

Wide Perihilar Region


I

Caudate Hypertrophy
I
I
I

Wide Large Right c
Fissure Gallbladder
Fossa Caudate / Right Lobe Ratio
Increased with C irrhosis
“Posterior
Hepatic Notch”

307
All this squeezing can lead to portal hypertension. Portal hypertension is usually the result o f
increased hepatic resistance from pre-hepatic {portal vein thrombosis, tumor compression),
hepatic {cirrhosis, schistosom iasis) and post hepatic (Budd-Chiari) causes. Obviously, most
cases are hepatic with schistosomiasis being the m ost com m on cause w orld-wide, and EtOH
cirrhosis being the m ost com mon cause in the US. Once portal venous pressure exceeds
hepatic venous pressure by 6-8 m m Hg - portal hypertension has occurred (variceal bleeding
+ ascites around > 12). In reaction to this increased resistance offered by the liver, collaterals
will form to decompress the liver by carrying blood away from it. These tend to be
esophageal and gastric varices. As a point o f trivia, in pre-hepatic portal hypertension,
collaterals will form above the diaphragm and in the hepatogastric ligaments to bypass the
obstruction.

The liver has a dual blood supply (70% portal, 30% hepatic artery) with com pensatory
relationships between the two inflows; arterial flow increases as portal flow decreases. This
helps explain the relationship betw een these two vessels with regard to D oppler US. As
fibrosis leads to portal hypertension, velocity in the hepatic artery increases.

Another phenom enon related to this “hepatic arterial buffer response” is the TH A D . These
Transient H epatic Attenuation Differences are typically seen in the arterial / early portal phase
- NOT on the equilibrium / delayed phases (hence the w ord “transient”). The easiest w ay to
think about them is that they are focal “arterial buffer responses.” In other words, in that tiny
little spot right there the liver feels like there isn ’t enough portal flow, so it responses by
increasing the arterial flow. This can happen for several reasons;

• C irrh o sis: A bunch o f scar / fibrosis deforms the hepatic sinusoids com pressing the tiny
little portal veins (think about arteries and veins in the neck or groin - pressure will
compress the vein first) - people call this “shunting.” This is m ost typical in the subcapsular
region.

• C lot: Venous blood flow could be com prom ised from a clot in a portal vein branch (these
enhancement patterns are typically larger, wedge shaped, and extend towards the periphery).

• M ass (B9 or M alignant): This can occur from two prim ary mechanisms:
1 - You could have the direct mass effect from the mass sm ashing the veins.
2 - The tum or could be recruiting / up-regulating arterial flow (VEGF e tc ...).

• Abscess / In fectio n : Also probably a m ixed m echanism. Some direct mass effect, but also
some elem ent o f “hyperem ia” - causing a “ siphon effect.” For clarity we aren’t ju st talking
liver abscess here, cholecystitis can also have this region effect.

308
The perfusion related changes you see with cirrhosis aren’t ju st local, you can also have global
patterns. Since the fibrosis blockade takes place at the level o f the central lobular vein (into
sinusoids), flow remains adequate for the central zones o f the liver, but not for the peripheral
zones. The arterial response produces enhancem ent o f the peripheral subcapsular hepatic
parenchyma with relative hypodensity o f the central perihilar area. The consequent CT pattern
is referred to as the “central-peripheral” phenom enon.

Sometimes you will see reversal o f flow (hepatofugal - directed aw ay fro m the liver). As an
aside, apparently “fugal” is latin for “flee.” So the blood is fleeing the liver, or running away
from it. Reversed flow in the portal system is seen in cirrhosis between 5-25% o f the time.

Why does the p o rta l vein reverse flo w instead o fju s t clotting o ff in the setting o f high
resistance to inflow?

The answer has to do with the unique dual hepatic blood supply (70% P / 30% A). As
mentioned above, in cirrhosis, the principal area o f obstruction to blood flow is believed to be
in the outflow vessels (the hepatic venules and distal sinusoids). The outflow obstruction also
partially shits on the hepatic artery, causing increased resistance as well. So why doesn’t the
artery clot or reverse ? The difference is that the portal system can decom press through the
creation o f collaterals, and the artery cannot. So the artery does something else, it opens up
tiny little connections to the portal system. The enlargem ent o f these tiny com m unications has
been referred to as “parasitizing the portosystem ic decom pressive apparatus.” If the
resistance is high enough, hepatic artery inflow will be shunted into - and can precipitate
hepatofugal flow in the portal vein. So, in patients with hepatofugal flow in the main portal
vein or intrahepatic portal vein branches, the shunted blood comes from the hepatic artery.
This is the long answ er for w hy cirrhotic changes can lead to THADs, and why these cirrhosis
related THADs tend to be subcapsular.

With increased resistance in the liver to the portal circulation, you also start to have colonic
venous stasis (worse on the right). This can lead to ‘^Portal H ypertensive C ohpathy, ” v^\\\ch
is basically an edem atous bowel that mim ics colitis. W hy is it worse on the right? The short
answer is that collateral pathways develop more on the left (splenorenal shunt, short gastrics,
esophageal varices), and decom press that side. The trivia question is that it does resolve after
transplant. The same process can affect the stom ach “Portal H ypertensive G astropathy”
causing a thickened gastric w all on CT, as w ell as cause upper G I bleeding in the absence o f
varices.

309
Earlier I mentioned that hepatocytes react to injury by turning into regenerative nodules. This is
how multi-focal HCC starts. Regenerative nodules -> Dysplastic nodules (increased size and
cellularity) -> HCC. As this process takes place, the nodule changes from preferring to drink
portal blood to only wanting to drink arterial blood. This helps explain why HCC has arterial
enhancement and rapid washout. The transformation also follows a progression from T2 dark
(regenerative) -> T2 bright (HCC). A buzzword is ‘‘nodule within nodule” where a central bright
T2 nodule has a T2 dark border. This is concerning for transformation to HCC.

Regenerative Dysplastic HCC

Contains Iron Contains Fat, G lycoprotein

T1 Variable, T2 Dark T1 Bright, T2 Iso to Dark T2 Bright

Does NOT Enhance Usually Does N OT Enhance Does Enhance

Another thing that happens with hepatocarcinogenesis is the decrease in a thing called the OATP
bile uptake transporter. This is the transporter that moves biliary contrast agents (example =
Eovist) into the cells. It’s the reason normal liver cells look bright on the delayed phase when
using a hepatocyte specific agent. It’s also the reason FNHs look super bright on delayed images
as they are basically hypertrophied hepatocytes. As hepatocytes become cancer they lose
function in this transporter and become dark on the delayed phase. The exception (highly
testable) is the well differentiated HCC which retains OATP function and is therefore bright on
the 20 min delayed Eovist sequence.

There is one last concept that I wanted to “squeeze” in. The squeezing that causes portal
hypertension also squeezes out most benign liver lesions (cysts, hemangiomas). So, lesions in a
cirrhotic liver should be treated with more suspicion.

Hepatic Contrast Phase Timing & Window

Arterial Phase Portal Venous Phase “Liver Window”


- 25-30 seconds after injection - 70 seconds Center: 100 Width: 200

This ("Late Arterial”) is the most


critical phase for HCC evaluation.
You can recognize this timing as
contrast in the hepatic artery and
portal vein (but none in the
hepatic veins).

310
- MR Contrast - Hepatobiliary Considerations -
I want to clarify a few issues that can be confusing (and may also be testable).

How they work: Gadolinium (which is super toxic) is bound to some fype of chelation
agent to keep it from killing the patient. The shape and function of the chelation agent
determine the class and brand name. The paramagnetic qualities of gadolinium cause a local
shortening of the T1 relaxation time on neighboring molecules (remember short T1 time =
bright image).

Types of Agents: I want you to think about MRI contrast in two main flavors:
(1) Extracellular and (2) Hepatocyte Specific.

Extracellular: These are nonspecific agents that are best thought of as Iodine contrast for
CT. They stay outside the cell and are blood flow dependent (just like CT contrast). The
imaging features in lesions will be the same as CT - although the reason they look bright is
obviously different - CT contrast increases the density (attenuation), MR contrast shortens the
T1 time locally - which makes T1 brighter. The classic imaging set up is a late hepatic arterial
phase (15-30 seconds), portal venous phase (70 seconds), and a hepatic venous or interstitial
phase (90 seconds - 5 mins) - just like CT.

Classic Example o f a Non-Specific Extracellular Agent = Gd-DTPA (Magnevist)

Hepatocyte Specific: Certain chelates are excreted via the bile salt pathway. In other
words, they are taken up by normal hepatocytes and excreted into the bile. This gives you great
contrast between normal hepatocytes and things that aren’t normal hepatocytes (cancer). The 20
min delay is the imaging sequence that should give you a homogenous bright liver (dark holes
are things that don’t contain normal liver cells / couldn't drink the contrast). The problem is that
it’s pretfy non-specific with a handful of benign things still taking it up (classical example is
FNH), and at least one bad thing taking it up (well-differentiated HCC). Plus, a handful of
benign things won’t take it up (cysts, etc..). There are at least three good reasons to use this
kind of agent: (1) it’s great for proving an FNH is an FNH - as most lesions won’t hold onto the
Gd at 20 mins, (2) it’s great for looking for bile leaks, and (3) once you’ve established a
baseline MRI (characterized all those benign lesions) it’s excellent for picking up new mets
(findings black holes on a white background is easy).

Classic Example o f a Hepatocyte Specific Agent = Gd-EOB-DTPA (Eovist).

Is Eovist a pure Hepatocyte Specific Agent ? Nope - It also acts like a non-specific extracellular
agent early on (although less intense). About 55% is excreted into the bile - and gives a nice
intense look at 20 mins.

What about Gd-BOPTA (Multihance) ? This is mostly an extra-cellular agent, but has a small
amount (5%) of biliary excretion. The implication is that you can use Multihance to look for a
bile leak you just have to wait longer (45mins-3 hours) for the Gd to accumulate.

What about Manganese instead o f Gd ? This is the old school way to do biliary imaging. It
works the same as Gd - by causing T1 shortening.

311
- liver Masses -
Hemangioma: The most common benign liver neoplasm, favoring women 5:1. They may
enlarge with pregnancy. On US will be bright (unless it’s in a fatty liver, then can be relatively dark).
On US, flow can be seen in vessels adjacent to the lesion but NOT in the lesion. On CT and MRI
tends to match the aorta in signal and have “peripheral nodular discontinuous enhancement”. It is
critical that it be discontinuous and not a complete ring. Should totally fill in by 15 mins.
Trivia: A hemangioma can change its sonographic appearance during the course of a single
examination. No other hepatic lesion is known to do this.
Trivia: Tc-99m-labeled RBCs can be used to diagnose hemangiomas - bigger than 2 cms
Trivia: Need to core for biopsy, FNA does not get enough tissue (only blood)
Flasii Filling
Typical Hemangioma: Giant Hemangioma: Hemagioma:
• Term is used for
• Classically Hyperechoic (bright) on • Technically not a
Hemangiomas larger than
ultrasound hemangioma, but
5 cms
• Enhanced thru transmission is common historically referred
• Similar CT findings to regular to as one.
• NO Doppler flow inside the lesion itself hemangioma peripheral
nodular discontinuous • They are smaller
• Calcifications are extremely rare (< 2 cm) and will
enhancement pattern
• On CT: peripheral nodular discontinuous which fills in on delays (just demonstrate a rapid
enhancement - which fills in on later phases takes longer to fill in - cuz its “flash filling” on
giant) arterial phase itself
• They otherwise
Kasabach-Merritt syndrome, retain contrast and
with its classic potential remain isodense to
complication consumptive
Peripheral Discontinuous Progressive blood pool. Thev
coagulopathy is a testable do not washout the
Nodular Enhancement filling in on
later phases association
way an HCC
would.
Focal Nodular Hyperplasia (FNH): The second most common benign liver neoplasm.
Believed to start in utero as an AVM. It is NOT caused by oral contraceptive (birth control) pills. It is
composed of normal hepatocytes, abnormally arranged ducts, and Kupffer cells (reticuloendothelial cells).
May show spoke wheel on US Doppler. On CT, should be “homogenous” on arterial phase - similar in
brightness to the IVC fnot the aorta) Can be a “Stealth” lesion on MRI - T1 and T2 isointense. Can have
a central scar. Scar will demonstrate delayed enhancement (like scars do).
Biopsy Trivia: You have to hit the scar, otherwise path results will say normal
hepatocytes.
Medicine Trivia: Can develop after chemotherapy treatment with
oxaliplatin (chemo for bowel cancer)
Sulfur Colloid is always the multiple choice test question (reality is that it’s only hot
30-40%). Unlike hepatic adenomas, they are not related to the use of birth control pills,
although as a point of confusing trivia / possibly poor MCQ writing, birth control pills Enhancement
may promote their growth (controversial - some say they don’t). intensity
should be equal
to that of the
IVC (not the
Aorta)

T1 - Stealth T2 - Stealth Arterial Homogenous Enhancement


-Also has a Central Scar
312
Hepatic Adenoma: Usually a solitary lesion seen in a female on OCPs. Alternatively could be
seen in a man on anabolic steroids. When it’s multiple you should think about glycogen storage
disease (von Gierke) or liver adenomatosis. Being a big fat pig with diabetes is also a described risk
factor. Think of a big fat diabetic girl named Von Gierke (short hair, probably has a nose ring).
No imaging methods can reliably differentiate hepatic adenoma from hepatocellular carcinoma.
Rarely, they may degenerate into HCC after a long period of stability. They often regress after OCPs
are stopped. The propensity to bleed sometimes makes them a surgical lesion if they won’t regress.

Gamesmanship: Adenoma Subtype Trivia:


Drop Out with In & Out of Phase - used to show fat
“In fla m m a to ry ”
• Most common
Trivia:
• Highest bleed rate
Q: Most common location for hepatic adenoma (75%)
A: Right Lobe liver “H N F -1 a lp h a m u ta ted "
• Second most common
Management: • Multiple masses
• Stop the OCPs and re-image, they should get smaller.
“B e ta c a te n in -m u ta te d ”
• Smaller than 5cm, watch them.
• Least common
• Larger than 5cm they often resect because • Anabolic steroids
• (1) they can bleed and • Glycogen storage disease
• (2) they can rarely turn into cancer. • Familial adenomatous polyposis

HCC: Occurs typically in the setting of cirrhosis and chronic liver disease; Hep B, Hep C,
hemochromatosis, glycogen storage disease. Alpha 1 antitrypsin. AFP elevated in 80-95%. Will often
invade the portal vein, although invasion of the hepatic vein is considered a more “specific finding.”
“Doubling Time” - the classic Multiple Choice Question. This is actually incredibly stupid to ask
because there are 3 described patterns of growth (slow, fast, and medium). To make it an even worse
question, different papers say different stuff. Some say; Short is 150 days, Medium to 150-300, and
Long is >300. I guess the answer is 300 - because it’s in the middle. Others define medium at
around 100 days. A paper in Radiology {May 2008 Radiology, 247, 311-330) says 18-605 days. The
real answer would be to say follow up in 3-4.5 months.
Other Random Trivia: HCCs like to explode and cause spontaneous hepatic bleeds.

Fibrolamellar Subtype of HCC: This is typically seen in a younger patients (<35) without
cirrhosis and a normal AFP. The buzzword is central scar. The scar is similar to the one seen in FNH
with a few differences. This scar does NOT enhance, and is T2 dark (usually). As a point of trivia,
this tumor is Gallium avid. This tumor calcifies more often than conventional HCC.

THIS vs THAT: THIS vs THAT:


Classic HCC vs Fibrolamellar HCC Central Scars of FNH and Fibrolamellar HCC

HCC FL HCC FNH FLHCC

Cirrhosis No Cirrhosis T2 Bright T2 Dark (usually)

Older (50s-60s) Young (30s)


Enhances on Delays Does NOT enhance
Calcifies
Rarely Calcifies
Sometimes Mass is Sulfur Colloid
Mass is Gallium Avid
Avid (sometimes)
Elevated AFP Normal AFP

313
A
Cholangiocarcinoma: Where HCC is a cancer o f Gamesmanship - They could
tell you the dude has ulcerative
the hepatocyte, cholangiocarcinoma is a cancer o f the bile
colitis, as a way to infer that he
duct. Cholangiocarcinoma believes in nothing Lebowski.
also has PSC.
It flicks you up, it takes the money (prognosis is poor).
Who gets it? The most classic multiple choice scenario
Buzzword = “Painless
would be an 80 year old man, with primary sclerosing
Jaundice.” (just like
cholangitis - PSC (main risk fa cto r in the West), recurrent
^ pancreatic head CA)
pyogenic “ o rien tal” cholangitis (main risk factor in the
East), Caroli Disease, Hepatitis, HIV, history o f
cholangitis, and fucking Liver Worms (Clonorchis).
Another risk factor is undergoing a semi-voluntary
THIS vs THAT:
cerebral angiogram performed by a Nazi with a cleft
asshole (in 1930s Germany, Thorotrast was the HCC = Invades the Portal Vein
preferred angiographic contrast agent). Cholangiocarcinoma = Encases the
Portal Vein
What does it look like? It is variable and the
described subtypes overlap. The easiest way to
conceptualize this thing is as a scar generating cancer.
F ibrosis (scar) is the main thing you are seeing - Classic Features:
either primarily as a mass that enhances on delayed
imaging (just like scar in the heart), or secondarily Delayed Enhancement
Peripheral Biliary Dilation
through the desmoplastic pulling o f the scar (example
Liver Capsular Retraction
capsular retraction and ductal dilation). The dilation
NO tumor capsule
o f ducts is most likely to be shown as unilateral and
peripheral, although if the lesion is central the entire
system can obstruct.

Klatskin Tumor: Cholangiocarcinoma that occurs at the bifurcation o f the right and left
hepatic ducts. It’s usually small but still causes biliary obstruction (“shouldering / abrupt
tapering” on MRCP). These things are mean as cat shit. It is a “named” subtype, so that
increases the likelihood o f it showing up on a multiple choice exam.
Staging Pearls: There are like 3-4 major systems, each one has rules on the subtype
(intrahepatic, extrahepatic, hilar/Klastskin), and honestly resectability is incredibly variable
depending on how much o f a gun slinger the surgeons at your institution are. This
combination o f factors makes specifics nearly untestable (under “fair” conditions). Having
said that, here are some potentially testable pearls:
• Proximal extent o f involvement is a key factor for surgical candidacy (more = bad).
• Atrophy o f a lobe implies biliary +/- vascular involvement o f that lobe (imaging often
underestimates disease burden).
• Typically combinations o f bilateral involvement (veins on the right, ducts on the left - vice
versa, etc.. e tc ... etc... ) is bad news.

This Could Be Useful

Cholangio: CEA ^ CA 19-9

Pancreatic CA: CEA CA 19-9

Colon CA: CEA^ CA19-9

314
Hepatic Angiosarcoma: This used to be the go to for thorotrast questions. Even though
everyone who got thorotrast died 30 years ago, a few dinosaurs writing multiple choice test questions
still might ask it. Hepatic Angiosarcoma is very rare, although technically the most common primary
sarcoma of the liver. It is associated with toxic exposure - arsenic use (latent period is about 25years),
Polyvinyl chloride exposure, Radiation, and yes... thorotrast. Additional trivia, is that you can see it
in Hemochromatosis andN Fl patients.

It’s usually multifocal, and has a propensity to bleed.

Biliary Cystadenoma Uncommon benign


cystic neoplasm of the liver. Usually seen in
middle aged women. Can sometimes present with
pain, or even jaundice. They can be unilocular or
multilocular and there are no reliable methods for
distinguishing from biliary cystadenocarcinoma
(which is unfortunate). Having said that - the
primary distinguishing characteristic of the
cystadenocarcinoma is a solid and nodular Cystadenocarcinoma: the primary distinguishing is
enhancing component. a solid and nodular enhancing component (arrow).

Mets to the Liver: This is way more common than a primary liver cancer (like 20-40x more
common). If you see mets in the liver, first think colon. Calcified mets are usually the result of a
mucinous neoplasm (colon, ovary, pancreas).

With regard to ultrasound: Hyperechoic mets are often hypervascular (renal, melanoma, carcinoid,
choriocarcinoma, thyroid, islet cell). Hypoechoic mets are often hypovascular (colon, lung, pancreas)
- this is the more common look. A halo of low density (target) is a classic description.

With regard to CT: Think low


density masses with a continuous rim
of enhancement (not discontinuous
like hemangioma). A background
fatty liver may result in the lesions
looking bright (higher density).

“Too Small To Characterize ” - even


in the setting of breast cancer (with no
definite hepatic mets) tiny
hypodensities have famously been Low Density Lesions Higher Density Lesions
shown to be benign 90-95% of the time. Normal Liver Background Fatty Liver Background

Lymphoma: Hodgkins lymphoma involves the liver 60% of the time (Non Hodgkins is around
50%), and may be hypoechoic.

Kaposi Sarcoma: Seen in patients with AIDS. Causes diffuse periportal hypoechoic infiltration.
Looks similar to biliary duct dilation.

315
Ultrasound Rapid Review

Anechoic with well defined, thin walls.


Single or multiple - less than 2 cm.

Things to suggest maybe it's not a cyst and


Cysts instead possibly a parasite (i.e. Hydatid)?
• Bigger than 2 cm.
• Presence of membranes.
• Abundant sediment in cysts.

Most common liver tumor.


Classic; well defined, homogeneously
echogenic. No intemal doppler flow (because it
is a slow flow lesion).
Hemangiom a Will see posterior enhancement (because like a
cyst, the blood is just slow flowing, so the signal
passes through it quickly.)
Things to suggest maybe it's not a hemangioma:
• Intemal flow

Young woman - most classic


Seen after chemotherapy treatment with
oxaliplatin (chemotherapy for bowel cancer)
Can occur with other lesions
Focal Nodular
(most commonly hemangioma)
Hyperplasia
Classic; Isoechoic to the liver and very difficult
to see - so they must either show the lesion;
• Bulging the liver,
• Presence of a central scar, or
• Classic spoke wheel appearance on doppler.

Development is induced by intake of anabolic


-4 0 %
hormones and oral contraceptives.
Hypoechoic
Association; Glycogen storage diseases
Hepatic Association: Being a fat ass with diabetes
Classic ; Round mass with well defined borders.
Adenoma Hvpoechoic halo of fatty sparing.
** Appearance is highly variably because of
hemorrhage, fat, calcification, necrosis ~ 30%
Hyperechoic
Classic Doppler: perilesional increased flow
(although this is also variable)

- Basically anything with a shit ton o f intemal


flow or peripheral flow on Doppler (other than
your typical spoke wheel pattern o f FNH) you
Hepatocellular gotta be worried.
Carcinoma - A thin peripheral halo of hypoechogenicity can
represent either focal fatty sparing (in the case of
an Adenoma) or... a fibrous capsule (target) as is
often seen in HCC - so this should also be Note the Cirrhotic
conceming. Liver Contour
(that is a huge hint)
Multiple lesions -
Mets Especially if there is a halo (target sign).

316
Benign Liver iViasses
Ultrasound CT MR Trivia

Kasabach-
Hyperechoic Merritt; the
Peripheral
with sequestration
Nodular Rare in
Hemangioma increased T2 Bright of platelets
Discontinuous Cirrhotics
through from giant
Enhancement
transmission cavernous
hemangioma

“Stealth Bright on
Homogenous
Lesion - Delayed
FNH Spoke Wheel Arterial Central Scar
Iso on T1 Eovist (Gd-
Enhancement
and T2” EOB-DTPA)

Fat OCP use,


Hepatic Containing Glycogen Can explode
Variable Variable
Adenoma on In/Out Storage and bleed
Phase Disease

Unlike renal
Hepatic T1/T2 Tuberous
Hyperechoic Gross Fat AML, 50%
Angiomyolipoma Bright Sclerosis
don’t have fat

Incidental Liver Lesion on CT - Algorithm

Size < 1cm Size > 1cm

\
Low Risk High Risk Benign Features Suspicious Features Flash Filing
(Not a Drunk, (Drinking, • Sharp margins • Blurry margins
IVD rug User, Whoring, • Density < 20 HU • Thick septa, nodular
or someone with IV Drug User, • Characteristic or heterogeneous 1.5 cm > L5 cm
Cancer) with Cancer) Benign • Density > 20 HU

I I
Enhancement on portal venous—
(i.e. discontinuous) phase imaging
MRI
No Follow up MRI
I /
No Follow up MRI Low Risk High Risk
(Not a Drunk, (Drinking,
IV Drug User, Whoring,
or someone IV Drug User,
with Cancer) with Cancer)

I \
No Follow up MRI

317
-Congenital Liver-
Cystic Kidney Disease (both AD and AR): Patient’s with AD polycystic kidney disease
will also have cysts in the liver. This is in contrast to the AR form in which the liver tends to have
fibrosis.
Hereditary Hemorrhagic Telangiectasia (Osier-Weber-Rendu) Autosomal
dominant disorder characterized by multiple AVMs in the liver and lungs. It leads to cirrhosis and a
massively dilated hepatic artery.
Trivia: The lung AVMs set you up for brain abscess.

-Liver infections-
Infection of the liver can be thought of as either viral, abscess (pyogenic or amoebic), fungal,
parasitic, or granulomatous. As previously mentioned, the long intra-hepatic course of the right
portal vein results in most hematogenous infection favoring the right hepatic lobe.
Viral: Hepatitis which is chronic in B and C, and acute with the rest. A point of trivia is that HCC
in the setting of hepatitis can occur in the acute form of Hep B (as well as chronic). Obviously,
chronic hep C increases risk for HCC. On ultrasound the “starry sky” appearance can be seen.
Although, this is non-specific and basically just the result of liver edema making the fat surrounding
the portal triads look brighter than normal.
Pyogenic: These can mimic cysts. For the purpose
infection Buzzwords
of multiple choice, a single abscess is Klebsiella, and
multiple are E. Coli. The presence of gas is highly
suggestive of pyogenic abscess. Viral Hepatitis Starry Sky (US)

“Double Target” sign with central low density, rim Pyogenic Double Target (CT)
enhancement, surrounded by more low density is the Abscess
classic sign of a liver abscess on CT. Candida Bull’s Eye (US)
Hydatic Cyst (Echinococcus) favors the liver (2nd most
Amoebic “Extra Hepatic
common is the lung), and is often asymptomatic. If it is Abscess Extension”
symptomatic the classic history is “enlarging liver”. It
has several classic imaging patterns depending on the
age / stage of disease including: the water lily, sand Hydatid
storm, and the daughter cyst. Treatment is surgical Disease
(usually). Sand Daughter Water
Storm Cysts Lilly
Amebic Abscess: A special situation (potentially testable)
is the amebic abscess in the left lobe. Those needs to be Schistosomiasis Tortoise Shell
emergently drained (they can rupture into the
pericardium).

Fitz-Hugh-Curtis Syndrome - This syndrome is seen exclusively


in women of questionable moral standard (“free spirits”). It manifests on
multiple choice exams in the setting of known pelvic inflammatory
disease (Gonococcal salpingitis and/or Chlamydia infection), with right
upper quadrant pain.

The classic imaging features are enhancement of the anterior liver


capsule, perihepatic ascites, and peritoneal septations. They could show
you a tubo-ovarian abscess in the pelvis to cue you in on the
excessively promiscuous behaviors this young lady has been engaging in.

318
- Diffuse Liver Processes -
F s t t y L iV G r : Very common in America. Can be focal (next to gallbladder or ligamentum teres),
can be diffuse, or can be diffuse with sparing. You can call it a few different ways.
For CT: If it’s a non-contrast study, 40 HU is a slam dunk. If it’s contrasted, some people say you can
NEVER call it. Others say it’s ok if (a) it’s a good portal venous phase (b) the HU is less than 100,
and (c) it’s 25 H.U. less than the spleen.
On US: If the liver is brighter than the right kidney you can call it. Hepatosteatosis is a fat liver.
NASH (hepatitis from a fat liver) has abnormal LFTs.
On MRI: Two standard deviation difference between in and out of phase imaging. Remember the drop
out is on the out of phase images (india ink ones - done at T.E 2.2 ms - assuming 1.5T).
Fuckery: This signal drop out assumes there is more water than fat in the liver. As such, the degree
of signal loss is maximum when the fat infiltration is 50% (exactly 1:1 signal loss). When the
percentage of fat grows larger than 50% you will actually see a less significant signal loss on out of
phase imaging, relative to that maximum 50%.
What causes it? McDonalds, Burger King, and Taco Bell. Additional causes include chemotherapv
(breast cancer), steroids, cystic fibrosis.

Hemochromatosis: Iron overload. They can show this three main ways:

(1) The liver and spleen being T1 and T2 dark.


(2) They show a single T2. The spleen is absent and the liver is darker (lower signal) than muscle.
On T2 a normal liver should always have a signal higher than muscle. In the setting of chronic
hemolytic anemia (a condition requiring multiple blood transfusions - that can cause
hemochromatosis) the spleen is often removed surgically.
(3) The second (and more likely) way this will be shown is in and out o f phase changes the opposite
o f those seen in hepatic steatosis. Low signal on in phase, and high signal on out of phase.
(“Iron on In-phase”)

Watch out now — this is the opposite of the fat drop out
**FAT - Drop out on OUT of phase (india ink one - T.E. 2.2 ms) -1.5 T
**IRON - Drop out on IN phase (non india ink one - T.E. 4.4 ms) - 1.5 T

The second main piece of trivia is to tell


primary V5 secondary. Hemochromatosis

Primary is the inherited type, caused by more GI uptake, with Primary Secondary
resulting iron overload. The key point is the pancreas is
involved and the spleen is spared. Genetic - Acquired -
increased chronic illness,
Secondary is the result of either chronic inflammation or absorption and multiple
multiple transfusions. The body reacts by trying the “Eat the transfusions
Iron,” with the reticuloendothelial system. The key point is
the pancreas is spared and the spleen is not. Liver, Pancreas Liver, Spleen

“Primary = Pancreas” , “Secondary = Spleen” Heart, Thyroid,


Pituitary

319
Budd Chiari Syndrome: Classic multiple choice scenario is a pregnant woman, but can
occur in any situation where you are hypercoagulable {most common cause is idiopathic). The
result o f hepatic vein thrombosis.

The characteristic findings o f Budd-Chiari syndrome include hepatic venous outflow


obstruction, intrahepatic and systemic collateral veins, and large regenerative (hyperplastic)
nodules in a dysmorphic liver. The caudate lobe is often massively enlarged (spared from
separate drainage into the IVC). In the acute phase, the liver will show the classic ‘‘flip-flop
pattern ” on portal phase with low attenuation centrally, and high peripherally. The liver has
been described as “nutmeg” with an inhomogeneous mottled appearance, and delayed
enhancement o f the periphery o f the liver.

Who gets a “Nutmeg L iver" ??? Arterial: Central Enhancement


• Budd Chiari Peripheral Minimal
• Hepatic Veno-occlusive disease
• Right Heart Failure (Hepatic Congestion) Portal V: Central Washout
• Constrictive Pericarditis Peripheral Enhancement

Regenerative (hyperplastic) nodules can be difficult to distinguish from multifocal


hepatocellular carcinoma. They are bright on T1 and typically dark or iso on T2. M ultiple big
(>10cm) and small (<4cm) nodules in the setting o f Budd-Chiari suggest a benign process. T2
dark also helps (HCC is usually T2 bright).

Presentation can be acute or chronic. Acute from thrombus into the hepatic vein or IVC. These
guys will present with rap id onset ascites. Chronic from fibrosis o f the intrahepatic veins,
presumably from inflammation.

Who gets massive caudate lobe hypertrophy???


• Budd Chiari
• Primary Sclerosing Cholangitis
• Primary Biliary Cirrhosis

Hepatic Veno-occlusive Disease: This is a form o f Budd Chiari that occurs from
occlusion o f the small hepatic venules. It is endemic in Jamaica (from Alkaloid bush tea). In
the US it’s typically the result o f XRT and chemotherapy. The main hepatic veins and IVC will
be patent, but portal waveforms will be abnomial (slow, reversed, or to-and fro).

Passive Congestion: Passive hepatic congestion is caused by stasis o f blood within the
liver due to compromise o f hepatic drainage. It is a common complication o f congesfive heart
failure and constrictive pericarditis. It is essentially the result o f elevated CVP transmitted from
the right atrium to the hepatic veins.

Findings include:
Refluxed contrast into the hepatic veins
Increased portal venous pulsatility
Nutmeg liver

320
-Misc Liver Conditions-
Portal Vein Throm bosis: Occurs in hypercoaguable states (cancer, dehydration,
e tc ...). Can lead to cavernous transform ation, with the developm ent o f a bunch o f
serpiginous vessels in the porta hepatis w hich may reconstitute the right and left portal
veins. This takes like 12 months to happen {it proves p o rta l vein is chronically occluded).

Pseudo Cirrhosis: Treated breast cancer mets to the liver can cause contour changes
that mimic cirrhosis. Specifically, m ultifocal liver retraction and enlargem ent o f the caudate
has been described. W hy this is specific for breast cancer is not currently known, as other
mets to the liver don’t produce this reaction.

Cryptogenic Cirrhosis: Essentially cirrhosis o f unknow n cause. M ost o f these cases


are probably the result o f nonalcoholic fatty liver disease.

Liver Transplant: The liver has great ability to regenerate and may double in size in as
little as 3 weeks, making it ideal for partial donation. Hepatitis C is the m ost com m on
disease requiring transplantation (followed by EtOH liver disease and cryptogenic
cirrhosis). In adults, right lobes (segments 5-8) are m ost com m only implants. This is the
opposite o f pediatric transplants, which usually donates segments 2-3. The m odem surgery
has four connections (IVC, artery, portal vein, CBD).

Contraindications include, extrahepatic malignancy, advanced cardiac disease, advanced


pulm onary disease, or active substance abuse. Portal HTN is NOT a true contraindication
although it does increase the difficulty o f the surgery and increase mortality.

Norm al Transplant US Syndrom e o f Im pending Throm bosis


• Normal Doppler should have a 3-10 days post transplant
RAPID systolic upstroke (1st) Initial N orm al W aveform
• Diastolic -> Systolic in less (2nd) N o diastolic flow
than 80msec (0.08 seconds) (3rd) Dampening Systolic flow
Resistive Index is Norm ally Tardus Parvus
between 0.5 - 0.7 RI < 0.5
Hepatic Artery Peak Velocity (4‘h) Loss o f Hepatic W aveform
should be < 200 cm/sec

As mentioned before, the normal liver gets 70% blood flow from the portal vein, m aking it
the key player. In the transplanted liver, the hepatic artery is the king and is the prim ary
source o f blood flow for the bile ducts (which undergo necrosis with hepatic artery failure).
Hepatic artery throm bosis comes in two flavors: early (< 1 5 days), and later (years). The
late form is associated with chronic rejection and sepsis.

Trivia: Tardus Parvus is m ore likely secondary to stenosis than throm bosis

321
-Biliary-
THIS vs THAT: Portal Venous Gas vs Pneumobilia — These are the two patterns of
branching air in the hver. The classic way to distinguish between the two is Central (Pneumobilia) vs
Peripheral (PVG). The way to remember this is that bile is draining out of the liver into the bowel - so
it is flowing towards the porta-hepatis and should be central. Portal blood, on the other hand, is being
pumped into the liver - so it will be traveling towards the periphery. The potential trivia question is
how peripheral is peripheral — and that is 2cm. Within 2cm of the liver capsule = portal venous gas.
Other things to remember - gas in the bile is usually related to a prior procedure (anything that fucks
with the sphincter of Oddi). Gas in the portal system can be from lots of stuff (benign things like
COPD or bad things - the most classic being bowel necrosis -look for pneumatosis*).

Jaundice: You always think about common duct stone, but the most common etiology is actually
from a benign stricture (post traumatic from surgery or biliary intervention).

Bacterial Ciiolangitis: Hepatic abscess can develop secondary to cholangitis, usually as the
result of stasis (so think stones). The triad of jaundice, fever, and right upper quadrant pain is the step
1 question.

Primary Sclerosing Cholangitis (PSC): Chronic cholestatic Dilated intrahepatic


liver disease of unknown etiology characterized by progressive bile ducts is very
inflammation which leads to multifocal strictures of the intra and /or rare in all forms of
extrahepatic bile ducts. The disease often results in cirrhosis, and is cirrhosis
strongly associated with cholangiocarcinoma. The buzzword for the
EXCEPT
cirrhotic pattern is “central regenerative hypertrophy”. It is
associated with inflammatory bowel disease (Ulcerative Colitis 80%, Primary Sclerosing
Crohn’s 20%). It is an indication for transplant, with a post transplant Cholangitis
recurrence of about 20%.

-S B PSC Buzzwords:
.“Withered Tree” - The appearance on MRCP, from abrupt narrowing of the branches
.“Beaded Appearance” - Strictures + Focal Dilations

AIDS Cholangiopathy: Infection of the biliary epithelium {classically Cryptosporidium) can


cause ductal disease in patients with AIDS. The appearance mimics PSC with intrahepatic and/or
extrahepatic multifocal strictures.

The classic association/finding is papillarv stenosis (which occurs 60% of the time).

THIS vs THAT: AIDS Cholangiopathy vs Primary Sclerosing Cholangitis

AIDS PSC
Focal Strictures o f the extrahepatic duct > 2cm Extrahepatic strictures rarely > 5mm

Absent saccular deformities o f the ducts Has saccular defoiiiiities o f the ducts

Associated Papillary Stenosis

322
O r ie n ta l C h o l a n g i t i s (Recurrent pyogenic cholangitis): Com m on in Southeast Asia (hence
the culturally insensitive name). They always show it as dilated ducts that are full of
pigm ented stones.

Buzzword: “straight rigid intrahepatic ducts.” The anatom ically longer,


flatter left biliary system
The cause o f the disease is not known, but it may be tends to m ake the disease
associated with clonorchiasis, ascariasis, and nutritional burden left dom inant
deficiency. These guys don’t do as well with endoscopic (the opposite o f
decom pression and often need surgical decom pression. hem atogenous processes
which fa v o r the right
Remember this is a m ajor risk factor for lobe).
cholangiocarcinoma in the East.

Primary Biliary Cirriiosis: A n autoim m une disease that results in the destruction o f
small & medium bile ducts (intra not extra). It prim arily affects m iddle-aged wom en, who
are often asymptomatic. In the early disease, normal bile ducts help distinguish it from PSC.
In later stages, there is irregular dilation o f the intrahepatic ducts, with norm al extrahepatic
ducts. Classically has a lace-like pattern o f fibrosis. There is increased risk o f HCC. If caught
early it has an excellent prognosis and responds to m edical therapy w ith ursodeoxycholic acid.
The step 1 trivia is “antim itochondrial antibodies (A M A )” w hich are present 95% o f the
time.

Long Common Channel:


An anatomic variant in w hich the
common bile and pancreatic duct
fuse prem aturely at the level o f the
pancreatic head (prior to the
sphincter o f Oddi complex).

The testable consequence is the


increased incidence o f pancreatitis -
as reflux o f enzymes is more
common.

There is also an association Normal Long


w ith Type 1 Common
choledochocysts. Channel

Type 1
F o c a l Dilation o f
the C B D

323
Choledochal Cysts I C aroli’s: Choledochal cysts are congenital dilations o f the bile
ducts -classified into 5 types by some dude nam ed Todani.

The high yie ld trivia is type 1 is fo c a l


dilation o f the CBD a n d is by fa r the most
common.

Type 2 and 3 are super rare. Type 2 is


basically a diverticulum o f the bile duct. Normal Type 1 Type 2
*most
Type 3 is a “choledochocele.” common

Type 4 is both intra and extra.

Type 5 is C aroli’s. and is intrahepatic only.

Type 3 Type 4 Type 5


*Caroli’s

Caroli’s is an A R disease associated with


polycystic kidney disease and m edullary
sponge kidney. The hallm ark is intrahepatic
duct dilation, that is large and saccular.
B uzzw ord is “ c e n tra l d o t sign” which
corresponds to the portal vein surrounded by
dilated bile ducts.
Central Dot
Complications W -V essel in a dilated duct
Cholangiocarcinoma
Cirrhosis
Cholangitis
Intraductal Stones

Gamesmanship: If they give you im aging o f dilated biliary ducts and a history o f
repeated cholangitis, think choledochal cyst. These things get stones in them and can
be recurrently infected.

324
- Ductal High Yield Sum m ary -

Caroli’s Primary Sclerosing Cholangitis


-Communicates with the ducts -40 year old Male with U.C.
-Type 5 Cyst -Withering Tree
-Central Dot Sign (on CT, MR, US) -Beading
-Associated with Polycystic Kidney, -Mild Dilation
Medullary Sponge Kidney, -Strongly Associated with Ulcerative
Cholangiocarcinoma Colitis & Cholangiocarcinoma

Oriental Cholangiohepatitis AIDS Cholangiopathy


(Recurrent Pyogenic) -Related to Cryptosporidium or
-Associations with clonorchiasis, cytomegalovirus.
ascariasis -Segmental Strictures (looks like PSC)
-Lots of Stones -Ducts look like P S C + Papillary
-Favors the left ductal system Stenosis
- Strongly Associated with - Associated with Cholangiocarcinoma
Cholangiocarcinoma

Association:
Cholangiocarcinoma VS B9 Strictures:
If you can’t remember what
the association is, and it’s CA Strictures tends to be long, with “shouldering.’
ductal pathology, always B9 strictures tend to be abrupt and short.
guess Cholangiocarcinoma.

325
S E C T IO N 4:
G a llb la d d e r

Normal Gallbladder
The normal gallbladder is found inferior to the interlobar fissure between the right and left lobe. The
size varies depending on the last meal, but is supposed to be < 4 x < 10cm. It is important to
understand that the gallbladder will distend when fasting and be empty after eating. So a mildly
distended gallbladder is “normal” in a fasting state. More than 4cm in width is dilated - and can
make you think about obstruction — usually from a stone.
The wall thickness should be < 3mm. A thick wall is nonspecific, and in general means that it is
inflamed or swollen. Inflammation can be from the gallbladder or the adjacent liver. This is why
hepatitis (inflammation of the liver) can cause gallbladder wall thickening. Edema - like you think
about in a CHF / fluid overloaded patient - that can also cause a thickening gallbladder wall.
The lumen should be anechoic — looking like a simple cyst.

Normal Anatomy: These things here (that


you can sometimes see
The gallbladder (like on MRCP) have a
the stomach) has Body testable name:
named parts that could
be potentially tested. Fundus “Spiral Valves
I’ve grey scale color of Heister”
coded them here in
this amazing
illustration:

Variants/Congenital
Duct of Luschka: An accessory cystic duct. This can cause a big problem (persistent bile leak)
after cholecystectomy. There are several subtypes - which is not likely to be tested.

Cystic Common
Duct Hepatic Duct

Common
Bile Duct

NORMAL Accessory Subvesicular Hepaticocystic


(most common)

Phrygian cap: A phrygian cap is seen when the GB folds on itself It means nothing.

Intrahepatic Gallbladder: Variations in gallbladder location are rare, but the intrahepatic
gallbladder is probably the most frequently recognized variant. Most are found right above the
interlobar fissure.

Duplicated Gallbladder: It can happen.

326
GBPathology Part 1- The last Wish
GB Wall Thickening (> 3mm):
Very non-specific. Can occur irom -Gallbladder Shadowing-
biliary (Cholecystitis, AIDS, PSC...) or
(1) Gallbladder full of stones - *Clean Shadowing.
non-biliary causes (hepatitis, heart
failure, cirrhosis, etc....). (2) Porcelain Gallbladder -* Variable Shadowing

Gallstones: Gallstones are found in (3) Emphysematous Cholecystitis - *Dirty Shadowing.


10% of asymptomatic patients. In kids
you should reflexively think about sickle
cell. Most (75%) are cholesterol, the
other 25% are pigmented.

On ultrasound, they cast shadows.


Reasons a stone might not cast a shadow
• It’s not a stone
• It’s a stone, but < 3mm in size
• The sonographer is an amateur
(Bush league psyche-out stuff. C lean S h a d o w (stone)Dirty S h a d o w (air in wail)
Laughable, man)

Acute Cholecystitis:
This is similar to the pathophysiology of a pimple (or an appendicitis). The Most Sensitive
gallbladder is full of dirty stuff and it needs to intermittently drain. If it gets Ultrasound Findings
clogged up (typically by a stone) then it dilates (which causes pain). Bile
salts begin to concentrate and inflame the gallbladder mucosa (causing it to • Sonographic
thicken). Eventually enteric bacteria, promising to usher in a marxist utopia, Murphy and
plot a bolshevik coup d’etat against the gallbladder mucosa - which if not • Cholelithiasis
stopped can lead to gangrenous cholecystitis, rupture, abscess, and the death
of free will and liberty.
In this way you can remember that acute cholecystitis should have:
• Pain — the sonographic Murphy sign (maximal tenderness from the ultrasound probe over the
visualized gallbladder). The pain is typically constant - as opposed to biliary colic with is intermittent.
• Stones - around 95% of cases are caused by a stone obstructing the cystic duct
• Dilation of the Gallbladder (more than 4cm in width)
• Wall thickness (from inflammation — this can increase in severity with time)
Gallstone impacted
Gangrenous Cholecystitis - This is an advanced subtype of cholecystitis with
necrosis of the gallbladder wall. Some useful trivia:
• Murphy Sign can be absent (necrosis of the nerve supply)
• Sloughed Membranes or “Cobwebs” is a buzzword for the sonographic
appearance. I ’ve attempted to illustrate that here (arrows):

Emphvsematous Cholecystitis - Infection with gas-forming organisms


(in the wall & lumen of gallbladder). Some useful trivia:
• Common in elderly diabetics
• More likely to perforate
• “Dirtv Shadowing” is the classic sonographic appearance.

327
GBPathology Part 2 - The Sword of Dosnny
Acute A calcu lou s Cholecystitis:
In most cases the absence of gallstones is going to make the diagnosis of cholecystitis unlikely. As we
discussed on the prior page, the process is typically obstructive - with a stone being the most conmion
cause of obstruction. Having said that, there is a specific situation where you can have a pissed off
gallbladder without a stone.
This process is similar to non-occlusive mesenteric ischemia (NOMI) in the bowel. Where most cases of
bowel ischemia are caused by a clot obstructing arterial or venous flow — or a direct pinch from a hernia
or an abnormal twist — there is a cause where none of that shit is happening. NOMI occurs after a severe
hypotensive event (massive blood loss, sepsis, near drowning or strangling yourself while masturbating).
In these near death events, there is often massive vasoconstriction of the bowel to shunt blood to the brain
and heart. This is the same idea with acute acalculous cholecystitis. My point here is that you want this
patient to be super sick or just narrowly escaped death final destination style. Some useful trivia:

• Classic scenario = following heart vale replacement surgery


• Thick wall and adjacent fluid are considered more important that gallbladder dilation
• HIDA scan will be positive (gallbladder not visualized) same as you’d expect with an obstructive stone.

Mirizzi Syndrome: This occurs when the common hepatic duct is obstructed
secondary to an impacted cystic duct stone. The stone can eventually erode into the
CHD or GI tract. The legend states that Mirizzi occurs more in people with a
low cystic duct insertion (normal variant), allowing for a more parallel course
and closer proximity to the CHD.

Key point: Increased co-incidence of gallbladder CA (5x more risk) Cystic Duct Stone
Obstructing CBD

Bouveret Syndrome:
Gastric outlet obstruction that occurs after impaction of a gallstone in the pylorus or proximal duodenum.
The idea is that a gallstone can erode through the gallbladder and form a fistula into the nearby small
bowel. If it gets stuck in the proximal duodenum or stomach it causes upstream dilation and a distended
stomach. This is not the classic location for a gallstone ileus (usually it is the the ileocecal valve) - so
don’t get confused — this is not a common thing - but it does have a French sounding name so you better
know it for the exam.
Some useful trivia:
• Bouveret Syndrome is a gallstone ileus - in its most proximal form.
• French sounding name “Bouveret” infers the syndrome is very high yield for multiple choice exams
• Bouveret sounds like belch to me (maybe because I don’t speak French) — distended stomachs need to
belch. Belching is rude. French people are famously rude. It is very rude of a gallstone to erode into
the proximal small bowel and cause a gastric outlet obstruction.

There is resulting
In a fashion
upstream dilation
typical of a rude
of the stomach
Frenchman, a
secondary to the
gallstone creates
very rude
a fistula into the
gallstone
adjacent
obstructing
proximal small
outflow.
bowel

328
GBPathologyPart 3-TheBlood of Elves

Clinically M eaningless THIS vs THAT:


Gallbladder Adenom yom atosis VS Cholesterolosis

N othing makes an A cadem ic Radiologist happier than spending time deploying intense
focused concentration targeted at distinguishing betw een two very similar appearing
com pletely benign (often incidental) processes. They believe this “adds value” and will
save them from the eventual avalanche o f reim bursem ent cuts w ith subsequent A! takeover.

Classic Example: Adenom yom atosis vs G allbladder Cholesterolosis. These things are
actually different - and even though it m akes zero difference clinically, this is ju st the kind
o f thing people who write questions love to write questions about.

G allbladder Adenom yom atosis: You have hypertrophied m ucosa and m uscularis propria,
with the cholesterol crystals deposited in an intralum inal location (w ithin Rokitanskv-
A schoff sinusesV

Cholesterolosis: Cholesterol and triglyceride deposition is within the substance o f the


lamina propria, and associated with form ation o f cholesterol polyps.

Adenomyomatosis: Results from hyperplasia o f


the wall with formation o f intramural mucosal
diverticula (Rokitansky-A schoff sinuses) which
penetrate into the wall o f the gallbladder. These
diverticula becom e filled with cholesterol crystals -
which m anifest from the unique acoustic signature
as comet-tail artifact Thighlv specific for
adenomyomatosis).

Comes in 3 flavors: Segmental (annular), Fundal


(localized or adenom yoma), and G eneralized (diffuse)
The Localized form can’t be differentiated from GB
cancer.

S e g m e n ta l, L o ca lize d , D iffuse

Gamesmanship: D on’t be tricked into selecting “Adenom yosis ” as a distractor. That


shit is in the uterus. Rem em ber the larger w ord is in the sm aller organ. Or, you can
think about the two Ms in M yoM at - turned inw ard sorta looks like a gallbladder.

n n
329
GBPathologyPart 4-Timeof Contempt
Porcelain Gallbladder:

• Extensive wall calcification.


• The key point is increased risk o f GB Cancer.
• These are surgically removed.

Gallbladder Polyps:

These can be cholesterol (by far the most


THIS vs THAT: Gallbladder Polyps
common), or non cholesterol (adenomas,
papillomas). Cholesterol polyps aren’t real Benign Malignant
polyps, but instead are essentially enlarged
> 1cm
papillary fronds full o f lipid filled < 5mm
♦between 5mm-10mm
*these are nearly always usually get followed for
macrophages, that are attached to cholesterol polyps growth
the wall by a stalk.
Pedunculated Sessile
The non-cholesterol subtypes are almost Solitary
Multiple
always solitary and are typically larger. The
Enhancement on CT/
larger polyps may have D oppler flow. They are Comet Tail Artifact on
MRl greater than the
NOT mobile and do N OT shadow. Once they Ultrasound (seen in adjacent gallbladder
cholesterol polyps) wall. Flow on Doppler.
get to be 1 cm, people start taking them out.

Gallbladder Cancer:

• Classic vignette would be an elderly w om en with nonspecific RUQ pain, w eight loss,
anorexia and a long standing history o f gallstones, PSC, or large gallbladder polyps.
• M ost GB cancers are associated with gallstones (found in 85% o f cases)
• M irizzi syndrome has a well described increased risk o f GB cancer
• Other risk factors include sm oldering inflam m atory processes (PSC, Chronic
Cholecystitis, Porcelain Gallbladder! and large polyps (“large” = bigger than 1cm)
• Unless the cancer is in the fundus (w hich can cause biliary obstruction) they often present
late and have horrible outcomes with 80% found with direct tum or invasion o f the liver or
portal nodes at the time o f diagnosis.

330
S E C T IO N 5:
HEPATIC D O P P L E R

B rief introduction to terminology.

• ‘‘Duplex ” means color.


• “Spectral” means color with a waveform.

- Concept of Arterial R esistance -

Some organs require continuous flow (brain), whereas others do not (muscles). The body is
smart enough to understand this, and will make alterations in resistance / flow to preserve energy.
When an organ needs to be “on,” its arteriolar bed dilates, and the waveform becomes low
resistance. This allows the organ to be appropriately perfused. When an organ goes to “power
save” mode, it directs the arterioles to constrict. The result is a switch in waveform to high
resistance as blood flow is diverted to other more vital organs.

To help quantify this low resistance high


resistance thing, we use this

“Resistive Index (R I)”


- which is defined as V1-V2 / V I.

Just remember that things that need blood all


the time, will have continuous diastolic flow
- and thus a low resistance wave form.
RI = PSV - EDV / PSV

What is this ‘‘Tardus Parvus” ?

Tardus: Refers to a slowed systolic


upstroke. This can be measured by
acceleration time, the time from end
diastole to the first systolic peak. An
acceleration time > 0.07 sec correlates with
>50% stenosis of the renal artery

Parvus: Refers to decreased systolic


velocity. This can be measured by
calculating the acceleration index, the
change in velocity from end diastole to the
first systolic peak.

An acceleration index <3.0 m/sec-


correlates with >50% stenosis of the renal Normal H. Artery
artery

331
- Understanding Stenosis -

The vocabulary o f “Upstream vs D ownstream ” is som ewhat confusing. Try and remember,
that the flow o f blood defines the direction.

* Upstream = Blood that has NOT yet passed through the stenosis
* Downstream = Blood that has passed through the area o f stenosis

So there are direct and indirect signs o f stenosis.

Direct Signs: The direct signs are those found at the stenosis itself and they include elevated
peak systolic velocity and spectral broadening (im m ediate post stenotic).

Indirect Signs: The indirect signs are going to be tardus parvus (downstream ) - with tim e to
peak (systolic acceleration) > 70msec. The RI dow nstream will be low (< 0.5) because the
liver is starved for blood. The RI upstream will be elevated (> 0.7) because that blood needs
to overcome the area o f stenosis.

Increased Low RI
High RI Peak Decreased
Velocity Peak
Velocity

332
- Hepatic Veins -

Flow in the hepatic veins is complex, with alternating forward and backw ard flow. The bulk
o f the flow should be forward “antegrade” (liver -> heart). Things that mess with the
w aveform are going to be pressure changes in the right heart which are transm itted to the
hepatic veins (CHF, Tricuspid Regurg) or com pression o f the veins directly (cirrhosis).

Anything that increases right atrial


Above the line is Retrograde
pressure (atrial contraction) will (towards the liver)
cause the wave to slope upward.
“A” represents atrial contraction.

Anything that decreases right atrial


pressure will cause the wave to
Below the line is Antegrade
slope downward.
(towards ttie heart)
Abnormal Hepatic Vein
Waveforms can m anifest in one o f three main Increased HV Decreased HV
Pulsatility Pulsatility
categories:
Tricuspid Regurg Cirrhosis
(1) More Pulsatile

(2) Less Pulsatile Hepatic Venous


Right Sided CHF Outflow Obstruction
(any cause)
(3) Absent = Budd Chiari

LOW YIELD - TRIVIA - Study only on second time through


THIS vs THAT: Tricuspid Regurg vs Rigiit Sided CHF

A V
A lthough certain dinosaur radiologists still
love this stuff, it’s unlikely to show up on
the test, and if it does it probably w on’t be
more than one question. If they ask it at all
- 1 would bet the question is something
regarding the “D Wave to S W ave”
relationship.

• Tricuspid Regurg - D deeper than S


• Right Heart Failure - S deeper than D

333
- Portal Vein -

Flow in the portal vein should always be towards the liver (antegrade). You can see some
normal cardiac variability from hepatic venous pulsatility transmitted through the hepatic
sinusoids. Velocity in the normal portal vein is between 20-40 cm/s. The waveform should be a
gentle undulation , always remaining above the baseline.

NORMA
You have three main patterns:

(1) Normal

(2) Pulsatile PULSATILE

(3) Reversed

REVERSED

Causes o f Portal Vein Pulsatility: Right-sided CHF, Tricuspid Regurg, Cirrhosis with Vascular
AP shunting.

Causes o f Portal Vein Reversed Flow: The big one is Portal HTN (any cause).

Absent Flow: This could be considered a fourth pattern. It’s seen in thrombosis, tumor invasion,
and stagnant flow from terrible portal HTN.

Slow Flow: Velocities less than 15 cm/s. Portal HTN is the most common cause. Additional
causes are grouped by location:

• Pre - Portal Vein Thrombosis


• Intra - Cirrhosis (any cause)
• Post - Right-sided Heart Failure, Tricuspid Regurg, Budd-Chiari

Final Doppler Trivia:

An ultra-com m on quiz question is to ask “w hat should A


the Doppler angle be?” N ow even though ultrasound
physics is covered in m ore detail in the dedicated section
o f the War M achine this is a high yield enough point to
warrant repetition. The answer is “less than 60. ”

Why? D oppler strength follows the cosine o f the angle.


For example, Cos 90 = 0, Cos 60 = 0.5, Cos 0 = 1.0
- the doppler strength follows the Cos.

334
S E C T IO N 6:
PANCREAS

Classic US Trivia: The


Trivia regarding the pancreas can be broadly
Pancreatic Echogenicity should be
clustered into: Solid Lesions, Cystic Lesions,
GREATER than the normal liver.
Pancreatitis, and M isc Trivia (mostly
developmental stuff).
Classic MRI Trivia: The
Pancreas should be the brightest
- Misc Pancreas Trivia - organ on non contrast T 1 .

Anatomy: The pancreas is a retroperitoneal structure (the tail may be intraperitoneal).

Cystic Fibrosis: The pancreas is involved in 85-90% o f CF patients. Inspissated


secretions cause proxim al duct obstruction leading to the two main changes in C F :
(1) Fibrosis (decreased T1 and T2 signal)
(2) Fatty replacem ent (increased T l) - the more com m on o f the two

Patients with CF, who are diagnosed as adults, tend to have more pancreas problem s than
those diagnosed as children. Just rem em ber that those w ith residual pancreatic exocrine
function tend to have bouts o f recurrent acute pancreatitis (they keep getting clogged up with
thick secretions). Small (1-3 mm) pancreatic cysts are common.

High Yield Trivia:


• Complete fa tty replacem ent is the m ost
common imaging finding in adult CF
• M arkedly enlarged with fatty replacem ent
has been term ed lipomatous
pseudohypertrophy o f the pancreas.
*This is a buzzword.
• Fibrosing C olonopathy: Wall thickening
o f the proxim al colon as a com plication o f
enzyme replacem ent therapy.
CF - Fatty Replacement of the Pancreas

Shwachm an-Diam ond Syndrom e: The 2"^ m ost com m on cause o f


pancreatic insufficiency in kids (CF #1). Basically, it’s a kid with diarrhea,
short stature (m etaphyseal chondroplasia), and eczema. Will also cause
lipomatous pseudohypertrophy o f the pancreas.

335
Pancreatic Lipomatosis:
THIS vs THAT:
Pancreatic Agenesis vs
M ost com m on pathologic condition involving the Pancreatic-Lipom atosis
pancreas. The m ost com mon cause in childhood is
CF (in adults it’s Burger King). Agenesis Lipomatosis

Additional causes worth knowing are Cushing Does N O T have Does have a
Syndrome, Chronic Steroid Use, Hyperlipidemia, a duct duct
and Shwachman-Diamond Syndrome.

Dorsal Pancreatic Agenesis: - All you need to know is that (1) this sets you up for
diabetes (most o f yo u r beta cells are in the tail), and (2) it’s associated with polysplenia.

Annular Pancreas: Essentially an em bryologic screw up (failure o f ventral bud to


rotate with the duodenum), that results in encasem ent o f the duodenum. Results in a rare
cause o f duodenal obstruction (10%), that typically presents as duodenal obstruction in
children and pancreatitis in adults. Can also be associated w ith other vague symptom s (post­
prandial fullness, “symptoms o f peptic ulcer disease” , e tc ...).

• Rem ember in adults this can present with pancreatitis (the ones that present earlier - in
kids - are the ones that obstruct).
• On imaging, look for an annular duct encircling the descending duodenum.

Pancreatic Trauma: The pancreas sits in front o f the vertebral body, so it’s susceptible
to getting smashed in blunt trauma. Basically, the only thing that m atters is integrity of
the duct. If the duct is damaged, they need to go to the OR. The m ost com m on delayed
com phcation is pancreatic fistula (10-20% ), followed by abscess formation. Signs o f injury
can be subtle, and may include focal pancreatic enlargem ent or adjacent stranding/fluid.

Im aging Pearls:
• Remember it can be subtle with ju st focal enlargem ent o f the pancreas
• I f you see low attenuation fluid separating two portions o f the enhancing pancreatic
parenchym a this is a laceration, N OT contusion.
• The presence o f fluid surrounding the pancreas is not specific, it could be from injury or
just aggressive hydration — on the test they will have to show you the liver and IVC to
prove it’s aggressive fluid resuscitation.

High Yield: Traumatic Pancreatitis in a kid too young to ride a bike = NAT.

Suspected Pancreatic D uct Injury? - N ext Step - M RCP or ER C P

336
- Pancreatitis -
Acute Pancreatitis:

Etiology: By far the most com mon causes are gallstones and EtOH which com bined make
up 80% o f the cases in the real world. However, for the purpose o f multiple choice tests, a
bite from the native scorpion o f the island o f Trinidad and Tobago is m ore likely to be the
etiology. Additional causes include ERCP {which usually results in a m ild course),
medications {classically valproic acid), traum a {the m ost common cause in a child),
pancreatic cancer, infectious {post viral in children), hypercalcem ia, hyperlipidem ia,
autoimmune pancreatitis, pancreatic divisum , groove (para-duodenal) pancreatitis, tropic
pancreatitis, and parasite induced.

Clinical Outcomes: Prognosis can be estim ated with the “Balthazar Score.” Essentially, you
can think about pancreatitis as “m ild” (no necrosis) or “severe” (having necrosis). Patients
with necrosis don’t start doing terrible until they get infected, then the m ortality is like
50-70%.

Key Point: Outcomes are directly correlated with the degree o f pancreatic necrosis.

Severe Pancreatitis: Severe acute


Week: 3-4
pancreatitis has a biphasic course. With Week: 1-2 SIR S /A R D S
Infection / Sepsis
the first two weeks being a pro-
inflammatory phase. This is a sterile
response in which infection rarely occurs. / Inflammatory |

The third and fourth w eeks transition to / Response |


^ Anti- 1 'y
an anti-inflammatory period in which the \ Inflammatory 1 ^
risk o f translocated intestinal flora and the X Pesponse ▼^

subsequent developm ent o f infection


increases.

Let’s Practice:

Which o f the fo llo w is the m ost “sa lie n t” (important) ? S o u n d s scary, but necrosis is
A: Hemorrhage in the Pancreas ^ m ore im portant
B: N ecrosis in the Pancreas
C: Fluid Collection 2nd B est O ption - I’d pick
D: Infection in Necrosis this if D w a s n ’t a C h o ice

B est O ption - R e m e m b e r to
R e a d A ll the A n s w e r C h o ice s!

337
VOCAB - Radiologist LOVE to Argue over Words — therefore high yield

< 4weeks A c u te Peripan creatic

Fluid C o llectio n

NO Necrosis >4^33^3 Pseudocyst

A c u te N e cro tic C o lle ctio n

Necrosis
W a lle d -O ff Necrosis

Vascular Com plications:


• Splenic Vein and Portal Vein Thrombosis
O Isolated gastric varices can be seen secondary to splenic vein occlusion
• Pseudo-aneurysm o f the GDA and Splenic Artery

Non-Vascular Com plications:


• Abscess, Infection, e tc ... as discussed
• Gas, as a characteristic sign o f an infected fluid collection, is detected in only 20% o f
cases o f pancreatic abscesses.

Random Imaging Pearl:


• On Ultrasound, an inflam ed pancreas will be hypoechoic (edem atous) w hen
com pared to the liver (opposite o f normal).

- Pancreatic Divisum -

Anatomy Refresher: There are two ducts, a major (Wirsung), and a minor (Santorini). Under
“normal” conditions the major duct will drain in the inferior of the two duodenal papilla
(major papilla). The minor duct will drain into the superior of the two duodenal papilla
(minor papilla). The way I remember this is that “Santorini drains Superior”, and “Santorini is
Small,” i.e. the minor duct.

Pancreatic Divisum is the most common


anatomic variant of the human pancreas,
and occurs when the main portion of the
pancreas is drained by the minor or
accessory papilla. The clinical
relevance is an increased risk of Nornnal (85%) Divisum (~15%)
pancreatitis.
-Bifid System with
Rudim entary D o rsa l Duct-

338
- Chronic Pancreatitis -
CP represents the end result o f prolonged inflam m atory change leading to irreversible
fibrosis o f the gland. Acute pancreatitis and chronic pancreatitis are thought o f as different
disease processes, and most cases o f acute pancreatitis do not result in chronic disease. So,
acute doesn’t have to lead to chronic (and usually doesn’t), but chronic can still have
recurrent acute.

Etiology: Same as acute pancreatitis, the most com m on causes are chronic alcohol abuse
and cholelithiasis which together result in about 90% o f the cases. (EtOH is #1)

Im aging Findings: Findings can be thought o f as early or late:

Earlv:
• Loss o f T1 signal (pancreas is normally the brightest T1 structure in the body)
• Delayed Enhancem ent
• Dilated Side Branches ~

Late:
• Comm only small, uniform ly atrophic
- but can have focal enlargement
• Pseudocyst formation (30%)
• Dilation and beading o f the pancreatic
duct with calcifications
* * most characteristic finding o f CP.

“Chain o f L akes”
Dilated and Beaded appearance o f the pancreatic
duct, with Intraductal Calcifications.

THIS vs THAT; Chronic Pancreatitis D uct Dilation


vs Pancreatic M alignancy D uct Dilation

CP C an cer

Dilation is Irregular D ilation is uniform (usually)

Duct is < 50% o f the AP gland diam eter Duct is > 50% o f the AP gland diam eter
(obstructive atrophy)

Complications: Pancreatic cancer (20 years o f CP = 6% risk o f Cancer) is the m ost crucial
com plication in CP and is the biggest diagnostic challenge because focal enlargem ent o f the
gland induced by a fibrotic inflam m atory pseudotum or m ay be indistinguishable from
pancreatic carcinoma.

339
- Cystic Pancreatic lesions -
Pseudocyst: W hen you see a cystic lesion in the pancreas, by far the m ost common cause
is going to be an inflammatory pseudocyst, either from acute pancreatitis or chronic
pancreatitis.

Sim ple Cysts: True epithelial lined cysts are rare, and tend to occur with syndromes such
as VHL, Polycystic Kidney Disease, and Cystic Fibrosis.

Serous Cystadenom a fGrandmaJ.- The form er term “m icrocystic adenom a” helps


me think o f a little old lady, which is appropriate for a lesion prim arily found in elderly
ladies. The lesion is benign, and classically described as a heterogeneous, m ixed-density
lesion made up o f multiple small cysts, which resembles a sponge. They are m ore com monly
(70%) located in the pancreatic head (mucinous is alm ost always in the body or tail). A n
additional key distinction is that it does N O T com m unicate w ith the pancreatic duct (IPM Ns
do). About 20% o f the time they will have the classic central scar, w ith or w ithout central
calcifications {mucinous calcifications are peripheral).

Rarely, they can be unilocular. W hen you see a unilocular cyst w ith a lobulated contour
located in the head o f the pancreas, you should think about this m ore rare unilocular
m acrocystic serous cystadenom a subtype.

Trivia: Serous Cystadenom a is associated w ith Von Hippel Lindau


M emory Aid: “GRAN D M A Serous is the HEAD o f the household”

Mucinous C y stic Neoplasm (IVlotherJ.- This pre-m ahgnant lesion is “always” found
in women, usually in their 50s. All are considered pre-m alignant and need to come out. They
are found in the body and tail {serous was more common in the head). There is generally no
com m unication with the pancreatic duct {IPMNs w ill communicate). Peripheral calcifications
are seen in about 25% o f cases {serous was more central). They are typically unilocular.
W hen multilocular, individual cystic spaces tend to be larger than 2 cm in diam eter {serous
spaces are typically sm aller than 2 cm).

M emory Aid: “M UCINOUS in the M OTHER”

Solid Pseudopapillary Tumor of the Pancreas - (Daughter^; Very rare, low


grade m alignant tum or that occurs alm ost exclusively in young (30s) females (usually Asian
or Black). It is typically large at presentation, has a predilection for the tail, and has a “thick
capsule.” Sim ilar to a hem angiom a it m ay dem onstrate progressive fill-in o f the solid
portions.

340
Uncommon Types and C au ses of Pancreatitis

Autoimmune Associated with Absence of Attack Responds to Sausage Shaped


Pancreatitis elevated lgG4 Symptoms steroids Pancreas, capsule
like delayed rim
enhancement around
gland (like a scar).
No duct dilation.
No calciflcations.

Groove Looks like a Less likely to cause Duodenal stenosis Soft tissue within the
Pancreatitis pancreatic head obstructive and /or strictures of pancreaticoduodenal
AKA Cancer - but with jaundice (relative the CBD in 50% of groove, with or
Paraduodenal little or no biliary to pancreatic CA) the cases without delayed
pancreatitis obstruction. enhancement

Tropic Young Age at onset, Increased risk of Multiple large calculi


Pancreatitis associated with adenocarcinoma within a dilated
malnutrition pancreatic duct

Hereditary Young Age at Onset Increased risk of SPINK-1 gene Similar to Tropic
Pancreatitis adenocarcinoma Pancreatitis

Ascaris Most commonly Worm may be seen


Induced implicated parasite within the bile ducts
in pancreatitis

Wiien 1 Say - Autoimm une Pancreatitis

I Say Autoim mune Pancreatitis You Say IgG4

Autoim m une Pancreatitis


Retroperitoneal Fibrosis
I Say IgG4 Sclerosing Cholangitis
Inflam m atory Pseudotum or
R iedel’s Thyroiditis

THIS vs THAT:
Autoimm une Pancreatitis vs Chronic Pancreatitis

A utoim m une Pancreatitis Chronic Pancreatitis

No ductal dilation Ductal Dilation

No calciflcations Ductal Calciflcations

341
IPMN - Intraductal Papillary M ucinous Neoplasm: These guys are mucin-
producing tum ors that arise from the duct epithehum . They can be either side branch, main
branch, or both.

Side Branch Features


“The pulmonary nodule Main Branch Concerning For
o f the pancreas ” Malignancy

•Common and usually meaningless •Produces diffuse dilation of the •Main duct >10 mm
•Typically appear as a small cystic main duct (some sources say 1.5 cm)
mass, often in the head or uncinate •Atrophy of the gland and •Diffuse or multifocal
process dystrophic calcifications may be involvement
•If large amounts of mucin are seen - •Enhancing nodules
produced it may result in main mimicking Chronic Pancreatitis •Solid hypovascular
duct enlargement •Have a much higher % of mass
•Lesions less than 3cm, are usually malignancy compared to side
benign branch
•All Main Ducts are considered
malignant, and resection should be
considered

- Cystic Pancreatic Lesion Summary -

Solid Pseudopapillary
- D a u g h te r Lesion
- Solid with C ystic
parts, e n h a n c e s like a
Main Branch IPMN h e m a n g io m a
- “T h e B ad O n e ” - C a p s u le
- H a s m align an t potential

Mucinous Cystic
- M o th er Lesion
Side Branch IPMN
- P rem a lig n an t
- “C o m m o n O n e ”
- B o d y /T a il - 9 5 %
- H a s m uch less
- U nilocular with thick
m alig nan t
w all sep tatio n s
potential
- O ften in head /
Serous Cystic
uncinate
- G ra n d m a Lesion
- C o m m u n ic a te s
- B enign
with duct
- M icrocystic, with
cen tral calcifications

342
- Solid Pancreatic Lesions -
Pancreatic Cancer basically comes in two flavors. (1) Ductal Adenocarcinoma - which is
hypovascular and (2) Islet Cell / Neuroendocrine which is hypervascular.

Ductal Adenocarcinoma: In the setting of a multiple choice Peak Parenchyma


test, the finding of an enlarged gallbladder with painless jaundice is Enhancement
highly suspicious for pancreatic adenocarcinoma, especially when
combined with migratory thrombophlebitis {Trousseau’s syndrome). Pancreas = 40 Seconds
The peak incidence is in the 7* or 8* decade. The strongest risk Liver = 60 Seconds
factor is smoking.
Two-thirds of these cancers arise from the pancreatic head. On ultrasound or MRCP, obstruction of
both the common bile duct and the pancreatic duct is referred to as the “double duct sign”. On CT, the
findings are typically a hypo-enhancing mass which is poorly demarcated and low attenuation
compared to the more brightly enhancing background parenchyma. The optimal timing is on a
pancreatic phase (40 seconds). MRI will show a low T1 signal mass (normal pancreas is T1 bright).

Non-Contrast Contrast (40 second) Contrast (Delay)


-Ductal Dilation -Ductal Dilation -Ductal Dilation
-Hypo-enhancing Mass -Mass now iso-intense

The key to staging is assessment of the SMA and celiac axis, which if involved make the patient’s
cancer unresectable. Involvement of the GDA is ok, because it comes out with the Whipple.

Additional Trivia Points about Pancreatic Adenocarcinoma:


• Tumor Marker = CA 19-9
• Hereditary Syndromes with Pancreatic CA:
• HNPCC, BRCA Mutation, Ataxia-Telangiectasia, Peutz-Jeghers
• Small Bowel Follow Through: Reverse impression on the duodenum “Frostburg’s Inverted 3 Sign”
or a “Wide Duodenal Sweep.” They would have to actually fin d a case o f the inverted 3 to show it,
but could ask it in words. The “Wide Duodenal Sweep ” could actually be shown.

Periampullary Tumor: Defined as originating within 2cm of


the major papilla. These can be cancers of the distal common bile
duct, pancreas, or duodenum. It can be difficult to differentiate from a
conventional pancreatic adenocarcinoma as both can obstruct the bile
duct, and present as a mass in the region of the pancreatic head.
MRCP/MRI is the exam of choice to try and get a good look at this
region. They can try and treat them with a Whipple and in general
they have a better prognosis than pancreatic adenocarcinoma.

Trivia: There is an increased incidence of ampullary carcinoma in Within 2cm of the Major Papilla
Gardner’s Syndrome.

343
Islet Cell / Neuroendocrine:
N euroendocrine tumors are uncom m on
tumors o f the pancreas. Typically
hypervascular, w ith brisk enhancement
during arterial or pancreatic phase. They
can be thought o f as non-functional or
functional, and then subsequently further
divided based on the horm one they make.
They can be associated with both M EN 1
Hyper-Enhancing Tumor
and Von Hippel Lindau.

Insulinoma: The most com mon type (about 75%). They are alm ost always benign (90%),
solitary, and small (< 2cm).

Gastrinoma: The second m ost common


type overall, but m ost com m on type
- G astrinom a Triangle -
associated with M EN 1. They are m alignant The anatomical region where most (90%) of
like 30-60%. They can cause increased gastrinomas arise.
gastric acid output and ulcer formation - Boundaries
Zollinger-Ellison syndrome.
Superior:
The Buzzword is Jejunal Ulcer = Zollinger- Junction of
the cystic and
Ellison common bile
ducts
Non-Functional: The m ost com mon Inferior: Start
type, usually m alignant (80%), and are of the third
portion of the
usually large and m etastatic at the time o f duodenum
diagnosis. Medial: Start
of the body of
I say “non-functional, ”
the pancreas
you say Large with Calcification

Intrapancreatic Accessory Spleen


It is possible to have a pancreatic mass that is actually just a piece of spleen. The typical
scenario is that of post traumatic splenosis. Look for the question stem to say something like
“history o f trauma. ” Another hint may be the absence of a normal spleen.
Imaging Findings:
-Follows spleen on all image sequences (dark on T l, and bright on T2 - relative to the liver).
-It will restrict diffusion (just like the spleen).
-The classic give away, and most likely way it will be shown is as a tiger striped mass on arterial
phase (tiger striped like the spleen on arterial phase).

Trivia: Nuclear medicine tests - (1) Heat Treated RBCs, and (2) Sulfur Colloid can be used to
prove the mass is spleen (they both take up tracer — just like a spleen).

344
S E C T I O N 7:
S u rg ical

The Whipple Procedure:

The standard Whipple procedure involves resection o f the pancreatic head, duodenum, gastric
antrum, and almost always the gallbladder. A jejunal loop is brought up to the right upper
quadrant for gastrojejunal, choledochojejunal or hepaticojejunal, and pancreatojejunal
anastomosis.

An alternative method used by some surgeons is to perform a pancreatoduodenectomy and


preserve the pylorus when possible. There is debate in the surgery literature with regard to
which method should be the standard. In this pylorus-preserving pancreatoduodenectomy, the
stomach is left intact and the proximal duodenum is used for a duodenojejunal anastomosis.

S to m ach

CBD

P an crea tic

Whipple
Complications:

Delayed gastric emptying {needfor NG tube longer than 1- day) and pancreatic fistula {amylase
through the surgical drain >50 ml fo r longer than 7-10 days), are both clinical diagnoses and
are the most common complications after pancreatoduodenectomy.

Wound infection is the third most common complication, occurring in 5% -20% o f patients.

345
- Transplant -
Pancreas transplant (usually with a renal transplant) is an established therapy for severe type 1
diabetes - which is often complicated by renal failure. The vascular anatomy regarding this
transplant is quite complicated and beyond the scope o f this text. Just know that the pancreas
transplant receives arterial inflow from two sources: the donor SMA, {which supplies the head
via the inferior pancreaticoduodenal artery) and the donor splenic artery, {which supplies the
body and tail). The venous drainage is via both the donor portal vein and the recipient SMV.
Exocrine drainage is via the bowel {in older transplants via the bladder).

The number one cause o f graft failure is acute rejection. The number two cause o f graft failure is
donor splenic vein thrombosis. Donor splenic vein thrombosis usually occurs within the first 6
weeks o f transplant. Venous thrombosis is much more common than arterial thrombosis in the
transplant pancreas, especially when compared to other transplants because the vessels are
smaller and the clot frequently forms within and propagates from the tied-off stump vessels.

Both venous thrombosis and acute rejection can appear as reversed diastolic flow. Arterial
thrombosis is also less o f a problem because o f the dual supply to the pancreas (via the Y graft).
A point o f trivia is that the resistive indices are not o f value in the pancreas, because the organ
lacks a capsule. The graft is also susceptible to pancreatitis, which is common < 4 weeks after
transplant and usually mild. Increased rates o f pancreatitis were seen with the older bladder
drained subtype.

‘‘Shrinking Transplant” is a buzzword for chronic rejection, where the graft


progressively gets smaller in size.

346
S E C T I O N 8:
Spleen

Normal Trivia

By the age o f 15 the spleen reaches its normal adult size. The spleen contains both “red
pulp” and “white pulp” which contribute to its tiger striped appearance during arterial phase
imaging. The red pulp is filled w ith blood (a lot o f blood), and can contain up to one liter o f
blood at any time. The spleen is usually about 20 HU less dense than the liver, and slightly
more echogenic than the liver (equal to the left kidney). The splenic artery (which usually
arises from the celiac trunk) is essentially an end vessel, with m inim al collaterals.
Occlusion o f the splenic artery will therefore result in splenic infarction.

Pathology involving the spleen can be categorized as either congenital, acquired (as the
sequela o f traum a or portal hypertension), or related to a “m ass.” A general rule is that m ost
things in the spleen are benign with exception o f lym phom a or the rare prim ary
angiosarcoma.

Normal Spleen on MRI:

It’s Bright on T 2 It’s D a rk on T1 Just like a lym ph node it


-R e la tiv e to the liver -R e la tiv e to th e liver will restrict diffusion

The spleen is basically a big waterv lymph node. It restricts diffusion (like a lymph node).

347
Heterotaxy Syndromes and Sickle Cell: I discuss these in the Peds chapter.
Accessory Spleens (Splenule): Can be called “splenunculus” for the purpose of fucking with
you on the exam. These are common (you probably have one). Some potential fuckery:
• Can mimic a pancreatic tail mass or a bad lymph node - sulfur colloid can tell the difference.
• They will often follow the spleen on different CT contrast phases (they stripe on the arterial phase).
• They can hypertrophy — classic scenario is post splenectomy for something like ITP or autoimmune
hemolytic anemia. The trick is that in that scenario, hypertrophy of an accessory spleen can result in a
recurrence of the original hematologic disease.
Wandering Spleen: A normal spleen that “wanders” off and is in an unexpected location. Because
of the laxity in the peritoneal ligaments holding the spleen, a wandering spleen is associated with
abnormalities of intestinal rotation. The other key piece of trivia is that unusual locations set the spleen up
for torsion and subsequent infarction. A chronic partial torsion can lead to gastric varices.
Trauma: The spleen is the most common solid organ injured in trauma. This combined with the fact
that the spleen contains a unit or so of blood means splenic trauma can be life threatening. Remember the
trauma scan is done in portal venous phase (70 second), otherwise you’d have to tell if that is the normal
tiger-striped arterial-phase spleen or it is lacerated.
Splenosis: This occurs post trauma where a smashed spleen implants and then recruits blood supply.
The implants are usually multiple and grow into spherical nodules typically in the peritoneal cavity of the
upper abdomen {but can be anywhere). If the diaphragm has been ruptured - they can be pleural based
nodules. It’s more common than you think and has been reported in 40-60% of trauma. Again, Tc Sulfur
colloid (or heat-treated RBC) can confirm that the implants are spleen and not ovarian mets or some other
terrible thing.
Gamesmanship: If they show you an absent spleen (or small nodular looking spleen) and they show you
lots of soft tissue masses - think splenosis... *especially if the history is prior trauma. If you are just
about ready to say “mets” — check and make sure the spleen is normal.

Sarcoidosis: Sarcoid is a disease of unknown etiology that results in noncaseating granulomas


which form in various tissues of the body {complete discussion in the chest section o f this text). The
spleen is involved in 50% - 80% of patients. Splenomegaly is usually the only sign. However,
aggregates of granulomatous splenic tissue in some patients may appear on CT as numerous discrete
l-2cm hypodense nodules. Rarely, it can cause a massive splenomegaly and possibly rupture. Don’t
forget that the gastric antrum is the most common site in the GI tract.

Gamna Gandy Bodies (Siderotic Module): Peliosis: This is a rare condition


These are small foci of hemorrhage in the splenic characterized by multiple blood filled cyst-
parenchyma that are usually associated with spaces in a solid organ {usually the
portal hypertension. They are T2 dark. -peliosis hepatitis). When you see it
in the spleen it is usually also in the liver
Gradient is the most (isolated spleen is extremely rare). The
sensitive sequence. j |^ etiology is not known, but for the purpose
of multiple choice tests it occurs in women
on OCPs, men on anabolic steroids, people
with AIDS, renal transplant patients (up
to 20% ), and patients with Hodgkin
lymphoma. It’s usually asymptomatic but
can explode spontaneously. €

348
-Splenic Vascular Abnormalities-

Splenic Artery Aneurysm is the most common visceral arterial aneurysm. Pseudoaneurysm
can occur in the setting of trauma and pancreatitis. The incidence is higher in women of child bearing
age who have had two or more pregnancies {4x more likely to get them, 3x more likely to rupture).
It’s usually saccular and in the mid-to-distal artery. Usually treated when they get around 2-3 cm.

Colossalfuck up to avoid: Don’t call them a hypervascular pancreatic islet cell mass and biopsy them.

Splenic Vein Thrombosis frequently occurs as the result of pancreatitis. Can also occur in
the setting of diverticulitis or Crohn’s. Can lead to isolated gastric varices.

Infarction can occur from a number of conditions. On a multiple choice test the answer is sickle
cell. The imaging features are classically a wedge-shaped, peripheral, low attenuation defect.

-Splenic infections-
Most common radiologically detected splenic infection is histoplasmosis (with multiple round
calcifications). Splenic TB can have a similar appearance (but much less common in the US).
Another possible cause of calcified granuloma in the spleen in brucellosis, but these are usually
solitary and 2 cm or larger. They may have a low density center, encircled by calcification giving the
lesion a “bull’s eye” appearance.

In the immunocompetent patient, splenic abscess is usually due to an aerobic organism. Salmonella
is the classic bug - which develops in the setting of underlying splenic damage (trauma or sickle
cell). In immunocompromised patients, unusual organisms such as fungi, TB, MAI, and PCP can
occur and usually present as multiple micro-abscesses. Occasionally, fungal infections may show a
“bulls-eye” appearance on ultrasound.

- Splenic Size - Too smaii vs Too BIG -


small Spleen BIG Spleen

Sickle Cell Passive Congestion (heart failure, portal HTN, splenic vein thrombosis)

Post Radiation Lymphoma

Post Thorotrast Leukem ia

M alabsorption Syndromes
Gauchers
(ulcerative colitis > crohns)

Felty’s Syndrome - abnormality of granulocytes, with a triad:


(1) Splenomegaly, ^
(2) Rheumatoid Arthritis,
(3) Neutropenia

349
- Benign Masses of the Spleen -
Post traumatic cysts (pseudocysts) are the most common cystic lesion in the spleen. They can
occur secondary to infarction, infection, hemorrhage, or extension from a pancreatic pseudocyst. As a
point of trivia they are “pseudo” cysts because they have no epithelial lining. They may have a thick wall
or prominent calcifications peripherally.
Epidermoid cysts are the second most common cystic lesion in the spleen. They are congenital in
origin. As a point of absolutely worthless trivia, they are “true” cysts and have an epithelial lining. They
typically grow slowly and are usually around 10 cm at the time of discovery. They can cause symptoms if
they are large enough They are solitary 80% of the time, and have peripheral calcifications 25% of the
time.
Hydatid or Echinococcal cysts are the third most common cystic lesion in the spleen. They are
caused by the parasite Echinococcus Granulosus. Remember this thing favors the liver (75% of cases) and
the second most common site is actually the lung (not the spleen). As before, there are multiple described
signs including the Water Lilly (floating debris) and the Daughter Cysts.
Hemangioma is the most common benign neoplasm in the spleen. This dude is usually smooth and
well marginated demonstrating contrast uptake and delayed washout. The classic peripheral nodular
discontinuous enhancement seen in hepatic lesions may not occur, especially if the tumor is < 2 cm.
Lymphangiomas are rare entities in the spleen but can occur. Most occur in childhood. They may be
solitary or multiple, although most occur in a subcapsular location. Diffuse lymphangiomas may occur
(lymphangiomatosis).
Hamartomas are also rare in the spleen, but can occur. Typically this is an incidental finding. Most
are hypodense or isodense and show moderate heterogeneous enhancement. They can be hyperdense if
there is hemosiderin deposition.
Littoral Cell Angioma is a zebra that shows up occasionally in books and possibly on multiple
choice tests. Clinical hypersplenism is almost always present. Usually presents as multiple small foci
which are hypoattenuating on late portal phase. MR shows hemosiderin (low T1 & T2).

- Malignant Masses of the Spleen -


Most things that occur in the spleen are benign. Other than lymphoma (discussed below) it is highly
unlikely that you will encounter a primary malignancy of the spleen {but if you do it’s likely to be
vascular). For the purposes of academic discussion (and possible multiple choice trivia), angiosarcoma is
the most common.
Angiosarcoma: It is aggressive and has a poor prognosis. On CT it can manifest as a poorly defined
area of heterogeneity or low density in an enlarged spleen. They can contain necrosis and get big enough
to rupture (spontaneous rupture occurs like 30% o f the time). Contrast enhancement is usually poor. Yes,
these can occur from prior thorotrast exposure.
Lymphoma is the most common malignant tumor of the spleen, and is usually seen as a manifestation
of systemic disease. Splenomegaly is the most common finding (and maybe the only finding in low-grade
disease). Although both Hodgkins and Non-Hodgkins types can involve the spleen, Hodgkins type and
high-grade lymphomas can show discrete nodules of tumor. With regard to imaging, they are low density
on CT, T1 dark, and are PET hot.
Metastatic Disease to the spleen is rare. When it does occur, it occurs via common things (Breast,
Lung, Melanoma). Trivia: Melanoma is the most common primary neoplasm that mets to the spleen.

350
I low did I get here ? C^hopping w(x)d, carrying water.
"I low am I going to be a gold medalist? By chopping wood and carrying water."

— Thomas (Oilman , 2 0 2 1 World (Champion

I love this quote. Reminds me that there are no days off.


No amount of success and no amount of fame or money buys a day off.

Yesterday - chopping wood and carry ing water.


'I'oday - chopping w (X )d and carr\ang water.
1 bmorrow - chopping wood and carrying water.

'I'his is the wav.

351
PROMETHEUS
L iomhart, M.1>.

352
PROMETHEUS LIONHART, M.D.

URINAKY
6

W arn in g: M e n c an e x p e c t painful urination w h ile sitting,


e sp e c ia lly if th e pen is is c a u g h t b e tw e e n th e to ilet s e a t a n d th e bow l.

353
S E C T I O N 1:
A natom y / C o n g e n ita l

Normal Anatomy:
The spaces around the kidneys have names and are easily tested in the form of multiple choice questions;
including the classic “what space is this ? ” - arrow on a space question. As this is the laziest possible way
to write a GU question it is the most likely one to be asked. As is often the case with anatomy, this stuff
can get pretty fucking complicated (lots of synonymous terms and potential spaces) so I’m gonna try and
break it down for you — like a cardboard box that I’m folding up nicely to put in the recycling.
Now I've analyzed this anatomy with a team of Victoria secret underwear models, who all came to one
unanimous conclusion: that renal fascia is super boring. After this careful analysis, I decided to try and
explain this piss in 5 stages ~ then a bit more in section 2. Notice I called it “piss” because this is GU.

Part 1: Overly Simplified Spaces (Shaded Areas) - Note the vocab: pararenal and perirenal spaces
I’ve deliberately over simplified these spaces in a way that I think lends itself most easily to “what space
is this ? ” and "what is in this space ? ” style of lazy multiple choice questions.

Posterior Pararenal Space Perirenal Space Anterior Pararenal Space


Contains: only fat pads Contains: Kidneys, proximal Contains: Pancreas, Duodenum,
Ureters, Adrenals Ascending & Descending Colon

Part 2: Fascial Borders to the Spaces (dotted lined and arrows)


Next let us consider that the named fascial bands could be asked in the form of “what makes up the
border ? ” and “what is this fascia s French sounding surname ? ” Lets go front to back:

Posterior Parietal Lateroconal Anterior Renal Fascia Posterior Renal Fascia Transversalis
Peritoneum Fascia (Gerota’s Fascia) (Zuckerkandl’s Fascia) Fascia

Part 3: Potential Spaces The Retrorenal


(grey shaded area and arrows) plane (RRP)
between the
The Retromesenteric Plane perirenal space and
(RMP) between the anterior posterior pararenal
pararenal space and perirenal space space

354
Normal Anatomy - Continued
I could end this fascial anatomy discussion here, but I'm really just getting started. I do have to go to
traffic court soon though, I accidentally ran over that lady who goes jogging outside with a mask on.

Part 4:
Now this is where
it gets complex, my
lovelies - they can
show this same
piss in sagittal
planes
(those bastards).

Arrows and
gridded patterns

Posterior Parietal Anterior Renal Fascia Posterior Renal Fascia Combined


Peritoneum (Gerota’s Fascia) (Zuckerkandl’s Fascia) Interfascial
Plane
Part 5: Pathology in the spaces
Perirenal Space Anterior
• Renal Mass Pararenal Space
• Renal Hematoma Pancreatitis
• Renal Abscess - fluid collections
• Urine Leaks (pseudocysts & stuff),
(most are proximal) Duodenal Hematoma

The distinction between “PERI” renal & “PARA” renal spaces is critical knowledge for MCQ trivia

That is probably good enough.... probably. Now for normal renal anatomy.

The normal adult kidney is shaped like a bean,


with a smooth (often lobulated) outer border. The
kidney is surrounded by a thick capsule outlined Column
by echogenic perirenal fat. This echogenic fat is of Bertin
contiguous with the renal sinus, filling the middle
of the kidney. The cortex extends centrally into
the middle of the kidney, separated by slightly Medulla
less echogenic medullary pyramids. The normal (Pyramid)
kidney should be between 9 cm and 15 cm in
length.
Papilla
(tip of medulla
The echogenicity of the kidney should be equal to projecting into
or slightly less than the liver and spleen. If the calyx)
renal echogenicity is greater than the liver, this
indicates some impaired renal fijnction (medical
renal disease). Liver echogenicity significantly
greater than the kidney indicates a fatty liver.

355
Variant flnatomy and Renal Pseudomasses:
Dromedary Hump:
Focal bulge on the left kidney, which forms as the
result of adaptation to the adjacent spleen.
It can simulate a mass, but once you’ve seen
a few you won’t be fooled as easily.

Prominent (or hypertrophied)


Column of Bertin:
Normal variant in which hypertrophied cortical tissue located
between the pyramids results in splaying of the sinus. Other
than the hypertrophy it looks totally normal. It will enhance
the same as adjacent parenchyma. The classic location is
between the upper and middle calyces
(or just remember - the middle third).

Most likely to be shown on ultrasound, - looks like a big Same idea - drawn with
echogenic cortex. They could show power doppler - it will Bulge of
more detail to show that
look identical in flow / color to the normal cortex. “Normal”
hypertrophied cortical
Cortex into the
tissue located between
Renal Sinus
the pyramids
Junctional Parenchyma Defect:

This is a “pseudo-scar” that looks like an echogenic line seen in the upper
1/3 of the kidney. It is basically only seen on ultrasound in the longitudinal
plane.

Fetal Lobulation; The fetal kidneys are subdivided into lobes that are separated with grooves.
Sometimes this lobulation persists into adult life. The question is always:
THIS vs THAT: Fetal Lobulation vs Reflux Scarring;

Lobulation = Scarring =
Renal surface Renal surface
indentations indentations
overlie the overlie the
space between dilated / blunted
the pyramids calyces.

Normal for Tends to favor


Comparison the upper and
lower poles.

Horseshoe Kidney & Crossed Fused Renal Ectopia: Discussed in the Peds chapter

356
Renal Agenesis;
Congenital absence o f one or both kidneys.

When it is bilateral you should think about the “Potter Sequence,” which to my utter
disappointment never ends with the development o f magical skills. It does give you hypoplastic
lungs - which I imagine could be remedied with gillyweed [ref 1 - Magical Water Plants o f the
Highland Lochs].

Potter Sequence: No Kidneys No Pee No Amniotic Fluid Hypoplastic Lungs

When it is unilateral it is typically sporadic and often asymptomatic. Having said that, for the
purpose o f muhiple choice you’ll need to consider the various forms o f fuckery that can be
associated with this:
Mayer-Rokitansky-
Associated GYN anomalies in women:
Kuster-Hauser:
• 70% o f women with unilateral renal agenesis have associated
genital anomalies - unicornuate uterus. Mullerian duct
anomalies including
Associated anomalies in men: absence or atresia of the
• 20% with renal agenesis have absence of the ipsilateral uterus. Associated with
epididymis and vas deferens or have an ipsilateral seminal unilateral renal
vesicle cyst. agenesis.

Lying Down Adrenal or “Pancake Adrenal” Sign: - describes the elongated appearance of
the adrenal not normally molded by the adjacent kidney. It can be used to differentiate
surgically absent vs congenitally absent.

The normal adrenal is However, on the left,


shaped like a Y, as instead of the normal
shown on the right. “Y” shape , he has an
elongated pancake
adrenal (arrow).

Same guy - one or two


The side with the
levels down shows that
elongated pancake
the right side with the
adrenal is missing a
normal Y shaped
kidney.
adrenal has a kidney.
This is a sign of
congenital absence
(not semi-voluntary
donation by a Uyghur in
a Chinese labor camp)

357
Renal Contrast Phases and Related Trivia
The kidneys drink contrast in a predictable order and their appearance changes accordingly. This
has implications for pathologic sensitivity (looking for tumor, vs urine leak, or urothelial lesions,
etc.. .etc... so on and so forth). Plus, it could be the source of multiple choice trivia - so I’m
compelled and honor bound to discuss it.
There are 4 possible “normal” looks the kidneys can give you:
Corticomedullary
Non- r Nephrographic Excretory
Contrast A n g w n e p h ro g ra p h ic - i f y o u
w an n a so u n d lik e a p rete n tio u s p r ic k Phase (NP) Phase (EP)

25- 40 Seconds 70-180 Seconds 180 Seconds - 8 Mins

Contrast makes its way


Contrast is excreted into
Contrast in the vascular system and the through the loops of the urinary tract /
extracellular interstitial space Henle and the collecting
collecting system
tubules (“gets filtered”)
Progressive decrease in
Fairly uniform the nephrogram
Cortex is enhanced the medulla is not
enhancement of the (depending on the timing
(hence the name).
cortex and medulla of the exam)
Utility: Utility: Utility:
• Characterizing renal tumor • Detect renal tumors • Evaluated morphology
enhancement (relative to the cortex of the papilla (necrosis
• Especially useful for
— clear cell similar to cortex, etc...)
small / central tumors
— papillary less than cortex) • Evaluate for urothelial
• Evaluation of renal arteries and Nephrographic phase is cell / TCC lesions
veins the most sensitive phase
for the detection of • Best phase to see the
• Optimal phase to detect tumor
renal cell carcinoma striated nephrogram
invasion of the renal veins
classic for infection
(important for staging / treatment
(but non-specific)
planning)
If CMP lasts longer the
Phase can be delayed (last longer) if NP will have delayed
the kidnevs are shit (failure, renal onset.
artery stenosis), obstructed, or in the The NP can onset earlier
setting of cardiac failure. if the contrast rate of
infusion is increased.

358
S E C T I O N 2:
Renal Masses

Renal Cell Carcinoma -


The most common primary renal malignancy. RCC till proven T im ing
otherwise; (a) Enhances with contrast (> 15 HU), (b) calcifications in Nephrogram phase
a fatty mass. Risk factors include tobacco use, syndromes like VHL, (^0 seconds) = Most
chronic dialysis (> 3years), family history. These dudes make sensitive for
hypervascular mets. They are ALWAYS lytic when they met to the detection of RCC
bones.
• Pseudoenhancement: A less than 10 HU increase in attenuation is considered within the
technical limits o f the study and is not considered to represent enhancement. M ore rare once
a cyst is larger than 1.5 cm.
• Can RCC have fa t in it? - Oh yeah, for sure -especially clear cell. This leads to the potential
sneaky situation o f a fat containing lesion in the liver (which can be a RCC met). Now to
make this work they’d have to tell you the patient had RCC - or show you one. A helpful
hint is that RCCs with macroscopic fat nearly always have some calcification/ossification - if
they don’t it’s probably an AML.

Subtypes:
• Clear Cell - Most common subtype in the general population. This is also the one associated with
VHL. It is typically more aggressive than papillary, and will enhance equal to the cortex on
corticomedullary phase. The most classic look is a cvstic mass with enhancing components.
• Papillary - This is the second most common type. It is usually less aggressive than clear cell (more
rare subtypes can be very aggressive). They are less vascular and will not enhance equal to the cortex
on corticomedullary phase. They also are in the classic T2 dark differential (along with lipid poor AML
and hemorrhagic cyst). Risk of primary renal malignancy in the transplanted kidnev is six times that of
the regular Joe. A point of testable trivia is that these cancers are usually papillary subtypes.
• Medullary - Associated with Sickle Cell Trait. It’s highly aggressive, and usually large and already
metastasized at the time of diagnosis. Patient’s are usually younger.
• C hrom ophobe - All you need to know is that it’s associated with Birt Hogg Dube.
• Translocation - Most common RCC subtype in kids. A history of prior cytotoxic chemotherapy is
classic.
Subtype Syndrome /Association
Overall Most Common in Gen Pop
Clear Cell
Conventional RCC Staging: Von Hippel-Lindau
Stage 1; Limited to Kidney and < 7 cm Hereditary Papillary Renal
Stage 2: Limited to Kidney but > 7 cm Papillary Carcinoma
Stage 3: Still inside Gerota’s Fascia Transplant Kidney
A: Renal Vein Invaded
B: IVC below diaphragm Chromophobe Birt Hogg Dube
C: IVC above diaphragm
Stage 4: Beyond Gerota’s Fasica Medullary Sickle Cell Trait
Ipsilateral Adrenal
Most common in Pediatric Setting
rans oca ion of Cytotoxic Chemotherapy

359
Sneaky Move
Does AD Polycystic Kidney Disease increase your risk for RCC???? - Well No, but sorta.
The genetic syndrome does NOT intrinsically increase your risk. However, dialysis does.
W ho gets dialysis?? People with ADPKD. It w ould be such a crap way to ask a question -
but could happen. If you are asked, I ’m recom m ending you say no to the increased risk, -
unless the question w riter specifies that the patient is on dialysis.

Renal Lymphoma: This can


literally look like anything. H aving said
that the most com mon appearance is
bilaterally, enlarged kidneys, with small,
low attenuation cortically based solid
nodules or masses ( “in filtrative soft
tissue in the renal hilum ”), and
associated lymph nodes. A solitary mass
is seen in about 1/4 o f the cases.

Trivia: Out o f all the renal masses -


lym phoma is the m ost likely to preserve
i Renal Lymphoma
the normal reniform shape.

Trivia: Lymphoma is the m ost common m etastatic tum or to invade / infiltrate the kidneys

Renal Leukemia: The kidney is the m ost com mon visceral organ involved. Typically
the kidneys are smooth and enlarged. Hypodense lesions are corticallv based only, with little
if any involvement o f the medulla.

Angiomyolipoma (AML); This is the m ost com m on benign tum or o f the kidney.
Almost all (95%) o f them have macroscopic fat, and this is the defining feature. They are
usually incidental (in the real world).

The things to know about them are:


(1) They are associated with Tuberous Sclerosis —
Tuberous Sclerosis can be called Bourneville Disease (for the purpose o f fucking with you)
(2) They can bleed if they get big enough (> 4cm). It’s controversial if they grow or bleed
more in pregnancy (if they ask you, I guess you should say yes - because th at’s the old
knowledge but some m odem papers are saying not for sure).
(3) They should never have calcifications (that’s probably a RCC).
(4) They can be lipid poor (about 5% are), and those are T2 dark.
G am esm anship: Traditional Spectral Fat Sat or In and Out o f Phase (India Ink) can be used
to suggest an AML. Just remember, rarely RCCs can have fat:

F at with Calcifications = RCC, F at with N o Calcifications = Probably AM L

360
Oncocytoma - This is the second most common benign Oncocytoma
tumor (after AML). It looks a lot like a RCC, but has a
central scar 33% o f the time (and 100% o f the time on
multiple choice). There will be no malignant features (such
as vessel infiltration). They cannot be distinguished from
RCC on imaging and must be treated as RCC till proven
otherwise.

If they want to ask about an Oncocytoma they can show it 3


ways: (1) Solid Mass with central scar - CT or MRI, (2)
“Spoke wheel” vascular pattern on US, (3) Hotter than the
surrounding renal cortex - on PET CT.

Gamesmanship: I have encountered two types o f GU radiologists in my life. The first type is the
practical type - he/she doesn’t EVER even mention oncocytoma, because enhancing renal masses
have to come out. Even if you biopsy it and get oncocytes, it doesn’t matter because RCCs can
have oncocytic features.

The second type is more o f your classic academic type. To try


and sound impressive this person will often include this (and
other rare entities) in the differential. Another common The PET Trick:
psychopathology in these types is excessive word nazism
(“don’t say that, say this”). I ’m fairly certain you have met RCC is typically COLDER
this person - there is usually one in every section. than surrounding renal
parenchyma on PET,
So if you are shown an enhancing renal mass with a central
scar, how do you decide if it’s a RCC or an oncocytoma? The Oncocytoma is typically
way to figure it out is simple - just read the mind o f the person HOTTER than surrounding
who wrote the question. If it’s a practical type then all renal parenchyma on PET,
enhancing renal masses are RCC till proven otherwise. If it’s
the academic type then central scar = oncocytoma. You may
also think... which o f these two people is more likely to
volunteer to write board questions?

Trivia: A syndrome associated with bilateral oncocytomas is Birt Hogg Dube


(they also get chromophobe RCC).

Multilocular Cystic Nepiiroma -


“Non-communicating, fluid-filled locules, surrounded by
thick fibrous capsule.” By definition these things are
characterized by the absence o f a solid component or
necrosis.
Buzzword: “protrudes into the renal pelvis.”
The question is likely the bimodal occurrence
(4 year old boys, and 40 year old women).

I like to think o f this as the Michael Jackson lesion - it loves


young boys and middle aged women.

**Remember this is the "older" nomenclature. It is not the preferred nomenclature dude.
Nerds ruined my joke. Now these are “Adult MLCNs" and “Pediatric MLCNs.” See page 229.

361
- More Retroperitoneum Fascial Anatomy -

I continued my analysis of this anatomy with a team of experts in the field of true love: who once again
came to one unanimous conclusion; that fascia is super boring and that I should get a life.

The retroperitoneum is “behind


the peritoneum” as the name
implies.

It is bordered anterior by the


anterior pararenal space (black
arrows) and posterior by the
transversalis fascia (white
arrows) - stuff we covered in
section 1.

Below the level of the kidneys, there is a blending of the


fascial planes which allows for potential spread of disease
between the retroperitoneum and the pelvis.

Illustrated with the light grey part (with the black arrows)
demonstrating the inferior extension / communication of
the retroperitoneum.

• White = Peritoneum
• Light Grey Around the White =
Compartments Communicating with the
Retroperitoneum

Anatomic Trivia: The RP contains the lower esophagus, most of the duodenum, the ascending and
descending colon, the kidneys, ureters, adrenals, pancreas (minus the tail), aorta, IVC, and the upper 2/3
of the rectum (some people say only the upper 1/3 is completely RP).

The classic tricks for multiple choice are:

• The pancreatic tail is NOT being part of the RP.

• Lower 1/3 of the rectum is NOT being part of the RP — it is “sub” peritoneal.

Pathology: -75% of the primary retroperitoneal neoplasms are malignant. Any tumor in this location is
guilty until proven otherwise. Having said that, there is an enormous amount of path that can occur in
this location — I’m gonna try and focus on what I think is probably the highest yield. The chart on the
following page does not include adrenal tumors - I’ll cover those in the endocrine chapter (and peds). RP
Fibrosis is mentioned briefly- it is discussed in detail later in the chapter. Neurogenic tumors will be
covered in the neuro chapter.

362
Rhabdo-
Lipomatosis Liposarcoma
Myosarcoma
Seen in big fat people Usually seen in the thigh of an old person - but is also the Most common soft
- with the classic most common primary malignant RP in adults. tissue sarcoma in
history of “incomplete children.
These things are notorious assholes with a high rate
bladder emptying”
(around 2/3) of local recurrence — hence the endless
You see a soft tissue
You can also see this surveillance studies you end up reading post treatment.
in homeless people, mass (in a kid) - you
Don’t call it a comeback (I’ve been here for years - should always be
who go to soup
rocking my peers puttin' suckers in fear). Also - Don’t call thinking about this.
kitchens... that
it a lipoma - no matter how simple and homogenous it
specialize in ice cream
looks.
soup.
Overgrowth of benign The deeper a fat containing lesion is - the more likely it is About half of them will
fat in the pelvis to be a bad actor. Fat-containing retroperitoneal lesions be in the neck, and
classically perirectal should be thought of like a male resident on the about 1/4 of them in
and perivesicular mammography service- guilty of all crimes until proven the pelvis around the
spaces. innocent. bladder or testicles.
The bladder is Anything that makes them look more complex -
displaced anterior and calcifications, solid components, not fat sating out - all No surprise - it is
superior - and is “pear that makes them even more likely to be bad (not just more gonna look like a
shaped” or inverted likely to be a cancer, but more likely to metastasize). tumor. Heterogenous,
tear drop shaped. enhancing, possibly
If you see something you think is a giant fucking AML -
but you aren’t totally sure it is coming from the kidney destroying nearby
AND it has calcifications - you should think Liposarcoma. bone.

Extra - Medullary
Lymphoma RP Hemorrhage
Hematopoiesis
Abnormal deposits of The Most Common RP malignancy.
hematopoietic tissue outside
Tons of Big Nodes or a Confluent Soft Tissue
the bone marrow.
Mass. Classically people talk about NHL vs HL. Most Common Cause =
The look is super non­ -NHL nodes are more likely to be larger, non- Over-Anticoagulation
specific -just a bunch of soft continuous, and involve the mesentery. (high PT/INR)
tissue masses in the para­ -HL nodes are more likely to involve the para­
2nd Most Common
vertebral region. aortic region early, and be more continuous
Cause = Rupture /
History is the only fair way to Leaking Aorta
test this — they have to tell
PET/CT is excellent for Lymphoma (in 3rd Most Common
you (or somehow show you)
particular it is very useful for disease vs Cause = Bleeding RCC
that the patient has a history
of hemoglobinopathy, treated residual scarring — with disease being or AML
myelofibrosis, leukemia - FDG avid).
etc...

“Mantle Like Soft Tissue Mass Around the Aorta , IVC, and/or Ureters”
- This has a classic DDx of:
L ym ph om a R P F ib ro sis E rdheim C h ester

Can displace the aorta forward, Like lymphoma RPF can be hot on Huge Zebra. Gamesmanship would
and be seen above the renal PET. It does not usually displace be to show you plain films of the
arteries. Tends to push things the aorta anterior, is uncommon legs with bilateral symmetric
rather than tether and obstruct. above the renal arteries, and tends sclerosis of the metaphysis
to tether and obstruct. (sparing the epiphysis).

363
S E C T I O N 3:
C ystic D isease

Bosniak Cyst Classification:


•Class 1: Simple - less than 15 HU with no enhancement
•Class 2: Hyperdense (< 3 cm). Thin calcifications, Thin septations
•Class 2F: Hyperdense (> 3 cm). Minimally thickened calcifications (5% chance cancer)
•Class 3: Thick Septations, Mural Nodule (50% chance cancer)
•Class 4: Any enhancement (>15 HU)

Class 1: Class 2: Class 2F: Class 3: Class 4:


-Simple Anechoic -Hyperdense (<3cm) -Hyperdense (>3cm) -Thick Calcifications -Any Enhancement
- 0% chance of CA -Thin Calcifications -Thin Calcifications -Mural Nodule -100% chance of CA
-Thin Septations - < 5% chance of CA -50% chance of CA This is your classic lool<
- 0% chance of CA for a clear cell R C C

Hyperdense Cysts: Basically, if the mass is greater than 70 HU and homogenous, it’s benign
(hemorrhagic or proteinaceous cyst) 99.9% of the time.
Autosomal Dominant Polycystic Kidney Disease (ADPKD) - Kidneys get
progressively larger and lose function (you get dialysis by the 5* decade). Hyperdense contents &
calcified wall are frequently seen due to prior hemorrhage. What you need to know is: (1) it’s
Autosomal Dominant “ADuU”, (2) They get cysts in the liver 70% of the time, (3) they get seminal
vesicle cysts (some sources say 60%), and (4) they get Berry Aneurysms. As mentioned before, they
don’t have an intrinsic risk of cancer, but do get cancer once they are on dialysis.
Autosomal Recessive Polycystic Kidney Disease (ARPKD)-T h eseg u y s getHTN
and renal failure. The liver involvement is different than the adult form. Instead of cysts they have
abnormal bile ducts and fibrosis. This congenital hepatic fibrosis is ALWAYS present in ARPKD.
The ratio of liver and kidney disease is inversely related. The worse the liver is the better the kidneys
do. The better the liver is, the worse the kidneys do. On ultrasound the kidneys are smoothly enlarged
and diffusely echogenic, with a loss of corticomedullary differentiation.
Seriously Motherfuckers ? - A nasty trick is to show you a nasty looking cystic lesion that
makes you think cystic neoplasm. The trick is a subacute or chronic renal abscess can look just like a
cystic renal neoplasm - and this differentiation can be very difficult - especially on a single image with
no history (prompting you to refiexively think - “seriously motherfuckers ?”).
History of fever, leukocytosis, or previously treated
urinary tract infection - all should activate your
spider-sense.

If that isn’t available, look for peri-renal stranding


and thickening of the fascia - as shown in this
awesome illustration I made (arrow).
More on this in the next section.

364
Lithium Nepiiropathy - Occurs in patients who take lithium long term. Can lead to diabetes
insipidus and renal insufficiency. The kidneys are normal to small in volume with multiple
(innumerable") tiny cysts, usually 2-5 mm in diameters. These "microcysts" are distinguishable
from larger cysts associated with acquired cystic disease of uremia. They are probably going to
show this on MRI with the history of bipolar disorder.

Uremic Cystic Kidney Disease - About 40% of patients with end stage renal disease
develop cysts. This rises with duration of dialysis and is seen in about 90% in patients after 5 year
of dialysis. The thing to know is: Increased risk of malignancy with dialysis (3-6x).
Trivia: The cysts will regress after renal transplant.

Von Hippei Lindau-Autosom al dominant multi-system


disorder. 50-75% have renal cysts. 25-50% develop RCC Cysts in Kidneys
(clear cell). ADPKD
1Liver are BIG
Pancreas: Cysts, Serous Microcystic Adenomas,
Neuroendocrine (islet cell) tumor Cysts in
VHL 1
Adrenal: Pheochromocytoma (often multiple) Pancreas
CNS: Hemangioblastoma of the cerebellum, brain stem,
and spinal cord. Endolymphatic sac tumors. Acquired Kidneys
Urogenital: Epididymal cysts, cystadenomas (Uremic) are small
i

Tuberous Sclerosis - Autosomal dominant multi-system disorder. You have hamartomas


everywhere (brain, lung, heart, skin, kidneys). The renal findings are multiple bilateral
angiomyolipomas. They also have renal cysts, and occasionally RCC (same rate as general
population, but in younger patient population). With regard to other organ
systems:
Tuberous Sclerosis can
Lung - LAM - thin walled cysts and chylothorax
Cardiac - Rhabdomyosarcoma (typically involve cardiac septum) be referred to as
Bourneville Disease
Brain - Giant Cell Astrocytoma, Cortical and Subcortical Tubers,
(for the purpose o f
Subependymal Nodules
fucking with you)
Renal - AMLs, RCC (in younger patients)

Calyceal Diverticulum: The classic look is shown on a


contrasted CT delayed (excretory phase). There will be a round
Fluid-Fluid Level near the collecting system. The levels you arc
seeing are excreted contrast layered under urine without contrast
within the diverticulum.

A common trick is to show a non-contrasted study that also appears


to have a fluid-fluid level. However, in this case it is actually a
fluid - debris level, representing “Milk of Calcium” in that same
calyceal diverticulum.

More on this in section 5.

365
T2 Dark Renal Cyst
Cysts are supposed to be T2 bright. If you see the “T2 Dark Cyst” then you are deahng with the
classic differential of:

(1) Lipid Poor AML


• A small percentage of AMLs are lipid poor in the
general population.
• For the purpose of multiple choice - If you see a lipid
poor AML (especially if you see a bunch of them)
you need to think about Tuberous Sclerosis - about
30% of their AMLs are lipid poor.

(2) Hemorrhagic Cyst


• These will likely be T1 bright

(3) Papillary Subtype RCC


• Remember that clear cells (the most common sub-type) are T2 HYPER Intense.
• Both Clear Cell and Papillary will enhance, - but the clear cell enhances more avidly (equal
to cortex on cortico-medullary phase).

Multicystic Dysplastic Kidney:


This is a peds thing (and is discussed in that chapter).
Quick refresher - this is the situation where you have multiple tiny
cysts forming in utero from some type of insult.

What you should know:


• "No functioning renal tissue, ” - shown with MAG 3 exam.
• Contralateral renal tract abnormalities occur like 50% of the time.
Typically you think of reflux (VUR) and UPJ Obstructions
UPJ
Obstruction
THIS vs THAT: MCDK vs Bad Hydro Contralateral
Kidney
In hydronephrosis, the cystic spaces are seen to communicate.
In difficult cases renal scintigraphy can be useful. MCDK will show no excretory function.

THIS vs THAT:Peripelvic Cyst vs Parapelvic Cyst

Para (beside): Originates from parenchyma, may compress the collecting


system. These look a lot like the cortical cysts that you see all the time, but
instead of bulging out - they bulge in.

Peri (around): Originates from renal sinus, mimics hydro. If you didn’t
have a pyelogram (delayed) phase - might be tricky to tell apart.

366
S E C T I O N 4:
In f e c t i o n

Pyelonephritis - This is a clinical diagnosis.


However, you do end up diagnosing it. The trouble Striated Nephrogram DDx:
starts in most cases as a retrograde infection - germs Acute ureteral obstruction
climb up into the kidney via the ureter from the Acute pyelonephritis
bladder. They enter the bladder from the urethra. The Acute renal vein thrombosis
most common bug is E. Coli (the poop bug) - use your Radiation nephritis
imagination how it gets into the urethra. Acutely following renal contusion
Vague focally reduced areas of enhancement with Hypotension (bilateral)
associated periphephric stranding is the described Infantile polycystic kidney (bilateral)
appearance on CT.
Striated Nephrogram is not specific for
However, pyelonephritis is most classically shown as a infection - infact it is seen in like a million
striped kidney (striated nephrogram) - seen best on an billion ba-gillion different things — but for
excretory phase . These wedge shaped areas are related the purpose o f multiple choice, think about
to decreased perfusion. anything that fucks with bloodflow or
causes edema

The exact appearance of the striated nephrogram stripes is a common source of trivia. In the setting
of acute pyelonephritis, the areas of reduced enhancement (stripes) classically involve a complete wedge
of renal parenchyma, extending from medulla peripherally all the wav to the capsule.

This is not the case with renal


infarcts - which tend to spare the
cortex. The preserved thin rim of
enhancement is called the “cortical
rim sign” and helps you distinguish
between infarct and infection (on
multiple choice exams). In the real
world the cortical rim is seen maybe
50% of the time with infarcts.
Infection - extends all the way to Infarct - spares the capsule
the capsule

( Pyonephrosis ) Pyelonephritis with Obstruction:


Pyelonephritis is classically associated with stones, but remember the actual N e x t S te p :
cause is a retrograde infection. Having said that, if infection is present in the • U rg en t
setting of obstruction (for example secondary to a stone) this becomes an D ec o m p res sio n
emergency - and a percutaneous nephrostomy is often indicated. (N e p h ro sto m y)
Pyonephrosis is the vocab word used for an infected or obstructed
collecting system. Fluid-Fluid level in the collecting system can be seen on
US.

Abscess - Pyelo may be complicated by abscess, which can present on N e x t S tep:


CT as round or geographic low attenuation collections that do not enhance • B ig g er T h an
centrally, but do have an enhancing rim. Bigger than 3cm and these guys 3 c m = D rain ag e
might visit the IR section for drainage.

367
Spectrum: Pyelonephritis Lobar Nephronia (ALN) Abscess
( Acute Focal Nephritis )
The spectrum of
disease is a Usually
source of described in
potential multiple Pediatrics (but
can occur in
choice trivia.
adults) ►
Here is my best
attempt at helping
you know which Poorly defined low density. Focal but not Well defined.
vocab words to Not round or oval. well defined. Cystic core, often
use along this Striated on excretory phase. Low density, but has a thick, irregular
spectrum.
not yet cystic. wall.

Chronic Pyelonephritis - Sort of a controversial entity. It is not clear whether the condition is
an active chronic infection, arises from multiple recurrent infections, or represents stable changes from
a remote single infection. The imaging findings are characterized by renal scarring, atrophy and
cortical thinning, with hj^ertrophy of residual normal tissue. Basically, you have a small deformed
kidney, with a bunch of wedge defects, and some hypertrophied areas.

Emphysematous Pyelonephritis: Emphysematous Pyelitis:


This is a life threatening necrotizing infection This is less bad relative to
characterized by gas formation within or surrounding emphysematous pyelonephritis.
the kidney. What you need to know: The gas is localized to the
collecting system.
(1) It is really bad - it believes in nothing Lebowski
(2) Diabetics almost exclusively get it - often fat smelly
It’s more common in women,
diabetics with one jagged front tooth (“it sure is
diabetics, and people with urinary
good for peeling oranges”).
obstruction. Radiographic finding
(3) Echogenic foci with dirty shadowing on ultrasound.
is gas outlining the ureters and
If there is air in the peri-nephric space , that’s
dilated calyces.
associated with a miserable shit outcome.

N e x t S te p :
• U rg en t
A n tib iotics
fo llo w e d by
N e p h re c to m y

Em physematous Pyelonephritis Em physem atous Pyelitis

368
Papillary Necrosis:

This is ischemic necrosis of the renal papillae, most commonly


involving the medullary pyramids.

Diabetes is the most common cause, but can occur with


pvelonephritis (especially in kids), sickle cell. TB. analgesic use.
and cirrhosis.

Filling defects might be seen in the calyx.

The appearance of a necrotic cavity in the


papillae with linear streaks of contrast inside
the calyx has been called a “lobster claw
sign. ”
Of
Trivia: 50% of sickle cell patients
develop papillary necrosis
Linear Streak in Calyx

Xanthogranulomatous
Pyelonephritis (XGP);
Chronic destructive granulomatous process that
is basically always seen with a staghom stone
acting as a nidus for recurrent infection. You
'im
can have an associated psoas abscess with
minimal perirenal infection. It’s an Aunt
Minnie, with a very characteristic “Bear Paw”
appearance on CT. The kidney is not
functional, and sometimes nephrectomy is done
to treat it. Xanttiogranulomatou$ Pyelonephritis - Bear Paw

HIV Nephropathy- This is the most common cause of renal impairment in AIDS (CD4 < 200)
patients. Although the kidneys can be normal in size, they are classically enlarged, and bright
(echogenic). Some sources will go as far as saying that normal echotexture excludes the disease (this
entity is essentially always bright). Loss of the renal sinus fat appearance has also been described (it’s
edema in the fat, rather than loss of the actual fat).

Just think - BIG and BRIGHT kidney in HIV positive patient who is clinically in nephrotic
syndrome (massive proteinuria).

Gamesmanship: To show you the kidney is big (longer than 12 cm) they will have to put

A calibers on the kidney. Calibers on anything should be a clue that the size being displayed is
relevant.

Final diagnosis is going to be via biopsy of the big bright kidney.

Disseminated PCP in HIV patients can result in punctate (primarily cortical) calcifications.

369
TB- The most common extra-pulmonary site of infection is
the urinary tract. TB in the kidneys is similar to TB in the
lungs with prolonged latency (years after exposure) and
“reactivation.” You could be shown imaging findings that
occur along a spectrum of severity. For the purpose of
multiple choice strategy the more severe disease would lend
itself better to imaging, and the less severe findings would
be more likely to be asked as trivia questions.

Calyceal blunting (“moth eaten calices”) is the earliest


finding. Remember the normal calyx should have a “gentle Normal IVP (from 1960)
cup shape” — not all pointy and shit. -Cup Shaped Calyces
-Symmetry of the Renal Pelvis
Distortion o f the calyx / papillary necrosis will result in
deep cups, and “w” shaped central necrosis patterns.
This is nonspecific - ischemia, diabetes, lots of things can do this
- but it is the “earliest described sign” which makes it testable.

Normal Calyx Progressed Focal stenosis Ureteral


Distortion Necrosis of an Stenosis can
/ Papillary leading to infUndibulum Cause
Necrosis “cavity” can result in Infundibulum Stenosis Generalized
formation. the absence of can also be diffuse and Hydronephrosis
This is the most opacification result in caliectasis
characteristic of the calyx Scarring at the renal
sign of renal (phantom pelvis can cause an
TB. calyx) uplifted appearance and
the classic “Kerr Kink”
at the renal pelvis.

Renal calcifications, which are very common with TB, can be


punctate, curvilinear, or replace the entire kidney. This
extensive calcification is a classic (and very testable) look for
TB - called the “Putty Kidney” - or an autonephrectomized end
stage TB kidney.

Another nonspecific finding - that can help you zero in on the


choice of TB is the presence of multiple calcified mesenteric
lymph nodes — arrows in my cartoon. Calcified adrenal glands
could also be seen.

370
Contrast Induced Nephropathy (CIN)
An Infectious Propaganda

The more you read about this the more you reahze it’s probably com plete (or at least
near complete) bullshit. Unfortunately a num ber o f academics (m ostly nephrologists)
have made a career on this and can be pretty defensive w hen the subject is brought into
question. For example, ju st the other day I had the following interaction w ith one such
mem ber o f the nephrology community.

Nephrologist: Hey! Do you believe in the plague o f Contrast Induced N ephropathy ?

Prometheus: I don’t know. There seems to be some controversy in the hterature...

Nephrologist: D on’t jerk me around ! It’s a simple question! A baby could answer it!

Prometheus: I guess so ...

Nephrologist: Oh, you made a wise choice, my friend! If you had said no, I w ould have
bitten your ear o f f ! I w ould have come at you like a tornado m ade o f arms and teeth.
A nd - and fingernails.

*Adaptedfrom SNL transcript 1/10/98

In your abundant free time, read this paper and becom e enlightened. I will warn you,
don’t let the nephrologists catch you reading it.

Davenport, M atthew S., et al. "The challenges in assessing contrast-induced


nephropathy: where are we now?." A m erican Journal o f Roentgenology 202.4 (2014):
784-789.

For the Exam — Trivia to know:


• Allergic reactions are N O T considered a risk factor for CIN.
• “Risk Factors” for CIN include pre-existing renal insufficiency, diabetes mellitus (even
more so with pre-existing renal insufficiency), cardiovascular disease with CHF,
dehydration, and myeloma.
• Hydration via IV with 0.9% normal saline 6-12 hours before and continuing 4-12
hours after contrast adm inistration supposedly decreases the incidence o f CIN in
patients with chronic renal insufficiency (true m echanism is diluting Cr levels).
Oral hydration has been shown to not w ork as well.

371
S E C T I O N 5:
C alcifications

Staghorn Stones
Nephrolithiasis (kidney stones)
- A mini Promethean Dialogue -
There are several different stone types. The most
The big branching “staghom” stones that you
Hkely testable trivia for each is: sometimes see filling the entire collecting system
• Calcium Oxalate - most common type (75%) are usually (but not always) “struvite” stones.
• Struvite Stone - More common in women and These struvite stones require alkaline urine to
associated with UTI form and grow. As such, these patients often have
• Uric Acid - “Unseen” on x-ray. a urease producing infection (Proteus or
• Cystine - Rare and associated with congenital Klebsiella) - hence their classic presentation is
disorders of metabolism fever and flank pain (not renal colic). These
• Indinavir - Stones in HIV patients which are stones are described as “insidious,” forged in the
the ONLY stones NOT seen on CT. dark mountains of Mordor, and growing slowly
into branching casts of the collecting system.
Their large size , branching morphology, and
Treatment Trivia: There are two pieces of trivia affiliation with the Necromancer of Dol Guldur
that matter with regard to treatment. makes passage rare. If they are to come out - it
Size Matters: often requires intervention.
• Stones measuring 5 mm or smaller have a high TLDR; Struvite Stones form Staghoms. They are
likelihood of spontaneously passing. Insidious in their background of Infection and
• Stones measuring 1 cm or larger have a high prefer to cause Fever rather than the Fearsome
likelihood of NOT passing spontaneously. pain of urinary colic.

Composition (Uric Acid vs Not Uric Acid):


• Uric acid stones very rarely will require any kind of invasive intervention (lithotripsy, etc...).
The reason is they are very pH dependent. Big Fat People and/or diabetics tend to have more
acidic urine (from all that Mountain Dew) which leads to an increase in uric acid stones. They
can be treated with medical therapy (potassium citrate or sodium bicarbonate) to increase the pH
and melt the stones. You can’t melt a calcium stone by messing with the pH.

Diagnosis of Uric Acid Stones:

Since identification of a uric acid stone is going to change management that makes it a target for
trivia on multiple choice. There are 2 things that I would know:
(1) Uric Acid Stones tend to have lower attenuation (< 500 HU).
(2) Uric Acid Stones will have little if any change in H.U. with dual energy CT. The reason is they
are composed of “light elements.” The larger atoms (Calcium, Phosphorous, Magnesium, and
Sulfur) tend to have a larger change - which is the basis of dual energy CT (80 kv, and 140 kv)
identification of stone composition.

Trivia: Non Uric Acid Stones will have higher HU at 80 kVp relative to 140 kVp.
Trivia: Uric Acid Stones will be very similar at 80 kVp relative to 140 kVp. *If they do show a
small change it will be the opposite - with a slightly higher HU at 140 kVp.
Trivia: Calcium stones are going to show the biggest HU change between high and low energies.
In general, low/high energy ratios are going to be around 1.1 for uric acid, 1.25 for cystine, and >
1.25 for calcium.

372
Milk of Calcium -
I touched on this briefly in section 3 in the setting of a calyceal diverticulum.
Milk of calcium is a suspension of
calcium crystals which frequently
masquerades as renal calculi within
a cyst or calyceal diverticulum. This
can be an easily tested source of
radiologic trivia - perfect for a lazy
question writer.

The classic look: Ultrasound


showing the layering “fluid - debris
level” that changes with positioning
of the patient.

Cortical Nephrocalcinosis
This is typically the sequela of cortical necrosis, which can be seen after
an acute drop in blood pressure (shock, postpartum, bum patients, etc...).
It starts out as a hypodense non-enhancing rim that later develops thin
calcifications.
Mimic is disseminated PCP.
Also remember TB can have a variable calcification pattern as discussed
earlier in the chapter.

Medullary Nephrocalcinosis
Hyperechoic renal papilla / pyramids which may or may not shadow on
ultrasound.
Causes:
• Hyperparathyroidism - Most people say this is the most common.
• Medullary Sponge Kidney - Some people say this is the most common.
• Lasix - Common cause in children.
• Renal Tubular Acidosis (distal subtype - type 1)
Trivia: RTA and Hyper PTH - tend to cause a more dense calcification
than medullary sponge.

Medullary Sponge Kidney


A congenital cause of medullary nephrocalcinosis (usually asymmetric). The underlying mechanism
is a cystic dilation of the collecting tubules of the kidney - so the testable association with Ehlers-
Danlos makes sense. The association with Caroli’s also sorta makes sense. The association with
Beckwith-Wiedemann doesn’t really make sense (and therefore is the most likely to be tested).

Think about medullary sponge kidney with unilateral less dense medullary nephrocalcinosis.

373
S E C T I O N 6:
PER FU SIO N / VASCULAR

Page Kidney
- This is a subcapsular hem atom a which causes
renal com pression and com plex fuckery with
the renin-angiotensin system. The result is
hypertension.

The capsule is the real issue here. That capsule


is tough and w on’t expand so the hem atoma
puts the squeeze on the “m eat” o f the kidney.
You could never get a “page pancreas” because
the pancreas has no capsule. This is the same
reason w hy resistive indices are worthless in
a pancreas (no capsule) but sometimes useful Subcapsular Hematoma + HTN = Page
in a kidney (which has a capsule).

Classic Clinical History: H ypertension post biopsy, lithotripsy, or trauma.

THIS vs THAT D e lay e d vs P e r s i s t e n t N e p h ro g ra m


Delayed Nephrogram - One kidney enhances and the other doesn’t (or does to a lesser
degree). Basically this is happening from pressure on the kidney, either extrinsic from a
Page kidney situation, or intrinsic from an obstructing stone.

Persistent Nephrogram - This is seen with hypotension/shock and ATN. They can show
this two ways, the first would be on a plain film o f the abdomen (with dense kidneys), the
second would be on CT. The tip offs are going to be that they tell you the time (3 hours
etc...) and it’s gonna be bilateral.

I
Delayed Nephrogram Persistent Nephrogram

374
Renal Infarct

So wedge shaped hypodensities in the kidney can be seen with lots o f stuff (infarct, tumor,
infection, e tc ...). Renal infarcts are m ost easily identified on post contrast im aging in the
cortical phase. I f the entire renal artery is out, w ell then it w on’t enhance (duh).

Two tricks that they could pull are the:

(1) “Cortical Rim Sign ” - which is absent


immediately after the insult, but is seen 8 hours
to days later. You have a dual blood supply,
which allows the cortex to stay perfused.
(2) “Flip Flop E nhancem ent” can be seen
where a region o f hypodensity / poor
enhancement on early phases becom es
relatively hyperdense on delayed imaging.

Renal Vein Thrombosis

Numerous causes; including dehydration, indwelling um bilical venous catheters (most


common in neonates), and nephrotic syndrome (m ost com m on in adults). This can mimic a
renal stone; presenting w ith flank pain, an enlarged kidney, and a delayed nephrogram.

On Doppler they are going to show you Reversed arterial diastolic flow and absent venous
flow. *This is discussed again in the subsequent transplant section.

Acute Cortical Necrosis


• Rare type o f acute renal failure with necrosis involving
only the renal cortex.
• Occurs secondary to severe hypovolem ia (think shock,
severe sepsis, intravascular hem olysis, and OBGYN
complications such as abruption etc).
• Classically shown w ith the R everse Rim Sign - non­
enhancing dark cortex with norm al enhancem ent o f the
renal medulla.
• A veeeery thin rim o f contrast enhancem ent (cortical
rim sign) can persist and this should not be m istaken for
sufficient perfusion.
Reverse Rim (the cortex is dark)
• In the chronic phase, the renal cortex becom es calcified.

375
S E C T I O N 7:
T ransplant

Renal transplant is the best treatment for end stage renal disease, and the quality o f life is
significantly better than that o f a long term dialysis patient (which fucking sucks !!!). The
transplanted kidney is most commonly placed in the extraperitoneal iliac fossa so that the
allograft can be anastomosed with the iliac vasculature and urinary bladder.
The way they do this surgery depends on where the kidney comes from (living vs cadaveric). The
main thing to know is that a kidney “harvested” from a hobo found floating in the river will have
not only the kidney and renal artery removed but also a segment o f the aorta - that can be used for
end-to-side anastomosis to the recipient external iliac artery. In a living donation they aren’t gonna
carve on the aorta (cuz you need your aorta to live). In both cases, end to side anastomosis is
preferred for the vein and artery - typically to the external iliac vein and artery (although you can
see the internal iliac used in some situations).
Normal: The superficial location o f the transplant in the iliac fossa makes ultrasound the
modality o f choice for evaluation. A transplant kidney is ju st like a native kidney. It should
have low resistance (it’s always “on”). The upstroke should be brisk, and the flow in diastole
forward (remember it’s always “on”).

- Understanding Renal RIs -


There are two major points to know first when thinking about RIs. The first is the kidney has
a capsule, and that capsule is unforgiving (it believes in nothing Lebowski). The second is a
sick kidney is a swollen kidney.
Peak Systolic - End Diastolic “lowest diastolic”
Rl =
Now lets look at this formula:
Peak Systolic

If the meat (parenchyma) o f the kidney is sick and


swollen, but can’t expand because it is wrapped in a Peak S V - End DV
tight unforgiving capsule you can imagine the Ri = = RI
blood vessels going through that kidney are going PSV
to get the squeeze.
You can also probably imagine that the passive
diastolic flow would be more impaired (compared
to the active systolic flow) by this squeeze. Low Resistance -

If the meat o f the kidney becomes “sick” from High Resistance


whatever the cause might be {rejection, infection, *decreased
inflammation, etc...) it swells increasing resistance. diastolic flow

R I’s should stay below 0.7. The higher the RI the sicker the kidney. This is why RIs are useful,
and this is why an upward trend in RI is worrisome. It is important to remember, RIs are not
specific since elevation occurs with basically every pathology. For the purpose o f multiple
choice, you should never use elevated RIs to exclude answers (unless the answer is normal).
Elevation in RIs does tell you something is wrong, especially if there is an upward trend.

376
The 3 Flavors of Complications - (a) Urologic, (Id Vascular (cl Cancer

- Urologic Complications - (more common than vascular complications)


Obstruction - The ureter must also be surgically implanted (ureteral neocystostomy to the bladder
dome). Just like in a native kidney the ureter can get obstructed. Don’t be fooled by the ultimate asshole trick of
showing you mild hydro on a transplant (especially one with normal labs) and trying to get you to call obstruction.
Transplanted kidneys pretty much all have some mild hydro - this is related to denervation of the transplant, and
floppy tone to the ureter. If there is a true obstruction it is usually at the site of ureteral implantation to the bladder.
The common causes are post operative edema, scarring, or technical errors leading to kinking. Stones, clots, etc.. are
less common. Having said that - transplants are more likely to have stones compared to the general population - it is
just that other stuff (edema, scar, k in k , etc... is more common).

Hematoma - Common immediately post op. Usually resolves spontaneously. Large hematoma can produce
hydro. Acute hematoma will be echogenic, and this will progressively become less echogenic (with older
hematomas more anechoic and septated).

Urinoma - This is usually found in the first 2 weeks post op. Urine leak or urinoma will appear as an
anechoic fluid collection with no septations, that is rapidly increasing in size. Most leaks (urine extravasation)
are going to be at the ureterovesical anastomosis. MAG 3 nuclear medicine scan can be used to demonstrate
this (or the cheaper ultrasound).

Lymphocele - Lymphoceles typically occur 1-2 months after transplant. They are caused by leakage of
lymph from surgical disruption of lymphatics or leaking lymphatics in the setting of inflammation. The fluid
collection is usually medial to the transplant (between the graft and the bladder). They are actually the most
common fluid collection to cause transplant hydronephrosis.

Buzzword = Ipsilateral lower extremity edema from femoral vein compression.

Hematoma Urinoma Abscess Lymphocele


Immediate Post Op - Around Day 10 Weeks to Months 2 weeks - 6 Months
Till about 1 week
Think Simple Collection Think Simple Collection
Think Complex between the bladder and (may have tiny septa).
Collection the kidney Think Complex
Hyperemic Collection CT: If you do a delayed
Heterogenous, Septa, CT: If you do a delayed phase you will NOT see
Etc... phase you can see leakage CT: Peripheral leakage of contrast
of contrast Enhancement
CT: Appearance will
Nukes; Tracers like
depend on how acute it is. Nukes; Tracers like US: Hyperemia (increased MAG3 and DTPA will
Acute = more dense. MAGS and DTPA will flow at the periphery) NOT accumulate outside
accumulate outside the the expected location of
MRI: T1 Bright (usually) expected location of the the bladder.
bladder.
Fluid Cr > Serum Cr Fluid Cr ~ Serum Cr
Fluid K+ > Serum K+ Fluid K+ ~ Serum K+

Fever and Elevated WBC They will Not usually


would be obvious clues - drain these things (they
These things happen from just come back). If they
although I expect the
ischemia to the ureter or do opt to treat them its
assholes would
obstruction (usually)
deliberately withhold that usually via sclerosing
information agent.

377
- Urologic Complications Continued ■ (more common than vascular complications)
Rejection is complicated business with a bunch of fancy sounding French words (maybe Latin) associated
with numerous overlapping biopsy related classification criteria and which subtype of the T-Cell mediated
pathway blah blah blah. None of that shit matters to Radiologists. Rejection workup involves, labs,
considering the time interval, ultrasound, maybe nukes, and in many cases a biopsy to actually prove it.

Hyperacute Rejection is an immediate failure of the graft - and you rarely see this imaged. It is
basically a dead on arrival transplant.
Acute Rejection is usually seen around week 1-3 (it is actually rare in the first 3 days). There is
overlap between the antibody mediated types (which occur early) and the T-cell activated types (which
occur later) - but again that shit is irrelevant to Radiologists. Up to 20% of transplant patients will have
some early rejection. The graft may swell and RIs will go up. Rejection vs ATN is the common question -
and MAG3 can help (see chart on the next page). Regardless of the Nukes Exam, most sources say
“biopsy” is the standard for differentiating the two.
Acute Tubular Necrosis (ATN) is common and occurs to variable degrees on basically every
transplant. The mechanism is ischemia in the kidney after they carve it out of the Hobo (presuming the
transplant is from the usual donor - Hobo found floating in the river). So in the time it takes to carve it
out of the Hobo and sew it into an affluent celebrity (Selena Gomez, Tracy Morgan, etc..) there is going to
be some ischemia - and therefore ATN.
Lingo: “Delayed Graft Function (DGF) ” = transplant requiring dialysis in the first week. The amount of
"cold ischemia” (how long the hobo kidney is on ice) is said to be the best predictor.
Cyclosporin Toxicity (Calcineurin Inhibitor) - Immunosuppressive therapy necessary to
keep the body from rejecting the graft can ironically end up poisoning the graft. The timing is usually
later than ATN (around a month). The MAG3 exam can also look like ATN (normal perfijsion, with
retained tracer) but will NOT be seen in the immediate post op period.

Chronic Rejection is a gradual progress process which occurs months to years after transplant. The
kidney may enlarge, and you can lose corticomedullary differentiation. The RIs will elevate (> 0.7),
which is nonspecific.

Acute Chronic
ATN Cyclosporin Toxicity
Rejection Rejection
... ?'
US “RIs” Elevated Elevated Elevated * Fkvdtcd
Antibody /
Ischemia During Nephrotoxic Reaction to Cellular Immune.
Mechanism Cell
“Harvesting” Immunosuppressive (T-Cell) Mediated
Mediated
Timing First Week First Week Month Months
Mag 3 - Normal to Mild
Crap Normal to Mild Delay Crap
Flow Delay
Mag 3 - Crap / Pretty Mueh
Pretty Much Normal i Crap/Delayed
Uptake Delayed Normal
Crap (slow Ciap (^siuW Crap (slow
Mag 3 - Crap (slow progressive
progressive progressive progressive
Making Piss excretion)
excretion) excretion) excretion)

378
- Vascular Complications -
Renal Artery Thrombosis always seen within the first month (usually minutes to
hours post odV resulting from technical factors - kinking or torsion of the vessel. Unless the patient
is undergoing rejection, or has renal artery stenosis (which has progressed to full on thrombosis) it is
pretty fucking rare to see this outside the early post-operative period. As a point of trivia - this is
different than hepatic artery transplant thrombosis — which is described as a later complication (>
than 1 month post op) — so don’t get it twisted and let the bastards trick you.

Renal Artery Stenosis - Typically seen within the first year after transplant (usually weeks
to months). Easily the most common vascular complication of transplant. This usually occurs at
the anastomosis (especially end-to-end types). CMV is a risk factor. The clinical / scenario buzzword
is going to be “refractory hypertension.” Criteria include:
• PSV > 200-300 cm/s. (some people say 340-400 cm/s)
• PSV ratio > 1.8-2.5x (Stenotic Part vs Non Stenotic Part)
• Tardus Parvus: Measured at the Main Renal Artery Hilum (NOT at the arcuates)
• Anastomotic Jetting

Renal Vein Thrombosis - Typically seen within the first week. Typically the kidney is
swollen. Instead of showing you the Doppler of the renal vein (which would show no flow), they will
most likely show you the artery, which classically has reversed diastolic flow.

Reversed
Diastolic
Flow

Renal Vein Throm bosis - with the artery showing reversal o f diastolic
flow. Some people call this the “reverse M sign.”

Arteriovenous Fistula (AVF) - These occur secondary to biopsy. They occur about 20% o f
the tim e post biopsy, but are usually sm all and asym ptom atic. They will likely show it w ith tissue
vibration artifact (perivascular, m osaic color assignm ent due to tissue vibration), w ith high arterial
velocity, and pulsatile flow in the vein.

Pseudoaneurysm - These also occur secondary to biopsy, but are less com m on. They can also
occur in the setting o f graft infection, or anastom otic dehiscence. They w ill m ost likely show you the
classic “yin-yang” color picture. Alternatively, they could show D oppler w ith biphasic flow at the
neck o f the pseudoaneurysm .

379
EARLY INTERMEDIATE LATE
Renal Allograft
Compartment Syndrome Renal Vein Thrombosis Renal Artery Stenosis (RAS)
(RACS)

Seen immediately - usually


Usually the first 5 days
< 2 hours post 3 months - Two Years
(peak at 48 hours)
transplantation

This is usually an operative


complication where the
kidney was too big for the Depending on who you
Depending on who you ask - this is the most
pelvic extraperitoneal space ask - like 30% of kids
common vascular complication in a transplant.
they decided to jam it and have graft failure because
when they stitch the fascia of this...
Classic History is “hvpertension refractor/ to
back up it puts the squeeze
treatment”
on the kidney. You can The incidence is less that
imagine if there was a fluid RAS - but the morbidity
The lesion is usually at the level of the surgical
collection nearby that would associated with it is still
anastomosis.
make it worse high.

Doppler: PSV > 250 cm/s


Doppler: Absent or almost or 1.8-2.5x increase from the “normal” vessel.
Doppler: Flow in the
totally absent cortical flow There is a trend towards using 340-400 cm/s for a
vein is gone. Reversed
in the kidney (color or more specific call. Which number will the test
diastolic arterial flow.
power Doppler) writer want you to use? Simply read his or her
mind to find out and answer accordingly.

Rx: Most places will try and angioplasty the


Transplant Size matters - a stenosis first. Having said that, if it is right over the
large transplanted kidney is Grey Scale: Swollen /
anastomosis - you will hear people say surgery
the main risk factor enlarged kidney.
might be safer / better (higher risk of ripping the
associated with RACS.
thing in half when you stretch it)

Occurring at Anytime After Trauma or Procedure


Renal Artery Thrombosis
AV Fistula Pseudoaneurysm Traumatic Hematoma
Usually very early (mins to
hours) as a post op Feeding artery with a To-and-Fro pattern of This can occur after
complication (clamp injury high velocity low- blood flow within the biopsy or from blunt
etc...) resistance waveform neck trauma.

It is rare to see this late (in Vein with a turbulent Yin-yang sign of Transplant kidneys don’t
the absence of progressive arterial appearance. swirling blood within have ribs and are fairly
stenosis or raging rejection). the sac superficial - so they can
Tissue Vibration Artifact get banged up in minor
Doppler: Flow is gone trauma.

Grey Scale: Might see Usually - no clinically Often need an Complex Collection
wedge shaped hypoechoic significant intervention - especially
infarcts hemodynamic if large (historically > 2 Heterogenous, Septa,
consequence and no cm) or increasing in size Etc...
intervention needed

380
-Cancer-
The prolonged im m unosuppression therapy that renal transplant patients are on places them
at significantly (lOOx) increased risk o f developing some type o f cancer. In particular, they
get more nonmelanomatous skin cancer, lym phoma, and colon cancer. In fact - annual skin
exams are a recom m endation for all renal transplant patients — that is kinda random .... and
possibly testable.

RCC - Increased risk, with most o f the cancers (90%) actually occurring in the native
kidney. Etiology is not totally understood; m aybe it’s the im m unosuppression or the fact that
many transplant patients were on dialysis (a know n risk factor) that leads to the cancer risk.
In reality it doesn’t matter, and is probably both.

Risk o f prim ary renal malignancy in the transplanted kidney is six times that o f the regular
Joe. A point o f testable trivia is that these cancers are usually papillary subtypes.

Post Transplant Lymphoproliferative Disorder (PTLD) - This is an


uncom mon com phcation o f organ transplant, associated with B-Cell prohferation. EBV is a
risk factor and that is one o f the main reasons they screen for it - and will put people on
Rituximab (if they m ention that drug - they could be hinting that patient was EBV positive).

It is m ost com mon in the first year post transplant, and often involves multiple organs. The
most typical look is a mass lesion encasing / replacing the hilum - although the appearance is
notoriously variable. The treatment is to back o ff the im munosuppression.

Renal Transplant + BK Virus = Urothelial Malignancy

WTF is “B K Virus” ? It is some random virus that pretty much everyone gets and doesn’t even
notice. Nephrologists love to write papers on this critter. S upposedly... (yes I read their stupid
papers) it is usually the donor kidney that has it and then it reactivates som ething crazy once the
patient is immunosuppressed. Sometimes it even m im ics rejection.

Just know BK = U rothelial Cancer.

Cyclophosphamide - As a point o f trivia, significant exposure to cyclophospham ide


(less com mon now w ith the developm ent o f cyclosporin A) is associated with increased risk
o f urothelial cancer.

Femoral Head AVN After Renal Transplant — This used to be a classic MSK cross over
A question because it was very common (-20%). The rate has decreased significantly in
modem time (most sources say something like 4-5%) because drugs like cyclosporine
allowed for reduced steroid doses. Having said that, boards questions tend to be written by
A those who trained during the Cretaceous period - so it is still probably a high yield trivia.

381
S E C T I O N 8:
RENAL T rauma

Obviously the kidney can get injured in traum a (seen in about 10%). Injury can be graded
based on the presence o f hem atom a -> laceration -> involvem ent o f the vein, artery, or UPJ
obstruction.

Gamesmanship: A good “N ext Step” type question in the setting o f renal traum a
(or pelvic fracture) w ould be to prom pt you to get delayed imaging - this is helpful
C to dem onstrate a urine leak.

Terminology:
• “Fractured Kidney” - A laceration, w hich extends the full
depth o f the renal parenchyma. By definition the laceration m ust
connect two cortical surfaces - so think about it going all the way
through.

“Shattered Kidney” - This is a more severe form o f a


fractured kidney. A kidney with 3 or more fragments - this is the
m ost severe form o f renal fracture.

Renal Trauma - Rapid Pearls

Wedge Shaped Perfusion Diffuse NonPerfusion- Persistent Nephrogram


Abnormality - T h in k T h in k D e v a s c u la riz e d — T h in k R en a l Vein
S e g m e n ta l A rtery Injury K idney Injury / T h ro m b o sis

Trivia: A transplant kidney is at increased risk o f injury in m ost traum a because o f its superficial
location (and loss o f the norm al rib protection).

382
Grading Trauma - A Source of Potentially Testable Trivia
Like many things you would simply look up on call, the American Association for the Surgery of Trauma
(AAST) Renal Injury Scale is a source of potential multiple choice trivia. I will attempt to summarize
and highlight the most easily tested trivia.
Indications: Typically any “trauma” that triggers the classic Delays (5-15 mins)
trauma protocol. More specific indications include gross Shows urine leaks (will be hyper
hematuria or micro-hematuria with a low blood pressure dense - and outside the collecting
(SBP < 90). Lower rib fractures or flank bruising in the system)
setting of trauma are also typically listed in trauma texts.
Focal Collections of contrast that
Protocol: Standard trauma protocol is an arterial phase (aorta
DECREASE in attenuation on
timed) through the chest and portal venous phase (~70
delays: think pseudoaneurvsm or
seconds) through the abdomen and pelvis. If a ureteral injury AV fistula
or bladder injury is suspected excretory phase “delays” should
be obtained (5-15 minutes). You will also hear people say to Focal Collections of contrast that
get arterial imaging of the renal vessels if a renal vascular INCREASE in attenuation on
injury is suspected. delays: think active bleeding

GRADE 1: GRADE 2:
Hematoma
around the
Subcapsular kidney is
Hematoma within the
NO Laceration peri-renal
space

Mild (< 1cm) Laceration


GRADE 3:
Management
Grades 1, 2, and 3 are
typically managed
conservatively

Grades 4 and 5 are


NO involvement typically managed
Moderate Active bleeding only surgically
(> 1cm) Laceration of the renal pelvis, within the perirenal
NO urine leak space

GRADE 4: GRADE 5:

Laceration Segmental “Shattered Renal Hilum Devascularised


that extends Infarcts Bleeding Kidney” Injury (urine Kidney
into the hilum (from outside leak, or main
“Fractured vascular the perirenal renal artery /
Kidney ” injury) space vein injury)

383
Developmental Ureteral Anomalies: D iscussed in the Peds chapter.

Ureter Infection / Inflammation:


Stones - Stones tend to lodge in 3 spots: UPJ, UVJ, pelvic brim.

Ureteral Wall Calcifications - Wall calcifications should make you think about
two things: (1) TB, (2) Schistosomiasis (worms).

Ureteritis cystica - N um erous tiny subepithelial fluid-filled cysts within the w all o f
the ureter. The condition is the result o f chronic inflam m ation (from stones and/or chronic
infection). Typically this is seen in diabetics w ith recurrent UTI. There may be an increased
risk o f cancer.

Ureteral Pseudodiverticulosis - This is sim ilar to ureteritis cystica in that both


conditions are the result o f chronic inflam m ation (stones, infection). Instead o f being cystic
filling defects, these guys are multiple small outpouchings. They are bilateral 75% o f the
time, and favor the upper and m iddle third. There is an association with malignancy.

Malakoplakia ( “The A ccu rsed ”) - Form er Lord o f the D ark Elves o f Svartalfheim and
rare chronic granulom atous condition, this pathology can create soft tissue nodularity /
plaques in the bladder and ureters (bladder m ore often). It is seen in the setting o f chronic
UTIs (highly associated with E.ColiV often in fem ale im m unocom prom ised patients.
There is also a more remote association w ith the Casket o f A ncient Winters. Since
malakoplakia m ost frequently m anifests as a mucosal mass involving the ureter or bladder,
the m ost com mon renal finding is obstruction secondary to a lesion in the low er tract.
Step 1 buzzw ord = M ichaelis-G utm ann Bodies.

The m ost easily tested piece o f trivia is ------------------------------------------------------------


this: M alakoplakia is Leukoplakia = Premalignant
NOT prem alignant, and usually gets Malakoplakia = NOT Premalignant
better with antibiotics.

Leukoplakia - This is essentially squamous m etaplasia secondary to chronic irritation


(stones or infections). The bladder is m ore com m only involved than the ureter. Imaging
findings are unlikely to be shown, but w ould be m ural filling defects.

The m ost easily tested piece o f trivia is this: Leukoplakia is considered prem alignant and the
cancer is squamous cell.

Trivia: Leukoplakia is associated w ith squamous cell carcinom a N O T transitional cell.

384
Retroperitoneal Fibrosis
This condition is characterized by proliferation o f aberrant fibro-inflam m atory tissue, which
typically surrounds the aorta, IVC, iliac vessels, and frequently traps and obstructs the
ureters. It is idiopathic 75% o f the time. O ther causes include prior radiation, m edications
(methyldopa, ergotamine, methysergide), inflam m atory causes (pancreatitis, pyelonephritis,
inflammatory aneurysm), and m alignancy (desm oplastic reaction, lymphoma).

Things (trivia) to know:


• M ostly (75 %) idiopathic A KA “O rm ond D isease”
• Associated with IgG4 disorders (autoim m une pancreatitis, Riedel's thyroiditis,
inflamm atory pseudotum or)
• Classically shown with medial deviation o f ureters
• It’s more com mon in men
• M alignancy associated RP fibrosis occurs about 10% o f the time (some people advocate
using PET to find a primary)
• The Fibrosis will be Gallium avid, and PET hot in its early stages and cold in its late
stages (mirroring its inflam m atory stages). M etabolically active RP fibrosis will show
increased FDG and Gallium uptake, regardless o f a benign or m alignant underlying
cause

Subepitheiiai Renal Pelvis Hematoma:


This tends to occur in patients on long-term anticoagulation or a history o f hem ophilia. You
are going to have a thickened upper tract wall - which is a classic mimic for TCC.

Gamesmanship: I would expect pre and post contrast images so that you can make
the classic findings o f hyperdense clot on the pre-contrast that does NOT
enhance. Although a non-contrast alone (showing blood in the urinary pelvis) with
the history o f hem ophilia should also be enough to seal the deal.

THIS vs THAT: Deviation of the Ureters


Medial Deviation
Lateral Deviation of the “Waisting” of the Ureters
Ureters
Retroperitoneal Fibrosis
Retroperitoneal
Adenopathy Retrocaval Ureter
(right side)
Aortic Aneurysm Pelvic Lipomatosis
Psoas Hypertrophy Psoas Hypertrophy
(proximal ureter) (distal ureter)

385
S E C T I O N lO:
U reteral Masses

Transitional Ceil Carcinoma (Urotiielial Carcinoma) - R ISK FACTORS


This histologic subtype makes up a very large majority (90%) of the Smoking
collecting system cancers. Imaging buzzword is “goblet” or Azo Dye
“champagne glass sign” on CT IVP. Cyclophosphamide
Aristolochic acid (Balkan
High Yield Statistical Trivia Nephropathy - see below)
— a way to remember this is where ever the urine sits static the longest Horseshoe Kidney
is more likely to have the cancer. Stones
So: Bladder > Renal Pelvis > Ureter. If you are getting cancers in the Ureteral
ureter than you probably also have them in the bladder Pseudodiverticulosis
Hereditary Non-Polyposis
Ureter is the least common location for TCC of the urinary tract Colon Cancer (type 2)
TCC of the renal pelvis is 2x -3x times more common than ureter
TCC of the bladder is lOOx times more common than ureter
In the ureter 75% of the TCCs are in the bottom 1/3
If you have upper tract TCC there is a 40% chance of developing a bladder TCC
If you have bladder TCC there is a 4% chance of developing a Renal Pelvis or Ureteral TCC
Ureteral TCC is bilateral 5%

Balkan Nephropatiiy - This is some zebra degenerative nephropathy endemic to


the Balkan States. The only reason I mention it is that it has a super high rate of renal
pelvis and upper ureter TCCs. It’s thought to be secondary to eating aristolochic acid
(AA) in seeds of the Aristolochia clematitis plant (herb).

Squamous Cell - This is much less common than TCC (in the US anyway). The major
predisposing factor is schistosomiasis (they both start with an “S”).

Hematogenous Metastasis - Mets to the ureters are rare but can occur (GI, Prostate, Renal,
Breast). They typically infiltrate the periureteral soft tissues and demonstrate transmural
involvement.

Fibroepitlielial Polyp - This is a benign entity presents as a filling


defect in the renal pelvis or proximal ureter - which mimics a TCC (blood
clot or radiolucent stone). The diagnosis is typically made post THISvsTHUT:
nephrectomy - since the assumption is nearly always TCC.
P o lp TCC
Gamesmanship: For the purpose of multiple choice (and real life), renal Younger Older
pelvis filling defects should always be assumed to be either clot, calcium (30-40) (60-70)
(stone), or cancer. The only way I can think that a polyp question could be
ask would be something like “which features would make a polyp more Smooth / Irregular
likely?” It has to be a trivia question, they couldn’t (in good conscious) Oblong
expect you to pick polyp over TCC with imaging alone — even if the Mobile Fixed
findings and demographics were perfect - that would be teaching a terrible
clinical message.

386
S E C T I O N 1 l:
B ladder

Normal Anatomy: Normal bladder is an extraperitoneal structure, with 4 layers. The dome
of the bladder has a peritoneal cover. It’s lined with transitional urothelium.

- Developmental Anomalies -
Prune Belly (Eagle Barrett Syndrome)
This is a malformation triad which occurs in males. Classically
shown on a babygram with a kid shaped like a pear (big wide
belly).

Triad.
• Deficiency o f abdominal musculature
• Hydroureteronephrosis
• Cryptorchidism
(bladder distention interferes with descent o f testes)

Urachus : This is also discussed in the Peds chapter. I will briefly


mention that the primary concern is the development o f a midline
adenocarcinoma. Most o f the time the presence o f a midline mass makes
it obvious although calcification within any urachal soft tissue should
make you think cancer.

Bladder Diverticula - These are more common in boys, and can be seen in a few situations.
Most bladder diverticula can also be acquired secondary to chronic outlet obstruction (big
prostate). There are a few syndromes (Ehlers Danlos is the big testable one) that you see them in
as well.

‘^Hutch Diverticulum:” Also discussed in the Peds chapter.

Quick refresher on some key points.


• NOT associated with posterior urethral valves or neurogenic bladder.
• Hutch Diverticula is associated with ipsilateral reflux - and referred to as “secondary reflux”
• Bladder Diverticula typically arise from the lateral walls or near the ureteral orifices
• Diverticula at the anterior / superior bladder are more likely to be urachal diverticula
• Most Diverticula are acquired (not congenital)
• Ureters more commonly deviate medially adjacent to a diverticula

Bladder Ears - “Transitory extraperitoneal herniation o f the bladder” if you want to sound
smart. This is not a diverticulum. Instead, it’s transient lateral protrusion o f the bladder into the
inguinal canal. It’s very common to see, and likely doesn’t mean crap. However, some sources
say an inguinal hernia may be present 20% o f the time. Smooth walls, and usually wide necks can
help distinguish them from diverticula.

387
-Bladder Cancer-
Gamesmanship: GROSS hematuria confers a 4x greater risk than microscopic hematuria. If the
question header specifically indicates “GROSS” hematuria - think bladder cancer first.

c Next Step: Along these lines if the history is GROSS hematuria, and the patient is 50 or older ■
they should get a CT Hematuria Protocol / Urography (pre and post, with delays), and also
cystoscopy.

What Does Marcelus Wallus’ Bladder Cancer Look Like ? Short answer = soft tissue in the bladder.
If you are looking at a well distended bladder (which is pretty much required to say shit about the
bladder) focal wall thickening or nodules should be considered cancer till proven otherwise.

What about diffuse circumferential bladder wall thickening ? This isn’t usually cancer, especially in
the world of multiple choice. This is probably more of an inflammation or infection situation - or chronic
partial outlet obstruction (if the prostate is enormous). I’d only call a cancer in this situation if there was
really asymmetric nodular thickening superimposed on circumferential thickening.

What about enhancement ? You will hear people refer to bladder cancers as hypovascular tumors - but
they can and often do enhance, especially on early arterial phases. Any focal enhancement should trigger
you to think cancer - unless you’ve got good reasons to think otherwise. Having said all that - most
people will say the delaved phase is the most important for identifying bladder cancers - and that is the
choice I would recommend if you are forced to choose (white background of contrast - makes soft tissue
masses easier to se e ).

Types of Cancer I Mimics:

Rhabdomyosarcoma - This is the most common bladder cancer in humans less than 10 years of age.
They are often infiltrative, and it’s hard to tell where they originate. “Paratesticular Mass” is often a
buzzword. They can met to the lungs, bones, and nodes. The Botryoid variant produces a polypoid
mass, which looks like a bunch of grapes.

Transitional Cell Carcinoma (Urothelial Carcinoma) - As stated above, the bladder is the most
common site, and this is by far the most common subtype. All the risk factors, are the same as above.
If anyone asks “superficial papillary” is the most common TCC bladder subtype.

Squamous Cell Carcinoma - When I say Squamous Cell Bladder, you say Schistosomiasis. This is
convenient because they both start with an “S.” The classic picture is a heavily calcified bladder and
distal ureters (usually shown on plain film, but could also be on CT). Another common association
with squamous cell cancer of the bladder in the presence of a longstanding Suprapubic catheter. This
also starts with an “S.”

Adenocarcinoma of the Bladder - This is a common trick question. When I say Adenocarcinoma of
the Bladder, you say Urachus. 90% of urachal cancers are located midline at the bladder dome.
Bladder Exstrophy is also associated with an increased risk of adenocarcinoma.

Leiomyoma (“bladder fibroid”) - Benign .


tumor (not cancer - even though the section is . (smooth, solid, homogeneous)
bladder cancer). It s often incidentally . young/Middle Age People (usually)
discovered (most common at the trigone). . clinical Buzzword “urinary hesitancy” or “dribbling"

388
Types of Bladder Cancer
Transitional Cell - also
known as Urothelial Squamous Cell Adenocarcinoma Rhabdomyosarcoma
Carcinoma
By far the Most Common Second Most Third Most Most common bladder
Type (like 90%) Common Type (like ~ Common Type tumor in Peds.
8 %) (like 2%)
Smoking is the classic risk Classic Associations: Classic They are often
factor, but other poisons • Recurrent urinary Associations: infiltrative, and it’s
including arsenic, aniline, tract infections and • Urachal Remnant hard to tell where they
benzidine - etc or as they stone disease • Bladder originate.
call it in Flint Michigan “Tap • Schistosoma Exstrophy
Water” - have a documented Hematobium “Paratesticular
relationship (asshole jungle Mass” is often a
worm) buzzword.
Bladder Diverticulum - • Longstanding
2-10% increased risk (related Suprapubic They can met to the
to stasis). In this setting Catheter lungs, bones, and
early perivesical fat invasion nodes.
is classic (because a This is convenient
diverticulum has limited because they both
muscle in the wall to slow start with an “S.”
the invasion)
Favors the base 90% of urachal
(inferior posterior) cancers are located
midline at the
bladder dome.
Sub-divided into The classic picture is The classic picture The classic look is
Papillary vs a heavily calcified is a large midline Grape-like polypoid
Non-Papillary. bladder and distal mass associated masses — this is the
ureters (usually with a urachal sarcoma botryoides
Papillary ones look like shown on plain film, remnant with variant
shrubs “frond like” and tend but could also be on scattered
to be low grade. CT). calcifications.

Non-Papillary tends to be Tumor favors the 70% of cases have


more aggressive. trigone and lateral calcification (if
walls you see
calcifications in a
The ones NOT urachal remnant it
associated with should make you
Schistosomiasis tend think about an
to be more early cancer).
aggressive.

389
- Bladder Hap Hematoma -
This isn’t a bladder surgery but rather a surgical “complication” from a c-section. Remember that the
uterus sits directly behind the bladder and can easily be injured directly during a wild west / 3rd world
style panic c-section (“the trainee caused a complication”). More common than the bladder being directly
lacerated by the Attending (and blamed on the resident) is the formation of a bladder flap hematoma.
So what is a bladder flap hematoma ? This is essentially blood (hematoma) that drapes over the top and
back of the bladder after a c-section performed with the common technique of a lower uterine incision.
As shown in these awesome illustrations I made, the uterus is cut open at the lower segment to access this
big headed rascal. After he’s been removed from the warm safety of his mother’s womb and plunged into
a cold callus universe, there will remain a potential defect in the uterus. If bleeding occurs there, it will
droop (ooze) down on top of the bladder forming the legendary bladder flap hematoma. This basically
always happens and is considered “normal” in OB literature as long as it is under 4 cm. They go away on
their own with time. The only thing you worry about is super infection. If the history says they got a
fever , etc... etc... few days later and there is gas in the collection — it’s probably infection.

Defect in uterus allows Blood (black)


Incision is made in Classic Look on Ultrasound
blood to ooze down (arrow) accumulates behind
lower uterine segment Hematoma flapped over the
the bladder (B)
bladder

-Dhieislon Surgery-
After radical cystectomy for bladder cancer there are several urinary diversion procedures that can be
done. People generally group these into incontinent and continent procedures. There are a ton of these
(over 50 have been described). I just want to touch on the big points, and focus on complications (the
most testable subject matter). The general idea is that a piece of bowel is made into either a conduit or
reservoir, and then the ureters are attached to it.

Early Complications: Late Complications ( > 30 days)


• Alteration in bowel function: • Urinary infection: This can be early or late.
Adynamic ileus is the most common • Stones: Remember to look on the non-contrast study.
early complication, occurring in
almost 25% of cases. In about 3% of • Parastomal Herniation: This occurs about 15% of the
cases you can get SBO, usually from time with ileal conduits. Obesity is a contributing
adhesions near the enteroentcric factor. Most don’t matter, but 10% will need a surgical
anastomosis. fix.
• Urinary Leakage: This occurs in about • Urinary stricture: The left side is higher risk than
5% of cases, and usually at the the right, secondary to the angulation (it’s brought
ureteral-reservoir anastomosis. A through or under the mesentery).
urinoma can develop when the leaked • Tumor Recurrence: The more advanced the original
urine is not collected by urinary drains. disease, the higher the risk for recurrence. The
• Fistula: This is uncommon and seen incidence is between 3-15%, and can present as a soft
more in patients who have had pelvic tissue mass at the ureter, bladder, or pelvic lymph
radiation. node.

390
- Psoas Hitch -
The “psoas hitch” procedure results in an Aunt M innie appearance o f the bladder, m aking it
uniquely testable.

This procedure is done in the situation where you have had an injury or pathology (stricture,
cancer, e tc ...) involving a long segment o f the distal ureter. N orm ally you w ould ju st cut that
shit out and re-im plant into the bladder. But w hat if the left over portion o f the ureter is too
short? The solution is to stretch the ipsilateral portion o f the bladder towards the short ureter
and sew it (“hitch it”) to the psoas muscle. That w ay you can get away w ith a short ureter,
because you stretched the bladder to bridge the gap.

Key Points:

• fVhy i t ’s done? Used for people with long segm ent distal ureter injury / disease
• Aunt M innie Appearance on CT IVP or Plain Film IVP (with contrast filling the bladder).

Aunt Minnie Appearance of the


Hitched Bladder filled with
contrast on a delayed IVP.

The “hitched” side has an upward


projection towards the psoas
muscle.

391
- “Acquired ” — Infectious / infiammatien -
Emphysematous Cystitis - Gas forming organism in the wall o f the bladder. More
than h alf the time it’s a diabetic patient. It’s usually from E. Coli. It’s gorma be very obvious
on plain film and CT. Ultrasound would be sneaky, and y o u ’d see dirty shadowing.

TB - The upper GU tract is more com monly effected, with secondary involvem ent o f the
bladder. Can eventually lead to a thick contracted bladder - buzzword: ‘Thim ble’ bladder.
Calcifications m ight be present.

Schistosomiasis - Comm on in the third world. Eggs are deposited in the bladder wall
which leads to chronic inflammation. Things to know: the entire bladder will calcify (often
shown on plain film or CT), and you get squamous cell cancer.

Fistula - This occurs basically in 3 Fistula - Most Common Etiology


conditions; (1) diverticulitis,
(2) Crohns, (3) Cancer. This is more Colovesicial Fistula = D iverticular Disease
common in men, although w om en are Ileovesical Fistula = Crohns
at significantly increased risk after Rectovesical Fistula = N eoplasm or Trauma
hysterectomy (the uterus protects the
bladder).

Neurogenic Bladder - This comes in two flavors: (a) small contracted


bladder, (b) atonic large bladder. The buzzw ord / classic sign is “p in e cone ”
bladder, because o f its appearance. It can lead to urine stasis, and that stasis
can predispose to bladder CA, stones, and infection.

Acquired Bladder Diverticula - As m enfioned above, these can be acquired mainly


via outlet obstruction (just think big prostate). They are m ost com m on at the UVJ. They can
lead to stasis, and that stasis can predispose to bladder CA, stones, and infection.

Bladder Stones - These guys show up in two scenarios: (1) they are bom as kidney
stones and drop into the bladder (2) they develop in the bladder secondary to stasis (outlet
obstrucfion, or neurogenic bladder). They can cause chronic irritation and are a know n risk
factor for both TCC and SCC.

“Pear Shaped Bladder” - This is m ore o f a sign than a pathology.


Think two things (I) pelvic lipomatosis, and (2) hem atoma.

or

392
-Bladder Trauma-
In the setting of bladder trauma - Cystography (scanning the bladder after filling it with contrast
“retrograde” via foley) is the gold standard — this can be done under fluoro or CT. I’ve head people say
this is “ 100%” sensitive for bladder rupture. It’s pretty obvious — if it looks intact ... its intact. If it
looks like a deflated balloon — its ruptured. Having said that — here is a testable piece of trivia, you
must distend the bladder — that means 300-400 mL of diluted water soluble (not barium!) contrast.
Inadequate bladder distention = loss of sensitivity.

What they want you to know is; extra versus intra peritoneal rupture. CT Cystography (contrast
distending bladder) is the best test - make sure the bladder is distended (300-400 mL).

Extraperitoneal-This one is more common (80-90%). Almost always associated with pelvic fracture.
This can be managed medically.

• If there is a pelvic fracture, then the chance of a bladder rupture is 10%.


• If there is a bladder rupture, there is almost always a pelvic fracture
• Molar Tooth Sign: Contrast surrounding the bladder, in the prevesicle space of Retzius. This indicates
extraperitoneal bladder rupture.

Trauma!
“Normal” EP Rupture Molar Tooth Appearance
Potential P rev es ic le S p a c e Contrast from the B la d d e r filling
A nterior to the B ladder the P re v e s ic le S p a c e (Rezius)

Intraperitoneal - This one is less common. A direct blow to a full bladder, basically pops the balloon and
blows the top off (bladder dome is the weakest part). The dude will have contrast outlining bowel loops
and in the paracolic gutters. This requires surgery.

Tf Trauma!
Normal IP Rupture Contrast
Outlines Bowel

“Pseudo Azotemia” (Pseudo Renal Failure) - If the bladder is ruptured the creatinine in urine can be
absorbed via the peritoneal lining. This will massively elevate the creatinine making it seem like the
patient is in acute renal failure. The kidneys are normal.

393
S E C T IO N 12:
URETHRA

- Male -
Normal Anatomy (most commonly seen on a RUG), is high yield. Here are the basics:

• The length is highly variable with most texts using the following graded scale;
(itty bitty ^ teeny tiny ^ tiny ^ small ^ medium ^ large extra large ^ Lionhart sized).
• Anatomists divide the thing into two main parts - anterior and posterior. The anterior part is made
up of the penile urethra + the bulbar urethra. The posterior part is made up of the membranous and
prostatic urethra.
• The most anterior portion of the urethra is termed the fossa navicularis which has a Latin / French
sound to it, so it is probably testable.
• The “Verumontanum” (another fancy sounding term) is an ovoid mound that lies in the posterior
wall of the prostatic urethra. An additional testable piece of trivia is that in the center of this thing is
the prostatic utricle (which is an embryologic “mullerian” remnant).
• The anterior part fills with a retrograde study (RUG). The posterior part fills with an antegrade
study (voiding urethrography). “Dynamic Urethrography” is the term used when these studies are
combined. You can fill the whole thing with a RUG - but that requires pressure to overcome the
normal spasms of this cruel and unusual procedure.
• There are two methods for identifying the bulbar-membranous junction (which is important for
delineating pathology - anterior vs posterior). The first is to find the “cone” shaped appearance of
the proximal bulbar urethra. The cone will taper into the membranous portion. The second (used if
you can’t opacify the urethra) is to draw a line connecting the inferior margins of the obturator
foramina.
Trauma:

Trauma + Blood in the meatus = “N ext Step” RUG.

Anterior Injury - Classic = Crashed your bicycle (or tricycle), i.e. = Straddle Injury. Unicycle
w reck is less often associated with urethral injury (because o f the lack o f cross bar) but is
often associated with various juggling, fire eating, and sword swallow ing injury patterns - as
well as mania, and histrionic personality disorder.

Posterior Injury - Classic = Crashed your Ferrari. Testable trivia = Often associated with a
pelvic fracture and bladder injury. I speculate that crashing one’s Ferrari is also associated
with sudden and severe atrophy o f the penis.

1970s Classification System - Fair Game for the “ Exam o f Future.’

Type1 Type 2 Type 3 Type 4 Type 5


(most common)

“STRETCHED" Membraneous Membraneous + Bladder Base Bulbous Urethra


Urethra Tear Bulbar Urethra injury extending “Anterior”
Tear into Prostatic
Urethra

Posterior Posterior Anterior + Posterior Anterior


Posterior

Urethra is intact A BOVE an UG Diaphragm is


and normal on INTACT UG ruptured.
RUG Diaphram

Extraperitoneal Extraperitoneal Straddle Injury


contrast is contrast is
present present

No perineum Perineum
contrast (intact contrast (UG
UG diaphragm diaphragm is
prevents this) torn). Contrast in
the scrotum.

Associated with Associated with Associated with


Incontinance Incontinance Incontinance

395
- Urethral Strictures -
THIS vs THAT: The Bicycle Crossbar Injury VS
The Injury From a Woman of Questionable Moral Standard

Straddle Injury: The m ost


com mon external cause o f
traum atic stricture is this type o f
mechanism. The physiology is
com pression o f the urethra
against the inferior edge o f the
pubic symphysis. The bulbous
urethra is the site o f injury
(this is the m ost likely
question).

Gonococcal Urethral Stricture: This tends to be a long irregular stricture


(the straddle stricture was short). It occurs in the distal bulbous urethra.

Straddle Injury Gonococcal


S h o rt S e g m e n t - B ulbous U reth ra Long S e g m e n t & Irreg u lar- B ulbous U reth ra

396
other Male Urethral Pathologies:
Pancreatic Transplant: This has been known to cause urethral injury, if the drainage is to the
bladder (the old way of doing it). Extravasation from urethral injury is said to occur in about 5% of
cases and is secondary to pancreatic enzymes jacking the urethra.

Condyloma Acuminatum - Multiple small filling defects seen on a RUG should make you
think this. Although, instrumentation including a retrograde urethrography is actually not
recommended because of the possibility of retrograde seeding.

Urethrorectal Fistula: This may occur post radiation, and is classically described with
brachytherapy (occurs in 1% of patients).

Urethral Diverticulum: In a man, this is almost always the result of long term foley
placement.

Cancer: Malignant tumors of the male urethra are rare. When they do occur, 80% are squamous
cell cancers (the exception is that prostatic urethra actually has transitional cell 90% o f the time).

Urethral Diverticulum Cancer: Cancer in a urethral diverticulum is nearly ALWAYS


adenocarcinoma (rather than squamous cell).

female Urethra:
Female Urethral Diverticulum: Urethral diverticulum is way more common in females.
They are usually the result of repeated infection of the periurethral glands (classic history is
“repeated urinary tract infections”).

In case books and


conferences this is
classically shown as
a Sagittal MRI. The
majority of urethral
diverticula are located
in the middle third of
the urethra and
involve the
posterolateral wall

It often coexists with stress urinary incontinence (60%) and urinary infection.

The buzzword is “saddle-bag” configuration, which supposedly is how you tell it from the urethra.

Stones can also develop in these things. All this infection and irritation leads to increased risk of
cancer, and the very common high yield factoid is this is most commonly adenocarcinoma (60%).

397
- GU Cancer Blitz! -
Renal Cancer (Adenocarcinoma)

Subtype Syndrome /Association

Enhances equal to cortex Overall Most Common in Gen Pop


Clear Cell
on CM Phase Von Hippel-Lindau

Enhances less than cortex Hereditary Papillary Renal Carcinoma


Papillary
on CM Phase Transplant Kidney

Chromophobe Birt Hogg Dube

Medullary Sickle Cell Trait

Denser than cortex Most Common in Kids


Translocation
on non-con CT Hx of prior cytotoxic chemotherapy is classic.

Ureteral Cancer (Transitional Cell - now called Urothelial)

Location - think about w here you get the m ost stasis


• Renal Pelvis - Twice as com mon as Ureter
• Distal Third o f U reter - M ost com mon site
• M iddle is 2nd, and Proximal is Third

Relationship to Bladder CA
• Bladder CA is w ay more com mon (like lOOx more).
• So if you have bladder CA you don’t need upper tract CA.
Since upper tract CA is not all that com mon, if you smoked
enough M arlboro Reds to get renal pelvis CA, you probably
smoked enough to get multifocal disease including the
bladder.
• Bottom Line: Bladder can be isolated , U reteral CA usually
also has bladder CA

398
Bladder Cancer

Transitional Cell Squamous Cell Adenocarcinoma


(Urothelial Carcinoma) - Think this when you see - Midline Location is Classic
- “The usual one” - like 90% Calcifications - Association with Urachal
- Favors the base (inferior posterior) - Favors the Trigone and Lateral Remnant
- 40% of them are multiple Walls - Association with Bladder
- “Frond like ” Papillary Tumors are - Association with Exstrophy
the Most Common Subtype Schistosomiasis
- Known to be multicentric with - Association with Chronic
synchronous and metachronous Suprapubic Catheter / Urinary
bladder and upper tract lesions — Status
but remember the bladder is the
most common site.

Urethral Cancer

Urethral
Diverticulum
-A d e n o c a rcin o m a

Prostatic Urethra
-Tran sition al C ell (9 0 % )
-T h in k ab o u t it like a B la d d e r C A

Bulbar / Penile Urethra


-S q u a m o u s C ell C A (8 0 % )

399
■PROMETHEUS
' L iomhact, m ,t>
.

400
401
SE C T IO N l:
C o ng en ital

The Uterus - Changes During Life

Neonate - Uterus is larger than you w ould think for a baby (maternal / placental hormones
are still working). If you look close, the shape is a little w eird with the cervix often larger
than the fundus.
Prepuberty - The shape o f the uterus changes - becom ing m ore tube-like, w ith the cervix
and uterus the same size.
Puberty - The shape o f the uterus changes again, now looking more like an adult (pear-like)
- with the fundus larger than the cervix. In puberty, the uterus starts to have a visible
endom etrium - with phases that vary during the cycle.

The Ovaries - Changes During Life


Just like with the uterus, infants tend to have larger ovaries (volum e around I cc), w hich then
decrease and rem ain around or less than 1 cc until about age 6. The ovaries then gradually
increase to norm al adult size as puberty approaches and occurs.

Turner Syndrome - The XO kids.


Besides often having aortic coarctations,
and horseshoe kidneys they will have a
pre-puberty uterus and streaky ovaries.
Wolffian Ducts
(should regress)
Embryology:
The quick and dirty o f it is that the
Mullerian Ducts
mullerian ducts make the uterus and
upper 2/3 o f the vagina. Urogenital
Sinus

The urogenital sinus grows up to meet


the m ullerian ducts and m akes the
bottom 1/3 o f the vagina.

Wolffian ducts are the boy parts, and


should regress com pletely in girls.

M ullerian Ducts W olffian Ducts Urogenital Sinus


Uterus Vas Deferens Prostate
Fallopian Tubes Seminal Vesicles Low er 1/3 o f the Vagina
U pper 2/3 o f the Vagina Epididymis

402
My idea for teaching this somewhat confusing topic is to tap into the thought process o f
embryology to help understand why anom alies happen, and w hy they happen together. The
embryology I’m about to discuss is not strict and doesn’t use all the fancy French / Latin
words. It’s more concept related ...

• • • •
v,»V
f •
O o o O
(»000
O -L>1>
S te p 1: S te p 2: S te p 3: S te p 4

Im ag in e that A s d ev e lo p m e n t N o w th e bottom But b e c a u s e th e y a re just


the stuff that occurs, this soup tw o p u d dles of m a sh e d tog eth er, th e y d o n ’t
m a k e s the g ets poured spilled / poured h a v e a cen tral cavity
kidneys and the d ow n the back of soup begin to n e c es s ary to carry a baby.
uterus is all the the belly. “fu s e ” form ing
s a m e soup. o n e puddle So, th e re is a c lea n -u p
T h e upp er part (u te ru s ). o p eration (c le a v a g e ), and this
You h av e tw o m aking the occurs from bottom to top -
bow ls of this kidney, and the like zipping up a jacket.
stuff - half on bottom part
the left, and m aking the
half on the uterus.
right.

So there are 3 main w ays this whole process can get screw ed up.

(1) You can have only soup on one side. This is a “failure to form ” As you can imagine,
if you don’t have the soup on one side you d o n ’t have a kidney on that side. You also
don’t have h alf o f your uterus. This is w hy a unilateral absent kidney is associated with
U nicom uate Uterus (+/- rudim entary horn).

(2) As the soup gets poured down it can fail to fuse completely. This can be on the
spectrum o f mostly not fiised - basically separate (Uterus Didelphys) or m ostly fused
except the top part - so it looks like a heart (Bicom uate). Because the B icom uate and
Didelphys are related pathologies - they both get vaginal septa (Didelphys more often
than Bicom uate - easily rem em bered because it’s a m ore severe fusion anomaly).

(3) The clean up operation can be done sloppy (“failure to cleave”). The classic example
o f this is a “Septate uterus,” where a septum rem ains betw een the two uterine cavities.

403
Failure to Form”
Vocab
• Mullerian Agenesis (Mayer-Rokitansky-Kuster-Hauser (in case you don’t speak
syndrome); Has three features: (1) vaginal atresia, (2) absent or French or whatever)
rudimentary uterus (unicomuate or bicomuate) and (3) normal Cornus = Uterus
ovaries. The key piece of trivia is that the kidneys have issues Collis = Cervical
(agenesis, ectopia) in about half the cases.

Unicomuate Uterus - The unicomuate uterus can be subdivided into 4 variants.

Isolated
Unicom uate +
Unicomuate
Communicating U nicom uate + f U nicom uate +
Cavitary Non- ■ Noncavitary Most Common
Rudimentary Communicating Rudimentary Subtype (35%)
Horn Cavitary Horn Horn

If you see a unicomuate utems the classic teaching is to look for a mdimentary horn. The reason is the
rudimentary homs can have endometrium - and if present can cause lots of phantom female belly pain
problems (dysmenorrhea, hematometra, hematosalpinx, etc..., etc..., so on a so forth). Endometrial tissue
in a rudimentary hom (communicating or not) - increases the risk of miscarriage. An additional problem
could be a pregnancy in the rudimentary hom - in both the communicating and noncommunicating types-
although especially bad in the non-communicating sub-type because it nearly always results in
mdimentary hom mpture (life-threatening bleeding).
Renal agenesis contralateral to the main uterine hom (ipsilateral to the rudimentary horn) is the most
common abnormality.

Failure to Fuse’
uterus Didelphys Bicornuate T-Shaped

This is a complete This comes in two flavors (one cervix This is the DES related
uterine duplication “unicollis", or two cervix “bicollis”). There anomaly. It is historical
(two cervices, two will be separation of the utems by a deep trivia, and therefore
uteri, and two upper myometrial cleft - makes it look “heart extremely high yield for the
1/3 vagina). shaped”. Vaginal septum is seen around 25% “exam of the future.” DES
of the time (less than didelphys). Although was a synthetic estrogen
A vaginal septum is they can have an increased risk of fetal loss, given to prevent miscarriage
present 75% of the it’s much less of an issue compared to Septate. in the 1940s. The daughters
time. If the patient Fertihty isn’t as much of a “size thing” as it is of patients who took this
does not have vaginal a blood supply thing. Remember you can have dmg ended up with vaginal
obstmction this is 8 babies in your belly at once and have them clear cell carcinoma, and
usually live... live long enough to take part in your uterine anomalies -
asymptomatic. reality show. classically “T-Shaped.”

404
Failure to Cleave '

• S e p ta te - T his one has tw o end o m etrial canals separated


by a fibrous (o r m u scu lar) septum . F ib ro u s vs M u scu lar can
be determ ined w ith M R I an d this distin ctio n ch an g es surgical
m anagem en t (d ifferen t approaches). T h ere is an in creased risk
o f infertility an d recu rren t sp o n tan eo u s abortion. T he septum
has a shitty b lo o d supply, and i f th ere is im p lan tatio n on it - it
w ill fail early. T h ey can resect the sep tu m - w h ich im proves
outcom es.

This is the m o st co m m o n M u llerian d u ct an o m aly asso ciated


w ith m iscarriage. T his is im p ro v ed w ith resectio n o f the
septum .

• A rc u a te U te ru s - M ild sm o o th co n cav ity o f th e uterin e


fundus (instead o f n o rm al straig h t or co n v ex ) T his is n o t really
a m alform atio n , b u t m ore o f a n o rm al v a ria n t. It is N O T
associated with infertility or obstetric complications.

THIS vs THAT: Bicornuate VS Septate

Bicornuate

• “Heart Shaped” - Fundal contour is


less than 5 mm above the tubal ostia

• No significant infertility issues

• Resection of the “septum" results in


poor outcom es

Septate

• Fundal contour is Normal; more than


5 mm above the tubal ostia

• Legit infertility issues - implantation


fails on the septum (it’s a blood
supply thing)

• Resection of the septum can help

405
Hysterosalpingogram (HSG)
HSGs are most typically done as part of an infertility workup,
although it can also be used to make sure a tubal ligation or
obstruction procedure worked. The idea is that you squirt contrast
into the uterus to (1) visualize a normal uterine cavity and (2) make
sure the tubes are patent. Evaluation of uterine bleeding should be
done with ultrasound or in some cases MRI. Bicomuate vs Septate is
tough on HSG - you need MRI or 3D Ultrasound to evaluate the N d rm a l
outer fundal contour - otherwise you could totally get uterine
malformation cases - as previously discussed..
Technique: Study is best performed during the proliferative phase (day
7-12), when the endometrium is the thinnest and makes pregnancy the
0-7 “Rag Week”
least likely. A catheter (usually 5F) is positioned in the cervical canal
7-14 Proliferative,
and a balloon is inflated to allow for contrast instillation. WATER-
14-28 Secretory
SOLUBLE contrast (never barium) is slowly squirted into the uterus.
*Day 14 Ovulation
The normal exam will show a normal uterine contour and free
intraperitoneal spillage of contrast (this proves the tubes are patent).

Contraindications: (1) Pregnancy, (2) Active Pelvic Infection, (3) Active bleeding (“rag week” /
“moon blood”), (4) Contrast Allergy

Irregular Filling Defects: The most common cause is synechiae / scarring / intrauterine
adhesions usually secondary to trauma from prior curettage. Endometrial infections (secondary
to the behaviors of questionable moral conduct and/or being a “free spirit”) are also classically
implicated. The look is irregular, often linear, filling defects arising from one of the uterine
walls. When the scars / adhesions cause clinical infertility then you get to deploy the term
“Asherman syndrome.” — see the
next page.

Round / Oval Filling Defects:


Endometrial polyps or Fibroids can
both look like round / oval filling
defects that are best seen during the
early filling stage. Fibroids be BIG
and totally distort the internal
contour. Bubbles may also appear as
round filling defects but they should
Polyp Bubbles
be mobile and transient.

Tubal Occlusion: Infection / PID


(secondary to questionable moral
standards and/or being a “free spirif’)
is the most common cause.
-------------- 1
The classic look is either:

Treatment: In general - proximal


obstruction (A) can be treated with
tubal catherization - where distal
obstruction / hydrosalpinx (B) cannot. (A) an abrupt cutoff of (B) a dilated tube(s) with
contrast with no distal no free intraperitoneal
opacification - with a spillage of contrast.
proximal obstruction

406
S E C T I O N 2:
ACQUIRED

Salpingitis Istiimica Nodosa (SIN):

This is a nodular scarring o f the fallopian tubes that


produces an Aunt Minnie Appearance. As trivia, it
usually involves the proximal 2/3 o f the tube. This is
o f unlmown etiology, but likely post inflammatory /
infectious (i.e. being a woman o f questionable moral
standard / “free spirit”).
It’s strongly associated with infertility and ectopic
pregnancy and that is likely the question.

Uterine AViVI - These can be congenital or acquired, with acquired types being way more
common. They can be serious business and you can totally bleed to death from them. The typical
ways to acquire them include; previous dilation and curettage, therapeutic abortion, caesarean
section, or just multiple pregnancies.
Doppler ultrasound is going to show: serpiginous and/or tubular anechoic structures within the
myometrium with high velocity color Doppler flow.

intrauterine Adliesions (Asiiermans) -


This is scarring in the uterus, that occurs secondary to
injury: prior dilation and curettage Tmost common I
surgery, or pregnancy. If the patient was a liberal arts
major - you can assume infection / PID. A history of
international travel would make you think about the classic
GU TB.
This is typically shown on HSG, with either (a) non filling
o f the uterus, or (b) multiple irregular linear filling defects
(lacunar pattern), with inability to appropriately distend Irregular Filing Defects / Synechiae
the endometrial canal. MRI would show a bunch o f T2 - in this waitress from Hooters
dark bands. Clinically, this results in infertility. (former liberal arts major).

Endometritis - This is in the spectrum o f PID. You often see it 2-5 days after delivery,
especially in women with prolonged labor or premature rupture. You are going to have fluid and
a thickened endometrial cavity. You can have gas in the cavity (not specific in a postpartum
women). It can progress to pyometrium, which is when you have expansion with pus.

407
Pelvic Floor
Getting old and having a bunch of kids can sometimes make stuflfhang out of your vagina and cause you to
pee your pants (when you don’t actually want to pee your pants). It is important to make that distinction
between “prolapse” (stuff hanging out of your vagina), and “relaxation” (peeing and/or pooping your pants
when you sneeze). Both are bad... although one is worse - I’ll let you decide which one that is.
Anatomy Review: This anatomy is complicated - buncha fascial bands “ligaments” muscles etc... creating a
“sling” which keeps all this stuff from falling out the bottom. The best way to think about the pelvic sling of a
female is to group it into 3 functional compartments: Anterior compartment (bladder and urethra), Middle
compartment (vagina, cervix, uterus, and adnexa), and Posterior compartment (anus and rectum). This anatomy
is incredibly complex - but a few of these vocab terms could make easy questions:
Endopelvic fascia: Buncha ligaments / fascia (pubocervical fascia, rectovaginal fascia, cardinal ligaments,
etc..) most of which have vaginal or cervix in the name. Main support for the anterior & middle compartments.
Levator ani: This is the main muscular component of the pelvic floor composed of the puborectalis,
pubococcygeus, and iliococcygeus. This muscle groups constant contraction maintains the pelvic floor height.
Urogenital diaphragm: This is the most caudal or superficial musculofascial structure. It does not have a
marketable sex toy name (unlike Levator Ani). This thing usually finds it way into multiple choice exams as
the anatomic landmark used in the classification of urethral injury - as discussed in the GU chapter.

Pelvic floor relaxation has two


components (pelvic floor descent and Normal
widening) that can be graded during
maximal strain on sag MRI:
• Hiatal enlargement (H line) —less
than 6cm - measurement of widening
• Pelvic floor descent (M line) - less
than 2cm - measurement of descent
Wider H or longer M = Worse.
PCL (pubococcygeal line) = drawn from As the pelvic floor
Organs hanging out = worse.
inferior margin of the symphysis pubis to
collapses - you will
the junction between the first and second see widening and
MRI Protocol: coccygeal elements.
Steady State - T2 - 3 planes descent evidenced by
H Line = drawn from the inferior margin these measurements.
Dynamic State - Rapid T2 with Fat of the symphysis pubis to the posterior
Sat, Sag Plane Only This is nonspecific -
aspect of the puborectalis muscle sling. it just means the
—performed during Valsalva, Kegel,
and/or taking a shit (seriously)
M Line = shortest distance between the thing don’t work
posterior aspect of the puborectalis right.
muscle sling and the PCL

Axial image through the Ischioanal space (Triangle of fat lateral and caudal to the levator
ani - could show a loss of the normal “H shaped” vagina or direct defects / asymmetric
thinning in the muscular sling. Having said that - for the purpose of multiple choice - this
anatomy is usually demonstrating an anal fistula in the setting of Crohns.

Anterior Compartment Middle Compartment Posterior Compartment


Cystocele - Bladder Descent > 1 cm Uterine prolapse - Descent of the Rectocele - Abnormal rectal
below the pubococcygeal line. cervix or posterior vaginal fornix > 1 bulging (typically anteriorly).
Urethral hvpermobilitv - what you cm below the pubococcygeal line. Due to weakening
say if the urethra is rotated A big turd can prop up the uterus - so of the rectovaginal fascia.
horizontally. This changes the it is best to measure them with an
treatment from retropubic empty rectum (post defecation phase). They describe them by how
urethropexy (for normal stress Axial images could show the vagina far they bulge relative to the
incontinence) to a pubovaginal sling. lose its normal "H" shape - hanging anal canal.
Risk Factors: Squeezing a bunch of low like the sleeve of a wizard (or the Risk Factors; Vaginal surgery.
kids out of your vagina - can cause tongue of a tired dog). Hysterectomy, Chronic
rupture of the pubocervical fascia Risk Factor: Hysterectomy Constipation, Being Old as
Dirt.

408
SE C T IO N 3:
M a s s e s a n d T u m o r s
O F THE U TE R U S AND VAGINA

Fibroids (Uterine Leiomyoma): These benign smooth muscle tumors are the most common
uterine mass. They are more common in women of African ancestry. They hke estrogen and are most
common in reproductive age (rare in prepubertal females). Because of this estrogen relationship
they tend to grow rapidly during pregnancy, and involute with menopause. Their location is
classically described as submucosal (least common), intramural (most common), or subserosal.
Typical Appearance: The general rule is they can look like anything. Having said that, they are
usually hypoechoic on ultrasound, often with peripheral blood flow and shadowing in the so called
“Venetian Blind” pattern. On CT, they often have peripheral calcifications (“popcorn” as seen on
plain film). On MRJ, T1 dark (to intermediate), T2 dark, and variable enhancement. The fibroids
with higher T2 signal are said to respond better to IR treatment. A variant subtype is the
lipoleiomyoma, which is fat containing.

Fibroid Types T1 T2 Tl+C

Hyaline (Classic) Most common type Dark Dark Homogenous

Densely packed smooth muscle (without much


Hyperceliular Dark Bright Homogenous
connective tissue). Respond well to embolization

Rare fat containing subtype (maybe the result of


Bright
degeneration). This thine will be hvperechoic on Maybe Rim
Lipoleiomyoma (dark if Bright Enhancement
ultrasound. Will look like a fatty uterine mass on
fat sat)
CT. Will drop sipial on fat saturation sequences.

Degeneration: 4 types of degeneration are generally described. What they have in common is a lack
of / paucity of enhancement (fibroids normally enhance avidly). The process of degeneration (basically
a fibroid stroke) can cause severe pain as well as fever and/or leukocytosis.

Fibroid Degeneration T1 T2 Tl+C T2 Dark

Most common type.


The fibroid outgrows
Hyaline : its blood supply, and
Variable
Heterogeneous
(Classic) (usually None
you end up getting the (usually dark)
Degeneration dark)
accumulation of
proteinaceous tissue.
T1 Bright Rim

This one occurs during


pregnancy - caused by
Red Peripheral
venous thrombosis.
(Carneous) Rim of Variable None
The classic imaging
Briaht T1
finding is a peripheral
rim of T1 high signal.

Myxoid
Uncommon Dark Bright Minimal
Degeneration

Cystic Uncommon Dark Bright None Cystic ''


Degeneration

409
uterine Leiomyosarcoma - The risk of malignant transformation to a leiomyosarcoma is
super low (0.1%). These look like a fibroid, but rapidly enlarge. Areas of necrosis are often seen.

Adenomyosis - This is endometrial


tissue that has migrated into the NORMAL ANATOMY REVIEW
myometrium. You see it most commonly
in multiparous women of reproductive Uterus anatomy questions are nearly always
age, especially if they’ve had a history of shown with a sagittal T2. The reason is because
uterine procedures (Caesarian section, T2 separates the layers out (on T1 these layers are
dilatation and curettage). hard to tell apart).
3 - Myometrium
Although there are several types,
adenomyosis is usually generalized, 2 - Junctional Zone
favoring large portions of the uterus 1 - Endometrium
(especially the posterior wall), but
sparing the cervix. It classically causes
marked enlargement of the uterus, with
preservation of the overall contour.
The dark one is the
They can show it with Ultrasound or MRI. Junctional Zone.
Ultrasound is less specific with findings
including a heterogeneous uterus
(hyperechoic adenomyosis, with When is it normal to NOT see this 3 zone pattern ?
hypoechoic muscular hypertrophy), or just (A) Younglings - premenarchal girls
enlargement of the posterior wall. MRI is (B) Oldlings - postmenopausal women
the way better test with the most classic
feature being thickening of the junctional (C) During Pregnancy: the junctional zone becomes
zone of the uterus to more than 12 mm more bright. This makes it harder to tell 1 & 2
(normal is < 5 mm). The thickening can be apart.
either focal or diffuse. Additionally, the (D) Postpartum: It takes about 6 months after
findings of small high T2 signal regions pregnancy has ended for the junctional zone to
corresponding to regions of cystic change return to its normal T2 dark color.
is a classic finding.

T2 Dark Junctional Zone


T2 Dark Junctional Zone
Normal (< 5mm = Normal) Adenomyosis Abnormally thick ( > 12mm)

T2 Bright Urine -
Clue that this is a T2 exam.

410
-Endometrium-Normal and Abnormal Thickening -
Normal: Later in the chapter we will discuss how the normal hormonal fluctuations change the
appearance of the ovaries. This also happens with the uterus - and lends itself easily to multiple
choice questions. So let us take a quick look at this:

Proliferative Secretory
Remember j r
the stripe is
measured without
including any
fluid in the canal.

Normal Trilaminar Appearance Thick Hyperechoic Homogenous


Thin Bright Center, Dark Middle, Can thicken up to ~ 16mm
Surrounding Echogenic Layer (this is normal thickening)
~ 4 - 12 mm
Thick vs Thin: Usually you are looking at the uterus
because the patient is having abnormal bleeding. The cause of Postmenopausal Bleeding:
this bleeding is your job as a radiologist to figure out (or at Is it from atrophy or cancer?
least try and help). Your general options are going to be: •Endometrium less than 5 mm =
submucosal fibroids, polyps, atrophy, or cancer — these are — Probably Atrophy
the things that can cause abnormal (non-rag week) bleeding.
•Endometrium > 4-5 mm =
Atrophy and Cancer are the ones you worry about in older
------ Hypertrophy vs cancer
(postmenopausal) patients. Telling the difference between
atrophy and cancer is easily solved with an ultrasound ► ---------- Gets a biopsy

• Focal or generalized thickening in post menopausal women


greater than 5mm should get sampled.
• Premenopausal endometriums can get very thick - up to 20mm can be normal.
Trivia:
• Estrogen secreting tumors - Granulosa Cell tumors of the ovary will thicken the endometrium.
• Hereditary Non-Polyposis Colon Cancer (HNPCC) - 30-50x increased risk of endometrial CA

Endometrial Cancer: Basically all uterine cancers are adenocarcinoma (90%+). The only
possible exception for the purpose of multiple choice would be the rare “leiomyosarcoma” - which looks
like a giant fucking fibroid, as discussed on the prior page.

Typical Scenario: A postmenopausal patient (60s) with bleeding. First Step Postmenopausal
Ultrasound shows an endometrium that is too thick (most people Bleeder = Ultrasound
say 4- 5mm) then it gets a biopsy. Almost always this will be Too Thick ( > 4-5mm) ? = Biopsy
stage 1 disease, and no further imaging will be done.
Extent of Local Disease = MRI
Occasionally extent of disease will be staged with MRI (not CT -
which is dog shit for looking at the cervix). Distal Mets = PET CT

Trivia: The older the patient, the more aggressive these tend to be.

411
Endometrial C an ce r - Part 2 - MRI Staging Trivia
Key M RI Findings
MRI is used for assessing myometrial invasion depth and cervical (Most Likely Tested)
involvement. To make these findings you will need to understand how For Endometrial Cancer
these studies are done and how they can show this to you.
(1) Myometrial Invasion:
There are 3 basic sequences you need to know. less or greater than half?

(1) Diffusion - the tumor will restrict, and it is most hkely to be shown on (2) Cervical Stromal
ADC (remember that true restricted diffusion has low ADC). Diffusion Invasion.
is good for “Drop mets” into the vagina, and finding lymph nodes.

(2) T2 - Both T2 and Post Contrast T1 can be shown in


sagittal or the axial oblique. The axial oblique is
obtained from images perpendicular (solid lines) to the
long axis of the endometrial cavity (dotted line).
Tumor on T2 tends to be intermediate signal - brighter
than the adjacent dark junctional zone. Axial Oblique

(3) Post Contrast T l: As above this can be shown in sagittal or axial oblique. The myometrium should
enhance fairly homogeneously. The tumor will actually be the thing that enhances LESS than the
adjacent myometrium (normally you think about cancer enhancing more than nearby tissue).

Stage la: Tumor is


Less than 50% of
Myometrium

Stage lb: Tumor is


More than 50% of
Myometrium
T2 : Tumor is Tl+C: Tumor Enhances
Intermediate Less than Adjacent
Myometrium

Who Gets Nodal Mets ?


Short Answer =
“Deeper” and Bigger Tumors
Superficial Tumor = ~5% Risk
Deep Myometrial Invasion = -45%
Risk
/ i Cervical Invasion
Normal T2 Cervix
The middle of the cervix Tumor < 2cm = -5% Risk
Abnormal T2 Cancer
(“endocervical canal”) is Tumor > 4cm = ~ 35% Risk
Invades Cervix (arrows)
bright with dark inner Stage 2 (cervical invasion)
stroma and intermediate Best Study For Nodal Mets ?
outer stromal layers Short Answer = PET CT

412
Can cer - Part 3 - More Staging Trivia
• Moving from stage lA (<50% myometrium) to stage IB (>50% of the myometrium) increases the risk
of lymph node disease.
• Some sites will do lymph node sample at stage lA, and radical lymph node dissection at stage IB.
• You will often hear people say that “extension of the tumor into the myometrium is the single most
important morphologic prognostic factor”
• Stage 2 disease is defined as cervical stroma
invasion. This also increases the risk for lymph node
mets.
• The diagnostic key is the post contrast imaging
(obtained 2-3 mins after injection). If the cervical
mucosa enhances normally, you have excluded
stromal invasion.
• You have to wait 2-3 mins after injection because the
normal cervix doesn’t enhance symmetrically (the
endocervix enhances quickly, whereas the outer and
inner fibrous stromal layers enhance gradually). Stage 1: Stage 2:
• Stage 2 is probably going to change management by T1+C: Normal dark Tl+C: Tumor
adding pre-op radiation to the cervix, plus a change cervical stroma (star). Invasion o f the
from TAH to radical hysterectomy (obviously this Enhancement of the cervical Cervix
varies from center to center). mucosa (arrows) excludes
invasion.

Tamoxifen Changes: This is a SERM (acts like estrogen in the


pelvis, blocks the estrogen effects on the breast). It’s used for breast
cancer, but marginally increases the risk of endometrial cancer (1% per
year). It will cause subendometrial cysts, and the development of
endometrial polyps (30%). Normally, post menopausal endometrial
tissue shouldn’t be thicker than 4mm, but on Tamoxifen the endometrium
is often thick (some papers say the mean is 12 mm at 5 years).
When do you biopsy? If post menopausal and bleeding. If the patient is
not symptomatic there is no routine screening and no role for ultrasound.
Gamesmanship: An easy way to test this is by showing you classic
tamoxifen changes (as above) and making you guess the history of breast cancer.

Endometrial Polyps: These things cause bleeding and can mimic a thickened endometrium.
The typical look is a solitary bright (hyperechoic) lesion, sometimes shown with a single vessel feeding it
on doppler imaging. The classic way this is tested is to get you to ask for (or just show you) an
ultrasound with saline infused into the uterus (sonohysterography).

Trivia:
Sonohysterography
should be performed
during the early
proliferative phase
(day 4-6) when the
endometrium is at its
thinnest.
Ultrasound: Bright and Focal Sonohysterography: Bright intracavitary
mass outlined by the fluid

413
Cervical Cancer-
It’s usually squam ous cell, related to H PV (like 90%). The big thing to know is param etrial
invasion (stage lib). Stage Ila or below is treated with surgery. Once you have param etrial
invasion (stage Ilb), or involvem ent o f the lower 1/3 o f the vagina it’s gonna get chem o/
radiation. In other words, m anagem ent changes so that is the m ost likely test question.

Cervical Cancer Staging Pearls

Stage IIA Spread beyond the cervix, but NO parametrial invasion Surgery

Stage II B Parametrial involvement but NOT extension to pelvic side wall. Chemo/ Radiation

How can I tell If there is Param etrial Invasion ?


I don Heven know what the hell the “parametrium ” is. ...

What is this parametrium ? The


parametrium is a fibrous band that separates
the supravaginal cervix from the bladder. It
extends between the layers of the broad
r
ligament.
No Invasion (HA)
Why is it so important ?The uterine artery
runs inside the parametrium, hence the need Parametrial Invasion (MB)
for chemo - once invaded.

How do you tell if it’s invaded ? Normally the cervix has a T2 dark ring. That thing
should be intact. If the tumor goes through that thing, you gotta call it invaded.

T2 Dark Ring Intact T2 Dark Ring Disrupted - arrow


(No Parametrial invasion) (Parametrial invasion)

Endometrial Fluid - In premenopausal women this is a common finding. In postmenopausal


women it means either cervical stenosis or an obstructing mass (usually cervical stenosis).

414
Vagina -
Could also be referred to as the “Petal-soft F o ld o f Womanhood, ” “P earl o f Passion, ” or
“D oor o f F em ininity” - if the question w riter is a fan o f Romance Novels.

Solid Vaginal Masses:

An uninvited solid vaginal mass is usually a bad thing. It can be secondary (cervical or
uterine carcinoma protruding into the vagina), or prim ary such as a clear cell adenocarcinom a
or rhabdomyosarcoma.

Leiomyoma - Rare in the vagina, but can occur (m ost com m only in the anterior wall).

Squamous Cell Carcinom a - The m ost com mon cancer o f the vagina (85%). This is
associated with HPV. This is ju st like the cervix.

Clear Cell A denocarcinom a - This is the zebra cancer seen in w om en w hose m others took
DBS (a synthetic estrogen thought to prevent miscarriage). That plus “T-Shaped U terus” is
probably all you need to know.

Vaginal R habdom yosarcom a - This is the m ost com m on tum or o f the vagina in children.
There is a bim odal age distribution in ages (2-6, and 14-18). They usually come o ff the
anterior wall near the cervix. It can occur in the uterus, but typically invades it secondarily.
Think about this when you see a solid T2 bright enhancing mass in the vagina / lower uterus
in a child.

Mets Trivia:
• A m et to the vagina in the anterior w all upper 1/3 is “a lw a ys” (90%) upper genital tract.
• A m et to the vagina in the posterior w all lower 1/3 is “always ” (90%) fro m the G I tract.

415
Cystic Vaginal I Cervical Masses:

Nabothian Cysts - These are usually on the cervix and you see them all the time. They are
the result o f inflammation causing epithelium plugging o f mucous glands.

Gartner Ducts Cysts - These are the result o f incom plete regression o f the W olffian ducts.
They are classically located along the anterior lateral wall o f the upper vagina. If they are
located at the level o f the urethra, that can cause mass effect on the urethra (and
symptoms).

Bartholin Cysts - These are the result o f obstruction o f the Bartholin glands (mucin-
secreting glands from the urogenital sinus). They are found below the pubic symphysis
(helps distinguish them from Gartner duct). Treatment — usually nothing, but if they get
infected or cause pain (or mess up your “only fans” donations) they can be dealt with via
marsupialization (basically sutured open to allow for continual drainage).

Skene Gland Cysts - Cysts in these periurethral glands, can cause recurrent UTIs and
urethral obstruction.

Nabothian Cyst

— “Bartholin is near the Butthole.

416
S E C T IO N 4:
O vary / A dnexa

Before we begin, a few general tips (1) never biopsy or recommend biopsy o f an ovary,
(2) on CT if you can’t find the ovary, follow the gonadal vein, and (3) hemorrhage in a
cystic mass usually means it’s benign.

A quick note on ovarian size; ovarian volume can be considered normal up until 15 ml
(some say 20 ml). The post menopausal ovary should NOT be larger than 6 cc.

Let’s talk about ovulation - to help understand the normal variation in the ovary.

Ovulation Luteal Phase


m
O

LH LH Surge

Follicular Phase Almost Ovulation Corpus


*Note Dominant Follicle Luteum

Follicles seen during the early menstrual cycle are typically small (< 5 mm in diameter). By
day 10 o f the cycle, there is usually one follicle that has emerged as the dominant follicle.
By mid cycle, this dominant follicle has gotten pretty big (around 20 mm).

The size isn’t surprising because it contains a mature


Meaningless Vocab (High Yield)
ovum. The LH surge causes the dominant follicle to
Premenopausal Ovarian Cyst
rupture, releasing the egg. The follicle then regresses
< 1 cm = Follicle
in size, forming a Corpus Luteum. A small amount o f
1-2 cm = Dominant Follicle
fluid can be seen in the cul-de-sac. Occasionally, a
follicle bleeds and re-expands (hemorrhagic cyst) - > 3 cm = Cyst
more on this later.

417
Cumulus Oophorus

This is a piece of anatomy trivia. It is a collection of


cells in a mature dominant follicle that protrudes into the
follicular cavity, and signals imminent ovulation (its
absence means nothing).

Cum ulus Oophorus


Fertility Meds

Medications such as a Clomiphene Citrate (Clomid), force the maturation of multiple


bilateral ovarian cysts. It is not uncommon for the ovaries of women taking this drug to
have multiple follicles measuring more than 20 mm in diameter by mid cycle.

Theca Lutein Cysts - this is a type of functional


cyst (more on that below), related to overstimulation
from b-HCG. What you see are large cysts (~ 2-3 cm)
and the ovary has a typical multilocular cystic “spoke-
wheel” appearance.

Think about 3 things:

•Multifetal pregnancy,
•Gestational trophoblastic disease (moles),
•Ovarian Hyperstimulation syndrome. Theca Lutein Cyst

Ovarian Hyperstimulation Syndrome - This is a complication associated with fertility


therapy (occurs in like 5%). They will show you the ovaries with theca lutein cysts, then ascites,
and pleural effusions. They may also have pericardial effusions. Complications include increased
risk for ovarian torsion (big ovaries) and hypovolemic shock.

Paraovarian (Paratubal) Cyst = Cyst that is in the adnexa but not within the ovary. Instead
these things are located adjacent to the ovary or tube. If the cyst is simple (not septated or nodular) and
clearly not ovarian they will not need followup — is doesn't matter how big it is, as they have incredibly
low rate of malignancy.

TLDR; Simple paraovarian cysts do not require follow up.

418
THIS vs THAT: O ld vs Y o u n g

Premenstrual:

• The ovaries o f a pediatric patient stay small until around age 8-9.

• Ovaries may contain small follicles.

Premenopausal:

• A piece o f trivia; premenopausal ovaries m ay be HOT on PET


(depending on the menstrual cycle).

• This is why you do a PET in the first week o f the menstrual cycle.

Postmenopausal
( > one yea r after menses stops): If the cyst is simple, regardless of age it’s almost
certainly benign.
• Considered abnorm al if it exceeds the
Gamesmanship: What if they don’t tell you if the
upper limit o f normal, or is twice the size patient is pre or post menopausal ? You can use 50
o f the other ovary (even if no mass is years old as a cut off. Under 50 Pre, 50 & up Post.
present).
Incidental Simple Appearing Ovarian Cyst
-Shown on CT-
• Small cysts (< 3 cm) are seen in around
20% o f post menopausal women. PreMenopausal: < 3 cm = Call it Normal Follicle
PreMenopausal: > 3 cm = Get an US
• In general, postmenopausal ovaries are
PostMenopausal: < 1cm = Call it Normal Cyst
atrophic, lack follicles, and can be PostMenopausal: > 1cm = Get an US
difficult to find with ultrasound.
Incidental Simple Appearing Ovarian Cyst
• The ovarian volume will decrease from -Shown on US-
around 8cc at age 40, to around 1cc at
PreMenopausal: < 7 cm = No Follow Up
age 70. PreMenopausal: > 7 cm = Follow Up (3 months)

• The maximum ovarian volum e in a PostMenopausal: < 5 cm = No Follow Up


post m enopausal wom an is 6 ml. PostMenopausal: > 5 cm = Follow Up (3 months)

• Unlike premenopausal ovaries, post Cyst is not simple (irregular septations, papillary
projections, or solid elements) = GYN consult.
m enopausal ovaries should NOT be
hot on PET.

419
In most clinical practices, the overwhelm ing m ajority o f ovarian masses are benign (don’t
worry, I ’ll talk about cancer, too).

Physiologic and functioning follicles


Corpora lutea
Hemorrhagic cysts
Endom etriom as
Benign cystic teratomas (dermoids)
Polycystic ovaries

Functioning Ovarian Cysts: Functioning cysts (folhcles) are affected by the menstrual
cycle (as I detailed eloquently above). These cysts are benign and usually 25 mm or less in
diameter. They will usually change / disappear in 6 weeks. If a cyst persists and either does not
change or increases in size, it is considered a nonfunctioning cyst (not under hormonal control).

Simple cysts that are > 7 cm in size may need further evaluation with MR (or surgical
evaluation). Just because it’s hard to evaluate them completely on US when they are that big,
and you risk torsion with a cyst that size.

Corpus Luteum: The normal corpus luteum arises from a dominant follicle (as I detailed
eloquently above). These things can be large (up to 5-6 cm) with a variable appearance (solid
hypoechoic, anechoic, thin-walled, thick-walled, cyst with debris). The most common
appearance is solid and hypoechoic with a “ring o f fire” (intense peripheral blood flow).

THIS THAT: Corpus Luteum VS Ectopic Pregnancy


They both can have that “ring of fire” appearance,
Corpus
but please don’t be an idiot about this. Most ectopic Ectopic
Luteum
pregnancies occur in the tube (the corpus luteum is
an ovarian structure). If you are really lucky, a RI < 0.4, or > 0.7 RI 0 .4 -0 .7
“hint” is that the corpus luteum should move with the
THICK Thin
ovary, where an ectopic will move separate from the
Echogenic Rim Echogenic Rim
ovary (you can push the ectopic away from it). Also,
the tubal ring of an ectopic pregnancy is usually more “Ring of Fire” “Ring of Fire”
echogenic when compared to the ovarian
parenchyma. Whereas, the wall of the corpus luteum
Moves Separate Moves
is usually less echogenic. from the Ovary with the Ovary
A specific (but not sensitive) finding in ectopic
pregnancy is a RI of <0.4 or >0.7.

420
Endometrioma:

This targets young women during their reproductive years and can cause chronic pelvic pain associated
with menstruation. The traditional clinical history of endometriosis is the triad of infertility,
dysmenorrhea, and dyspareunia. Laparoscopy is the diagnostic gold standard for endometriosis.

The classic appearance is a rounded mass with homogeneous low level internal echoes and increased
through transmission (seen in 95% of cases). Fluid-fluid levels and internal septations can also be seen.
It can look a lot like a hemorrhagic cyst (sometimes).

As a general rule, the more unusual or varied the echogenicity and Q: What is the most sensitive
the more ovoid or irregular the shape, the more likelv the mass is
imaging feature on MRI for the
an endometrioma. Additionally, and of more practical value, they
are not going to change on follow up (hemorrhagic cysts are). In diagnosis of malignancy in an
about 30% of cases you can get small echogenic foci adhering to endometrioma ?
the walls (this helps make the endometrioma diagnosis more
likely). Obviously, you want to differentiate this from a true wall A: An enhancing mural nodule
nodule.

The complications of endometriosis (bowel obstruction, infertility, etc...) are due to a fibrotic reaction
associated with the implant. The most common location for solid endometriosis is the uterosacral
ligaments. They can also occur in or near C-section scars. Most people will tell you that if vou see an
Endometrioma on MRI that vou should look for “dropped implants” in the deeper pelvis and along the
abdominal scar. This can be shown with MRI (TI bright) or ultrasound of the c-section scar (well defined
solid lesion).

Do Endometriomas Ever Become Cancer? About 1% of endometriomas undergo malignant


transformation (usually endometrioid or clear cell carcinoma). How do you tell which one is which???
Malignancy is very rare in endometriomas smaller than 6 cm. They usually have to be bigger than 9 cm.
Additionally, the majority of women with carcinoma in an endometrioma are older than 45 years. So risk
factors for turning into cancer: (a) older than 45, (b) bigger than 6-9 cm.

Pregnancy Trivia: There is a thing called a “decidualized


endom etrioma. ” This is a vocab word used to describe a solid
nodule with blood flow in an endometrioma of a pregnant girl.
Obviously this is still gonna get followed up - but is a mimic
of malignancy. The thing never to forget is that if the patient
is NOT pregnant and you see a solid nodule with blood flow -
that is malignant degeneration - period - no hesitation, next
question.

Endometrioma on MRI: Will be TI bright (from the blood).


Fat saturation will not suppress the signal (showing you it’s
not a teratoma). Will be T2 dark! (from iron in the
endometrioma).

The shading sign is a buzzword for endometriomas on MR


imaging. On T2 you should look for “shading.”

The shading sign describes T2 shortening (getting dark) of


a lesion that is TI bright.
Endometrioma - “Shading Sign”

421
Hemorrhagic Cysts:

As mentioned on prior pages, sometimes a ruptured follicle bleeds internally and re-
expands. The result is a homogenous mass with enhanced through transm ission (tumor
w o n ’t do that) with a very sim ilar look to an endom etriom a. A lacy “fishnet appearance”
is sometimes seen and is considered classic. D oppler flow will be absent. The traditional
way to tell the difference between a hem orrhagic cyst vs endom etriom a, is that the
hem orrhagic cyst will go away in 1-2 m enstrual cycles (so repeat in 6-12 weeks).

Hemorrhagic Cyst on Mi?/ - Will be T I bright (from the blood). Fat saturation will not
suppress the signal (showing you it’s not a teratoma). The lesion should N O T enhance.

Hemorrhagic cysts in o ld ladies? Postm enopausal w om en may occasionally ovulate, so


you don’t necessarily need to freak out (follow up in 6-12 weeks). Now, late
postmenopausal w om en should N EV ER have a hem orrhagic cyst and if you are shown
something that looks like a hem orrhagic cyst in a 70 year old - it’s cancer till proven
otherwise.

THIS vs THAT; Endometrioma vs Hemorrhagic Cyst

Endometrioma Hem orrhagic Cyst


Homogeneous with Low Level Echoes Lacy Fishnet Appearance

422
Dermoid:

These things typically occur in young women (20s-30s), and are the most common ovarian
neoplasm in patients younger than 20. The “Tip o f the Iceberg Sign” is a classic buzzword and
refers to absorption o f most o f the US beam at the top o f the mass. The typical ultrasound
appearance is that o f a cystic mass, with a hyperechoic solid mural nodule, (Rokitansky nodule or
dermoid plug). Septations are seen in about 10%.

T1 T1FS T2
Dermoid on MRl: Will be bright on T 1
(from the fat). There will be fat Endometrioma
suppression (not true o f hemorrhagic
cysts, and endometriomas).
Dermoid
O
\
Hemorrhagic
Cyst

Do Dermoids Ever Become Cancer? About 1% o f dermoids can undergo malignant


transformation (almost always to squamous cell CA). Again, risk factors are size (usually
larger than 10cm), and age (usually older than 50).

R a re C a n c e r T ra n s fo rm a tio n S u b ty p e s

Endom etriom a Clear Cell

Dermoid Squamous

Dermoid Gamesmanship = Gross Fat containing ovarian mass on CT

Dermoid Gamesmanship = The Old Tooth Trick - shown on plain film, CT, or even as
susceptibility (dark stuff) on MR. Remember Dermoids are basically teratomas, and
teratomas grow all kinds o f gross shit including teeth, hair, finger nails etc... The tooth is
obviously the classic one.

Dermoid Gamesmanship = “Dot -dash” pattern has been described for hair within a cyst.

Polycystic Ovarian Syndrome:


—Typically an overweight girl with infertility, acne, and a pencil mustache (not a full Ron Swanson)

The imaging criteria is:

Ten or more peripheral simple cysts (typically small < 5 mm)


Usually Characteristic 'string-of-pearls'
Ovaries are typically enlarged (>10 cc),
although in 30% of patients the ovaries have a normal volume

423
S E C T IO N 5:
O varian C a n c e r

Ovarian cancers often present as com plex cystic and solid masses. They are typically intra-
ovarian (most extra-ovarian m asses are benign). The role o f im aging is not to come down
hard on histology (although the exam may ask this o f you), but instead to distinguish benign
from malignant and let the surgeon handle it from there.

Think Cancer if:

* Unilateral (or bilateral) com plex cystic adnexal m asses with thick ( > 3 mm)
septations, and papillary projections (nodule with blood flow).
* Solid adnexal masses with variable necrosis
* Ancillary findings present: ascites, evidence o f invasive spread, LAD

Knee Jerks:

M ultiple thin or thick septations = Call the Surgeon


N odule w ith Flow = Call the Surgeon
Solid Nodules W ithout Flow =
o Get an M R to make sure it’s not a derm oid plug,
o If it’s not a dermoid, then call the surgeon

Serous Ovarian I Cystadenocarcinom a / Cystadenom a

Serous tumors are the m ost com m on type o f


ovarian malignancy. A bout 60% o f serous
tumors are benign, and about 15% are considered
borderline (the rest are malignant). They favor
women o f childbearing age, with the malignant
ones tending to occur in older women. They
typically are unilocular w ith few septations. They
are frequently bilateral (especially when
malignant). Papillary projections are a com mon
finding, and are suggestive o f malignancy. If you
see ascites, they have mets (70% have peritoneal
involvement at the time o f diagnosis). Serous Ovarian Neoplasm
-L a rg e , U nilocular, F e w S e p ta tio n s

424
Mucinous Ovarian C y s ta d en o c a rc in o m a

Often a large mass. They are typically m ulti-loculated (although septa are often thin).
Papillary projections are less com m on than with serous tumors. You can see low level
echos (from mucin). These dudes can get Pseudom yxom a peritonei with scalloping
along soHd organs. Smoking is a known risk factor (especially for m ucinous types).

TH IS vs THAT: Serous vs M ucinous

Serous: Mucinous:

Unilocular (few er septations) M ulti-locular (more septations)

Papillary Projections Common Papillary Projections Less Common

Endometrioid Ovarian Cancer: Gamesmanship:


Ovarian Mass +
This is the second m ost com m on ovarian cancer (serous Endometrial Tiiickening
number one, mucinous num ber three). These things are
This is a w ay to show both
bilateral about 15% o f the time.
Endom etrioid CA (which
What to know: often has both ovarian and
endom etrial CA), and
• 25% o f women will have concom itant endom etrial cancer,
G ranulosa-Theca Cell
with the endom etrial cancer as the prim ary (ovary is met).
Tumor (which produce
• Endometriomas can turn into endom etrioid cancer
estrogen - and cause
• 15% are bilateral
endom etrial hyperplasia)

B.F.M’s - for Adults


It’s useful to have a differential for a B .F.M .
(Big Fucking M ass) in an adult and a child. I
discuss the child version o f this on page 81.
For adults think about 3 main things:

(1) Ovarian M asses - M ucinous and Serous


(2) Desmoids - Rem em ber Gardner Syndrome
(3) Sarcomas

425
Fibroma I Fibrothecoma:

The ovarian fibroma is a benign ovarian tumor, most commonly seen in middle aged women.
The fibrothecoma / thecoma spectrum has similar histology. It’s very similar to a fibroid. On
ultrasound it’s going to be hypoechoic and solid. On MRI it’s going to be T1 and T2 dark, with
a band of T2 dark signal around the tumor on all planes. Calcifications are rare.

Similar or Related Conditions:

• Meigs Syndrome: This is the triad o f ascites, pleural effusion, and a benign ovarian tumor
(most commonly fibroma).

•Fibromatosis: This is a zebra. You have tumor-like enlargement of the


ovaries due to ovarian fibrosis. It typically hits girls around the age o f 25.
It’s associated with omental fibrosis and sclerosing peritonitis. You are
going to get dark T1 and T2 signal. The buzzword for that T2 signal is
k i “black garland sign. ” The condition is benign, and sometimes managed
with surgical removal o f the ovaries.

Brenner Tumor: Epithelial tumor o f the ovary seen in women in their 50s-70s. It’s fibrous
and T2 dark. Unlike Fibromas, calcifications are common (80%). They are also sometimes
referred to as "Ovarian Transitional Cell Carcinoma ” for the purpose o f fucking with you.

Struma Ovarii:

These things are actually a subtype o f ovarian teratoma. On imaging you are looking for a
multilocular, predominantly cystic mass with an INTENSELY enhancing solid component. On
MRI - the give away is very low T2 signal in the “cystic” areas which is actually the thick
colloid. These tumors contain THYROID TISSUE, and even though it’s very rare (like 5%), I
would expect that the question stem will lead you to this diagnosis by telling you the patient is
hyperthyroid or in a thyroid storm.

Metastatic Disease to tlie Ovary


Around 10% o f malignant ovarian tumors are mets. The primary is most commonly from
colon, gastric, breast, lung, and contralateral ovary. The most common look is bilateral solid
tumors.

Krukenburg Tumor

- This is a metastatic tumor to the ovaries from the GI tract (usually stomach).

426
S E C T IO N 6:
R a n d o m O varian P ath

-Ovarian Torsion

Rotation of the ovarian vascular pedicle (partial or complete) can result in obstruction to venous
outflow and arterial inflow. Torsion is typically associated with a cyst or tumor (anything that makes
it heavy, so it flops over on itself).

^ Critical Point = The most constant finding in ovarian torsion is a large ovary.

Features:

Unilateral enlarged ovary (greater than 4 cm)


Mass on the ovary
Peripheral Cysts +/- fluid-debris levels
Free Fluid
Lack of arterial or venous flow

The Ovary is Not a Testicle: The ovary has a dual blood supply. Just because you have flow, does
NOT mean there isn’t a torsion. You can torse and de-torse. In other words, big ovary + pain =
torsion. Clinical correlation recommended.

-Hydrosalpinx

Thin (or thick in chronic states) elongated tubular structure in the pelvis.

The buzzword is “cogwheel appearance,” referring to the normal longitudinal folds of a


fallopian tube becoming thickened. Another buzzword is “string sign” referring to the
incomplete septae. The “waist sign” describes a tubular mass with indentations of its opposing
walls (this is suppose to help differentiate hydrosalpinx from an ovarian mass).

There are a variety of causes, the most common is being a skank, infidel, or free spirit (PID).
Additional causes include endometriosis, tubal cancer, post hysterectomy (without salpingectomy /
oophorectomy), and tubal ligation. Rare and late complication is tubal torsion.

- Pelvic Inflammatory Disease (PID) A plague upon the “dirty, slovenly, untidy woman”

Infection or inflammation of the upper female genital tract. It’s usually secondary to the cultural
behaviors of trollops and strumpets (collectors of Gonorrhea / Chlamydia). As a hint, the question
writer could describe the patient as “sexually disreputable. ” The question could also describe the
patient as recently appearing as a guest on the Maury Show (the "Not the Father!” show — google if
unfamiliar, it could be on the exam).

On ultrasound you are gonna see a Hydrosalpinx. The margin of the uterus may become ill defmed
(“indefinite uterus” - is a buzzword). Later on you can end up with tubo-ovarian abscess or pelvic
abscess. You can even get bowel or urinary tract inflammatory changes.

427
- Paraovarian Cyst

This is a congenital remnant that arises from the Wolffian duct. They are more common than you
think with some texts claiming these account for 10-20% of adnexal masses. They are classically
round or oval, simple in appearance, and do NOT distort the adjacent ovary (key finding). They
can indent the ovary and mimic an exophytic cyst, but a good sonographer can use the transducer to
separate the two structures.

- Ovarian Vein Thrombophlebitis

This is seen most commonly in postpartum women, often


presenting with acute pelvic pain and fever +/- 10 days post
delivery. For whatever reason, 80% of the time it’s on the
right. It’s most likely to be shown on CT (could be
ultrasound) with a tubular structure with an enhancing wall
and low-attenuation thrombus (arrow) in the expected location
of the ovarian vein. A dreaded sequela is pulmonary embolus.
Treatment is anticoagulation (and antibiotics - if fever is
present).

- Peritoneal Inclusion Cyst

This is an inflammatory cyst of the peritoneal cavity that occurs when adhesions envelop an ovary.
Adhesions can be thought of as diseased peritoneum. Whereas the normal peritoneum can absorb
fluid, adhesions cannot. So, you end up with normal secretions from an active ovary confined by
adhesions and resulting in an expanding pelvic mass. The classic history is patient with prior pelvic
surgery (they have to tell you that, to clue you in on the presence of adhesions), now with pain.

They could get tricky and say history of PID or endometriosis (some kind of inflammatory process to
piss off the peritoneum). In that case, it is likely they would show an ultrasound (or MR) with a
complex fluid collection occupying pelvic recesses and containing the ovary. It’s not uncommon to
have septations, loculations, and particulate matter within the contained fluid.

Key Features:

• Lack of walls. “Passive shape” that conforms


to and is defined by surrounding structures.

• Entrapment of an ovary. Ovary will be either


in the collection, or at the periphery.

Classic Vignette: A woman of reproductive age


with a history of endometriosis, pelvic surgery,
and pelvic inflammatory disease. Accompanied
by images (most likely ultrasound, less likely CT
or MR) or a fluid-filled mass that conforms to the Peritoneal Inclusion Cyst -
shape of the pelvis and surrounds an ovary. Adhesions around an Ovary

428
- Gestational Trophoblastic Disease
Think about this with marked elevation o f B-hCG. They will actually trend betas for tumor
activity. Apparently, elevated B-hCG makes you vomit —so hyperemesis is often part o f the
given history. Other pieces o f trivia is that moles are more common in ages over 40, and prior
moles makes you more likely to get another mole.

Hydatidiform Mole

This is the most common form, and the benign form o f the disease. There are two subtypes:

• Complete Mole (classic mole) (70%); This one involves the entire placenta. There will be no
fetus. The worthless trivia is that the karyotype is diploid. A total zebra scenario is that you
have a normal fetus, with a complete mole twin pregnancy (if you see that in the wild, write it
up). The pathogenesis is fertilization o f an egg that has lost its chromosomes (46XX).

First Trimester US: Classically shows the uterus to be filled with an echogenic, solid,
highly vascular mass, often described as “snowstorm” in appearance.

Second Trimester US: Vesicles that make up the mole enlarge into individual cysts
(2-30 mm) and produce your “bunch of grapes” appearance.

• Partial Mole (30%): This one involves only a portion o f the placenta. You do have a fetus, but
it’s all jacked up (triploid in karyotype). The pathogenesis is fertilization o f an ovum by two
sperm (69XXY). Mercifully, it’s lethal to the fetus.

US: The placenta will be enlarged, and have areas o f multiple, diffuse anechoic lesions.
You may see fetal parts.

Remember I mentioned that Theca Lutein cysts are seen in molar pregnancies.
Theca Lutein Cyst Trivia: Most commonly bilateral and seen in the second trimester

Invasive Mole

This refers to invasion o f molar tissue into the myometrium. You typically see it after the
treatment o f a hydatidiform mole (about 10% o f cases). US may show echogenic tissue in the
myometrium. However, MRI is way better at demonstrating muscle invasive. MRI is going to
demonstrate focal myometrial masses, dilated vessels, and areas o f hemorrhage and necrosis.

Choriocarcinoma (the guacamole has gone bad)

This is a very aggressive malignancy that forms only trophoblasts (no villous structure). The
typical attacking pattern o f choriocarcinoma is to spread locally (into the myometrium and
parametrium) then to spread hematogenous to any site in the body. It’s very vascular and bleeds
like stink. The classic clinical scenario is serum p-hCG levels that rise in the 8 to 10 weeks
following evacuation o f molar pregnancv. On ultrasound, choriocarcinoma (at any site) results in
a highly echogenic solid mass. Treatment = methotrexate.

429
S E C T IO N 7:
DONG BO NE

Could also be referred to as the “Penis. ”

Anatomy o f this thing in cross section:


Superficial Fascia
Deep Fascia

Tunica Albuginea

Corpus Cavernosum

Urethra Corpus Spongiosum

Fractured Penis: This is one o f the most tragic situations that can occur in medicine.
There are several potential mechanisms o f injury. Anecdotally, it seems to be most common
in older men participating in extra-marital relations with strippers named “Whisper.” There
is at least one article stating “im potence” is protective - w hich m akes sense if you think
about the pathophysiology.

They can show it on ultrasound (look for hematoma) or MRI.

Key Trivia: Defined by fracture o f the corpus


cavernosum and its surrounding sheath, the tunica
albuginea (black line outlining the dong bone).

Cartooned T1 axial through the D ong Bone (Penis):


Interruption o f the black line (tunica albuginea) - arrow.
It’s helpful to look for hemorrhage (T1 bright) in the
corpus cavernosum (the primary stabilizing strut o f this ■N
battering ram).

Stigmata o f this injury can include a sub optimal


angulation (Peyronie disease) from fibrous scar formation.

430
S E C T IO N 8:
The P e o p l e ’s P r o s t a te

Cancer: Biopsy o f the prostate is a terrible terrible situation, worse than anything you can
imagine in 1000 years o f hell. MRI o f the prostate (instead o f biopsy) is probably a little better
although many sites use an endorectal c o il.. .my God this endorectal coil! You can use prostate
MRI for high risk screening (high or rising PSA with negative biopsy), or to stage (look for
extracapsular extension).

First, let’s talk about prostate anatomy: Anatomists like to use


“zones” to describe locations, and it actually helps with Adenocarcinom a:
pathology. The anterior fibromuscular gland is dark on T1 and Peripheral Zone: 70%
T2. The central and transitional zones (together called the Transition Zone: 20%
“central g la n d ”) are brighter than the anterior muscular zone, Central Zone; 10%
but less bright than the peripheral zone on T2. In other words
the peripheral zone is the most T2 bright.

------ -
A nterior
Transitio n al Z o n e
Fibrom uscular
Central Zone
Zone Median Lobe

Peripheral Zone

Don’t Confuse
D ark Stuff = C entral Gland “Zones” and
(this is where BPH nodules live) “Glands”

Vocab interposition
B r ig h t S tu ff = P e r ip h e r a l Z o n e
is classic multiple
(th is is w h ere c a n ce r lives)
choice fuckery.

431
Prostate Cancer Continued:

M R l fin d in g fo r Prostate CA: C a n c e r is d a r k on T2 (background is h ig h ), restricts on


diffusion (low on ADC), and enhances early and washes out (type 3 curve - ju st like a breast
cancer).

Dark on T2 Dark on ADC Enhances


(restricts diffusion)

Bone scan is the m oney for prostate mets (vertebral body mets).

Trivia: PSA can be useful w hen considering risk o f bone mets. There is at least 1 paper that
says a PSA < 20 has a high predictive value in ruling OUT skeletal mets. In other words,
PSA tends to be high when disease is aggressive enough to go to the bones.

Staging: The main Stage II S tage III


thing to know is
stage II vs stage III,
Confined by capsule (T2) Extension through capsule (T3a)
as extra capsular
extension is the
most important A butm ent o f the capsule Bulging o f the capsule,
factor governing without bulging or frank extension through it
treatment.

Seminal vesicles (T3b) and the nerve bundle are also right behind the prostate and can get
invaded (urologists love to hear about that).

432
PSA, Gleason Score, and PI-RADS

Key Concept: Prostate Cancer is a lot like Breast Cancer - in that it is


very common and often a “benign” entity ( l i n 6 men will be diagnosed Clinically Significant Cancer:
with prostate cancer , 3% will die of it). There are tons and tons of
Gleason Score > 7
elderly people with low grade prostate CA and breast cancers (DCIS)
that will never even know they have it unless somebody does a biopsy Cancer Volume > 05cc
on them... or worse performs screening on them. Which brings us to
the PSA. When family medicine doctors started doing PSA checks on Extension through the Capsule
everybody suddenly there was this enormous spike in the incidence of (loss of normal T2 signal in the
prostate cancer. Suddenly everyone has cancer (I blame the liberal seminal vesicle, bulging of the
media). capsule / frank invasion through
it, loss of fat between prostate
Who Dies of Prostate Cancer ? There are 3 factors - Gleason Scores and rectum, neurovascular
8-10, Advanced Clinical Stage (mets), and PS A > 20. People with bundle asymmetry).
Gleason scores less than 6, PSA < 10, and staging < 2a (less than half
the lobe) are unlikely to die of the prostate cancer.

Management ? It is gonna depend a lot on who you ask. Most people will add hormone therapy around stage
2B (more than half the lobe) but it is complicated.

PSA: This is an antigen produced by the normal prostate and incorporated into the ejaculate (from the window
to the wall), for the purpose of dissolving cervical mucus etc... It also leaks out into the blood in small amounts
in normal men and in larger amounts when the prostate is abnormal (cancer, infected, riding a bicycle, sticking
stuff up your ass that don’t belong up your ass, and benign hypertrophy).Family Medicine docs willscreen
people starting at 50. Some numbers to have a vague familiarity with include: Normal < 4.Low Risk Category
< 10. High Risk Category > 20. After prostatectomy normal is zero, if it rises to 0.2 think recurrence. After
radiation anything over 2.0 is concerning for recurrence (although it’s a little more complicated than that). PSA
< 20 = bone mets unlikely.

Gleason: There is a “grade” a “score” and a “group” - you better fucking believe the distinction is fair game..

• Gleason Grade: This refers to the histological patterns in the sample “ 1” is normal, “5” is very very not
normal. 2-4 are in the middle.

• Gleason Score: This is the sum of the two most common “grades.” The more common pattern is always first.
So “A” 3+5 = 8, and “B” 5-1-3 = 8. “B” has more of 5 than “A” and is therefore worse off. Total scores less
than 6 aren’t usually reported.

• Gleason Group: This uses pattern scores to reflect the actual risk. This removes the confusion over one 7
being worse than another 7. For example 3+4 is grade 2, and 4+3 is grade 3. Grade is 1-5

PI-RADS: Scores are calculated by using data from DWI, T2, and Enhancement. Tumor in the Transition zone
is determined primarily from T2 (t for t). Peripheral zone is determined primarily from DWI.

PZ-DWI PI-RADS TZ-T2

1 Normal 1 1 - Normal N odules - full capsule

2 - Linear / Wedge 2 2 - A typical Nodule - no capsule


^oca’
3 - Vague Patchy 3 3 - Heterogenous, blurry margins
ent ?.e‘

4 - Focal < 1.5 cm 4 4 - Low Signal < 1.5 cm

5 - Focal > 1.5 cm or with extracapsular ext 5 5 - Low Signal > 1.5 cm or w ith extracapsular ext

433
Benign prostatic hyperplasia (BPH): Obviously this is super common, and makes
old men pee a lot. Volume o f 30cc is one definition. M ost com m only involves the
transitional zone (cancer is rare in the transitional zone - 10%). The central gland enlarges
with age. The median lobe com ponent is the one that hypertrophies and sticks up into the
bladder. It can cause outlet obstruction, bladder wall thickening (detrusor hypertrophy), and
development o f bladder diverticulum.

The IVP buzzw ord is “J shaped”, “Fishhook”, or “Hockey stick” shaped ureter - as
the distal ureter curves around the enlarged prostate.

With regard to the BPH nodules you


see on MRI, they are usually;
In the Transitional Zone
(Central Gland)
• T2 Heterogenous
Can Restrict Diffusion
May enhance and washout

Post Biopsy Clianges: Classically T1 bright stuff in the gland. It’s subacute blood.

Th e People’s Prostate Pathology Sum m ary Chart


So You Can More Easily Smell What the Prostate is Cooking

T2 ADC Enhancem ent

Peripheral Zone Early Enhancement,


Dark Dark
Tumor Early Washout

Peripheral Zone
Hemorrhage Dark
(sometimes Dark (less dark) None
*Typically T1 bright)
Post Biopsy

Central Gland /
Early Enhancement,
Transitional Dark “Charcoal” Dark
Early Washout
Zone Tumor

BPH Dark “Well Defined” Less Dark Can Enhance

434
S E C T IO N 9:
M iscella n eo u s Ma le

Calcified Vas Deferens:

You see this all the time in bad diabetics.


W ho is a “bad” diabetic ? Depends on who you ask
I guess. If you ask me, diabetics who have those
gross long ear hairs are “bad diabetics.”

This can be shown on plain film or CT.

P©lviC C y s ts : First think about lower pelvic


cysts in a male as either midline or lateral. Male Pelvic Cysts

Midline Lateral

Prostatic Seminal
Utricle Vi'sick' Cyst

Divcrticulosis
Mullerian of the ampulla
Duct Cyst (1| \I1S
deferens

Ejaculatory
Lateral Midline Duct Cysts

LATERAL:

Seminal Vesicle Cyst:


The classic look is a unilateral cyst that is lateral to the prostate. If they get large they
can look midline, but if they show you a large one you w on’t be able to tell it from a utricle
cyst. They can be congenital or acquired.
Congenital Trivia:
• Associated with renal agenesis
• Associated with vas deferens agenesis
• Associated with ectopic ureter insertion
• Associated with polycystic kidney disease
Acquired Trivia:
• Obstruction often from prostatic hypertrophy, or chronic infection/scarring
• Classic history is prior prostate surgery

435
M ID L IN E :

Prostatic Utricle Cyst: iViuiierian Duct Cyst


Things to Know Things to Know
This represents a focal dilation in the prostatic urethra This represents a failed
(remnant of the Mullerian duct) - as such they communicate regression of the caudal ends
with the urethra and can cause “dribbling” (both on and off of the Mullerian ducts (male
the basketball court). equivalent of the vagina /
Hypospadias is the most common associated condition - cervix).
which makes sense given the relationship with the urethra. Does not communicate with
Other Associations: Prune Belly Syndrome, Downs, the urethra and does not have
Unilateral Renal Agenesis. the same associations as
and my personal favorite - the Imperforate Anus utricle cyst.
The tendency towards superinfection is also explained by the
communication with the urethra.
A sneaky trick would be to show it on a RUG, where a
prostate utricle cyst would look like a focal out-pouching
from the prostatic urethra.

THIS VS THAT:

utricle Cyst Mullerian Cyst

Pear Shaped and Usually Smaller Tear Drop Shaped

Will NOT extend above the base of the prostate Will extend above the base of the prostate

Conununicates with the Urethra (Utricle), therefore Does NOT communicate with the Urethra, should
could opacify on a RUG not opacify on a RUG

Both have a tiny risk (mostly case reports) of malignancy (various types: endometrial, clear cell, squamous).

Prostate Abscess: This can cause a thick walled, septated, heterogenous, cystic lesion
anywhere in the prostate. It is usually bacterial (E. coli). When chronic it can have a more “swiss
cheese” appearance referred to as “cavitary prostatitis.” Usually this is imaged via transrectal
ultrasound - because it gives you (the urologist) the option to do an image guided drain.

436
-The Painful Scrotum-

Torsion of the Testicle:


Results from the testis and spermatic cord twisting within the serosal space leading to ischemia.
If it was 1950 you’d call in your nuclear medicine tech for scintigraphy. Now you just get a Doppler
ultrasound.

The grey scale findings are fairly straight forward. The testicle is going to be darker (hypo-echoic)
and asymmetrically enlarged - at least in the chronic setting. If it’s chronic then it will shrink up.

The doppler findings are somewhat complex. The most obviously / basic look would be to show you
absent arterial flow. This would be the equivalent of an underhand slow pitch. The curve ball would
be to show you preserved arterial flow BUT with increased resistance and a decreased diastolic flow
(or reversed diastolic flow).

That is correct my friends. Arterial flow does NOT need not be absent for torsion to be
present (depending on the duration and severity). This leads the way for some serious
fuckery if the test writer wants to be an asshole.

The best way to think about it is like this: Testicle = Brain.

Just like the brain requires continuous diastolic flow (the thing is never off), so does the testicle.

So when you look at the waveform for a rule out torsion case you need to remember that torsion has
three possible patterns:

1- Classic Absence of Arterial Flow Normal


2 - High resistance Arterial Flow (with
decreased or reversed diastolic flow)
3 - Monophasic Arterial Waveform (loss of Abnormal:
the normal dicrotic notch) Loss of Diastolic Flow
Fuckery: We are talking about testicular Abnormal:
artery wave forms here. The normal Reversal of Diastolic Flow
cremasteric artery will not have diastolic flow
(think about that thing as the artery to a
muscle) - it’s normally high resistance. Abnormal:
Monophasic

Cause: The “bell-clapper deformity,” which describes an abnormal high attachment of the tunical
vaginalis, increases mobility and predisposes to torsion. It is usually a bilateral finding, so the
contralateral side also gets an orchiopexy.

Viability: The viability is related to the degree of torsion (how many spins), and how long it has
been spun. As a general rule, the surgeons try and get them in the OR before 6 hours.

437
High Flow States / Hyperemia:

I f torsion dem onstrates decreased flow, it is useful to have an idea about what can
demonstrate increased flow (decreased R.I. or increased diastolic flow)

Two things w orth thinking about in this scenario: (1) Epididym o-orchitis (2) Detorsion.

The distinction betw een these two will be the clinical scenario.
Orchitis is painfiil. D etorsion is pain free.

Epididymitis: Inflam m ation o f the epididymis, and the m ost com m on cause o f acute
onset scrotal pain in adults. In high-school / college age m en (likely sexually active men) the
typical cause is chlam ydia or gonorrhea. In m arried men (not likely to be sexually active) it
is more likely to be e-coli, due to a urinary tract source. The epididym al head is the m ost
affected. Increased size and hyperem ia are your ultrasound findings. You can have infection
o f the epididymis alone or infection o f the epididymis and testicle (isolated orchitis is rare).

Typical S p re a d : T a il-------► Body -------► Head

Gamesmanship: Could be asked as “w here is the m ost com m on location” ? = Tail (because
in most cases it starts there).

Orcliitis: Typically progresses from epididym itis (isolated basically only occurs from
mumps and TB). It looks like asymm etric hyperemia.

Multiple small dark


Typical: (1) Epididymitis ► (2) Epididym itis + Orchitis
(hypoechoic) nodules
in enlarged testis is
Mumps: Straight to Orchitis the classic military
TB: Can also go orchitis first (then epididymitis) appearance of TB.

Im pending Infarct: The swelling o f the testicle can becom e so severe that it com prom ises
venous flow. In this case you will see loss o f diastolic flow (or reversal) - sim ilar to the
atypical torsion patterns. This is reported as a sign o f “im pending infarct.”

Testicular Trauma: The big distinction is rupture vs fracture. Surgical intervention is


required if there is testicular rupture. Intratesticular fracture, and hem atom as (small) do not
get surgery.

• Rupture: D isrupted tunica albuginea, heterogenous testicle, poorly defined testicular


outline

• Fracture: Intact tunica albuginea, linear hypoechoic band across the parenchym a o f the
testicle, well defined testicular outline.

438
S E C T I O N lO :
T esticu lar C a n cer

Risk Factors: Cryptorchidism (for both testicles), Gonadal Dysgenesis, Klinefelters, Trauma, Orchitis,
and testicular microlithiasis (maybe). The step 1 trivia is that cryptorchidism increases the risk of cancer
(in both testicles), and is not reduced by orchiopexy.

Most Classic: Male age 18-35, history of undescended testicle ^ seminoma.

Gamesmanship: For the purpose of muhiple choice, “non-palpable testicle” = undescended testicle.

/I Most Classic Sneaky Move: Most testicular


tumors met via the
Male age 18-35 with mystery retroperitoneal adenopathy. lymphatics
Next step ^ Ultrasound his balls (retroperitoneal
nodes at the level of
Possibly the only thing useful to remember from med school the renal hilum).
embryology is that retroperitoneal (para-aortic & paracaval) nodes are The testable
the initial landing station / blast off point for testicular cancers. exception is
choriocarcinoma,
Subtypes: which mets via the
Germ Cell (90%) Non Germ Cell (10%) blood.
Seminoma (50%)
^ Sertoli
- seen more in
the 4th decade
Leydig
Non- Seminoma (50%)
-Teratoma
-Yolk Sac
-Mixed Germ Cell (35%)

Work-Up: Ultrasound of the balls is done with a high frequency (7-10 MHz) probe. It is useful to
differentiate solid vs cystic masses that are within the ball (testicle) or outside it (extra testicular).

Intratesticular mass
Mass in ^ Sohd
the V (More likely Malignant)

\
Ball Sac Extratesticular mass

If it’s Cystic
extratesticular (More likely Benign)
and cystic, it’s
probably
benign.

In general, hypoechoic solid intratesticular masses should be thought of as cancer until proven
otherwise. Doppler flow can be helpful only when it is absent (can suggest hematoma - in the right
clinical setting) — more on that later.

439
Generalizations for the Purpose of Multiple Choice:
Seminoma: This is the most common Non-Seminomatous Germ Cell
testicular tumor, and has the best prognosis as Tumors: Basically this is not a seminoma.
they are very radiosensitive. They are much We are talking about mixed germ cell tumors,
more common (9x) in white people. The teratomas, yolk sac tumors, and
classic age is around 25. It usually looks like choriocarcinoma. They typically occur at a
a homogenous hypoechoic round mass, which young age relative to seminomas (think
classically replaces the entire testicle. On teenager). They are more heterogeneous and
MRI they are usually homogeneously T2 dark have larger calcifications.
(non-seminomatous GCTs are often higher in
signal). NSGCTs = heterogeneous with cystic spaces
and calcifications
Seminomas = hypoechoic and homogeneous

Burned-Out Testicular Tumor


(Shrinidng Tumor) - Germ cell tumors
can sometimes “bum out” and regress into a
small mass or tiny calcification. The idea is
that you’ve had spontaneous regression of a
germ cell testicular neoplasm, that is now
calcified - after growing too fast for its own
good. This can happen even in the setting of
widespread metastatic disease. An important Time Point 1: Time Point 2:
pearl is that there can still be viable tumor in Heterogenous Mass shrinks, now
there. Management is somewhat Mass in the Testicle has calcifications
controversial and unlikely to be asked (most
people pull them out).

Testicular Lymphoma: Can be primary or secondary. The look is highly variable - can be a
diffusely enlarged ill defined hypoechoic testicle or multiple hypoechoic masses. AGE is the
primary discriminator (most common over 60).
Bilateral disease is also a hint - but could be a great source of multiple choice fuckery. Most
testicular lymphoma is unilateral (60%) - BUT bilateral lymphoma is the most common bilateral
testicular tumor - so watch the wording of questions. Just be aware that lymphoma can “hide” in the
testes because of the blood testes barrier. Immunosuppressed patients are at increased risk for
developing extranodal/ testicular lymphoma.
Buzzword = multiple hypoechoic masses o f the testicle.

440
High Yield Testicle Tumor Trivia

Seminoma is the most common (Age 18-35) and has the best prognosis (it melts with radiation)

Lymphoma is the most common Age > 60

Lymphoma is usually Diffuse B- Cell Subtype

Lymphoma is usually unilateral (60%) - even though it is the most common bilateral ball cancer

Homogenous and Microcalcifications = Seminoma

Cystic Elements and Macrocalcifications = Mixed Germ Cell Tumor / Teratoma

Most testicular tumors met via the lymphatics to the retroperitoneal nodes

Choriocarcinoma mets via the blood - and tends to bleed like stink

Gynecomastia can be seen with Sertoli Leydig Tumors

Peutz-Jeghers Syndrome can be seen with Sertoli Tumors

Bilateral testicular cancer = Lymphoma

Bilateral hypoechoic masses = Lymphoma

Bilateral solid testicular masses + congenital adrenal hyperplasia = Adrenal Rests

Bilateral solid testicular & epididvmal masses = Testicular Sarcoid

Bilateral 1-6-mm, hyperechoic, avascular masses + Cowden Syndrome = Focal Lipomatosis

Elevated Beta hCG Elevated AFP

Seminoma M ixed Germ Cell (Non-S)

Choriocarcinom a (Non-S) Yolk Sac (Non-S)

Sneaky Moves:

• Male older than 60, bilateral testicular masses, with fever/weight loss = Lymphoma
• Male age 18-35, pelvic mass with an ipsilateral draining vein that empties into the inferior vena cava (if
right sided) or left renal vein (if left sided). Then they show you a scrotum with only one testicle.
Think cancer of an undescended testicle - whisper to yourself “nice try assholes.”

• Black guy with “lung problems” and uveitis. Multiple small hypoechoic testicle lesions that don’t
change over time. Think “sarcoid of the balls” —yes it is a thing. Involvement of the epididymis is
actually more common. Most likely involvement of the testicle will also have involvement of the
epididymis.

441
staging Pearl

Testicular mets should spread to the para-aortic, aortic, caval region (N1-N3).
It’s an embryology thing.

If you have mets to the pelvic, external iliac, and inguinal nodes - this is considered “non-
regional” i.e. M l disease. The exception is some kind of inguinal or scrotum surgery was done
before the cancer manifested - but I wouldn’t expect them to get that fancy on the test. Just
remember inguinal / pelvic nodes are non-regional and a higher stage (Ml).

Mimics: Don’t Cut Off My Bails (unless... you really need to)

Hematoma: No increased internal flow on Doppler within


Infarct: No blood flow on
the “masses” - History of Acute Pain - horrible fucking pain -
Doppler - History of Acute like 3 days ago, after getting kick in the nuts while training for
Pain - horrible flicking pain - no-gi Pans (standard elevator entry - butterfly to inside
like 5 days ago. No Trauma. sankaku). Follow up shows the same areas getting smaller.

Infection: Unlike infarcts and hematomas, infection can have increased blood flow and create a
closer mimic of tumor. However, the clinical history will be different. Focal orchitis hurts and these
patients have fever, WBC elevation, etc... The question writer will need to give you some clinical hint
that it is infection and not a tumor. Otherwise, a good clue would be to show a follow up in 2-4 weeks
with improvement.

Epidermoid Cyst: This is a benign Tubular Ectasia of the Rete Testis:


mass of the testicle (no malignant potential), This is a common benign finding, resulting from
with an Aunt Minnie “onion skin” look, - obliteration (complete or partial) of the efferent ducts.
alternating hypoechoic and hyperechoic It’s usually bilateral - and in older men. The location of
rings. It’s relatively non-vascular relative to the cystic dilation is next to the mediastinum testis.
the rest of the testicle. Think about this as a normal variant. It requires no
follow up or further evaluation.

\
442
S E C T I O N 1 1:
Ma l e In f e r t i l i t y

Causes: Can be thought o f as Obstructive vs Non-Obstructive

• Obstructive: Congenital bilateral absence o f the vas deferens (seen in Cystic


Fibrosis), ejaculatory duct obstruction, prostatic cysts. Think about associated
renal anomalies (Zinner Syndrome).

* Non-Obstructive: Varicocele, Cryptorchidism , Anabolic Steroid Use, Erectile


D ysfunction and “The Liberal M edia.”

Varicocele: This is the most com m on correctable cause o f infertility. They can be
unilateral or bilateral. Unilateral is much more com m on on the left. Isolated right sided
should make you think retroperitoneal process com pressing the right gonadal vein.

Cryptorchidism : U ndescended testes. The testicle is usually found in the inguinal


canal. The testicle has an increased risk o f cancer (actually they both will - w hich is weird).
It’s most com m only seen in prem ature kids (20%).

M ajor complication association fo r cryptorchidism:


• M alignant degeneration - o f both the undescended and contralateral testicle
• Infertility
• Torsion
• Bowel Incarceration - related to the association o f indirect inguinal hernia

Gamesmanship: A good distractor w ould be “orchitis.” It’s a pathology that


involves the balls, so it’s not totally far fetched. O bviously they can get
orchitis... but not at a higher rate. It’s not a reported association - so don’t fall
for that.

Zebras and Syndromes A ssociated With M ale Infertility:

* Pituitary Adenoma m aking prolactin


*Kallman Syndrome (can ’t smell + infertile)
Is *Klinefelter Syndrome (tall + gynecom astia + infertile)
»S **
^ ■v Vv *Zinner Syndrome (renal agenesis + ipsilateral sem inal vesicle cyst)

443
S E C T IO N 12:
T r a n s g e n d e r M isc

This is an in vogue topic popular in articles of the various academic journals. As such, I feel
compelled (Jordan Peterson style) to at least touch on some of the basics related to this topic.
Vocab: Transgender: Gender self identity does not match their genetic / sex assigned identity at birth.
The alternative is a “Cisgender” - A man who identifies as a man (XY) or a woman who identifies as
a woman (XX). Gender and sexual orientation are different things. Sexual Orientation is the emotion
/ sexual attraction to others. Transgender people are not necessarily homosexual. Gender self identity
is different than who you want to fuck.
Transgender Man: A female (XX) who identifies as the masculine (male) gender
Transgender Woman: A male (XY) who identifies as the feminine (female) gender
“Top and Bottom” Surgeries: Slang for breast & genital procedures (gender-affirming surgery)
Vaginoplasty: Procedure to create a functional and cosmetically acceptable neovagina.
• Penoscrotal inversion (PIV): The most common procedure with the lowest complication rate. It
involves orchiectomy and “penile disassembly” in a method similar to the induction ceremony of the
feared Unsullied Army. The skin from the disassembled parts is inverted / folded back to create a
tunnel. The clitoris is constructed using the native penile neurovascular anatomy.
• Intestinal Interposition: Second line strategy which involves using a segment of bowel (usually
rectosigmoid colon) to create a neovagina by coloperineal anastomosis. They do this because there
uhhh - how best to say this - is “insufficient tissue” from the penis to make the tunnel.
Phalloplasty: Procedure to create a functional and cosmetically acceptable neopenis. Standing
urination is a typical metric of success. Several months prior to constructing the neopenis - these
patients typically undergo hysterectomy (+/- oophorectomy).
• Phalloplasty: Vaginectomy and urethroplasty are performed using vascularized vaginal mucosa to
try and elongate the urethra. Skin flaps often fail - but the “RPFF” or radial flap procedure is
probably the most common. Most of these skin grafts use a “tube in tube” strategy. I can’t believe
the details would be on the exam - but you can imagine they roll the skin and subcutaneous fat up to
make something that looks like a dick. Maybe not the most impressive of dicks - but a dick none
the less. Hey... you know what they say, it’s not the size of the dog in the fight - it’s the size of the
fight in the dog. Not sure if this expression applies to a surgically created neo-phallus but I ’m trying
to be positive.
• Metoidioplasty: An alternate technique to phalloplasty - which has a lower complication rate. The
downside is the length of the created neopenis is usually not enough to have sex, but they can still
pee standing up (a major metric to success of the procedure). This technique is performed by first
using hormones to hypertrophy the clitoris (like a female body builder). Then the urethra is
lengthened (by dividing various ligaments) and anastomosed to the clitoris, which serves as the
glans. Labia minora is gonna be the source for skin to construct the shaft.
Testicular prostheses / Scrotoplasty: Generally made of silicone (high density on CT) and placed
around 6 month post phalloplasty. Just like those dick pumps you sometimes see in diabetics - a
hydraulic pump apparatus can be placed - that thing will have tubing and be more water density.
Complications: Older technique didn’t resect the vagina - these patients were prone to fistula
between the neourethra and native vagina. DVT / PE is a post op risk if the patient is taking hormone
therapy. Bleeding, infection, urinary complication (urethral stenosis etc) all can occur - as one might
expect.
Some other surgeries that could come up on the exam include breast implants, and the various neck
surgeries to make a dude look less like a dude (thyroid chondroplasty / tracheal shave) and sound less
like a dude (glottoplasty, cricothyroid approximation).
444
S E C T IO N 13:

EarlyPregnancy-
Vocab:

•Menstrual Age: Embryologic Age + 14 days


•Embryo: 0-10 weeks (menstrual age)
•Fetus: > 10 weeks (menstrual age)
•Threatened Abortion - Bleeding with closed cervix
•Inevitable Abortion - Cervical dilation and/or placental and/or fetal tissue hanging out
•Incomplete Abortion - Residual products in the uterus
•Complete Abortion - All products out
•Missed Abortion - Fetus is dead, but still in the uterus.

Intradecidual Sign: This is the early gestational


sac. W hen seen covered by echogenic decidua is very
characteristic o f early pregnancy. You can see it
around 4.5 weeks. You w ant to see the thin echogenic
line o f the uterine cavity pass by (not stop at) the sac
to avoid calling a little bit o f fluid in the canal a sac.

Small amount of
fluid between

Double Decidual S a c Sign:

This is another positive sign o f early


pregnancy.

It’s produced by visualizing the layers


o f decidua.

Decidua Vera

Decidua Capsularis

Double Decidual Sac Sign


445
Yolk Sac: This is the first structure visible
within the GS. The classic teaching was you
should always see it when the GS measures
8mm in diameter. The thing should be oval or
round, fluid filled, and sm aller than 6 mm.

The yolk sac is located in the chorionic cavity,


and hooked up to the umbilicus o f the embryo
by the vitelline duct.

Yolk Sac Gone Bad: The yolk sac shouldn’t be


too big (> 6 mm), shouldn’t be too small Yolk Sac - in the chorionic cavity

(< 3 mm), and shouldn’t be solid or calcified.

The Amnion: The membranes o f the


amniotic sac and chorionic space
typically rem ain separated by a thin
layer o f fluid, until about 14-16 weeks
A= Amniotic Space
at which point fusion is normal. If the
amnion gets disrupted before 10 weeks Arrow = Amnion
the fetus m ight cross into the chorionic C= Chorionic Space
cavity and get tangled up in the fibrous
bands. This is the etiology o f am niotic
band syndrome, which can be terrible
(decapitation, limb am putation, etc...).

Double Bleb Sign: This is the earliest visualization o f the embryo. This is two fluid
filled sacs (yolk and amniotic) with the flat em bryo in the middle.

Double Bleb Sign

446
Crown Rump Length - This is typically used to estimate gestational age, and is more accurate
than menstrual history. *Embryo is normally visible at 6 weeks.

Anembryonic Pregnancy - A gestational sac without an embryo. When you see this, the
choices are (a) very early pregnancy, or (b) non-viable pregnancy. The classic teaching was you
should see the yolk sac at 8 mm (on TV). Just remember that a large sac (>8-10 mm) without a yolk
sac, and a distorted contour is pretty reliable for a non-viable pregnancy.

Pseudogestational Sac - This is not


the same thing as an anembryonic pregnancy.
This is seen in the presence of an ectopic
pregnancy. What you are seeing is a little bit
of blood in the uterine cavity with
surrounding bright decidual endometrium
(charged up from the pregnancy hormones).

Subchorionic Hemorrhage: These are very common.


The thing to know is that the percentage of placental
detachment is the prognostic factor most strongly associated
with fetal demise; hematoma greater than 2/3 the
circumference of the chorion has a 2x increased risk of
abortion. Other trivia: women older than 35 have worse
outcomes with these.

Implantation Bleeding: This is a nonspecific term referring to a small subchorionic hemorrhage that
occurs at the attachment of the chorion to the endometrium.

Criteria for Fetal Demise:

Diagnostic of Pregnancy Failure Suspicious for Pregnancy Failure

Crown-rump length of >7 mm and no heartbeat No embryo >6 wk after last menstrual period

Mean sac diameter of 16-24 mm and no


Mean sac diameter of >25 mm and no embryo
embryo

No embryo with heartbeat >2 wk after a scan No embryo with heartbeat 13 days after a scan
that showed a gestational sac without a yolk sac that showed a gestational sac without a yolk sac

No embryo with heartbeat >11 days after a scan No embryo with heartbeat 10 days after a scan
that showed a gestational sac with a yolk sac that showed a gestational sac with a yolk sac

447
Pregnancy of Unknown Location: This is the vocabulary used when neither a normal lUP
or ectopic pregnancy is identified in the setting of a positive b-hCG. Typically this just means it is a
very very early pregnancy, but you can’t say that with certainty. In these cases you have three
possibilities:

1 - Normal Early Pregnancy The management is follow up (serial b-hCG) and repeat US
2 - Occult Ectopic assuming the patient is hemodynamically stable.
3 - Complete Miscarriage

Ectopic: The following increase the risk of ectopic pregnancy: Being a free spirit (Hx of FID),
Tubal Surgery, Endometriosis, Ovulation Induction, Previous Ectopic, Use of an lUD.

The majority of ectopic pregnancies (nearly 95%) occur in the fallopian tube (usually the ampulla). A
small percentage (around 2%) are “interstitial” developing in the portion of the tube which passes
through the uterine wall. These interstitials are high risk, as they can grow large before rupture and
cause a catastrophic hemorrhage. It is also possible (although very rare) to have implantation sites in
the abdominal cavity, ovary, and cervix.

Always start down the ectopic pathway with a positive BhCG. At around 1500-2000 mlU/L you
should see a gestational sac. At around 5000 mlU/L you should see a yolk sac. As a general rule, a
normal doubling time makes ectopic less likely.

Tubal Ring Sign: An echogenic ring,


which surrounds an un-ruptured ectopic
pregnancy. This is an excellent sign of ectopic
pregnancy - and has been described as 95%
specific.

Heterotopic Pregnancy: This is a baby


in the uterus and a baby in the tube (or other
ectopic location). This is pretty rare, and
typically only seen in women taking ovulation
drugs, or with prior bad PID.

Th e Big 3 to Remem ber with E cto p ics (positive B-hCG)

(1) Live Pregnancy / Yolk Sac outside the uterus = Slam Dunk
(2) N othing in the uterus + anything on the adnexa (other than corpus luteum) =
75-85% PPV for ectopic
a. A moderate volum e o f free fluid increases this to 97% PPV
(3) N othing in the uterus + moderate free fluid = 70% PPV
a. More risk if the fluid is echogenic

448
Fetal Blometrv and Fetal Growth:
In the second and third trimesters, four standard m easurem ents o f fetal growth are made
(Biparietal, Head Circumference, Abdom inal Circumference, and Fem ur Length). The
testable trivia seems to include what level you make the m easurem ent, and what is and is not
included (see chart).

Fetal Measurement For Growth


M easurem ent M ade NOT including Trivia
Recorded at the level Affected by the shape
o f the thalam us from o f the fetal skull
Biparietal D iam eter the outeiinost edge o f (false large from
“BPD” the near skull to the brachycephaly, false
inner table o f the far small from
skull dolichocephaly)

Recorded at the same Does NOT include Affected less by head


Head Circum ference
slice as BPD the skin shape

Recorded at the level Does N OT include


Abdominal o f the junction o f the the subcutaneous soft
Circumference um bilical vein and tissues
left portal vein

Longest dim ension o f Fem oral epiphysis is


Fem ur Length
the femoral shaft N O T included

Estim ated Fetal Weight: This is calculated by the m achine based or either
(1) BPD and AC, or (2) AC and FL.

Gestational Age (GA): Ultrasound estim ates o f gestational age are the m ost accurate
in early pregnancy (and becom e less precise in the later portions). Age in the first trim ester is
made from crown rump length. Second and third trim ester estimates for age are typically
done using BPD, HC, AC, and FL - and referred to as a “com posite GA.”

Gestation Age (Less Good Later in the Pregnancy)


First Trimester - Crown Rum p Length Accurate to 0.5 weeks

Accurate to 1.2 w eeks (between 12 and 18 weeks)


2"^! and Trim ester - “Composite GA ”
A ccurate to 3.1 w eeks (between 36 and 42 weeks)

449
Intrauterine Growth Restriction:
*Baby is smaller than expected

Readings Suggestive of iUGR:

• Estim ated Fetal Weights Below 10'^ percentile


• Femur Length / Abdominal Circum ference Ratio (F /A C) > 23.5
• Umbilical Artery Systolic / Diastolic Ratio > 4.0

N ot A ll is lost: I f the kid is measuring small, he m ight ju st be a little guy. If he has


normal Doppler studies - most o f the time they are ok.

Maybe A ll is lost: If the kid is measuring small, suggesting IUGR, and he has
oligohydramnios (AFI < 5) or polyhydram nios, he/she is probably toast.

Trivia: M ost common cause for developing oligohydram nios during the 3rd trim ester
= Fetal Growth Restriction associated with Placental Insufficiency.

TH IS vs THAT: Sym m etric vs Asym m etric:

• Asym metric: Think about this as a restriction o f w eight followed by length. It


is the more com mon o f the two types. The head will be norm al in size, w ith the
body being small. Some people call this “head sparing,” as the body tries to
protect the brain. You see this m ainly in the third trim ester, as a result o f
extrinsic factors.

• The classic scenario w ould be normal growth for the first tw o trimesters, w ith a
normal head / small body (small abdom inal circum ference) in the third trim ester -
with a mom having chronic high BP / pre-eclampsia.

• There are a bunch o f causes. I recom m end rem em bering these three; High BP,
Severe M alnutrition, Ehler-Danlos.

Symmetric: This is a global growth restriction, that does NOT spare the head.
This is seen throughout the pregnancy (including the first trimester). The head
and body are both small. This has a much w orse prognosis, as the brain doesn’t
develop normally.

There are also a bunch o f causes. I recom m end rem em bering these: TORCH
infection. Fetal A lcohol Syndrom e / Drug Abuse, C hrom osom al
Abnorm alities, and Anemia.

450
Intrauterine Growth Restriction - Continued

Symmetric lUGR Asymmetric lUGR


-Small Head -Abdomen is small
-Skinny Body -Aching Belly (Malnutrition)
-Same Throughout Pregnancy -Abnormally High BP
-Syphilis {Among other TORCHs) -“Alastic” skin (Ehler Danlos)
-Scotch Whiskey (Fetal EtOH) -Asymmetric Time Interval
-Some Extra Chromosomes (primarily seen in 3rd trimester)

MCA Doppler:

Technique: Measurement should be done at the proximal 1/3 of the vessel.

Normal: The fetal MCA should be a high resistance waveform with


continuous forward flow of diastole (the space between the waveform
peaks).

ABnormal: When the fetal brain experiences hypoxia there is a


reflex response to protect the brain. This “brain-sparing reflex” will
manifest early on as an increase in diastolic flow (less resistance).

Cerebroplacental Ratio - This is a ratio of the pulsatility in the MCA and Umbilical Artery that is used to
evaluate the brain sparing reflex and predict outcomes. High MCA diastolic flow (from hypoxia induced
cerebral vascular dilation) combined with decreased diastolic flow in the umbilical artery (from increased
placental resistance) will result in a devastating outcome (i.e. - kid will have deficits in cognitive
functioning / poor academic achievement possibly resulting in a residency match to family medicine).
Cerebroplacental Ratio: >1:1 is normal

Fetal Anemia : There are a variety of causes (maternal alloimmunization and parvovirus are the most
common). MCA Doppler is useful with an increase in peak svstolic flow often occurring before hydrops.
Other imaging finds include enlargement of the liver and spleen.

451
Um bilical A rtery Systolic / Diastolic Ratio:

The resistance in the umbilical artery should progressively decrease with gestational age. The
general rule is 2-3 a t 32 weeks. The ratio should not be more than 3 at 34 weeks. An elevated
S/D ratio means there is high resistance. High resistance patterns are seen in pre-eclampsia and
lUGR. Worse than an elevated ratio, is absent or reversed diastolic flow - this is associated with
a very poor prognosis.

The way I remember this: I think about the kid starting out as a clump o f cells/mashed up soup.

jiL L im
Early on he/she is basically just a “muscle.” Then as he/she gets closer and closer to viable age
he/she becomes more like a “brain.” Once you think about it like that - muscle vs brain, it’s
much easier to understand why the diastolic flow goes up (S/D ratio goes down).

Remember the brain is always on, so it needs continuous flow. Muscles are only on when you
need to perform amazing feats o f strength. So more brain = more diastolic flow. This also
explains why absent or reversed diastolic flow is so devastatingly bad. In fact, the evil socialist
health care systems in Europe use carotid ultrasound as a cheap brain death test (no diastolic
flow in the ICA = brain dead). Coincidently, the absence o f diastolic flow in the ICA is also
used in many American Radiology Departments as hiring criteria for the QA Officer.

Biophysical Profile: This thing was developed to look for acute and chronic hypoxia. Points
are assigned (2 for norm al, 0 for abnormal). A score of 8-10 is considered normal. To call something
abnormal, technically you have to be watching for 30 mins.

Components of Biophysical Profile


Amniotic Fluid At least 1 pocket measuring > 2 cm in a vertical plane Assess Chronic Hypoxia
Fetal Movement 3 discrete movements Assess Acute Hypoxia
Fetal Tone 1 episode of fetal extension from flexion Assess Acute Hypoxia

Fetal Breathing 1 episode of “Breathing motion” lasting 30 sec Assess Acute Hypoxia
Non-stress Test 2 or more fetal heart rate accelerations of at least 15 Assess Acute Hypoxia
beats per minute for 30 seconds or longer

452
- Macrosomia -
Babies that are too big (above the 90* percentile). Maternal diabetes (usually gestational, but
could be type 2 as well), is the most common cause. As a point o f trivia, type 1 diabetic mothers
can also have babies that are small secondary to hypoxia from microvascular disease o f the
placenta. The big issue with being too big is complications during delivery (shoulder dystocia,
brachial plexus injury) and after delivery (neonatal hypoglycemia, meconium aspiration).

Erb’s Palsy:

Injury to the upper trunk o f the brachial plexus (C5-C6),


most commonly seen in shoulder dystocia (which kids with
macrosomia are at higher risk for).

If you see an aplastic or hypoplastic humeral head / glenoid


in a kid, you should immediately think about an Erbs Palsy.

Clinical Correlation Recommended.

- Afflniotic Fluid -
Early on, the fluid in the amnion and chorionic spaces is the result o f filtrate from the
membranes. After 16 weeks, the fluid is made by the fetus (urine). The balance o f too much
(polyhydramnios) and too little (oligohydramnios) is maintained by swallowing o f the urine
and renal function. In other words, if you have too little fluid you should think kidneys aren’t
working. If you have too much fluid you should think swallow or other GI problems. Having
said that, a common cause o f too much fluid is high maternal sugars (gestational diabetes).
Fine particulate in the fluid is normal, especially in the third trimester.

Amniotic Fluid Index.- Made by measuring the vertical height o f the deepest fluid
pocket in each quadrant o f the uterus, then summing the 4 measurements.

Normal is 5-20.

Oligohydramnios is defined as AFI < 5 cm.

Polyhydramnios is defined as AFI > 20 cm , or a single fluid pocket > 8 cm.

O ligohydram nios Oligohydramnios


Not Pee’n Enough is a frequent
-AFI < 5 -Bad Kidneys finding in growth-
restriction related
to placental
Polyhydram nios N ot Drinking insufficiency.
-AFI > 20 or Enough Pee
-Single Pocket > 8 cm -Bad GI

453
-Normal Development-
I’m going to briefly touch on w hat I think is testable trivia regarding normal development.

Brain: Choroid plexus is large and echogenic. There should be less than 3 mm o f
separation o f the choroid plexus from the medial wall o f the lateral ventricle (if more it’s
ventriculom egaly). The cistem a m agna should be betw een 2 m m -II m m (too small think
Chiari II, too large think Dandy Walker).

Face / Neck: The “fialcrum” o f the upper lip is normal, and should not be called a cleft lip.

Lungs: The lungs are norm ally hom ogeneously echogenic, and sim ilar in appearance to the
liver.

Heart: The only thing to know is that papillary muscle can calcify “Echogenic Foci in the
ventricle,” and although this is com mon and can mean nothing - it’s also associated w ith an
increased risk o f Downs (look hard for other things).

Abdominal: If you only see one artery adjacent to the bladder, you have yourself a two
vessel cord. Bowel should be less than 6mm in diameter. Bowel can be m oderately
echogenic in the 2"d and 3^^ trim ester but should never be more than bone. The adrenals
are huge in newborns, and are said to be 20x their relative adult size.

^ Tw o Vessel Cord - Gam esm anship ^

There are two main ways to show a two vessel cord. The first one is a single vessel
running lateral to the bladder down by the cord insertion. The second is to show the cord
in cross section w ith two vessels.

Cord in Cross Section


-Only Two Vessels

M ore on the 2 vessel cord in a few pages.

454
Classic Normal Pictures Welcome to your
nightmare, bitch! j
I H X T LOOK S C X K Y

Cystic Rhombencephalon:

The normal rhom bencephalon is present


as a cystic structure in the posterior fossa
around 6-8 weeks.

D o n ’t call it a Dandy-W alker


malformation, for sure that will be a
distractor.

Normal Cystic R hom bencephalon (6-8 weeks)

Physiologic Midgut Herniation:

The m idgut norm ally herniates into the um bilical cord around 9-11 weeks.

D o n ’t call it an omphalocele, for sure that will be a distractor.

Normal Midgut Herniation (9-11 weeks)

455
S E C T IO N 14:
P lacenta and C ord

h ttD ://www.wonnenshealthmag.com/mom/placenta-recipes
My opinion: Use two cups o f strawberries in the smoothie.

Normal: You can first start to see the placenta around 8 weeks (focal thickening along the
periphery o f the gestational sac). It should be shaped like a disc around 12 weeks. The
normal sonographic appearance is “granular” with a smooth cover (the chorion). Underneath
the basal surface there is a normal retroplacental com plex o f decidual and m yom etrial veins.

N orm al P lacen tal A ging: As the placenta ages it gets hypoechoic areas, septations, and
randomly distributed calcifications.

Venous Lakes: These are an incidental finding o f no significance. They look like focal
hypoechoic areas under the chorionic m em brane (or within the placenta). You can
sometimes see slow flow in them.

Variant Placental Morphology:

Increased risk o f type 2 vasa


Two near equal previa (vessel cross the internal
Bilobed sized lobes - os), post partum hem orrhage
Placenta connected by a
thin strip.
from retained placental tissue,
and velam entous insertion o f
the cord 0
One or more Increased risk o f type 2 vasa
Succenturiate
small accessory previa, post partum hem orrhage
Lobe
lobes from retained placental tissue

Rolled
Circumvallate placental edges High risk for placental
Placenta with smaller
1chorionic plate
abruption and lU G R

0
456
THIS vs THAT; Placental Thickness
Too Thin (< 1 cm) Too Thick (> 4cm)
Placental Insufficiency, Maternal Fetal Hydrops, M aternal DM, Severe
Hypertension, M aternal DM, Trisomy 13, M aternal Anemia, Congenital Fetal Cancer,
Trisomy 18, Toxemia o f Pregnancy Congenital Infection, Placental Abruption

Placental Abruption: — PAINFUL

This is a premature separation o f the placenta from the myometrium. The step 1 history was
always “mother doing cocaine,” but it also occurs in the setting o f hypertension. Technically,
subchorionic hemorrhage (marginal abruption) is in the category - as previously discussed.
Retroplacental Abruption is the really bad one. The hematoma will appear as anechoic or
mixed echogenicity beneath the placenta (often extending beneath the chorion).

Buzzword is “disruption o f the retroplacental complex.”

THIS vs THAT: Placental Abruption vs Myometrial Contraction /Fibroid

Placental Abruption will d is ru p t the i M yom etrial Contractions / Fibroids will


retroplacental com plex o f blood vessels I d isplace the retroplacental complex

Placenta Previa: — PAINLESS

This is a low implantation o f the placenta that covers part o f or all o f the internal cervical os. A
practical pearl is that you need to have an empty bladder when you look for this (full bladder
creates a false positive). Several subtypes - as seen in my awesome little chart below.

Buzzword is “painless vaginal bleeding in the third trimester.”

Low-Lying Marginal Complete Central


., ^ Extends to the edge of
Marein is within 2 cm oi ° , Covers the intemal os Centered over the
° the internal cervical os
the intemal cervical os intemal os
(but doesn t cover it)

457
Placenta Creta:
Most common (75%)
This is an abnormal insertion o f the placenta, which and mildest form. The
Placenta villi attach to the
invades the myometrium. The severity is graded with
Accreta myometrium, without
fancy sounding Latin names. The risk factors include
invading.
prior C-section, placenta previa, and advanced maternal
age. The sonographic appearance varies depending on Placenta Villi partially invade
the severity, but generally speaking you are looking for Increta the myometrium
a “moth-eaten” or “Swiss cheese” appearance o f the
The really bad one.
placenta, with vascular channels extending from the Villi penetrate through
placenta into the myometrium (with turbulent flow on the myometrium or
Placenta
Doppler). Thinning o f the myometrium (less than 1mm) beyond the serosa.
Percreta
is another sign. This can be serious business, with life Sometimes there is
invasion of the
threatening bleeding sometimes requiring bladder or bowel.
hysterectomy.

Risk factors are prior c-section. and placenta previa.


“Ad” =To “In” = Into “Per” = Through
Normal Accreta Increta Percreta

Endometrium

Myometrium

Serosa

Placenta Chorioangioma:

This is basically a hamartoma o f the placenta, and is the most


common benign tumor o f the placenta. These are usually well-
circumscribed hypoechoic masses near the cord insertion. Flow
within the mass pulsating at the fetal heart rate is diagnostic (they
are perfused by the fetal circulation). They almost always mean
nothing, but if they are large (> 4 cm) and multiple
(“choriangiomatosis”) they can sequester platelets, and cause a
high output failure (hydrops).

THIS vs THAT; Placental Chorioangiom a VS Placental Hem atom a

Chrorioangiom a has pulsating D oppler flow H em atom a does N OT

458
- Umbilical Cord -
Normal Cord: Should have 3 vessels (2 arteries, 1 vein).

Two Vessel Cord: This is a normal variant - seen in about 1% o f pregnancies. Usually the
left artery is the one missing. This tends to occur more in twin pregnancies and maternal
diabetes. There is an increased association with chromosomal anomalies and various fetal
malformations (so look closely). Having said that, in isolation it doesn’t mean much.

Velamentous Cord insertion:


This is the term for when the cord
inserts into the fetal membranes
outside the placental margin, and
then has to travel back through Velamentous
the membranes to the placenta Normal
(between the amnion and the
chorion). It’s more common
with twins, and increases the risk
of intra-uterine growth restriction
and growth discordance among
twins.

Marginal Cord Insertion:


This is basically almost a
velamentous insertion (cord is
within 2 cm o f the placental
margin). It’s also seen more in
twin pregnancies.

Vasa Previa: Fetal vessels that cross (or almost cross) the internal cervical os. It’s seen more
in twin pregnancies, and variant placental morphologies. There are two types:

•Type 1: Fetal vessels connect to a velamentous cord insertion within the main placental body

•Type 2: Fetal vessels connect to a bilobed placenta or succenturiate lobe.

Nuchal Cord: This is the term used to describe a cord wrapped around the neck o f the fetus.
Obviously this can cause problems during delivery.

Umbilical Cord Cyst: These are common (seen about 3% o f the time) and are usually
single (but can be multiple). As a point o f completely irrelevant trivia, you can divide these into
false and true cysts. True cysts are less common, but have fancy names so they are more likely to
be tested. Just know that the omphalomesenteric duct cyst is usually peripheral, and the allantoic
cyst is usually central. If the cysts persist into the 2"^ or 3''^ trimester then they might be
associated with trisomy 18 and 13. You should look close for other problems.

459
S E C T IO N 15:
C o n g en it a l F etal

- DOW NS-
Ultrasound Findings Concerning for Down Syndrome
More than h alf o f fetuses (or feti, if you prefer) with Downs
Congenital Heart Disease have congenital heart issues, - m ost com monly AV canal and
VSD

Most com m on intra abdom inal pathology associated with


Duodenal Atresia
Downs (hard to see before 22 weeks)

Short Fem ur Length N ot Specific


Not Specific (can be seen with obstruction, infection, CF,
Echogenic Bowel
ischemia, and lots o f other stuff)
N ot Specific, and actually seen more with Trisomy 18. It
Choroid Plexus Cyst should prom pt a close survey for other findings (normal if in
isolation)
Nuchal Translucency Translucency > 3mm in the first trimester.
Thickness > 6mm in the second trimester- nonspecific and
Nuchal Fold Thickness
can also be seen with Turners
Echogenic Focus in
N ot Specific, but increased risk o f Downs x 4
Cardiac Ventricle

Nuchal Lucency:

Measured between 9-12 weeks, this anechoic area


between the neck/ occiput and the skin should be
less than 3 mm.

Measurements > 3 mm are associated with Downs


(trisomy 21) or other chromosomal abnormalities.

Positioning o f the neck is critical to avoid false


positives. The ideal positioning is a neutral neck,
with the nasal bone visualized, and the head in the
mid-sagittal position. A well delineated skin edge.

Maternal blood sample also analyzed for free Beta


hCG and pregnancy associated plasma protein-
A (PAPP-A).

460
- OTHER HORRIBLE SHIT THAT CAN HAPPEN -
Amniotic Band Syndrome:

The fetus needs to stay in the amniotic cavity, and stay the hell out of
the chorionic cavity. If the amnion gets disrupted and the fetus wanders
/ floats into the chorionic cavity he/she can get caught in the sticky
fibrous septa. All kinds of terrible can result ranging from decapitation,
to arm/leg amputation.

Losing fingers like this is terrible - it’s much better to cut them off in a
more manly way. For example, drunken chainsaw lumberjack work or
trying to do that thing with the knife that the cyborg did in the movie
Aliens (youtube “Aliens: Bishop’s Knife Trick”). Not to menfion,
unless you are Jean Jacque Machado (youtube “Heart of the champion
documentary” ) your chances of becoming a world champion in Jiu
Jitsu are going to be significantly decreased.
-Am putated Fin gers
This is most likely to be shown in one of two ways:

(1) X-ray of a hand or baby gram showing fingers amputated or a hand/arm amputated - with the
remaining exam normal, or

(2) Fetal ultrasound with the bands entangling the arms or legs of a fetus.

Hydrops:
Fetal hydrops is bad news. This can be from immune or non-immune causes. The most common
cause is probably Rh sensitization from prior pregnancy. Some other causes include; TORCHS,
Turners, Twin Related Stuff, and Alpha Thalassemia. Ultrasound diagnosis is made by the
presence o f two o f the following: pleural effusion, ascites, pericardial effusion, and
Subcutaneous Edema. A sneaky trick is to instead show you a thickened placenta (> 4-5cm)
“placentom egaly” - they call it, although I think it’s much more likely to show a pleural effusion
and pericardial effusion.

Hydrops - Body Wall Edema, Pleural Effusion, Ascites

MCA Doppler is useful with an increase in peak systolic flow often occurring before hydrops.

461
-Chiari II /Open Neural Tube Defect

I think at least some general idea o f the mechanism for this pathology is helpful for
understanding the ultrasound findings. There are a bunch o f proposed mechanisms, and o f
course they all think they are right. I don’t give a shit which one is the “real m echanism ,” I
just picked the one that helps me understand the findings.

So this is the one I like: You have a hole in your back from a neural tube defect (Step 1 trivia
= not enough folate). The hole in your back (“m yelom eningocele”) lets CSF drip out. So
you end up with a low volum e o f CSF. The CSF volum e needs to be at a certain pressure to
distend the ventricular system. If it’s under distended then the hindbrain structures drop into
a caudal position. This caudal herniation o f the cerebellar vermis, brainstem, and 4th
ventricle is the hallm ark o f Chiari II.

This caudal herniation o f the cerebellum into the foramen m agnum obliterate the normal
contour o f the vermis, creating the contour o f a banana.

If you can think about a normal pressure in the developing ventricular system being
necessary for the brain to stretch into a normal shape, then it isn ’t a far stretch to think about
this normal pressure being needed to shape the skull correctly, too. The low pressure and
abnormal distention o f the developing brain results in incomplete stretching o f the rostral
(front part) skull. The result is a “lemon shaped” rostral skull. The key point (testable) is
that this lemon shape goes away in the 3rd trimester. So it’s only present in the 2nd trimester.
The way I rem em ber this is that the problem was from a lack o f volume. Once the brain
grows big enough (even if there isn ’t enough CSF distention) it still gets big enough to put a
normal curve on that rostral skull. So they “grow” out o f it.

Testable Trivia:
• Both banana and lemon signs are classic for the Chiari II / Spina Bifida Path
• The banana sign is present in both 2nd and 3rd trimesters
• The lemon sign is only in the 2nd trim ester (you grow out o f it).
• The banana sign is more sensitive and specific
• The lemon sign is less sensitive and specific; it can also be seen in Dandy Walker, Absent
Corpus Callosum, Encephaloceles, e tc ... Having said that if you see it on the test it’s
Chiari 2 + Open NTD.
• Hydrocephalus is also seen with Chiari II + Open N TD - but only later in gestation, and
only when it’s severe.

462
Open Neural Tube Defect

Low CSF Pressure During Development

Chiari II
t

Banana Lemon
-Loss of the normal bilobed -Flat /Concave
shaped of the cerebellum Frontal Bones

This is what it’s


supposed to
look like

Normal Bilobed Normal Shape


Shaped of the Cavum Septum of the Frontal
Pellucidum „
Cerebellum Bone

463
Ventriculomegaly - There are multiple causes including hydrocephalus (both
communicating and non-com m unicating), and cerebral atrophy. Obviously this is bad, and
frequently associated with anomalies.
Things to know:
• Aqueductal Stenosis is the m ost com m on cause o f non-com m unicating hydrocephalus
in a neonate
• Ventricular atrium diam eter > 10 mm = too big
• “Dangling choroid” hanging o ff the wall more than 3 m m = too big

Ventriculomegaly - Shows dangling choroid

Choroid Plexus Cyst -

This is one o f those incidental findings


that in isolation means nothing. Having
said that, the incidence o f this finding is
increased in trisom y 18. trisom y 21,
Turner’s Syndrome, and K linefelter
Syndrome.

Choroid Plexus Cyst

Facial Clefts - This is the m ost com m on fetal facial anomaly. A bout 30% o f the time
you are dealing with chrom osom e anom alies. A round 80% o f babies w ith cleft lips have
cerebral palsy. You can see cleft lips, but cleft palate (in isolation) is very hard to see.

Cystic Hygroma - If they show you a com plex cystic mass in the posterior neck, in
the antenatal period, this is the answer. The follow-up is the association with Turners and
Downs.

464
Anencephaly - This is the most com m on neural tube defect. You have total absence of
the cranial vault and brain above the level o f the orbits. O bviously this is not compatible
with life.

Anencephaly - No brain / vault above the orbits

Congenital Diaphragmatic Hernia - A bdom inal contents push into the chest.
Nearly all are on the left (85%). The things to know is that it (1) causes a high mortality
because o f its association with pulm onary hypoplasia, and (2) that all the kids are
malrotated (it messes with normal gut rotation). I f they show this it will either be (a) a
newborn chest x-ray, or (b) a 3rd trim ester MRI.

Echogenic intracardiac Focus (EIF) - This is a calcification seen in a papillary


muscle (usually in the left ventricle). You see them all the time, they don’t m ean that much
but are seen at a higher rate Trisom y 21 (12%) and Trisom y 13. So you are supposed to look
for more features.

H ighest Yield Trivia:

• It occurs in the
normal general
population - around
5%,

• It occurs more in
Downs patients -
around 12%.

Echogenic Intracardiac Focus (EIF)

Abnormal Heart Rate: Tachycardia is defined as a rate > 180 bpm.


Bradycardia is defined as a rate < 100 bpm.

465
Double Bubble:

Not the good kind o f bubble you get from


eating ice cream and doing squats (or buying
those stretchy pants).
This is the bad bubble - described in detail in
the Peds Chapter.
Just realize this can be shown with antenatal
ultrasound or MRI. It’s still duodenal atresia.
Double Bubble - Duodenal Atresia

Echogenic Bowel: This can be a norm al variant but can also be associated with
significant badness. N orm ally bowel is isoechoic to the liver. I f it’s equal to the iliac crest bone
then it’s too bright. The DDx includes CF, Downs and other Trisomies, Viral Infections, and
Bowel Atresia.

Sacrococcygeal Teratoma: This is the m ost com m on tum or o f the fetus or infant.
These solid or cystic masses are typically large and found either on prenatal imaging or birth. They
can cause mass effect on the GI system, hip dislocation, nerve com pression causing
incontinence, and high output cardiac failure. Additionally, they may cause issues with
premature delivery, dystocia, and hem orrhage o f the tumor. They are usually benign (80%).
Those presenting in older infants tend to have a higher m alignant potential. The location o f the
mass is either external to the pelvis (47% ), internal to the pelvis (9%), or dum bell’d both
inside and outside (34%).

Autosomal Recessive Polycystic Kidney Disease - The classic look is


m assively enlarged bilateral kidneys with oligohydram nios. Additional details in the Peds
chapter.

Posterior Urethral Valves: The classic look is bilateral hydro on either fetal US or 3rd
Trimester MRI.

Short Femur: A short femur (below the percentile) can make you think o f a skeletal
dysplasia.

466
S E C T IO N 16:
Ma t e r n a l D iso r d e r s

Incompetent Cervix: When shortened, the cervix is associated with a high


T hings T h a t
risk of premature delivery. You call it “short” when the canal is < 2.5 cm in length.
G row D u rin g
Hydronephrosis: occurs in 80% of pregnancies (mechanical compression of P regnancy:
the ureters is likely the cause). It tends to affect the right more than the left
(dextrorotation of the pregnant uterus). • Babies
• Splenic Artery
Fibroids: Fibroids tend to grow in early pregnancy secondary to elevated Aneurysms
estrogen. Progesterone will have the opposite effect, inhibiting growth, in later • Renal AMLs
pregnancy. Stretching of the uterus may affect the arterial blood supply and • Fibroids
promote infarcts and cystic degeneration.
Uterine Rupture: You see this most commonly in the 3'‘‘>trimester at the site of prior c-section.
Other risk factors worth knowing are the unicomuate uterus, prior uterine curettage, “trapped
uterus” (persistent retroflexion from adhesions), and interstitial implantation.
HELLP Syndrome: Hemolysis, Elevated Liver Enzymes, Low Platelets. This is the most severe
form of pre-eclampsia, and favors young primigravid women in their trimester. It’s bad news and
20-40% end up with DIC. If they are going to show this, it will be as a subcapsular hepatic hematoma in
pregnant (or recently pregnant) women.
Peripartum Cardiomyopathy: This is a dilated cardiomyopathy that is seen in the last month of
pregnancy to 5 months postpartum. The cardiac MRI findings include a global depressed fijnction, and
non-vascular territory subepicardial late Gd enhancement - corresponding to cellular lymphocytic
infiltration.
Sheehan Syndrome: This is pituitary apoplexy seen in postpartum female who suffer from large
volume hemorrhage (causing acute hypotension). The pituitary grows during pregnancy, and if you
have an acute hypotensive episode you can stroke it out (it bleeds). The look on MR is variable
depending on the time period, acute it will probably be T1 bright (if they show a picture). Ring
enhancement around an empty sella is a late look.
Ovarian Vein Thrombophlebitis: This can be a cause of postpartum fever. Risk factors
include C-section and endometritis. The right side is affected five times more often than the left. They
could show you an enlarged ovary and a thrombosed adjacent ovarian vein.

Retained Products of Conception: The typical clinical s t o r y ______________


is continued bleeding after delivery (or induced abortion). The most
common appearance is an echogenic mass within the uterine cavity. The
presence or absence of flow is variable, you can have lots or none. A
sneaky way to show this is irregular thickening of the endometrium
(>10mm) with some reflective structures and shadowing - representing
the fetal parts. You can also think about RPOC when the endometrial
thickness is > 5 mm following dilation and curettage. Testable
associations include: medical termination of pregnancy (abortion), 2nd
trimester miscarriage, and placenta acereta.
Endometritis - Broadly speaking, it is an inflammation or infection of the endometrium. The history
will be (if you are given one) fever, and uterine tenderness and recent c-section (or prolonged labor). On
ultrasound you will see a thickened, heterogenous endometrium, with or without fluid / air.

467
S E C T IO N 17:
MULTIPLE G esta tio n s

Placentation Terminology: So you can have monozygotic twins (identical) or dizygotic


twins (not-identical). The dizygotics are always dichorionic and diamniotic. The placenta of the
monozygotics is more variable and depends on the timing of fertilized ovum splitting (before 8 days =
diamniotic, after 8 days = monoamniotic). As a point of trivia, a late splitting (after 13 days) can
cause a conjoined twin. As a general rule, the later the split the worse things do (monoamniotics have
more bad outcomes - they get all tangled up, and the conjoined ones have even more problems).

M onochorionic IVionochorionic D ichorionic D ichorionic


M on o am n io tic D iam niotic D iam n io tic D iam niotic
Fu sed P la c e n ta S e p a ra te P lac en tas

THIS vs THAT: Monochorionic VS Dichorionic

Membrane Thickness: To differentiate the different types, some people use a method classifying
thin and thick membranes. Thick = “easy to see” 1-2 mm. Thin = “hard to see.” Thick is supposed
to be 4 layers (dichorionic). Thin is supposed to be 2 layers (monochorionic). Obviously this
method is very subjective.

Twin-Peak Sign: A beak-like tongue between the two membranes of dichorionic diamniotic
fetuses. This excludes a monochorionic pregnancy.

T Sign: Think about this as basically the absence of the twin peak sign. You don’t see chorion
between membrane layers. T sign = monochorionic pregnancy.

468
Twin Growth - You can use normal growth charts in the first and second trimester (but not the third).
The femur length tends to work best for twin age in later pregnancy. More than 15% difference in fetal
weight or abdominal circumference between twins is considered significant.
Chronic Feto-Fetal Transfusion
Monochorionic Twins share a
placenta. This placenta (if it has “Fast flow ” ^ ^ “Slow flow"
abnormal vascular connections) can TTTS/ TRAP TAPS
allow for blood to be transferred from -Characterized -No TOPS-
one fetus to the other - this is bad. by TOPS- -Hgb Differences-

Twin- Twin Transfusion Syndrome (TTTS) - This occurs in monochorionic twins when a
vascular communication exists in the placenta. You end up with one greedy fat twin who takes all the
blood and nutrients, and one skinny wimpy looking kid who gets the scraps. It is best to think about this
pathology like an episode of “My 600 pound life” (that reality show about big fat people — where the
skinny boyfriend feeds the gigantic fat girlfriend). The somewhat counter intuitive part is that the skinny
kid actually does better, and the fat one usually gets hydrops and dies.

You are going to have unequal fluid in the amniotic sacs, with the TOPS Twin Oligohydramnios
donor (skinny) twin having severe oligohydramnios and is Polyhydramnios Sequence
sometimes (*buzzword) “stuck to the wall of the uterus,” or
“shrink wrapped.” The fat twin floats freely in his Oligohydramnios for the donor
polyhydramniotic sac. This imbalance is referred to as “TOPS”. Polyhydramnios for the recipient

The donor (skinny) twin will also have a high resistance umbilical artery.

Twin Reversed Arterial Perfusion Syndrome (TRAP) - This is a severe variant of TTTS.
Similar to TTTS you have a greedy twin (which is described in OB literature as the “parasite twin”) and
an enabling “pump” twin — which I call “the skinny boyfriend twin.” In this more extreme situation the
greedy twin is so greedy that he/she doesn’t even bother to develop a heart. Instead, it just uses the heart
of the “pump twin.” Although you might think not having a heart would give you an advantage in life
(especially in business, politics, or various board games - like monopoly) these kids don’t do well. They
usually have wrecked / totally deformed upper bodies - so they probably wouldn’t make it very far in
politics and would have trouble rolling the dice in monopoly (since they don’t have arms) ... plus they all
die in utero.
The “pump twin” is usually normal, and does ok as long as the strain on his/her heart isn’t too much.
Although if the acardiac twin is a really big set of legs (> 70% estimated fetal weight of the co-twin) then
the strain can kill the pump twin too. They could show this as a Doppler ultrasound demonstrating
umbilical artery flow toward the acardiac twin, or umbilical vein flow away from the fucked up acardiac
twin ... who is basically a set of legs.

Twin Anemia Polycythemia Sequence (TAPS) - This is another syndrome related to feto-
fetal transfusion but this time there is no difference in amniotic fluid. Instead you essentially have a
transfer of hemoglobin (Hgb) with one anemic twin and one twin with polycythemia.
Classic Scenario: TAPS occurs after incomplete laser treatment for TTTS
Classic Finding: Discordant MCA Doppler with elevated peak systolic velocitv in the anemic twin

One Dead Twin - At any point during the pregnancy one of the twins can die. It’s a bigger problem
(for the surviving twin) if it occurs later in the pregnancy. “Fetus Papyraceous” is a fancy sounding Latin
word for a pressed flat dead fetus.
“Twin-Embolization Syndrome” is when you have embolized, necrotic, dead baby being
transferred to the living fetus (soylent green is people!). This can result in DIC, tissue ischemia, and
infarct. By the way, a testable point is that this transfer can only occur in a monochorionic pregnancy.

469
fROMETHEUS
Liom hart, M.1>.

470
I'm Here For M y
Thyroid Ultrasound

471
S E C T IO N 1:
A d r e n a l

Anatomy: The adrenal glands are paired retroperitoneal glands that sit on each kidney. The right
gland is triangular in shape, and the left gland tends to be more crescent shaped. If the kidney is
congenitally absent the glands will be more flat, straight, discoid, or “pancake ” in appearance. Each
gland gets arterial blood from three arteries (superior from the inferior phrenic, middle from the aorta,
and inferior from the renal artery). The venous drainage is via just one main vein (on the right into the
IVC, on the left into the left renal vein).

Step 1 Trivia:
Cortex There are 4 zones to the
Zona Glomerulosa adrenal, each of which
Medulla makes different stuff.
• Zona Glomerulosa:
Makes Aldosterone -
Zona Fasciculata prolonged stimulation
here leads to hypertrophy.
• Zona Fasciculata:

mm Zona Reticularis
Makes Cortisol
• Zona Reticularis:
Makes Androgens
Medulla
• Medulla: Makes
Catecholamines

Age related trivia: The relative size of the adrenal changes as you age.
• Second Trimester: Adrenal is Half the Size of the Kidney
• Third Trimester: Adrenal is 1/3 the Size of the Kidney
• Adulthood: Adrenal is 1/13 the Size of the Kidney

Normal Ultrasound:
In babies, the cortex is hypoechoic, and
the medulla is hyperechoic. This gives the
adrenal a triple stripe appearance (dark
cortex, bright medulla, dark cortex).

Normal Adrenal - Hypoechoic Cortex, Hyperechoic Medulla, Hypoechoic Cortex


- like an Oreo, with a cream filling.

All
Adrenal Ultrasound Gases - eamesmanshlp
If you see a Pediatric Adrenal Cases you should think about: (a) normal,
(b) congenital, (c) neuroblastoma, (d) hemorrhage, and (e) hyperplasia.

Normal : As discussed on the prior page, will have the “Y” shape and a
normal triple stripe appearance on Ultrasound.

Congenital : There are three classic congenital cases:

• Pancake (Discoid) - 1 discussed this in the renal chapter (page 357). If the
kidney is absent the “Y” shape doesn’t form. An elongated flat (pancake)
adrenal is seen with congenital absence of the kidney.

Horseshoe - Just like you can get a horseshoe kidney, you can get a horseshoe
adrenal. This occurs when the limbs of the adrenal glands fuse in the midline,
and is associated with asplenia (right isomerism). Remember, they can show
you bilateral trilobed lungs, a horizontal midline liver, an absent spleen,
malrotation, and congenital heart disease (total anomalous pulmonary venous
return - most commonly) — all in association with this horseshoe adrenal
gland. In fact, I would guess that is the most likely way this would be asked.
Example, show you a bilateral trilobed lung and ask what the adrenal look like.

Adrenal Cyst: I’m not sure this is truly congenital - maybe “developmental”
is a better term. Just like a kidney can get cysts, adrenals can also. They are
almost always benign. Having said that, in a newborn - they usually get
resected because you can’t tell them apart from cystic neuroblastoma. If there
has been a history of hematoma, sometimes they watch them to see if they
will resolve. In an adult, incidentally found, obviously they aren’t a
neuroblastoma - those are ok (probably).

Neuroblastoma: •#

I talk about this a ton in the peds chapter. To rehash the important
parts, they form in the adrenal medulla (usually), and typically look
like an enlarged gland with a hyperechoic component. Having said
that they can have cystic components and look like hemorrhage.
“Complex cystic mass” is a good way to describe these. For the
purpose of multiple choice I’d go with hyperechoic.

473
Hemorrhage:

This occurs m ost com monly in the setting o f traum a or stress


(neonates). W hat this typically looks like on ultrasound is an
enlarged gland with an anechoic component. W ith time, the
clot changes and it can be more and more anechoic. So
basically, it can look like anything but for the purpose o f
multiple choice I ’d go with anechoic.

Second order pa th type trivia/knowledge:

S tress: It’s classically seen after a breech birth, but can also be seen with fetal distress, and
congenital syphilis. Imaging features change based on the timing o f hemorrhage.
Calcification is often the end result (that could be shown on CT or MR). It should be
avascular. This can occur bilaterally, but favors the right side (75%).

Classic N ext Step: Serial ultrasounds (or MRI) can differentiate it from a cystic
neuroblastoma. The hem orrhage will get sm aller (cancer will not).

So which is it? Serial ultrasound or M RI? - If forced to pick you w ant serial ultrasounds. It’s
cheaper and doesn’t require sedation.

Trauma: This is going to be an adult (in the setting o f trauma). M ost likely it will be shown
on CT. It’s more com mon on the right.

W aterhouse-Friderichsen S yn d ro m e - Hem orrhage o f the adrenal in the setting o f


M fulm inant meningitis (from N eisseria M eningitidis).

Hyperplasia:

W hat this typically looks like on ultrasound is a “big adrenal”


that “looks like a brain.” So what does “b ig ” mean ? M ost
sources will say longer than 20mm, and a limb that is thicker
than 4 m m (although this is debated - and will likely not be
asked). For the purpose o f multiple choice I would say that if
they stick calibers on it. then it is too big.

So w hat does “looks like a brain ” m ean ? That means the


surface is wrinkled, like it has gyri and sulci.
Cerebriform Pattern

474
Adrenal Hyperplasia Trivia:
Bilateral Solid
21-Hydroxylase Deficiency: Congenital adrenal hypertrophy is Testicular Masses
caused by 21-hydroxylase deficiency in > 90% of cases. It will manifest
clinically as either genital ambiguity (girls) or some salt losing pathology
(boys). The salt losing can actually be life threatening. The look on Congenital Adrenal
imaging is adrenal limb width greater than 4mm. In some cases you lose Hyperplasia
the central hyperechoic stripe (the whole thing looks like cortex).

• I say “Genital ambiguity”, Adrenal Rests


you say 21-Hydroxylase Deficiency

Too Much Cortisol from overproduction of ACTH - which results in bilateral adrenal gland
hyperplasia. If someone wanted to be a real asshole they could get into the weeds with vocabulary.
For example, the “Disease” vs “Syndrome” THIS vs THAT:

• Cushing Disease: This is an overproduction of ACTH by a pituitary adenoma, resulting in


too much cortisol. This is actually the most common cause of excess cortisol (75%).

• Cushing Syndrome: The “syndrome” is basically a variety of causes resulting in common


symptoms. So you can have overproduction of ACTH by an ACTH secreting tumor (classic step
1 example is the small cell lung tumor), or overproduction of ACTH via an adrenal adenoma
(these cases will not have hyperplasia), or you can have straight up primary adrenal hyperplasia.
You could even get the “syndrome” by taking chronic high dose steroids. Any way you end up
with a fat moon face and big gross lines all over you belly counts as “syndrome.”

Adrenal M ass Mimic:

Infradiaphragmatic extralobar sequestration is the classic mimic of an adrenal mass on imaging.


The ultrasound will show a heterogeneous suprarenal mass. It is really hard to tell this from an adrenal
mass without any “hints.” Examples of hints:

Classic extra lobar sequestration history “male neonatal with respiratory distress and cyanosis. ”

CT picture clearly showing the blood supply from


the aorta (or branches of the aorta):

475
■Summary / Rapid Review—

Normal:
- Triple Stripe
- Hypoechoic Cortex,
■ Hyperechoic M edulla,
" H ypoechoic Cortex
- Smooth Surface

Hyperplasia:
- Big (longer than 20mm)
■ Looks like a brain (wrinkled surface)
- Can som etim es lose the central bright layer

- “Genital am biguity”, = 21-OH Deficiency

Hemorrhage:
- Big w ith an anechoic (or echogenic) com ponent
■ Gets sm aller over time

- Seen with “stress” or traum a

Neuroblastoma:
- Big w ith an echogenic (or anechoic) com ponent
- Does N O T gets sm aller over time

476
Adrenal Adenoma:
These things are easily the most com m on tum or in the adrenal gland. Up to 8% o f people
have them. Proving it is an adenom a is an annoying (testable) problem.

* Non-Contrast; Less than 10 HU

* Contrast: Two options:

Absolute Washout

Enhanced CT - Delayed CT
X 100 Greater than 60% = Adenoma

Enhanced CT - Unenhanced CT

Relative Washout

Enhanced CT - Delayed CT x 100 Greater than 40% = Adenoma

Enhanced CT

H ypervascular mets (usually renal, less likely HCC) can mimic adenom a washout.
Portal venous HU values > 120 should m ake you think about a met.

Along those lines Pheochrom ocytom as can also exhibit washout. The trick is the same,
if you are getting HU measurements > 120 on arterial or portal venous phase you can
NOT call the thing an adenom a.

MRI; Look for drop out on In and O ut o f phase T l.

Adrenal Adenoma -Signal drop out In and Out o f Phase

477
Adrenal Adenoma Continued
Real Life = Mass in Adrenal = Adenom a

M ultiple Choice = M ass in the Adrenal = Possible Fuckery

Although m ost adenom as are not functional, C ushing’s (too much cortisol) and C onn’s (too
much aldosterone) can present as functional adenomas.

Tips / Tricks:

Adenoma are usually homogeneous. I f they are showing you hem orrhage (in the absence o f
trauma), calcifications, or necrosis you should start thinking about other things.
Adenomas are usually small (less than 3 cm). The bigger the mass, the more likely it is to
be a cancer. H ow big? M ost people will say more than 4 cm = 70% chance cancer, and
more than 6 cm = 85% chance cancer. The exceptions are bulk fat (myelolipomas) or
biochemical catecholamines in the question stem (pheo) - those can be big.
Bilateral Small = Probably adenom a
Bilateral Large = Pheo or M et (Lung cancer)
Portal Venous Phase HU > 120 = Probably a met (RCC, HCC) or pheo.

“Collision Tumors” - Two different tum ors that smash together to look like one mass.
Usually one o f them is an adenoma. Rem em ber adenom a should be hom ogenous and small.
If you see heterogenous m orphology consider that you could have two tumors. FDG PET
and M RI can both usually tell if the tum or is actually a collision o f two different tum ors -
those w ould be the appropriate next steps.

Conn’s Syndrome - Syndrome o f excessive aldosterone production. This is most


com m only caused by a benign adenom a (70%). C ortical-carcinom a can also do it, but that
is much more rare and usually accom panied by hypercortisolism.

478
Pheochromocytoma
Uncommon in real life (common on multiple choice tests). They are usually large at presentation
(larger than 3 cm). The look is variable (heterogenous, homogenous, cystic areas, calcifications,
sometimes even fat). Having said that, the most classic look is a heterogeneous mass with AVID
ENHANCEMENT. On MRI they are T2 bright. Both MIBG and Octreotide could be used (but
MIBG is better since Octreotide also uptakes in the kidney).

Gamesmanship: The hyper


enhancement washout trick. They
could show you what looks like an
adenoma study (multiple phases to
calculate washout) - but with
mega enhancement (HU > 120).
Remember, I don't care what the
flick washout out numbers you get
- if they show you HU
measurements > 120 on arterial or
portal venous phase you can NOT
call the thing an adenoma.

Gamesmanship: This thing isn’t


always in the adrenals. They can Pheo at the O rg a n of Z u c k e rk a n d l - 1 2 Bright
be extra adrenal (organ of
Zuckerkandl - usually at the IMA).

“Rule of 10s”

10% are extra adrenal (organ o f Zuckerkandl - usually at the IM A), 10% are bilateral, 10%
are in children, 10% are hereditary, 10% are N O T active (no HTN).

“Syndromes” MEN 2 (subtypes 2a and 2b)

Both subtypes of MEN 2 are associated with


Associated syndromes: First think Von H ippel
Pheochromocytomas (50% of patients).
Lindau, then think M EN Ila and Ilb . O ther
In the case of MEN 2 they usually occur
things less likely to be tested include NF-1, multiple and bilateral.
Sturge Weber, and I S .
MEN 2 + Adrenal Mass = Pheo

•“Carney Triad”

•Extra-Adrenal Pheo, GIST, and Pulm onary Chondrom a (hamartoma).


% m S? «> •Don't confuse this with the Carney Complex
%V u -V
(Cardiac Myxoma, a n d Skin Pigmentation).

479
other Misc Adrenal Masses:
Myelolipoma

• Benign tumor that contains bulk fat.


• About Va have calcifications.
• If they are big (> 4 cm) they can bleed, and present with a retroperitoneal hemorrhage.
• Associated with endocrine disorders (Cushings, Congenital Adrenal Hyperplasia, Conns). Don’t get it
twisted, these tumors are NOT functional, they just happen to have associated disorders about 5-10% of
the time.

Gamesmanship:

The fat within these


tumors can cause a
speed displacement
artifact on ultrasound
and make the
diaphragm appear
discontinuous.

Sound travels slower discontinuous


in fat relative to soft diaphragm artifact
tissue. This confuses
the ultrasound machine
and results in the
discontinuous
diaphragm.

Adrenal Cyst: Calcifications: Cortical Carcinoma:


This is often the result of prior
You can get cysts in your • LARGE (4 cm -10 cm),
trauma or infection (TB).
adrenal. They are often
Certain tumors (cortical • May be fimctional (Cushings),
unilateral, and can be any
carcinoma, neuroblastoma, • Calcify in about 20% of cases.
size. The really big ones
myelolipoma) can have • Bad news & met everywhere
can bleed. They have a
calcifications. Melanoma (direct invasion often first).
thin wall, and do NOT
mets are known to calcify. • As a pearl, an adrenal carcinoma
enhance.
is not likely to be less than 5 cm
and often has central necrosis.
Mets:
Wolman Disease:
Think breast, lung, and This is a total Aunt Minnie
melanoma. They have no (and massive zebra / unicorn).
specific imaging findings Bilateral enlarged calcified
and look like lipid poor adrenals. It’s a fat
adenomas. If the dude metabolism error thing that
has a known primary kills (“booka” - drive by)
(especially lung, breast, or before the first year of life.
melanoma), and it’s not an
adenoma then it’s
probably a met.

480
S E C T IO N 2:
Syndrom es

MEN: “Multiple Endocrine Neoplasia”

There are three o f these stupid things, a n d people who write m ultiple choice tests love to
ask questions about them.

MEN 1: MEN Mnemonics


• Parathyroid Hyperplasia (90%),
• Pituitary Adenoma, M EN I (3 Ps)
• Pancreatic Tumor (Gastrinoma most commonly) - Pituitary, Parathyroid, Pancreas

MEN 2: M EN Ila (lM ,2P s)


• Medullary Thyroid Cancer (100%) M edullary Thyroid Ca,
• Parathyroid hyperplasia,
Pheochrom ocytom a, Parathyroid
• Pheochromocytoma (33%)
M EN Ilb (2Ms, IP )
MEN 2b:
• Medullary Thyroid Cancer (80%) - M edullary Thyroid Ca,
• Pheochromocytoma (50%) M arfanoid Habitus /m ucosal neuroma,
• Mucosal Neuroma, Pheochrom ocytom a
• Marfanoid Body Habitus

A Methodology to Remember MEN Syndromes

Each syndrome has 3 things:

M EN 1 = 1 Letter o f the alphabet w hich all three have in com m on is “P ” so there are 3
P ’s: Pi Para Pane (pituitary, parathyroid, and pancreas)

M EN 2 A = The A’s : Adrenal (pheochrom ocytom a). A ggressive thyroid CA


(medullary) and A lot o f Calcium (parathyroid)

M EN 2 B: it is a w annaBe (it wants to be like A). So, it is the same (adrenal pheo,
m edullary CA) BUT it has a b e e f w ith calcium, so instead o f parathyroid, it decided to
B a G angster (gangster for Ganglioneuroma).

481
Carcinoid Syndrome:

Flushing, diarrhea, pain, right heart failure from serotonin m anufactured by the carcinoid
tumor. The syndrome does not occur until the lesion mets to the liver (normally the liver
metabolizes the serotonin). The typical prim ary location for the carcinoid tum or is the GI
tract (70%). The m ost com m on prim ary location is the distal ileum (older literature says
appendix). The actual syndrome only occurs in 10% o f cases - and is actually very rare (in
real life - not on tests).

Carcinoid -Classic Mesenteric Involvement

Trivia: GI carcinoids are associated with other GI tum ors (GI adenocarcinoma).

Trivia: Urine Test for Carcinoid = 5-HIAA (5-hydroxyindoleacetic acid)

Trivia: Nukes Test o f Choice: "*In-Octreotide (Octreoscan)

Trivia: M IBG is also positive - but less than 25% o f the time (like 15%). G allium is positive,
but super non-specific.

Trivia: Systemic serotonin degrades the heart valves (right sided), and classically causes
tricuspid regurgitation

482
Hereditary Syndromes

Pheochromocytoma
. , Endolymphatic Sac Tumor IS (risk ~ 1 0% ). Less
Hemangiob astoma ,
. ^ ^ common (-1 0 % ). likely to be
IS the most
associated with
common tumor
Bilateral clear cell RCC catecholamine
VHL (seen in the
(-risk - 7 0 % ) production.
retina,
cerebellum, spinal
Papillary Cystadenoma of the Pancreatic Cysts
cord)
epididymis (-5 5 % ) (-7 5 % ) and serous
cystadeomas

Multiple head and


PGL Syndrome
neck Pheochromocytoma risk is variable depending on the
(Paraganglioma &
paragangliomas subtype
Pheochromocytoma)
(risk ~ 70% )

Cortical Tubers, Subependymal Renal AM Ls (risk


Nodules, -8 0 % )
Clinical Triad
(Facial
Tuberous Sclerosis Subependymal Giant Cell Cardiac
Angiofibroma,
(Bourneville Disease) Astrocytoma (SEGA) - found at Rhabdomyoma
Seizures, Retard
the caudothalamic groove
Brain)
adjacent to the foramen of Pulmonary Cysts
monro LA.M

Parathyroid Gastrinoma is the


Pituitary Primary Hyperparathyroidism is most common
Pancreas the most common (risk - pancreas /
100% ), related to adenomas duodenal tumor —
MEN 1
Less commonly jejunal ulcers and
carcinoids of the Prolactinoma is the most gastric fold
bronchus, and com mon pituitary tumor thickening (Zollinger
bowel Ellison)

Pheochromocytoma (-5 0 % )
Medullary Thyroid
MEN 2 A and often bilateral
Cancer
Primary Hyperparathyroidism Thyroid cancers
occur at younger
ages and are
Marfanoid Appearance
Medullary Thyroid multicentric
Mucosal Neuromas
Cancer
MEN2B Intestinal Ganglioneuromas
Elevated levels
These patients fart alot
Pheochromocytoma of calcitonin cause
(seriously)
flushing and
diarrhea similar to
Can be
carcinoid syndrome
Familial Medullary considered a
Thyroid (FMTC) subtype of MEN
2.

483
S E C T IO N 3:
T h y r o id

Anatomy: The thyroid gland is a butterfly shaped gland, with two lobes connected by an
isthmus. The thyroid descends from the foramen cecum at the anterior midline base o f the
tongue along the thyroglossal duct. The posterior nodular extension o f the thyroid (Zuckerkandl
tubercle) helps with locating the recurrent laryngeal nerve (which is medial to it).

Thyroid Nodules: Usually evaluated with ultrasound. Nodules are super super common and
almost never cancer. This doesn’t stop Radiologists from imaging them, and sticking needles
into them. Ultrasound guided FNA o f colloid nodules is a major cash cow for many body
divisions, that on very rare occasions will actually find a cancer. Qualities that make them more
suspicious include: more solid (cystic more benign), calcifications (especially
microcalcifications). Microcalcifications are supposed to be the buzzword for papillary
thyroid cancer. “Comet Tail” artifact is seen in Colloid Nodules. “Cold Nodules” on 1-123
scans are still usually benign but have cancer about 15% o f the time, so they actually deserve
workup.

Colloid Nodules: These are super super common. Suspicious features include
microcalcifications, increased vascularity, solid, size (larger than 1.5 cm), and being cold on a
nuclear uptake exam. As above, Comet tail artifact is the buzzword.

Ultrasound Characteristics of Thyroid Nodules

Margins & Size &


Calcifications Vascularity Echotexture
Shape Multiplicity

Increased Risk Complete Peripheral Solid composition has Nodule size is


• Micro x3 hypoechoic pattern of flow the highest sensitivity NOT predictive of
• Macro x2 halo is highly is suggestive of for malignancy malignancy.
suggestive of benign disease (although not specific
benign disease - many benign nodules Suspicious
Microcalcifications have Central pattern are also solid.) features should be
the highest accuracy, of flow is the primary
Pure cystic, or
specificity, and positive suggestive of consideration
spongiform with more
predictive value for malignancy when targeting for
than 50% specific are
detecting malignancy biopsy (not size)
likely benign.
Microcalcifications = the
“hallmark” of papillary CA

Macrocalcification = most Irregular, Solid Change over time Multiple nodules =


common calcification in spiculated, & hypervascular from solid to cystic decreased cancer
medullary CA microlobulated nodules are suggests benign rate per nodule —
margins more likely to disease but the overall
suggest be malignant rate of having
Macrocalcifications in a malignancy Most hyperechoic or cancer in one
single nodule = higher risk isoechoic nodules are nodule is fairly
benign. constant.
Macrocalcifications in a
Solid Hypoechoic =
multi-nodule goiter = lower
risk
feature of
malignancy

484
Thyroid Adenoma: These look just like solid colloid nodules on ultrasound. They can be hyper
functioning (hot on uptake scan). Usually if you have a hyper-functioning nodule (toxic adenoma),
your background thyroid will be colder than normal (which makes sense).

Goiter - Thyroid that is too big. In North America it’s gonna be a multi-nodular goiter or Graves.
In Africa it’s low iodine. You can get compressive symptoms if it mashes the esophagus or trachea.
These are often asymmetric - with one lobe bigger than the other.
Subacute Thyroiditis I De Quervains Thyroiditis: The classic clinical scenario is a
female with a painful gland after an upper respiratory infection. There is a similar subtype that
happens in pregnant women, although this is typically painless. You get hyperthyroidism (from
spilling the hormone) and then later hypothyroidism. As you get over your cold, the gland recovers to
normal function. Radiotracer uptake will be decreased during the acute phase.
Acute Suppurative Thyroiditis: This is an actual bacterial infection of the thyroid. It is
possible to develop a thyroid abscess in this situation. A unique scenario (highly testable) is that in
kids this infection may start in a 4‘'>branchial cleft anomaly (usually on the left), travel via a
pyriform fistula and then infect the thyroid. Honestly, that is probably too much for the exam - but
could show up on a certification exam under neuro.

Reideis Thyroiditis; This is one of those IgG4 associated diseases (others include orbital
pseudotumor, retroperitoneal fibrosis, sclerosing cholangitis). You see it in women in their 40s-70s.
The thyroid is replaced by fibrous tissue and diffusely enlarges causing compression of adjacent
structures (dysphagia, stridor, vocal cord palsy). On US there will be decreased vascularity. On an
uptake scan you are going to have decreased values. A sneak trick would be to show you a MR (it’s
gonna be dark on all sequences - like a fibroma).
Thyrogiossai Duct Cyst (TGDC): The
most common congenital neck cyst in Pediatrics. TGDC - Location Fuckery
This can occur anywhere between the foramen These are the general numbers to think
cecum (the base of the tongue) and the thyroid about:
gland (or below). It looks like a thin walled cyst.
Suprahyoid = 25%
Why Care? At the Hyoid = 30%
•They can get infected Infrahyoid = 45%
•They can have ectopic thyroid tissue
•Rarely, that ectopic tissue can get papillary So, where is the most common location?
thyroid cancer (if you see an enhancing nodule) Well, it depends on how they ask. If all
things are equal the answer is Infrahyoid
(which seems counterintuitive based on the
embryology but is nonetheless true).
BUT - if “at or above the hyoid” is a choice
- then that is actually the right answer
(25+30 > 45). As always, read every choice.

Ectopic and Lingual Thyroid: Similar to a thyrogiossai duct cyst, this can be found
anywhere from the base of the tongue through the central neck. The most common location (90%)
is the tongue base (“Lingual Thyroid”). It will look hyperdense because of its iodine content (just
like a normally located thyroid gland). If you find this, make sure you check for a normal thyroid
(sometimes this is the only thyroid the dude has). As a point of trivia, the rate of malignant
transformation is rare (3%).

485
Graves - A utoim m une disease that causes hyperthyroidism (m ost com mon cause). It’s
prim arily from an antibody directed at the TSH receptor. The actual TSH level will be low.
The gland will be enlarged and “inferno hot” on Doppler.
* Graves Orbitopathy. Spares the tendon insertions, doesn’t hurt (unlike pseudotumor).
Also has increased intra-orbital fat.
* Nuclear M edicine: Increased uptake o f 1-123 % RAIU usually 50-80%. Visualization o f
pyramidal lobe is accentuated.

Hashimotos - The most com m on cause o f goitrous hypothyroidism (in the US). It is
an autoimmune disease that causes hyper then hypo thyroidism (as the gland bum s out later).
It’s usually hypo - when it’s seen. It has an increased risk o f prim ary thyroid lymphoma.
Step 1 trivia; associated with autoantibodies to thyroid peroxidase (TPO) and anti-
thyroglobulin.

On Ultrasound.
There are two classic findings:

(a) Heterogeneous “giraffe sldn” appearance,

(b) W hite Knights - uniform hyperechoic nodules -


which are actually regenerative nodules.

White Knight

Level 6 Nodes - “Delphian Nodes”


These are the nodes around the thyroid in the front o f the neck.
You can com m only see them enlarged with Hashimotos.
However, for the purpose o f multiple choice tests, a sick looking level 6 node - or
“Delphian N ode” is a laryngeal cancer met.

486
Thyroid Cancer: You can get lots o f cancers in your thyroid. There are 4 m ain subtypes
o f prim ary thyroid cancer. Additionally you can get mets to the thyroid or lym phom a in your
thyroid - this is super rare and I ’m not going to talk about it.

Microcalcifications is M ets via the


The Most Common the buzzword and lymphatics. Has an
Papillary Subtype. “Papillary is key finding (seen in overall excellent
Popular” the cancer and prognosis, and
nodes). responds well to 1-131.

Mets hematogenously
to bones, lung, liver,
etc.. Survival is still
The second most
Follicular ok, (less good than
common subtype.
papillary). Does
respond to I-131.

Tendency towards
Association with local invasion, lymph
MEN II syndrome. nodes, and
Medullary Uncommon
Calcitonin production hem atogenous spread.
is a buzzword. Does NOT respond to
1-131.

Seen in Elderly. Seen Rapid growth, with


in people who have prim ary lymphatic
Anaplastic Uncommon
had radiation spread. Does NOT
treatment. respond to 1-131.

Does not take up


1-131 as well as
Hurthle Cell (variant
Uncommon Seen more in Elderly. normal follicular.
o f Follicular) FDG-PET is the way
to go fo r surveillance.

Metastasis: The buzzw ord is going to be m icrocalcifications in a node (with papillary).


The nodes are typically hyperechoic com pared to regular nodes, hyperenhancing on CT, and
T1 bright on MR. Rem em ber that thyroid cancer is hypervascular, and it can bleed like stink
when it mets to the brain. If there are mets to the lungs, the classic pattern is “miliary.” The
additional pearl with regard to lung mets is that they can be occult on cross sectional imaging,
and only seen on whole body scintigraphy. For the purpose o f multiple choice tests pulm onary
fibrosis is a risk o f treating with 1-131 i f yo u have diffuse lung mets.

487
S E C T I O N 4:
Pa r a -T h y r o id

Anatomy: There are norm ally 4 parathyroid glands located posterior to the thyroid. The
step 1 trivia is that the superior 2 are from the 4th branchial pouch, and the inferior 2 are
from the 3rd branchial pouch. The inferior two are more likely to be in an ectopic location.

Parathyroid Adenoma: This is by far the m ost com m on cause o f


hyperparathyroidism (90%). On ultrasound these things look like hypoechoic beans
posterior to the thyroid. A 4D-CT can be used to dem onstrate early w ash-in and delayed
wash-out. N uclear medicine can use two techniques (1) the single-tracer, dual-phase
Sestamibi, or (2) the dual tracer Sestamibi +1-123 (or Pertechnetate). These are discussed
in detail in the nukes section.

Early-Arterial

Parathyroid Adenoma - 4D C T shows early enhancement, and delayed washout

Parathyroid Carcinoma: This is pretty uncom m on, and only m akes up about 1% o f
the causes o f hyperparathyroidism . It looks exactly like an adenom a on imaging. The only
way you can tell on imaging is if they show you cervical adenopathy or invasion o f adjacent
structures.

High Yield Parathyroid Trivia:


Q: W hat are the causes o f hyperparathyroidism ?
A: H yperfunctioning Adenom a (85-90%),
M ulti-Gland Hyperplasia (8-10% ), Cancer (l-3%>).
Q: W hat factors does sestam ibi parathyroid im aging depend on ?
A: M itochondrial density and blood flow

488
You don't get w hat you deserve, yo u get w hat yo u earn.

-Tom Brands, Olympic Gold M edalist

489
fROMETHEUS
Liom hart, m.d.

490
PROMETHEUS LIONHART, M.D.

NUKES

9 ^

My friend, who has been in a coma for 3 m onths and presum ed dead, text me this

< Messages

I lived bitch

491
S E C T IO N l:
W H A T S CAN IS IT?

Ok, here is the scenario that I want you to be prepared for:

The plane has crashed. A ll the nuclear techs are dead. Prior to the plane crashing, they
completed several studies, but fo rg o t to label them or give indications. The bean counter (non-
MD) who is running the hospital is breathing down yo u r neck to read these studies now,
because the metrics he set up are gonna look bad at the next QA/QC meeting. So now you have
to interpret nuclear studies, and you d o n ’t know why they did them or even what tracer was
given.

Fortunately, you trained for this as part o f your preparation for the Exam.

Seriously, this is famously one o f the m ost common ways nuclear medicine is tested. It was
like that on the old oral boards, and probably still like that now (same knuckle heads writing the
questions). It’s such a ridiculous thing to ask and yet it almost always finds its way onto the
exam in one way or another.
My primary advice: D o n ’t Fight It. Games are Best Played As Games.

* Note the M IBG is under both heart and no heart - this is because i t ’s variable.
MIBG with 1-123 is more likely to have heart than 1-131.

492
This is an alternative pathway that some people prefer. This one focuses more on photon
output (how light or dark stuff is), and liver and spleen. It rem oves the confusion o f heart

A nother alternative w ay to w ork the bones pathw ay is to ask Lacrim al Glands? Gallium will
have them, W BC scans and Sulfur Colloid will NOT. The trick on Lacrim al Glands is free
Tc (but bones will be real w eak on that one). M IBG can have lacrimal activity , but again no
bones.

493
What Scan is it - Special Topics:
MIBG
I’ll talk about this more later in the chapter, but MIBG can be labeled with either 1-123 or 1-131.
The energy of the 1-123 (159 keV) is better for imaging, you can give a higher dose, and the
results are typically available within 24 hours (1-131 usually requires delays to optimize target to
background noise). Having said that 1-131 labeled MIBG is still used all over the place, especially
with adults. 1-131 may also be better for estimation of tumor uptake, for planning related to
MIBG therapy. The point of me rambling here is that you have two different MIBGs - so telling
which scan is which requires a little finesse.
It has variable cardiac uptake, so it finds itself on multiple branch points. Cardiac activity is more
often seen on an 1-123 MIBG scan (as opposed to 1-131 MIBG). Another thing that helps me
remember this stuff: when you do a MIBG you are often looking for neuroblastoma. If the kidney
was also hot it would be hard to tell a mass near the kidney from the kidney - so part of the
reason the study works is that the kidney does NOT take up MIBG.
Adrenal glands; Normal adrenal glands are not seen. However, you can have faint uptake in the
adrenals in about 15% of 1-131 patients, and around 75% of 1-123 patients. So, if you see
adrenals and you are sure they are normal (faint and symmetric) it’s more likely to be 1-123.

DARK SPLEEN + Tc WBC vs In WBC


DARK KIDNEYS Just like MIBG can be labeled with either 1-123 or 1-131
you can label WBCs with Tc or Indium.
The classic knee jerk for Octreotide.
In WBC vs Tc WBC: Both will TcWBC In WBC
This is a high count study, so the
have hot spleens. Additionally,
images should be clearer (relative to Renal NO Renal
Tc is a higher count study and
the normal “un-clear medicine.” GI NO GI
will typically look cleaner.
You can go down the “no bones”
pathway, but the trigger should be no Tc WBC: The trick here can be imaging at 4 hours vs
bones + liver + dark spleen + dark imaging at 24 hours. At 4 hours you can see lung
kidneys. uptake. At 24 hours the lungs are clearing up, but you
start to get some bowel uptake. Just like an In-WBC the
Hot Kidneys spleen is still darker than the Liver.
j Hot Spleen

(p5*.v ^

Tc-99 WBC at 4 hours Tc-99 WBC at 24 hours

494
M echanism s of Localization:
Just like
Nuclear Medicine is sorta like working for the Bounty Hunter Guild (but way
bounty
lamer). In the same way that the Guildmaster would provide you with a chain code
hunting.
and tracking device - you use tracer agents (things that give off gamma rays as they
Nuclear
decay) and attach them to things that mimic normal physiology. For example, if
Medicine is a
you want to look for infection you would attach a tracer (Indium 111) to neutrophils
complicated
because you know those things will localize to infection. In this way, agents have
profession.
“mechanisms of localization” - and those are highly testable.

Overview of Tracers and How They Localize


“Chemisorption” - Binds with More tracer in regions o f bone
Tc-99m Medronate Phosphate Analog hydroxyapatite on the bone remodeling (osteoblastic / active
(MDP)
surface lesions).

Facilitated Diffusion and Trapped in the cell via


F18-FDG Glucose Analog
phosphorylation till F I8 decays
Secondary Active Transport.

Transported into the cell by the Absorbed via the GI


Na/1 symporter “NIS” Na/I symporter is located at the
Iodine 123 & 131 Iodine Analog
Incorporated into the Thyroid basolateral membrane o f thyroid
Hormone follicular cells

Iodine Analog
Tc-99m Transported into the cell by the NOT Incorporated into the
(for thyroid scans
Pertechnetate anyway)
Na/I symporter “NIS” Thyroid Hormone

Thallium 201 Potassium Analog i Transported via Na/K ATPpump — Active Transport

WBC Analog Localized via the WBC Indium Oxine is lipophilic so it


Indium 111 (when tagged to chemotaxis, then uptake occurs will passively cross the cell
Neutrophils) via phagocytosis membrane.

Crosses the blood brain barrier on the first pass because it begins its journey in a
Lipophilic state. Distribution is proportional to regional blood flow in the brain (usually
Tc-99m HMPAO highest in the gray matter). Will then be metabolized to a hydrophilic form that is
oppressed by the tyranny o f brain tissue (it can’t diffuse out).

Mechanism is similar to HMPAO with the testable difference being that ECD has a more
Tc-99m ECD
rapid clearance from the blood pool.
Mitochondrial transport occurs
via active uptake.
Lipophilic Cationic (positively
Has a fetish for the charged) molecule passivley Mo Metabolism =
negatively charged diffused into cells, and then Mo Mitochondria =
Tc-99m Sestamibi mitochondrial latches onto the negatively Mo Localization
membranes. charged mitochondrial *in the same way that
membranes. Mo Money correlates strongly
with Mo Problems.
(Mo is Jive for More)

Active uptake via NE


Norepinephrine Stored in neurosecretory granules
MIBG transporter and passive
Analog
diffusion

Indium 111 Somatostatin Binds to somatostatin receptors 2,3,5


Pentetreotide Analog (G-Protem Coupled Receptors)

Transferrin and lactoferrin


Gallium 67 Iron Analog Acute Phase Reactant
receptor binding

495
S E C T IO N 2:
S keleton

Tc-MDP: The w orkhorse o f skeletal imaging is methylene diphosphonate (M DP) - which is


tagged with Tc-99m.

Mechanism: M DP is a phosphate analog that binds with the crystalline hydroxyapatite in


bone via a process called chemisorption. Chem isorption can also be described as
“physicochemical adsorption” on multiple choice exams - for the purpose o f fucking with
you. Distribution is based on osteoblastic activity and to a lesser extent blood flow.
Osteoclasts

This (arrow) is
where the MDP
goes - at the
mineralization front
of bone (osteoid)
near the osteoblasts.
NOT near the
osteoclasts

This is prim arily bone cortex


In the growing skeleton (peds), tracer/M DP deposition is also seen at epiphyseal growth
plates and osteochondral junctions. Hot growth plates are normal on M DP bone scan.

G amesmanship - M DP and HDP are both bone agents so don’t get confused if they
say HDP to purposefully confuse you.

MDP Uptake Depends On: Normal MDP Uptake / Localization Occurs At:
• OsteoBLASTIC activity Bone (duh), but also the Epiphyses in kids
(why pure lytic lesions can Kidney (not seen *or very faint = Super Scan) , Bladder
be cold) Breasts (especially in young women)
» Blood Flow Soft tissues - low levels

There is a preparation process (which I ’ll discuss on the next page w hen I talk about
labeling). A fter preparation o f the tracer (15-25m Ci) it is injected into the patient. Then
you wait 2-4 hours to let the tracer clear from the soft tissues (so you can see them bones).
Renal function is critical for this soft tissue clearing.

496
Dude... Are You Fucking This Up ?

There are 3 prim ary ways the standard bone scan gets flicked up. You as the resident / fellow
will be blam ed for all three, unless you can find a w ay to deflect culpability upon a more
junior resident or perhaps a non-English speaking m edical observer.

3 Ways to Fuck it Up:

(1) Bad Renal Function — If you can’t clear the tracer from the soft tissue, cuz your
kidneys are shit (all those donuts & cigarettes finally caught up with you) you w on’t be able
to visualize the bones. Too much noise (soft tissue signal), not enough signal (from bones).

The Fix: You may be able to prevent the problem by encouraging oral hydration during the
2-6 hours between injection and image acquisition. You can also try a 24 hours delay —
some people call this a “4th phase.” A potential problem with this is the h a lf life o f Tc99m
(6 hours). This means at 24 hours you only have ~ 6% o f the tracer left to give you signal.

(2) Shitty Labeling - The scan is prepared from a kit m


which has M DP and stannous ion. You add free
pertechnetate and the stannous ion reduces it so it will
bind to the MDP. I f you don’t have enough stannous ion
{or you get air into the vial or syringe —that can cause
oxidation) you m ight get fr e e Tc (salivary gland, thyroid,
stom ach u p ta ke ). Uplate Jn Bone Scan
'incidental note of sternal met
Gamesmanship: Bone scan with tracer in the stomach, breast CA
thyroid, or salivary glands = Free Tc

The Fix: D on’t leave air in the syringe you fucking crayon chewer

(3) The Flare Phenom enon - This is a scenario specific to bone mets that have been
treated with chemotherapy. The “flare” typically occurs around 2 weeks to 3 months post
treatment (some sources say up to 6 m onths), and m anifests as an increase in both the
num ber and size o f the lesions. In this way, a potentially good response (bone turnover from
metastatic lesions healing) could mimic a bad response (more / larger lesions). Some people
call this “pseudo-progression.”

H ow can you tell i t ’s fla re and not actually cancer getting worse?
* On plain film lesions should get more sclerotic
* After 3 months they should improve - confirm ing a response to therapy.

497
Abnormal Distribution
Increased focal uptake is very nonspecific, and basically is just showing you bone turn over. So a
metastatic deposit can do that (and this is the classic indication). But, you can also see it with
arthritis (classically shoulder) and healing fractures (most commonly shown with segmental ribs).
Let's look at some potentially testable scenarios.

Sneaky Situations:

• Skull Sutures: It’s normal to see some persistent visualization of the


skull sutures, BUT when this is marked you may be thinking about
renal osteodystrophy.

• Breast Uptake: Some mild diffuse breast uptake is normal (especially


in younger women), BUT focal uptake can be cancer.

• Renal CORTEX activity: You are supposed to have renal activity (not
seeing kidneys can make you think super scan), BUT when the renal Asymmetric Breast
Tissue Uptal<e
cortex is hotter than the adjacent lumber spine you should think
(Primary Breast CA)
about hemochromatosis.

Diffuse Renal Uptake: This often occurs in the setting of


chemotherapy (especially if the study is looking for bone mets). This
also can be seen with urinary obstruction.

Liver Uptake: This can be several things, but the main ones to think
about are (1) Too Much Al+3 contamination in the Tc, (2) Cancer -
either primary hepatoma or mets, (3) Amyloidosis, (4) Liver
Necrosis

Spleen Uptake: This is a common trick to show an auto-infarcted


spleen - common in sickle cell patients. These same patients are
Diffuse Renal Uptal<e
going to have scattered hot and cold areas from multiple bone (Ciiemotlierapy)
infarcts.

Lung Uptake: In most cases this is some type of heterotopic calcification (dystrophic or mets).
The classic MDP hot lung met would be an osteosarcoma. Ultimately, it’s not specific and can be
seen in a ton of other random situations (fibrothorax, primarily tung tumors, radiation changes,
sarcoid, berylliosis, alveolar microlithiasis, Wegener’s, etc...).

Muscle - Yes... MDP is for bones, but it will also localize to injured skeletal muscle. The classic
way to show this is very hot quads, calfs, shoulders in a marathon runner (or military recruit) - as
a way to show rhabdomyolysis.

Diffusely Decreased skeletal uptake: This can be seen with:


(1) free Tc, or (2) Bisphosphonate therapy.

498
Abnormal Distiibution - Part 2 - MOP’SRevenge

Sacral Insufficiency Fracture:

This is a hot geographic area, confined to the i


sacrum, often with a characteristic butterfly or
“H” shaped (Honda sign).

Osteoporosis is the most common cause, but it


can also occur in a patient who has had radiation.

Fractures in the Elderly (including elder


abuse): In older populations, bone scans may be
♦ H) “Honda Sign”

negative for several days. A bone scan obtained


at 1 week will exclude a fracture.

Special Topic - Hypertrophic Osteoarthopathy:

This is a “Tramline” along the periosteum of long bones, which is


associated with conditions of chronic hypoxia (CF, Cyanotic Heart
Disease, Mesothelioma, Pneumoconiosis). However, when you see
this - you need to think lung cancer.
The classic look is a non-uniform, broken up, linear line of increased
uptake along the long bones of the legs (and sometimes arms) -
resembling a “tram-line.”
Apparently it’s actually seen in 10% of patients
with lung cancer.

Next Step? CXR, Chest CT etc.. .to look for the lung cancer - or other
cause of chronic hypoxia.
i I “Tramline
Sign"
Special Topic - AVN:
AVN, as discussed in the MSK chapter, can occur from a variety of causes (EtOH, Steroids, Trauma,
Sickle Cell, Gauchers). It is also classic after a renal transplant (although less common now with
modem drugs / less steroid use).
MRI is first line for AVN. Bone scan is actually second line (better sensitivity that plain film). In
addition to a contraindication to MRI, it might be a good choice in situations where multiple bones
need to be imaged at once

Gamesmanship:
• The appearance on bone scan will change depending on the
timing. Early on it might be normal or even “cold” - related to
interrupted blood supply. Later in the disease (once you start
seeing sclerosis on the plain film) it will become hot.
• The classic look is the “donuf’ sign - hot on the outside and —-
cool in the center. “D o n u t S ig n ” o f F em oral A VN

499
Abnormal Distribution - Part 3 - Dream Warriors
Primary Bone Tumors: Both benign and malignant bone tumors can be HOT on bone scan.
Remember, bones are stupid, the only thing they know how to do is make bone. When something is
happening in a bone... the bone makes bone — regardless if that is a benign or malignant process.

HOT Lesions
• Fibrous D ysplasia • Osteoblastom a • Aneurysm al Bone Cyst
• Giant Cell Tumor • Osteoid Osteom a *donut sign (centrally cold)

“Donut Sign” - 1 mentioned this earlier with AVN in the hip. This is
not a specific sign. It just describes the hot circle with the cold center
(like a donut).

You see it mostly with “cystic” bone lesions; aneurysmal bone cyst,
simple bone cyst, giant cell tumor, and the bad boy telangiectatic
osteosarcoma.

PSA <10 ng/ml nearly


IVlets: The overwhelming majority of bone scans performed in the real excludes bone mets,
world are done for patients with known malignancy (breast, lung, and PSA> 100 ng/ml is
prostate) looking for bone mets. highly predictive
Classic: Hot, multiple, randomly scattered, and different sizes / shapes. of bone mets

Diffuse: This is the “super scan” where all the tracer is in those thirsty cancer laden bones and almost
no activity is seen in the kidneys or soft tissues.

Arthritis vs Mets: Arthritis tends to involve both sides of the joint —


cancer doesn’t do that (usually).
Most common single
metastatic lesion is in
The Single Lesion: When you see a single hot lesion, the false positive
the spine
rate for attributing the finding to a met is high. Only about 15% to 20%
of patients with proven mets have a single lesion (most commonly in the
Most “classic” single
spine). In other words, 80% of the time it’s benign. A classic exception
lesion is a breast
is a single sternal lesion in a patient with breast cancer. This is due
cancer met to the
to breast CA 80% of the time.
sternum.
Vertebral Body Fracture (Benign V5' Malignant): A horizontal linear
pattem of tracer uptake is the classic look for an osteoporotic fracture
(especially if they are multiple and of varying intensity). If the tracer extends
from the vertebral body into the posterior elements or just involves the pedicles
then you should think cancer. Lastly, followup of the osteoporotic fracture
should show tracer activity decreasing (cancer isn’t gonna do that).

Rib Lesions (Benign V5 Malignant): Multiple ribs in a vertical linear pattem


should make you think fractures (think about any trauma scan you’ve ever seen
with rib fractures - they tend to line up). Now, if the hot spots are multiple (3 or
more) and are jumping around, bilaterally, etc... you should think mets. The
big thing is that when cancer goes to bones it rarely just goes to the ribs. So
lesions only in the ribs are more likely to be benign - but when you have lesions Multiple Contiguous
in the ribs and other spots then you are likely dealing with mets. Lesions (Rib Fractures)

500
Abnormal Distribution - Part 4- Tbe Dream Master

Specific Cancers - Specific Trivia:


• Prostate Cancer Loves Bone M ets BAD BONES!
(85% o f dying patients have it) A ny bone uptake on:
• Prostate Cancer bone mets are uncom m on MIBG,
with a PSA less than 10 mg/ml M 3 1 , or
• Lung Cancer bone mets tend to be Octreotide
in the appendicular skeleton
is abnorm al. & concerning for mets.
• Lung Cancer can have
hypertrophic osteoarthropathy (10%)
• Breast Cancer bone mets are m ost com m on to the spine,
but the solitary sternal lesion is more specific
• N euroblastom a frequently mets to the bones (metaphysis o f long bones)
• 1-123 and 131 M IBG are superior for detection o f neuroblastom a bone mets

Bone Island vs Prostate Met:


• Both are sclerotic on plain film / CT
• The Prostate M et should be very HOT on Bone Scan
• The Bone Island should be cold, or faintly w arm
• Osteopoikilosis should be cold - clustered around joints, and for the m ost part spares the
spine & skull

Bone Scan vs Skeletal Survey (Trivia): Equivocal Lesion


Next Step ?
• Bone Scan is way better (more sensitive) than skeletal
survey for blastic mets If a bone scan “equivocal lesion”
is found the next step is a plain
• Skeletal Survey is superior (more sensitive)
film. If the plain film shows no
for Ivtic mets corresponding lesion this is
• Skeletal survey is the preferred evaluation for osseous MORE suspicious for mets. Next
involvement in myeloma step at that point would be a MRI.

Post Treatment:
Chemotherapy — I mentioned the “flare” phenomenon that occurs post chemotherapy earlier in the
chapter. Remember, that imaging around 3 months post treatment is key to avoiding this pitfall.

Radiation - Similar to chemotherapy, the acute (early) phase of radiation osteitis will typically be warm
(-20% above baseline) within the radiation port - peaking around 2-3 months. Late phase changes are
typically cold. As a general rule - if it is cold at 6 months, it is probably staying cold permanently (it
won’t go back to baseline). So, if it was cold at month 7 and now it is hot — that is recurrent disease.

501
Abnormal Dlstiibuaon - Pan 5 - The Dream Child

Super Scans:
This is a com mon trick, where the scan shows no abnormal focal uptake, That s my secret,
but you can’t see the kidneys. That is the secret, everything is hot. I ’m always angry.

This occurs in two


flavors:

• Diffuse Mets: Diffuse


skeletal metastatic
activity
(breast and prostate
are the common
culprits).

• Metabolic: From
metabolic bone *
pathology; including
hyperparathyroid,
renal osteodystrophy,
Pagets, or severe
thyrotoxicosis.

Diffuse Prostate Mets Hyper PTH (hot skull)

How can you tell them apart?

■ (1) The skull will be asym m etrically HOT on the metaboHc super scan.

■ (2) Besides the hot skull, metabolic scans tend to have a more “uniform ” look w ith tracer
uptake extending m ore distal into the arms / legs (appendicular skeleton).

D on Vg et it tw isted - A com mon sneaky m ove is to show you a bone scan, with no renals. But
it’s because there is a horseshoe kidney in the pelvis. Could be phrased as a next step question,
with the answer being look at prior CT to confirm normal anatomy.

502
Abnormal Distribution - Part 6 - Tbe Hnai Nightmaro

^ Back to Primary Bone Lesions:

Both Osteosarcoma and Ewings will be hot.


Primary utility o f the bone scan is to see extent o f
disease. W ith regard to benign bone tum ors the
only ones worth knowing are the HOT ones and
the COLD ones. Osteoid Osteoma is worth
knowing a few extra things about (because they
lend themselves easily to multiple choice
Osteosarcoma Osteoid Osteoma
questions).
-Double Density

Osteoid Osteoma: The lesion will be focal and three phase hot. A central hot nidus is often
seen (double density or hotter spot within hot area). A norm al bone scan excludes this
entity.

Fibrous Dysplasia: Be aware that in case books / case conference this is sometimes shown
as a super hot mandible. Could also be shown as a leg that looks similar to Paget.

Pagets:

Seen prim arily in older patients (8% at 80), it’s classically shown five ways:

1. Super Hot Enlarged Femur


2. Super Hot Enlarged Pelvis
3. Super H ot Skull
4. Expanded hot “entire” vertebral body m
5. M etabolic Superscan - from w idespread I
Pagets. Although, as a point o f
gam esm anship if they show you a
metabolic superscan the answ er is
probably hyper PTH.
Pagets

A Pagets Spine - Classically involves BOTH the vertebral body and posterior elements.

503
Abnormal Dlstribuaon - Part 7 - Wes Craven’s New Nightmare

“HO”
Normally when we think about the consequences of Cold Lesions
“HO” we typically think about nasty things like
herpes, crabs, and gonorrhea - but on multiple Late Radiation Therapy Changes /
choice HO can refer to something totally different - Osteitis (usually segmental)
heterotopic ossification. This describes bone Early Osteonecrosis (AVN)
(hence the term “ossification”) growing in soft Infarction (very early or late)
tissues. Typically you see this acquired after a Anaplastic Tumor (Renal, Thyroid,
muscular trauma. As discussed in the MSK chapter Neuroblastoma, Myeloma)
the pattern is typically outside to in, with the Artifact from prosthesis, pacemaker,
eventual development of mature cortical bone. In spine stimulator, etc....
some cases this abnormal bone proliferation can Hemangioma ** variable
reduce joint motility and cause pain. Therefore, in
Bone Cyst (without fracture)
some cases surgical resection is performed to
M ature Heterotopic Ossification
preserve the function of the joint.
Enter the MDP Bone Scan.
The main reason you image this is to see if it’s “mature.” Serial exams are used to evaluate if the
process is active or not (not = “mature”). If it’s still active it has a higher rate of recurrence after it’s
resected. The idea is you can follow it with imaging until it’s mature (cold), then you can hack it out
(if someone bothers to do that).

'■
^F Sodium Fluoride PET vs Tc-MDP Bone Scan
We have talked a lot about Tc-MDP scans, but you can actually do bone scans with several
different tracers - the main two are 18F Sodium Fluoride and Tc-MDP. If you were trying to
differentiate the two (perhaps in the plane crash scenario I discussed on the first page o f the
chapter), the primary take home point is F-18 NaF PET is way way way cleaner looking than
Tc-MDP. By “cleaner” I mean that the image quality and sensitivity o f FI 8 NaF is multiple
orders o f magnitude better than Tc-MDP. It also has a shorter examination time. So, why do
you never see NaF? Because it costs more, and insurances w on’t pay etc... Politics and
Finance are the reasons. Another potentially testable factoid would be which organ gets the
highest dose? The organ receiving the highest dose is Bone with MDP, and Bladder with
NaF (overall >*F NaF > Tc-MDP) — probably... honestly you 11 read different things in
different sources.

—Scan on Scan on Scan—


This is a common trick, popular in case books & case conferences - asking you to distinguish
F-18 NaF bone scan vs Tc-MDP bone scan vs PET-FDG with marrow stimulation.

This is how you do it;


•Tc-MDP will have bone and kidney uptake. It will be a blurry fiizzv piece of crap.
•■
'®FNaF - PET will be beautiful, super high resolution, and look like a MIP PET - but
WITHOUT the brain uptake.
• ■'8F FDG -PET with bone stimulation - In patients treated with the granulocyte
colony-stimulating factor (GCSF) or erythropoietin (EPO) diffuse bone marrow uptake is
common. The scan will look similar to the F-18, but will have brain uptake. Also, this can show
increased uptake in the spleen.

504
The Three Phase Scan;
Bone scans can be done in a single delayed phase, or in 3 phases (flow, pool, and delayed). A
lot o f things can be “3 phase hot”, including osteom yelitis, fracture, tumor, osteoid osteoma,
charcot joint, and even reflex sympathetic dystrophy.

Cellulitis vs Osteomyelitis:

The benefit o f using 3 phases is to distinguish betw een cellulitis (which will be hot on flow
and pool, but not delays), and osteom yelitis (which is 3 phase hot). In children, a whole body
bone scan is often perform ed to evaluate extent. Additionally, because o f subperiosteal pus/
edema you can actually have decreased vascularity to the infected area (cold on initial phases)
but clearly hot on delayed phases.

Trivia: A n extended delay or “4th phase” - can help localization.


Trivia: A normal perfusion (flow) phase makes infection very unlikely.

In the spine, gallium (com bined with bone scan) or M RI are the preferred im aging modalities.

Response:

You can also use a bone scan to evaluate response to treatment. Blood flow and blood pool
tend to stay abnormal for about 2 months, with delayed activity persisting for up to 2 years.

This is especially true w hen dealing with load bearing bones. Gallium*’? and I n d iu m 'W B C
are superior for m onitoring response to therapy.

Reflex Sympathetic Dystrophy (RSD):

Sometimes called “com plex regional pain syndrom e,” it can be seen after a stroke, trauma, or
acute illness. The classic description is increased uptake on flow and blood pool, with
periarticular uptake on delayed phase. The uptake often involves the entire extremity.
About one third o f adult patients w ith docum ented RSD do not show increased perfusion and
uptake (which probably means they are fa k in g it, a n d need a rheum atology consult fo r
fibrom yalgia). In children, sometimes you actually see decreased uptake.

505
Sulfur Colloid Bone Scan & WBC Imaging:
Tc can be tagged to sulfur colloid with the idea of getting a
normal localization to the bone marrow. You can actually
perform Tc sulfur colloid studies to map the bone marrow in In -WBC Scan
patients with sickle cell (with the idea to demonstrate marrow
expansion and bone infarct). However, the major utility is to Normal Distribution ;
use it in combination with tagged WBC or Gallium. • Spleen » Liver > Marrow

Both Tc Sulfur Colloid and In WBCs will accumulate in No Renal Activity. No GI activity
normal bone marrow, in a spatially congruent way (they Helpful findings: Hot Spleen,
overlap). The principal is that infected bone marrow will Relatively low-count study
become photopenic on Tc-Sulflir Colloid. Now, this takes
about a week after the onset of infection, so you have to be Contrasted to Gallium:
careful in the acute setting. WBC, on the other hand, will • Liver > Spleen, and
obviously still accumulate in an area of infection. Combined • Can have GI activity
Tc-Sulfur Colloid and WBC study is positive fo r infection if
there is activity on WBC image, without corresponding Tc
Sulfur Colloid activity on the bone marrow image.

Tc-Sulfur Colloid In-WBC

i
Less Uptake in More Uptake in
Infected Bone Infected Bone

Special Situation >The Spine


When imaging the spine, WBC frequently fails to migrate showing a photopenic area
fWBC = False Negative in the Spine). This is why gallium is better for osteomyelitis of the spine.

Tc-Sulfur Colloid
ft* In-WBC Gallium
Less Uptake in Less Uptake in More Uptake in
Infected Bone Infected Bone - Infected Bone
*Faise Negative

506
Prosthesis Evaluation:

Differentiating infection from aseptic loosening is challenging and the m ost com m on reason a
nuclear medicine doctor would get involved in the situation. Bone scan findings o f
periprosthetic activity is very nonspecific, because you can see increased tracer activity in a
hip up to 1 year after placem ent (even longer in cem entless arthroplasty). Typically, there
would be diffusely increased activity on im aging with Tc-MDP in the case o f infection (more
focal along the stem and lesser trochanter with loosening) - but this isn ’t specific either.
Combined Tc-Sulfur Colloid and W BC im aging is needed to tell the difference.

H elpful when negative - A negative bone scan excludes loosening or infection.

Neuropathic Foot:

M ost commonly seen in the tarsal and tarsal-m etatarsal joints (60%), in diabetics. W hen the
question is infection (w hich diabetics also get), it’s difficult to distinguish arthritis changes vs
infection with Tc-MDP. Again, com bined m arrow + W BC study is the w ay to go.

An additional pearl that could make a good “next step” question is the need for a fourth phase
in diabetic feet. As these patient’s tend to have reduced peripheral blood flow, the addition o f
a 4th phase at 24 hours may help you distinguish betw een bone and delayed soft tissue
clearance.

Instead of In-WBC, What about Tc99 HMPAO WBC ?

When w ould yo u consider Tc99 HMPAO instead o f In-W B C fo r infection ? Two main
reasons
(1) Kids - Tc99 will have a lower absorbed dose & shorter im aging time
(2) Small Parts - Tc99 does better in hands and feet

Why not use Tc99 HMPAO all the time ? The dow nsides to Tc99 HMPAO are:
(1) It has a shorter h a lf life -6 hrs- w hich limits delayed imaging, and
(2) It has normal GI and gallbladder activity w hich obscures activity in those areas.

In-WBC Chem and Pharm


The vast majority (like 90%) o f the labeled cells are Neutrophils. This allows you to
“trace” anything that triggers neutrophil m igration (inflam m ation / infection).
The radiation dose that the Indium deposits on the neutrophil doesn’t mess w ith its
function (supposedly). Lym phocytes on the other hand, tend to be killed by the radiation
(they don’t turn into cancer).
In the norm al situation the critical organ is the spleen.
If the cells becom e fra g m en ted - the Indium binds w ith transferrin and yo u see more
uptake in the liver a n d bone marrow.

507
S E C T IO N 3:
Pulm o n ary

If 1940 calls and wants to rule out a PE, you’ll want to get the angiography room ready. In 2014,
textbooks and papers still frequently lead with the following statement “Pulmonary angiography is
the definitive diagnostic modality and reference standard in the diagnosis o f acute PE. ” In reality,
pulmonary angiography is almost never done, and CTPA is the new diagnostic test of choice. V/Q
scan is usually only done if the patient is allergic to contrast or has a very low GFR. The primary
reason V/Q isn’t done is that it’s often intermediate probability, and the running joke is that if you
don’t know how to read one, just say it’s intermediate and you’ll probably be right.
The idea behind the test is that you give two tracers; one for ventilation and one for perfusion. If you
have areas of ventilated lung that are not being perfused, that may be due to PE. Normally
Ventilation and Perfusion are matched, with a normal gradient (less perfusion to the apex - when
standing).

Tracers:
Perfusion: For perfusion, Te-99m macroaggregated albumin (Tc-99m MAA) is the most common
tracer used. MAA is prepared by heat denaturation of human serum albumin, with the size of the
particles commercially controlled. You give it IV and the tracer should stay in the pulmonary
circulation (vein-> SVC-> right heart -> pulmonary artery -> lung *STOP). The tracer should light up
the entire lung. A normal perfusion study excludes PE. Areas of perfusion abnormality can be from
PE or other things (more on this later). The biologic half life is around 4 hours (they eventually fall
apart, becoming small enough to enter the systemic circulation to eventually be eaten by the
reticuloendothelial system).
Ventilation: There are two ways to do the ventilation; you can use a radioactive gas (Xenon-133) or a
radioactive aerosol (Tc-99m DTPA).
• Xenon 133: The physical half-life is 5.3 days, the biologic lialf-life is 30 seconds (you breath it
out). Because it has low energy (81 keV) it is essential to do this part of the test first (more on
this in the physics chapter). Additionally, because the biologic half life is so short, you only can
do one view (usually posterior) with a single detector (dual detector can do anterior and
posterior). There are 3 phases to the study: (1) wash in (single max inspiration and breath hold),
(2) equilibrium (breathing room air and xenon mix), and (3) wash out (breathing normal air).

• Tc-99m DTPA: This one requires patient cooperation because they have to breath through a
mouth guard with a nose clamp for several minutes. It is also essential to do this part of the test
first.

Quantitative Perfusion:
You can do quantitative studies typically to evaluate prior to lung resection, or prior to transplant.
You want to make sure that one lung can hold its own if you are going to take the other one out.

Testable Trivia: Quantification is NOT possible if you use Tc-99 DTPA aerosol. You can do it with a
combined Xe + Tc MAA because the Xe will not interfere with the Tc.

508
5 Classic Trivia Questions about Tc99m MAA:

(1) They show tracer in the brain: This is a classic w ay o f showing you a shunt (it
got into the systemic circulation somehow, m aybe an ASD, VSD, or Pulmonary
AVM).

(2) H ow big are the particles? A capillary is about 10 micrometers. You need your
particles to stay in the lung, so they can’t be sm aller than that. You don’t w ant
them to be so big they block arterioles (150 micrometers). So the answer is
10-100 micrometers.

(3) When do yo u reduce the particle am ount? A few situations. You don’t w ant to
block m ore than about 0.1% o f the capillaries, so anyone who has fewer
capillaries (children, people with one lung). Also you don’t w ant to block
capillaries in the brain, so anyone w ith a right to left shunt. Lastly anyone with
pulm onary hypertension (or who is pregnant).

(4) Is reduced particle the sam e as reduced dose? Nope. The norm al dose o f Tc can
be added to few er particles.

(5) They show yo u multiple fo c a l scattered hot spots: This is the classic w ay o f
showing “clum ped M A A ” , w hich happens if the tech draws blood into the
syringe prior to injection.

Classic Trivia Questions for Xenon 133:

(1) They show yo u persistent pulm onary activity during washout. This indicates
Air Trapping (COPD)

(2) They show yo u accumulation o f tracer over the RUQ\ This is fatty
infiltration o f the liver (xenon is fat soluble).

Classic Trivia Questions for Tc-99m DTPA

Xenon TC-99m DTPA

Quick Wash Out only one or two views Slow er Wash Out - m ultiple projections
Activity hom ogenous in the lungs “C lum ping” com m on in the mouth,
central airways, and stomach (from
swallowing).

509
-Gamesmanship-

Q: What i f yo u see tracer in the thyroid or stom ach on VQ Scan??


A: You should think 2 things: (1) Free Tc, or (2) R ight - to - Left Shunt

Q: What do yo u need to call a R ight - to - L eft Shunt ?


A: Tracer in the Brain

Q: I f you suspect a shunt (or the shunt is known) how do yo u alter the scan?
A: You reduce the num ber o f particles. I f the norm al am ount o f particles is around 500K,
you would reduce it to around lOOK.

Q: What i f the p a tient has pulm onary hypertension?


A: Same deal - reduce the particles

Q: Particle reduction is the sam e as dose reduction?


A: Nope. You keep the dose the same - otherwise the study is non-diagnostic.

Q: What about a neonate ? D o you do anything different?


A: Yes - m ajor particle reduction. Down to lOK- 50K particles (depending on who you
ask). The reason is that kids have less capillaries than adults. A n adult num ber o f
particles (500K) could functionally cause a PE - by blocking the m ajority o f the
capillaries.

Q: What i f you see a unilateral perfusion defect (o f the whole lung), but no ventilation
defect ?
A: Get a CT or MRI. DDx is gonna be a mass, fibrosing mediastinitis, or Central PE.

Q: O f those which is the M O ST COM M ON?


A: M ost sources will say “central obstructing m ass” i.e. “bronchogenic carcinom a”

Q: H ow do yo u grade this unilateral perfusion defect ( o f the whole lung), but no


ventilation defect ?
A: It’s technically low probability.

510
Gallium Scan

The body handles Ga+3 the same w ay it w ould Fe+3 - which as you may rem em ber from
step 1 gets bound (via lactoferrin) and concentrated in areas o f inflamm ation, infection,
and rapid cell division. Therefore it’s a very non-specific w ay to look for infection or
tumor. Back in the stone ages this was the gold-standard for cancer staging (now we use
FDG-PET). I should point out that Gallium can also bind to neutrophil m em branes even
after the cells are dead, which gives it some advantages over Indium W BC - especially in
the setting o f chronic infection.

Gallium is produced in a cyclotron via the bom bardm ent


o f Zn*58, at which point it’s com plexed with citric acid to
make Gallium Citrate. The h alf life is around 3 days
(78hours). It decays via electron capture, em itting gam m a
rays at 4 photopeaks:

93 keV - 40%
1 8 4 k e V -2 0 %
300 keV - 17%
393 keV - 5%

Images are not typically done sooner than 24 hours -


because background is too high. The critical organ is the
Normal Ga®^
colon. Rem em ber “critical organ” = the first organ to be
subjected to radiation in excess o f the m axim um allowable
amount.

N orm al localization: Liver {which is the highest uptake), bone m arrow ( “Poor M ans s
bone s c a n ’’) , spleen, salivary glands, lacrimal glands, breasts (especially if lactating, or
pregnancy). Kidneys and bladder can be seen in the first 24 (faintly up to 72 hours). Faint
uptake in the lungs can be seen in < 24 hours. A fter 24 hours you will see some bowel. In
children the grow th plates and thymus.

“Poor M a n ’s Bone Scan ” - Uptake is in both cortex (like regular bone scan) and marrow.
Degenerative change, fractures, growth plates, all are hot - ju st like bone scan.

Gallium uptake is nonspecific and can be seen with a variety o f things including infection,
but also CHF, atelectasis, and ARDS.

511
Sarcoidosis:

The utility o f Gallium in Sarcoidosis patients is to help look for active disease. Increased
uptake in the lungs is 90% sensitive for active disease (scans are negative in inactive
disease). Additionally, Gallium can be used to help guide biopsy and lavage - if looking to
prove the diagnosis. The degree o f uptake is graded relative to surrounding tissue (greater
than lung is positive, less than soft tissue is negative).

Classic Signs:

* Lambda Sign - The nuke equivalent to the “ 1-2-3 Sign” on Chest x-ray. You have
increased uptake in the bilateral hila, and right paratracheal lym ph node.

• Panda Sign - Prom inent uptake in the nasopharyngeal region, parotid salivary gland,
and lacrimal glands. This can also been seen in Sjogrens and Treated Lymphoma.

■r
Lacrimal Gland

Nasopharynx

Parotid

Ga 67 - Panda Sign

Other Noninfectious Things

* Gallium can be used to show early drug reaction from chem otherapy (Bleomycin) or
other drugs (Amiodarone).
* Gallium is elevated in IPF (idiopathic pulm onary fibrosis) and can be used to m onitor
response to therapy.

512
Immunosuppressed Patients

* PCP - Gallium Hot, Characteristic Gallium Pattern is Diffuse Bilateral Pulmonary


Uptake
* Kaposi Sarcom a - Gallium Negative, Thallium Positive
* Bacterial P n eum onia - Intense lobar configuration without parotid or nodal uptake.

Galium 6^ - Pneumonia
- Lung Uptake at 72 Hours

Misc Infections That Gallium Can Pick Up:

• Abdom inal and Pelvic Infections - In-111 W BC is superior to Gallium


(Gallium has some norm al GI uptake).

• M alignant Otitis M edia - Will be both G allium and Bone Scan (Temporal Bone) Hot.

• Spinal Osteomyelitis - Gallium is superior to Indium W BC for spinal infections.

513
S E C T IO N 4:
^ T hyroid

The thyroid likes to drink Iodine (it’s sort o f its job). Imaging takes advantage o f this with
Iodine analogs. The distinction between “trapping” and “organification” is a com mon
question.

• “Trapping” - A nalog is transported into gland. >231 ^ 13 and 99mXc all do this.

* "Organification ” - A nalog is oxidized by thyroid peroxidase and bound to tyrosyl


moiety. '23i and i^il do this. ’’mXc does N O T do this. Instead 99mTc slowly washes
out o f the gland.

Tracer options / pros and cons.

1-131: The m ajor advantage here is that it’s cheap as dirt. The disadvantage is that it has a
long h alf life (8 days), and that it’s a high energy (364 keV) beta emitter. The high energy
makes a crappy image with a '/i inch crystal. It’s ideal for therapy, not for routine imaging.
It’s contraindicated in kids and pregnant women.

Trivia: Thyroid form ation takes p lace in fe tu s at 8-12 weeks.

1-123: This guy has a shorter h alf life (13 hours) and ideal energy (159 keV). It decays via
electron capture and all around makes a prettier image. The problem is that it costs more.

Tc-99m: Rem em ber that this guy is trapped but not organified. Background levels are higher
because only 1-5% o f the tracer is taken up by the thyroid gland. A com m on scenario to
choose Tc over Iodine is when they’ve had a recent thyroid blocker on board (iodinated
contrast is the sneaky one).

Random Trivia on Breastfeeding


(You know they love this shit):

Tc-99m: You can resume breast fe ed in g in 12-24 hours


T123: You can resume breast fe ed in g in 2-3 days
1-131: You should not breast fe e d - pum p a n d dump.

514
Iodine Uptake Test:

You give either 5 micro Ci o f 131 or 10-20 m icro Ci o f 123. This is conventionally reported
at 4-6 hours, and 24 hours. N orm als are 5-15% (4-6 hours), and 10-35% at 24 hours. A
correction for background is done on m easurem ents prior to 24 hours (using the neck counts
- thigh counts).

Factors affecting the test


• Renal Function (increases stable iodine pool, reduces numbers)
• Dietary Iodine - variable and controversial
• M edications - thyroid blockers, Nitrates, IV Contrast, A m io d aro n e

Increased Uptake Decreased Uptake

Graves Primary or secondary causes of Hypothyroidism

Early Hashimoto Renal Failure

Rebound after Abrupt withdrawal of antithyroid Medications (thyroid blockers, Nitrates, IV


medication Contrast, Amiodarone)

Dietary Iodine Deficiency Dietary Iodine Overload

Graves Disease;

About 75% o f the time, if you have hyperthyroidism the


cause is going to be graves. Graves is an autoimm une
disease where an antibody to the thyrotropin receptor
stimulates the thyroid to produce hormone. TSH will be
very low, w here T3 and T4 will be high. The classic
clinical scenario is a middle aged w om en with a protracted !•
course, pre-tibial edema, and exophthalmos. Scintigraphy
is going to give you a hom ogeneously increased gland,
with uptakes increased at both 4 hours and 24 hours.
Sometimes the 24 hour uptake is lower than the 4 hours (or
even at a normal range) - this is from rapid thyroid
hormone production.

Visualization o f the pyram idal lobe: The pyram idal lobe is seen in about 10%
o f normal thyroids. In patients with Graves disease it is seen as m uch as 45%
o f the time. Therefore, it’s suggestive when you see it.

515
Multi-nodular Toxic Goiter (Plummer Disease): The classic scenario is an
elderiy w om en with w eight loss, anxiety, insomnia, and tachycardia. The gland is typically
heterogeneous, with uptake that is only m oderately elevated. The nodules will be hot on the
background o f a cold gland.

Toxic Multi-nodular Goiter vs Non-Toxic Multi-nodular goiter:


The toxic goiter will have hot nodules on a background o f cold thyroid. The N on-toxic one
will have w arm /hot nodules on a normal background o f the thyroid.

Graves Toxic M ulti-Nodular Goiter

Uptake High ; 70s (typically > 50%) Uptake M edium High: 40s (typically <50% )

Homogenous Heterogeneous

Hashimotos:
The most com m on cause o f goitrous hypothyroidism (in the US). It is an autoimm une
disease that causes hyper first then hypothyroidism second (as the gland bum s out later). It’s
usually hypo - when it’s seen. It has an increased risk o f prim ary thyroid lymphoma.
Step 1 trivia; associated w ith autoantibodies to thyroid peroxidase (TPO) and anti-
thyroglobulin. The appearance o f the hypothyroid gland is typically an inhom ogeneous gland
with focal cold areas. The hyperthyroid (acute) gland looks very much like Graves with
diffusely increased tracer.

Subacute Thyroiditis:
If you have a viral prodrome followed by hyperthyroidism , and then thyroid uptake scan
shows a DECREASED % RAIU you have de Quervains (G ranulom atous thyroiditis).
During this acute phase, the disease can mimic Graves w ith a low TSH, high T3 and high
T4. The difference is the uptake scan. A fter the gland bum s out, it m ay stay hypothyroid or
recover. If they ask you about this, it’s m ost likely going to try and fool you into saying
Graves based on the labs, but have a low % RAIU.
Solitary Nodules
20-40% Cold Nodules = Cancer
Hot Nodule vs Cold Nodule: < Warm Nodules = Cancer

M ost thyroid nodules are actually cold, and therefore m ost are benign (colloid, cysts, etc..).
In fact, cold nodules in a m ulti-nodular goiter are even less likely to be cancer com pared to
a single cold nodule. Having said that, cold nodules are m uch m ore likely to be cancer
when com pared to a functional (warm) nodule.

Discordant Nodule: This is a nodule that is HOT on Tc^^ but COLD on I ’23. Because some
cancers can maintain their ability to trap, but lose the ability to organify a hot nodule on Tc,
it shouldn’t be considered benign until you show that it’s also hot on I'23.

516
^ Gamesmanship: Iodine vs Tc ^
A classic move is to show you a thyroid that will take up Tc, but N O T Iodine on 24 hour
imaging. This can be from a couple o f things: (1) congenital enzyme deficiency that
inhibits organification, (2) a drug like propylthiouracil that blocks organification.

Now if they ju st show you an Iodine Thyroid with low uptake on 24 hours, this is de
Quervains, or a burned out Hashimotos.

Radioiodine Tiierapy
can be used to treat both m ahgnant and non-m alignant thyroid disease.

Cancer
Actual subtypes and pathology o f thyroid cancer have been discussed at length in the
endocrine chapter. However, ju st a few points that are relevant to discuss here. Papillary is
the m ost com m on subtype (papillary is popular), and it does well with surgery + 1-131.
M edullary thyroid CA (the one the M ENs get), does N O T drink the 1-131 and therefore
doesn’t respond well to radiotherapy. Prior treatm ent can also make you m ore resistant to
treatment, and re-treatm ent dosing is typically 50% more than the original dose.

Things that m ake yo u treatment resistant:


• M edullary Subtype CA (will not drink the tracer)
• History o f prior 1-131 (“easy gland has been killed o f f ’)
• History o f M ethim azole treatm ent (even if years ago)

So, norm ally the patient gets diagnosed and


then they go for surgery. A fter surgery they
will come to nuclear medicine. You expect IVledullary Subtype CA
that they will have some residual thyroid
Neuroendocrine in origin, so can occasionally
(it’s really hard to get it all out). Prior to (around 10%) have uptake on MIBG or
actually treating them you will give them a Octreotide. They will be cold on thyroid scan
tiny dose o f 1-131 to see how much thyroid and don’t drink the treatment 1-131. If forced
to pick - I ’d go with Octreoscan for medullary
they have left. If the uptake is less than 5%
CA.
this is ideal. Uptake m ore than 5% will
The more aggressive the tumor, the more
result in a painful ablation (may need
useful PET CT is. It trends with calcitonin:
steroids on top o f the NSA ID s) and may • If the calcitonin is > 1000 PET is sensitive.
need to go back to the OR. N ext, you will • If the calcitonin is < 500 , it is shit.
treat them. You want their TSH really
ramped up. The higher the TSH the thirstier Associated with MEN 2a and 2b
the cancer /residual thyroid tissue. A n ideal
TSH is like 50 (30 w ould be a minimum).

517
Radioiodine Therapv - Continued

How do you get the TSH up?


• There are two ways;
o (1) is to stop the thyroid hormone (post op they are obviously hypothyroid),
0 (2) is to give recombinant TSH “Thyrogen.”

How do you decide on dosing?


• Dosing is dependent on the stage o f the disease; 100 for thyroid only, 150 for thyroid
+ nodes, 200 for distal. They are told about the precautions e tc ... Then you give them
the dose. Before you let them go home, you test them to see if they need to go to the
hospital.

So when do patients need to be adm itted to the hospital?


• NRC limit is 7mR/h measured at 1 meter from the patient’s chest (some agreement
states use 5 mR/h). The number to remember is 33 mCi of residual activity (or 30
mCi in some strict agreement states).

Possible Side Effects o f Treatment:


• Can cause pulmonary fibrosis if given to patient with lung mets. This is really only
the case o f macro-nodular disease (as opposed to micronodular disease). That isn’t
necessarily a contraindication
• Sjogrens have a greater risk o f salivary gland damage
• Salivary gland damage is dose related - so cancer treatment patients have a greater risk

What routes does the body use to eliminate f ^ ’?


• Urine is the main way it is eliminated but sweat, tears, saliva, and breast milk are other
routes.

I f they d o n ’t need to be adm itted to the hospital, what precautions should they take?
* There is a whole bunch o f crap they are asked to do. Drink lots o f water (increase renal
excretion). Suck on hard candy (keep radiotracer from jacking your salivary glands).
Patients are encouraged to stay away from people (distance principal). Sleep alone for
3 days (no sex, no kissing - keep that dirty dick in your pants!). Good bathroom
hygiene (flush twice, and sit down if you are a guy). Use disposable utensils and
plates. Clothes and linens should be washed separately. M ost o f these things are done
for 3 days.

Is it ok to breast feed? Is it ok to try and get pregnant?


• No breast feeding. If y ou take your breast feeding days are over
(at least this time around).
• No getting pregnant for at least 6-12 months after therapy

518
Radioiodine Tiierapy - Continued
Other Trivia:
• If you participated in the therapy, you need your thyroid checked
Gamesmanship
24 hours later.
• If the patient got admitted to the hospital the RSO needs to
- Iodine Post Treatment -
inspect the room after discharge before the janitor can clean it or I f you see an Iodine Scan,
the next patient can move in.
and you see u p ta k e in th e
• Thyroglobulin is a lab test to monitor for recurrence. Anything
liv er , this is ALWAYS a post
over zero - after thyroidectomy, is technically abnormal,
although the trend is more important (going up is bad) treatm ent scan.
• Severe uncontrolled thyrotoxicosis and pregnancy are
absolute contraindications.

C lassic S cenario - Patient is on dialysis a n d needs Rx

Give 1131 immediately following dialysis to maxim ize the time the I'^i is on board. Decrease
dose as there is limited (essentially no) excretion until next dialysis. D ialysate can go dow n
sewer. D ialysis tu b in g needs to stay in sto rag e.

Hyperthyroidism:
Why not just treat if they are
I'3i can also be used to treat hyperthyroidism. Dosing depends
experiencing severe thyrotoxicosis ?
on the etiology; 15 mCi for Graves (more vascular), 30 mCi
for multi nodular (harder to treat the capsule). Again the TSH Supposedly you should never treat
must be high for the therapy to be effective. By 3-4 months, severe thyrotoxicosis because you risk a
there should be clinical evidence of resolution of signs and thyroid storm (fever, hypertension,
symptoms of hyperthyroidism, if 1-131 therapy was etc). Most people will say to block
them with meds, and cool them off.
successful.
Having said that, the risk of a true
As an aside, there is no such thing as an “emergent storm is probably exaggerated and you
hyperthyroid treatment.” You can always use meds to cool it will hear people say “fuck it dude, treat
down. The standard medication is Methimazole. However, if them” ... but even the wildest gun
there is an allergy to Methimazole, the patient is having WBC slingers will still give beta blockers to
issues (side effect is neutropenia) or the patient is pregnant reduce the risk of cardiac
complications.
-use propylthiouracil (PTU). PTU is recommended during
pregnancy.
What about Thyroid Eye Disease? It’s controversial, but some people believe that thyroid eye
disease will worsen after 1-131 treatment. If you are prompted, I would just have opthal look at
their eyes, bad outcome is likely severity related. *You might not want to treat a bug-eyed dude
(depends on who you ask).

Wolff-Chaikoff Effect: Since we are talking about hyperthyroid treatment, there is no better time than
to discuss the W.C. effect. Essentially, this is a reduction in thyroid hormone levels caused by ingestion
of a large amount of iodine. The Wolff-Chaikoff effect lasts several days (around 10 days), after which
it is followed by an "escape phenomenon.” The W.C. effect can be used as a treatment principle
against hyperthyroidism (especially thyroid storm) by infusion of a large amount of iodine to suppress
the thyroid gland. The physiology of the W.C. effect also explains why hypothyroidism is sometimes
produced in patients taking several iodine-containing drugs, including amiodarone.

519
SECTION 5:
P a r a t h y r o i d

What Causes Hyperparathyroidism ?

• Most common cause is a hyperfunctional adenoma (85%).


• Second most common cause is multiple gland hyperplasia (12%).
• Third most common cause is cancer (3%).

Nuclear medicine can offer two techniques to localize these lesions; dual phase, and dual tracer.

Dual Phase Technique

In dual phase technique, a single tracer (Tc^^.Sestamibi) is administered, and both early (10
mins) and delayed (3 hours) imaging is performed. The idea is that sestamibi likes things with
lots o f blood flow, and lots of mitochondria. Parathyroid pathology tends to have both o f
these things, so the tracer will be more avid early, and stick around longer (after the tracer
washes out o f normal tissue). SPECT can give you more precise localization.

W w

Initial Phase Delayed Phase


-B oth Thyro id and -Uptal<e R e m a in s in
P arathyroid A b no rm al P arathyroid
(H y p e rp la s ia in this c a s e )

Trivia: Sestamibi parathyroid imaging depends on mitochondrial density and blood flow

False Positives: Caused by things other than parathyroid pathology that like to drink Sestamibi.
• Thyroid Nodules
• Head and N eck Cancers
• Lymphadenopathy

520
Dual Tracer Technique

In dual tracer technique two different agents are used and then subtraction is done. The first
agent is chosen because it goes to both thyroid and parathyroid (options are either Tc^^-
Sestamibi or 201- Thallium Chloride). The second agent is chosen because it only goes to the
thyroid (options are either 1-123 or Pertechnetate). When subtraction is done, anything left hot
could be a parathyroid adenoma.

Problems:
• Mo Tracers, Mo Problems
• Motion: subtraction imaging can’t tolerate much motion
• Stuff Messing with the Thyroid Tracers: recent iodinated contrast, e tc ...

Tc - S estam ib i 1-123 Su b tractio n U ltraso u nd C o rrelatio n

P arathyroid A d e n o m a - S h o w n on D u al T ra c e r M eth od

The parathyroid gland can be surgically implanted into the forearm - typically done with hyperplasia
surgery where they carve out 3 1/2 glands.
Successful surgical treatment of hyperparathyroidism is often defined as intra-operative reduction of
PTH by 50%.
False Positives: Thyroid adenoma (most common), thyroid cancer, parathyroid cancer.
False Negative: Small sized adenoma (most common), 4 gland hyperplasia. A negative study in the
setting of abnormal / suspicious labs should raise concern for these things (multiple gland
hyperplasia or a small adenoma).

Gamesmanship - MIBl - Lymph Nodes

On any study, parathyroid or a heart, if the tracer is M IBI then you should NOT see lymph
nodes. If you see lym ph nodes they are suspicious (maybe cancer). N ext step would be
ultrasound to further evaluate them.

Oh, and don’t forget about focal breast uptake (also cancer), - Breast Specific Gamma
Imaging (BSGI) uses MIBI for a reason.

521
SECTION 6:
CNS

The goal o f brain imaging in nuclear m edicine is to evaluate function (more than anatomy).
Typically you are dealing with SPECT brain (seizures, ischemia), FDG Brain (dementia), and
Cistemogram s / shunt studies.

This is the general idea:

O N L Y F O R B R A IN D E A T H

PLANAR IM AGING

L IP O P H IL IC P E R F U S IO N A G E N T S
^ P R O P O R T IO N A L T O B L O O D F L O W
SPECT IM AGING

PET IM AGING P R O P O R T IO N A L T O M E T A B O L IS M

I want to start out talking about 3 o f the agents that are com m only used in CNS Nukes;
HMPAO, ECD, and DTPA. The focus o f course is on how questions can be asked about
these tracers. First let’s group them:

Extracted (Can be used for Parenchym al Imaging)


HMPAO
ECD
NOT Extracted (N ot used for Parenchym al Imaging - i.e. no SPECT)
DTPA

HMPAO and ECD are very similar, and any time you have sim ilar things, som eone will be
scheming about how to ask a question about the one or two ways they are slightly different.

General Mechanic of Localization - HMPAO I ECD:

Agents (HMPAO & Lipophilic Agents Distribution is Unstable Lipophilic


ECD) are cross the blood proportional to forms convert to a
administered in a brain barrier on regional blood flow Hydrophilic State —
lipophilic state the first pass. in the brain (usually Trapping Agents
highest in the gray
matter).

On the next page I ’m going to com pare and contrast these agents - and their potential uses.

522
Tc HMPAO (hexam ethylpropyleneam ine oxime) and Tc ECD (ethyl cysteinate dimer).
HMPAO and ECD can be used in both dem entia im aging and for seizure focus locahzation.

These studies are typically perform ed with SPECT. The idea behind brain SPECT is that you
can look at brain blood flow, which should m im ic metabolism . These two agents are neutral
and lipophilic, which lets them cross the blood brain barrier and accumulate in the brain.
W here and how much they accumulate should follow flow (and m etabolism).

As I m entioned previously, the two tracers have sim ilarities and differences, and the contrast
between them lends well to multiple choice tests.

HMPAO ECD
Neutral and Lipophilic N eutral and Lipophilic
Accumulate in the cortex proportional to A ccum ulate in the cortex proportional to
blood flow (Gray M atter > White M atter) blood flow (Gray M atter > W hite M atter)
B etter Blood Clearance
(better brain to background ratio)

Preferred in Stroke Imaging

Best im aged 15 min - 2 hour post injection Best im aged 15-30 mins post injection

Uptake favors the parietal and occipital lobes


Uptake favors the frontal lobe, thalamus, and
* M akes com parison between HMPAO and
cerebellum
EC D difficult

K ey points:
• Both agents pass blood brain barrier and stick to gray m atter proportional to CBF
• HMPAO washes out faster
• ECD washout is slower, has better background clearance, and does not dem onstrate
intracerebral redistribution.

T h e O th e r G uy - Tc- DTPA

Unlike HMPAO or ECD, this agent is lipoPHOBIC (hydrophilic) - and is best thought o f as
an “angiographic tracer” because it stays in the blood (or CSF if you put it there).

Key Points:
• DTPA does N O T cross the blood brain barrier and therefore cannot be used for brain
parenchym al imaging. *You can NOT do SPECT
• Has the advantage over HMPAO and ECD in that it can be repeated w ithout delay
• DTPAs main utility is fo r shunt studies, N PH , and Brain Death.

523
S e iz u r e F o c u s :

The goal o f nuclear im aging regarding


seizures is to attempt to localize a
seizure focus (sometimes they do ok if
they cut it out).

The idea is that a seizure focus will


be hot (hypermetabolic and ICTAL INTER-ICTAL

hyperperfiising) during the seizure


“ictal.” Then cold between seizures
“interictal.” You need to inject tracer
(HMPAO or ECD) w ithin 30 seconds
o f the seizure to get a good study.
PET can be used, but is less practical.

Thallium 201

Thallium is produced in a cyclotron, decays via electron capture, and has a h alf life o f
around 3 days (73 hours). The m ajor em issions are via the characteristic x-rays o f its
daughter product M ercury 201 - at 69 keV and 81 keV. The tracer is norm ally given as a
chloride and will therefore rapidly be rem oved from the blood

Thallium behaves like potassium , crossing the cell m em brane by active transport (Na+/
K pump). Tumors and inflam m atory conditions will increase the uptake o f this tracer.
The higher the grade o f tumor, the m ore uptake you get. As Thallium requires active
transport, it can be thought o f as a viability m arker - you need a living cell to transport it.

Normal Distribution: Thyroid, salivary glands, lungs, heart, skeletal muscle, liver, spleen,
bowel, kidneys, and bladder. Any muscle tw itching will turn hot.

If you are going to use it with Gallium, you m ust use the Thallium first as the Gallium
will scatter all over the Thallium peaks.

High Yield Generalizations / Uses:


• Toxoplasma Infection is Thallium N egative
Lymphoma is Thallium Positive
• Kaposi Sarcoma is Thallium Positive (Gallium Negative)
Tumor is Thallium Positive
• Necrosis is Thallium N egative

524
Tumor vs N ecrosis:
The tracers used for SPECT tum or studies are different than
Tumor vs Necrosis
those used for dem entia or seizures. The tum or tracers are
20ITI (more common) and 99017 ^ Sestamibi (less common).
2«iTI is a potassium analog, that enters the cell via the N a/K Thallium Hot,
pump. Inflam matory conditions will increase the uptake o f this Tumor
HMPAO Cold
tracer, but not as much as tumors. The higher the tum or grade,
the more intense the uptake. Thallium can be thought o f as a
marker o f viability, as it w ill localize in living tum or cells, and Thallium Cold,
Necrosis
not necrosis. The control is the scalp (abnormalities will have HMPAO Cold
greater uptake than the scalp). You can use Thallium in
combination with perfusion tracers (HMPAO).

CNS Lymphoma vs T oxoplasm osis:


As discussed in the neuro chapter CNS lym phom a
Typically CNS lymphoma, toxoplasmosis,
vs CNS Toxoplasmosis can be a diagnostic bacterial abscess, cryptococcus infection,
dilemma. Thallium has a role in helping to and tuberculosis are all positive on Ga-67
scintigraphy.
distinguish the two (Toxo Cold, Lymphoma
Hot). Please refer to the neuro chapter fo r CNS lymphoma will be positive on T1-',201
additional discussion.

Brain Death
You are looking for the absence o f intracerebral perfusion to confirm brain death. So that
you don’t keep G randm a around as a piece o f broccoli, you need to have a tourniquet on the
scalp - otherwise you m ight think scalp perfusion is brain perftision and say she’s still alive.
You have to identify tracer in the com m on carotid - otherw ise the study m ust be
repeated. In the setting o f brain death, tracer should stop at the skull base. The hot
nose sign, is seen secondary to perfusion through the external carotid to the m axillary
branches. As a point o f trivia - the hot nose sign cannot be used to call brain death , it is a
“Secondary Sign.” Some institutions will say you have to image the kidneys (to prove
adequate systemic circulation / perfusion) and the injection site (to prove the tracer didn’t
extravasate).

Brain
Death -
Hot Nose
Sign

525
stroke Luxury Perfusion

There is no reason, ever, under any circumstances known to This is a paradoxical finding
man, women, or beast to ever, ever use SPECT to diagnose that describes the vascular
stroke. Having said that, you can look at stroke with SPECT dilation / excessive blood
flow observed
and will therefore likely be asked questions about it.
within in the relatively
avascular infarcted area of
The big take home points are this; the brain.
• Acute Stroke is Cold
• Subacute Stroke can be warm - fr o m luxury perfusion Typically you seen this 48-72
(bloodflow is more than dead cells need) hrs after an ischemic stroke.
So, it is a finding associated
• Chronic Stroke is Cold
with “sub-acute” stroke.

Ischem ia (TIAs)
You can evaluate for cerebrovascular reserve by first giving acetazolamide (Diamox) - which
is a vasodilator, followed by a perfusion tracer. Normally you should get a 3-4x increase in
perfusion. However, in areas which have already maxed out their auto regulatory
vasodilation (those at risk for ischemia) you will see them as relatively hypointense. These
areas o f worsening tracer uptake may benefit from some revascularization therapy.

PRE-DIAMOX POST-DIAMOX
W orsening Uptake = Ischem ia
This might benefit from
revascularization. Next Step = Angio

Bottom Line = Ischemic Tissue Looks W ORSE (relatively) compared to surrounding tissue,
after vasodilation (Diamox / Acetazolamide)

Gamesmanship = Diamox = A cetazolam ide... don’t just remember one.


You don’t know which one they will use.

526
FDG-PET
FDG-PET General Perfusion Patterns
PET can assess perfusion ( ' 5O-H 2O) but
typically it uses >^FDG to assess
Higher Grade Tumor = Higher SUV
metabolism (which is analogous to
Radiation Necrosis = No Uptake (COLD)
perfiision). Renal clearance of '^FDG is
Stroke / Infarct = No Uptake (COLD)
excellent, giving good target to background
Interictal PET= Decreased Uptake at Seizure Focus
pictures. Resolution of PET is superior to
SPECT

It’s important to remember external factors can affect the results; bright lights stimulating the
occipital lobes, high glucose (>200) causes more competition for the tracer and therefore less uptake.

The most common indication for FDG Brain PET is dementia imaging. Because blood flow mimics
metabolism HMPAO, and ECD can also be used for dementia imaging and the patterns of pathology
are the same.

PET Technique For Dementia Workup:

• NO TALKING GRAMPS ! Patient should rest in a dimly lit room (without talking) after the
injection for around 30-45mins. This allows for optimal distribution.
• WAKE UP GRAMPS ! No sleeping during the PET — this increases variability in uptake.
• SIR! PLEASE STOP MASTURBATING! If sedation is needed wait till after the uptake period.

Dementia is discussed in detail in the neuroradiology chapter. Please refer to the masterpiece that is
the neuro chapter in volume 2 for a complete discussion — the below chart is for rapid review
purposes (and because I know how annoying it is to have to go grab another book).

FDG PET - Brain Dementia Patterns

Normal
Dementia with Picks /
Alzheimers Multi Infarct
Lewy Bodies Frontotemporal

“Things look Low posterior


Scattered areas of Low in lateral
symmetric” temporoparietal Low frontal lobe
decreased activity occipital cortex
Basal Ganglia is cortical activity
= to 15%
GREATER than -Identical to
cortex. Parkinson Preservation of
Cerebellum is Dementia the mid posterior
15% LOWER -Posterior cingulate gyrus
than cortex. Cingulate gyrus is (Cingulate
Thalamus the first area Island Sign)
should be equal abnormal
to cortex.

527
CSF Im a g in g

The principle involved in imaging the CSF consists o f intrathecal adm inistration that will
safely follow CSF and remains in the CSF com partm ent until it is absorbed through the
conventional pathways. The most com m on tracer used is - labeled DTPA. So, you
have to do an LP on the dude (it’s intrathecal).

Normal Examination
* Time Zero - You do the LP
* 2-4 hours it ascends and reaches the basal cisterns
• 4 hours - 24 hours it flows around the sylvian fissures and interhemispheric cistern
• At 24 hours it should clear from the basilar cisterns and be over the cerebral
convexities

1 Hr 5 Hr 24 Hr

Abnormal Examination (general principles)


* Tracer in the lateral ventricles
* Failure to clear from the cisterns and localize over the convexities by 24 hours

1 Hr 5 Hr 24 Hr

528
Communicating Hydrocephalus:
Normal pressure hydrocephalus is wet, wacky, and wobbly (incontinent, confused, and ataxic)
clinically, and demonstrates “ventricular enlargement out of proportion to atrophy” on CT.

On scintigraphy you are looking for:


• Early entry (4-6 hours) o f tracer into the lateral ventricles
• Persistence o f tracer in the lateral ventricle > 24 hours
• Delay in Assent to the parasagittal region > 24 hours

NPH-
Persistent
Tracer in the
Ventricles
> 24 Hours

Since radiotracer shouldn’t normally enter the ventricles, a radionuclide cistemogram cannot be used
to distinguish communicating from noncommunicating hydrocephalus. Historically (1930s) you could
tell by injecting the material directly into the lateral ventricles.

TH IS V5' THAT: NPH vs Non Obstructive (Communicating) Hydrocephalus.


NPH will have a normal opening pressure on LP.

CSF Leak:
You can use CSF tracers to localize a leak. The most common sites of CSF leak (fistulas) are between
the cribriform plate and ethmoid sinuses, from the sella turcica into the sphenoid sinus and from the
ridge of the sphenoid to the ear. The study is like a bleeding scan, in that the leak must be active
during the test for you to pick it up.

How is it done? You image around the time the CSF is at the basilar cisterns (1-3 hours) and also
image pledgets (jammed up the nose prior to the exam). You compare tracer in the pledgets to serum
(ratio greater than 1.5 is positive).

Shunt Patency:
There are a bunch of ways to do this. Most commonly, Tc labeled DTPA is used (>"In - labeled DTPA
could also be used). Usually, the tracer is injected straight into the tubing.

• Normal Test will show tracer in the peritoneum - shows distal end is patent.
• You can manually occlude the distal limb to force tracer into the ventricles - shows proximal end
is patent.
If the tracer fails to reflux into the ventricles, or it does but then doesn’t clear, you can think
proximal obstruction
• If there is delayed tracer flow into the peritoneum (> 10 minutes = delayed), this can mean partial
distal obstruction.

529
S E C T IO N 7:
GI N U K E S

Gastric Emptying : Believe it or not, this study is Standard M eal


actually considered the “gold standard” to evaluate gastric
4 oz o f Egg W hites
motor function. The primary indication is typically
gastroparesis (usually in a diabetic). The exam should be 2 Slices o f W hite Bread
performed fasting (at least 4 hours). Some texts say that it Strawberry Jam
should be done in the first 10 days o f the menstrual cycle to Water
prevent hormones from interfering (I’m sure this Tc Sulfur Colloid
recommendation is evidence based). M ost commonly Tc
labeled sulfur colloid is used on a standardized liquid meal,
solid meal (egg whites), or both. Solids are more sensitive, but you can have emptying
problems from liquids only and
normal emptying from solids. Solid Phase Liquid Phase

Gamesmanship: One sneaky way


to test this would be related to the
differences between solid food
and liquid food curves. The main
point is that solids have a “lag
phase” in which the stomach
helps grind up the food into
smaller parts (liquids don’t have
this). Lag Time can be increased Time
in diabetic patients.

Gamesmanship: Another possible question is that “attenuation correction” plays a role in


calculation o f emptying times, as movement from the back o f the stomach to the front can
increase counts due to attenuation.

Pharmacology Trivia:
• Prokinetic drugs (which enhance gastric emptying) metoclopramide (Reglan), tegaserod
(Zelnorm), erythromycin, and domperidone (M otilium) are stopped at least 2 days prior
to the test. - this can cause a false negative exam.
• Opiates (which delay gastric emptying) are stopped 2 days prior to the test. These can
cause a false positive exam.
• Anticholinergic/ Antispasmodic drugs such as Donnatal, Bentyl, Robinul, and Levsin, are
stopped for 2 days prior to the test
• Serotonin receptor antagonists - the classic one being Ondansetron (Zofran) are fine and can
be given prior to the exam.

Esophageal Transit: Used (rarely) in the evaluation o f esophageal motility disorders.


The supposed advantage is the ability to give quantitative information. The patient is made to
fast overnight, then fed Tc-99 sulfur colloid. Dynamic imaging is performed and transit time
and /or residual esophageal activity is measured.

530
Gl Bleeding:
GI bleed scan sensitivity
The goal of a GI bleeding scan is to localize the bleed (not to say there =0.7 ml/min
is one). Bleeding scan is sensitive to GI bleed rates as low as at 0.1 . •• • _
ml/min (Mesenteric angiography requires 1-1.5 ml/min bleeding). Angiogram sensitivity -
1.0 ml/min
First Some Technical Stuff (Very Boring and High Yield)

Before the Tc-99 can be tagged to a RBC {beta chain o f the hemoglobin)', it must first be reduced.
This is accomplished with stannous ion (tin). This is referred to as “tinning.” There are 3 methods:

In Vivo

1. Tin (stannous ion) is injected into the patient


2. Then Tc-99m pertechnetate is injected
3. Tin binds to the hem oglobin then reduces the Tc (which then binds)

A lthough the process is super simple, you only get about 60-80% o f it bound. So you have
a lot o f free Tc and a dirty image (poor target to background). Sometimes it fails m iserably
(via drug interaction - heparinized tubing, or recent IV contrast).The images are too
crappy for cardiac wall m otion studies, but can w ork for GI bleeding.

In Vivo - In Vitro (M odified M ethod)

1. Tin (stannous ion) is injected into the patient


2. A fter 15-30 mins, you pull 3-5 cc o f blood out o f an IV line into a syringe with both
Tc-99m pertechnetate and an anticoagulant
3. It’s then re-injected 10 mins later

This one does a little better, binding close to 85%. D rug interactions (like heparin) are the
m ost com mon cause o f failure.

In Vitro

Blood is w ithdraw n and added to a kit with both Tin (stannous ion) and Tc. It’s then re­
injected. This m ethod works the best (98% binding), but is the m ost expensive.

Image Acquisition: GI bleeding scan is acquired w ith D YNAM IC imaging (as opposed to 5
min static, transm ission, SPECT, or dual tracer protocol). This allows the detection o f
intermittent bleeds and better localization o f the origin o f the bleed.

531
Reading the Study: You are looking for; You can get faked out by a lot of stuff; renal
or bladder excretion (possibly with hydro),
(1) Tracer outside the vascular distribution transplant kidney (classic trick - but again it
won’t move), varices or angiodysplasia (these
shouldn’t move), a penis with blood in it (this
(2) Tracer that M oves like bowel
will look like a penis), hemangioma (this will
(can be antegrade or retrograde) be over the liver or spleen - and not move),
and the last trick - Free Tc in the stomach.
(3) Tracer that Increases Intensity over time
It you see gastric uptake* next look at the
salivary glands and thyroid to confirm it’s
The distal colon is an exception to the rule of free Tc, and not an actual bleed.
mobility — tracer here may not move.

5 min. 30 min.

Norm al Scan - N o Abnorm al Tracer A bnorm al Scan - A bnorm al Tracer


Accumulation. Normal U ptake in Blood accum ulation at the Hepatic Flexure,
Pool, Liver, & Heart. increases over time. SM A territory.

Rapid Review o f Vascular Territories (localization on GI bleed exam helps direct


angiographic therapy to the specific vascular territory — plus it’s an easy w ay to ask a
multiple choice question).

• Celiac: Distal Esophagus, Stomach, 1st part o f the D uodenum


• SMA: 2nd - 4th Parts o f the Duodenum, the rest o f the small and large bow el to the
transverse colon at the level o f the splenic flexure.
• IMA: Distal 1/3 o f the Transverse Colon to the Proxim al Rectum

Alternative (Stone Age) Way o f D oing A Bleeding Scan; Back w hen dinosaurs roam ed the
earth, they used to do bleeding scans with Tc Sulfur Colloid. This had a variety o f
disadvantages: fast clearance (had to do scan in 30 m ins), m ultiple blind spots (the stomach,
splenic flexure, and hepatic flexures - as sulfur colloid goes to the liver and spleen normally).
The only possible advantages are that it requires less prep and has good target to background.

532
M eckel Scan:

The Meckel Diverticulum is a rem nant o f the


omphalomesenteric duct located near the
distal ileum. These things can have ectopic
gastric mucosa and present with painless
bleeding in the pediatric population.
Pertechnetate is used because it is taken up
by gastric mucosal cells. So you are looking
for tracer uptake in the pelvis (usually RLQ)
around the same time as the stomach.

Only about 10-30% o f M eckels diverticulum


w ill have eastric m ucosa (these are the ones
m ost likely to bleed).

Meckel Scan

Here are the Tricks:

* You need to do the study when the patient is NOT bleeding (if they are bleeding -
then do a bleeding scan).

* Pre-Treatment. You can use a bunch o f different stu ff to make the exam better:
0 Pentagastrin - enhances uptake o f pertechnetate by gastric m ucosa (also
stim ulated GI activity)
O H2 Blockers (Cim etidine and Ranitidine) block secretion o f the pertechnetate
out o f the gastric cells m aking it stick around longer.
O Glucagon - slows gastric motility.

* False Positive: Can occur from bowel irritation (recent scope, laxative use)

* False Negative-. R ecent In vivo labeling o f RBCs , Recent Barium Study


(attenuated)

533
HIDA Scan:
Function and integrity of the biliary system can be evaluated by using Tc-99m labeled tracers that mimic
bilirubin’s uptake, transport, and excretion. All the tracers are basically analogs of this iminodiacetic acid
stuff. Trivia: You need higher doses o f tracer if the patient has hyperbilirubinemia

Prep for the test is diet control. You need to have not eaten within four hours (so your gallbladder is ready
to fill), and have eaten within 24 hours (so your gallbladder isn’t so full, it can’t let any tracers in). If you
haven’t eaten for over 24 hours, then CCK can be given. CCK makes the GB contract.
Normally, the liver will have prompt tracer uptake (within 5 minutes), then you will have excretion into
the ducts, then the bowel - pretty much the same time you see the gallbladder. If the gallbladder is sick
(obstructed), it will still not have filled within 60 min. This is the basic idea.

A. A %
1 min 10 min 20 min 60 min V ^
A cute C holecystitis: Almost always (95%) patients with acute cholecystitis have an obstructed cystic
duct. If you can’t get tracer in the gallbladder within 4 hours, this suggests obstruction.

■■
....

1 min
1
10 min 20 min

Rim Sign Cystic Duct Sign:


A curved area of increased activity along the gallbladder
This sign is seen with acute
fossa (hot rim, or pericholecystic hepatic activity sign)
cholecystitis. The sign describes a nub
suggests a more angry gallbladder - (supposedly seen in
of activity in the cystic duct, with the
20% of gangrenous cholecystitis).
remaining duct obstructed.
Mechanism of the Rim Sign: The mechanism is the
result o f inflammation causing regional hepatic
hyperemia, with more radiopharmaceutical being
delivered to this area o f hepatic parenchyma

534
Chronic Cholecystitis: This can be shown two ways; (1) delayed fiUing o f the GB (not
seen at 1 hour, but seen at 4 hours), or (2) w ith a low EF (< 30%) w ith CCK stimulation. A
reduced EF can also be seen in acute acalculous cholecystitis.

Testable Trivia
- The Dose o f CCK: 0.02 m icrogram /kg over 60 mins
A - The Dose o f M orphine 0.02-0.04 m g/kg over 30-60 mins

Biliary Obstruction: The classic w ay to show this is a lack o f visualization o f the


biliary tree - sometimes call a “L/ver Scan S ig n ” It’s caused by back pressure in an
obstructed CBD.

1 min 10 min 2 0 min 6 0 min

Prompt Uptake - With Delayed Excretion (Medication)

Cholestatic jaundice can be drug induced. The classic offenders:

Chlorprom azine, Erythrom ycin, Birth Control (Estrogens), Anabolic


Steroids, and sometimes Statins. This can mimic biliary obstruction.

TH IS vs THAT: Biliary Atresia VS Neonatal Hepatitis


If you see a hepatobiliary scan (HID A) in a kid, for sure this is the indication. Apparently,
these two things are hard to tell apart clinically. I f you see tracer in the bowel it’s hepatitis,
but ju st rem em ber that it m ight be slow so you need super delays (24 hours if necessary). If
you don’t see it in the bowel, you m ight still need to repeat the study if you did n ’t charge up
those hepatocytes with som e phenobarb (up regulates the cytochrome system). In other
words, a lot o f places pre-m edicate with phenobarbital to increase the utility o f the test. If
you operate early (Kasai procedure) they do a lot better, so it’s im portant not to screw this up.

Trivia: Dose o f Phenobarb to prim e the liver = 5 m g/kg x 5 days Technically it’s 2.5 mg/kg
“5 fo r 5 keeps the liver alive ” twice a day - but that
doesn’t rhyme

535
Bile L e a k

You can use HIDA tracer after


traum a or surgery to look for bile
leak. The trick is that you need
delayed images, and look in the right
paracolic gutter / pelvis. You can get
tracer in the gallbladder fossa,
mimicking a gallbladder.

Reappearing Liver Sign - Labeled


bile may track superiorly into the
peri-hepatic space and coat the
surface o f the liver. This can give the
appearance o f paradoxically
r f -r
increasing activity in tlie liver after
an initial decrease in activity from
liver em ptying into the bowel.

HIDA Scan - Rapid Path Summary:


No Bowel Activity, Persistent Blood Pool = Hepatocyte D ysfunction (Hepatitis)
No Bowel Activity, Blood Pool Goes Away N orm ally = Com m on D uct O bstruction
N o G allbladder Activity x 4 hours (or 1 hour + m orphine) = A cute Cholecystitis
Abnormal GB em ptying (EF < 30% ) = Chronic Cholecystitis

Narcotics
Most people will say something like - “hold the morphine for at least 6 hours — or 3 half lives -
prior to the HIDA scan.” Having said that narcotics alone should not prevent you from seeing the
gallbladder. What narcotics can do is delay bowel visualization by triggering the sphincter of oddi
to contract. This can mimic a biliary obstruction.
Don’t get it twisted - morphine is not the devil - it is a tool. Remember you will give morphine to
help promote visualization of the gallbladder in the scenario where there is dumping of the tracer
into the small bowel, but no GB activity seen.
Never inject CCK and morphine within 30 minutes of one another. Why Not ? It would be bad.
Try to imagine all life as you know it stopping instantaneously and every molecule in your
body exploding at the speed of light— Total protonic reversal.

Elevated Bilirubin
Elevated bilirubin (total > 5mg/dl) will increase the number of non-diagnostic/inconclusive and
false negative exams.
Gamesmanship: Increased renal activity can suggest elevated bilirubin
Next Step: Alternative agents DISIDA and BROMIDA are preferred over HIDA in the setting of
high levels of bilirubin

536
Sulfur Colloid Liver Scan

Not frequently done because of the modem invention of CT. Sulfur Colloid tagged with Tc is quickly
eaten by the liver’s reticuloendothelial system. It can be used to see “hot” and “cold” areas in the liver.

Classically, the multiple choice question is Focal Nodular Hyperplasia is Hot on sulfur colloid
(although in reality it’s only hot 30-40% of the time).

Sulfur Colloid Liver Scan

Hepatic Adenom a COLD

FNH 40% HOT, 30% COLD, 30% Neutral

Cavernous H em angiom a COLD (R B C S ca n H O T)

HCC COLD (G allium H O T )

Cholangiocarcinoma COLD

Mets COLD

Abscess COLD (G allium H O T )

Focal Fat C O U y (X e H O T )

Particle size is worth discussing briefly. Particles for this scan need to be 0.1 - 1.0 micrometers. This
is the right size for the liver to eat them. If they are too big the spleen will eat them, and if they are too
small the bone marrow will eat them. Also, realize that if they were too big they would get stuck in the
lungs like a VQ on the first pass through.

Colloid Shift - In a normal sulfur colloid scan, 85% of the colloid is taken up by the liver (10%
spleen, 5% bone marrow). In the setting of diffuse hepatic dysfunction, portal hypertension,
hypersplenism, or bone marrow activation you can see change in uptake - shift to the spleen and bone
marrow. The most specific causes of colloid shift are cirrhosis, diffuse liver mets, diabetes, and blunt
trauma to the spleen.

Diffuse Pulmonary Activity - This is not normal localization of sulfur colloid. This is non-specific
and can be seen with a ton of things (most commonly diffuse liver disease), but the first thing you
should think (on multiple choice) is excess aluminum in the colloid. It can also be seen in primary
pulmonary issues (reflecting phagocytosis by pulmonary macrophages).

Renal Activity on Sulfur Colloid = The most common cause is CHF {maybe due to decreased renal
bloodflow andfiltration pressure). Alternatively, in the setting of renal transplant - this can
indicate rejection {due to colloid entrapment within the fibrin thrombi o f the microvasculature).
Other more rare causes include coxsackie B viral infection, disseminated intravascular coagulopathy,
and thrombotic thrombocytopenic purpura.

537
Hemangioma Scan

You can “prove” a liver lesion is a hem angiom a with a Tc Labeled RBC scan.

Why w ould you do that instead o f using M R l or C T 7 W ell... lots o f reasons.

(1) M aybe you got sucked into an interdim ensional vortex and landed in an alternate reality
where Taco Bell makes better French Fries than M cDonalds, A dolf Hitler cured cancer,
MRI and CT were never invented, and only scintigraphy is available for the
characterization o f hemangiomas.
(2) O r... or., uhhh maybe you uhhh had a stroke and lost access to the part o f your brain that
remembers how to read M R or CT.
(3) Or., maybe you are trying to win a bet.

So, you can see why it w ould be reasonable to ask about this on an interm ediate level exam.

H ow is this study done ? Using Tc labeled RBCs w ith anterior and posterior projection
images. Delayed blood pool is typically done (30 mins - 3 hours).

What i f it is sm all ? “ Small” is actually the m ost com m on reason for a false negative exam.
You need the thing to be at least 1.5cm, otherwise your sensitivity really drops o ff (that’s
what she said) You could try SPECT but first you seriously need to consider w hat you are
doing with your life.

What is the classic look ? You w ant to see m arked HOT on delays, with no real hot spot
on im m ediate flow or im m ediate pool. A ngiosarcom a could be HOT on delays but w ould
also be hot on flow. A partially fibrosed hem angiom a may be a false negative.

Spleen Scan

You can use Tc Sulfur Colloid or heat dam aged Tc labeled RBCs to localize to the spleen. A
possible indication m ight be hunting ectopic spleen.

Fatty Changes

A fatty liver can alter the distribution o f several tracer. Examples:

• Reduced uptake in the Liver on Tc Sulfur Colloid - including the possible reversal o f
normal liver > spleen uptake pattern (Colloid Shift).
• There will be increased uptake in a fatty liver w ith X enon 133.
• Trivia - FDG-PET uptake in the liver is not altered by background steatosis.

538
SECTION 8:
GU N U K E S

Imaging o f GU system in nuclear medicine can evaluate function (prim ary role), or it can
evaluate structure.

Function (Dynamic):

Normal kidney function is 80% secretion and 20% filtration. Tracer choice is based on which
o f these param eters you w ant to look at.

Tc-DTPA: Alm ost all filtered and therefore a great agent for determ ining GFR. A piece o f
trivia is that since a small (5%) portion o f DTPA is protein bound (and not filtered) you are
slightly underestim ating GFR. Critical organ is the bladder.

Tc-MAG 3: This agent is alm ost exclusively secreted and therefore estimates effective renal
plasma flow (ERPF). It is cleared by the proxim al tubules. Critical organ is the bladder.

Tc-GH (glucoheptonate): This agent can be used for structural im aging (discussed later in this
section), or functional imaging as it is filtered. Cridcal organ is the bladder.

Tc DTPA Tc MAG 3 TcGH


Filtered (GFR) Secreted (ERPF) Filtered
Good For Native Kidneys with Concentrated better by kidneys Good for dynamic and cortical
Normal Renal Function with poor renal function imaging.
Critical Organ Bladder Critical Organ Bladder Critical Organ Bladder

There are essendally 5 indications for dynam ic (functional) scanning: (1) Differendal
Funcdon (2) Suspected Obstruction, (3) Suspected Renal Artery Stenosis, (4) Suspected
Com plicadon from Rental Transplant, (5) Suspected Urine Leak.

Some basics:

Images are obtained posteriorly (anterior if patient has a transplant or horseshoe).


Typically dynamic exams have 3 phases: blood flow phase, cortical phase, and clearance
phase.

539
Differential Function

This is a basic exam with the standard flow, cortical, and clearance phases.

Flow: Begins w ithin 20 seconds o f injection. Flow will first be seen in the aorta. Then as it
reaches the renal arteries, the kidneys should enhance sym m etrically and about equal to the
aorta (at that time).

Flow

D ecreased (symmetric) Technical Error - poor bolus

Renal Artery Thrombosis


Renal Vein Throm bosis
Decreased (asymmetric) Chronic High Grade Obstruction
Acute Rejection
Acute Pyelonephritis

Increased (asymmetric) Renal Artery Aneurysm

An important piece o f trivia is that ATN, Interstitial Nephritis, and Cyclosporin toxicity
will all have norm al perfusion/flow.

Cortical (parenchymal): This is the m ost im portant portion o f the exam (with regard to
differential function). An area o f interest is drawn around the kidneys and a background area
o f interest is also draw n (to correct for the background). This can be screwed up by drawing
your background against the liver or spleen (which is not true background since they will take
up some tracer). You want to measure this at a time w hen the kidney is really drinking that
contrast, but not so late that it is putting it in the collecting system. M ost places use around 1
min. A steep slope is good.

Clearance (excretory): Radiotracer will begin to enter the renal pelvis, collecting system, and
bladder. In a normal patient, you will be down to h alf peak counts at around 7-10 mins. If
you w anted to quantify retention o f tracer you could look at a 20/3 or 20/peak ratio.

20/3 or 20/peak ratio: This is a m ethod o f quantifying retention o f radiotracer by com paring
the peak count at 20 m inutes with the peak count at 3 mins (norm al < 0.8) or the peak count
(normal 0.3).

540
Suspected Obstruction (“The Lasix Renogram”)

The exam is perform ed the same as a standard dynam ic exam (blood flow, cortical, and
clearance), with a 30 minute w ait after clearance. If there is still activity in the collecting
system, a challenge is performed with Lasix. The idea is that a true obstruction will NOT
respond to Lasix, whereas a dilated system will em pty when overloaded by Lasix. The study
can be done with MAG-3 or DTPA. MAG-3 does better w ith patients w ith poor renal
function, and thus is used m ore commonly.

The exam is interpreted as follows:


* No obstruction = tracer clears from
collecting system w ithout need for
Lasix
• No obstruction = W ashout o f 50% o f
the tracer w ithin 10 m inutes o f Lasix
adm inistration
• Indeterminate = W ashout o f 50% o f
the tracer w ithin 10-20 minutes o f
Lasix administration
o The m ost common cause for
this indeterminate result is a
very dilated pelvis and
subsequent “reservoir effect.”
* Obstructed = W ashout taking longer
than 20 mins after Lasix adm inistration

Source o f False Positive for Obstruction:


• Poor response to Lasix - secondary to bad renal function, or dehydration at baseline
• “Reservoir Effect” - very dilated renal pelvis, delaying transit time
• Back Pressure Effects - Full or Neurogenic Bladder can generate back pressure and
not let the kidneys empty (can be resolved with a foley catheter).

541
Suspected Renal Artery Stenosis

The study can be perform ed in one


o f two ways (both using M AG 3
as the typical tracer). The first is
a standard dynamic study,
followed by ACE inhibitor. The
second is a baseline study with 'A
dose, followed by a full dose o f
ACE inhibitor. A “norm al
O
study” will occur if there is no
difference between the baseline
N orm al Exam - N o D ifference Betw een P re a n d P o s t
and the captopril studies.

The appearance o f RAS will vary depending on the tracer given:

• DTPA - Rem em ber


this is a GFR tracer. A
sick kidney will have
decreased uptake
and flow, because o f
loss o f perfusion
pressure.

POST-
M AG 3 - Rem ember T '
this is a Secreted tracer. S ick

i 1 r" ...--■« r-— —1 r - _ ~ i r— — i


marked tracer
retention, with a curve
• • ( *0 0 II 0 10 1
sim ilar to obstruction.
o - # o • m
M A G 3 - The S ick K id n e y (Holds onto the Tracer

I f it’s bilateral up or bilateral down, it’s not RAS. I f the baseline study has asym m etrically
poor function, that isn ’t positive for RAS, you need to see it w orsen (> 10%).

Trivia related to A C E inhibitor administration: they need to stop their ACE inhibitor prior to
the renal study (3-5 days if captopril). They should be NPO for 6 hours prior to the test (for
PO ACE inhibitors).

542
Suspected C om plication from R enal T ra n sp lan t

ATN vs Rejection: The most com mon indication for nuclear m edicine in the setting o f renal
transplant is to differentiate rejection from ATN. ATN is usually in the first w eek after
transplant, and is more common in cadaveric donors. There will be preserved renal perfusion
with delayed excretion in the renal parenchym a (elevated 20/3 ratio, delayed time to peak).
ATN usually gets better. There is an exception to this rule, but it will confuse the issue with
respect to multiple choice, so I’m not going to m ention it. Cyclosporin toxicity can also look
like ATN (normal perfusion, with retained tracer) but will N OT be seen in the im mediate post
op period. Rejection w ill have poor perfusion, and delayed excretion. A chronically rejected
kidney w on’t really take up
the tracer.
Immediate Post
Perfusion Excretion
ATN OP (3-4 days
Normal Delayed
post op)
In a N orm al DMSA or A T N
(with M AG 3 tracer), the Cyclosporin Perfusion Excretion
Long Standing
Toxicity Normal Delayed
nephrographic appearance
is the same. Tracer in the
Acute Immediate Post Poor Excretion
cortex, and th a t’s it. Rejection OP Perfusion Delayed

\ '"1
■I m V#

9 W ■

0 0 m 0
I »

Rejection
ATN
(flow sucks)

543
S uspected C om plication from Renal T ra n s p lan t C ont...

Fluid Collections: Fluid collections seen after a transplant include urinom as, hematomas,
and lymphoceles. All 3 can cause photopenic areas on blood pool imaging.

• Urinoma: U sually found in the first 2 weeks post op. Delayed im aging will show
tracer between the bladder and transplanted kidney. The prim ary differential in this
time period is the hematoma. A hem atom a is not going to have tracer in it.

O Urinoma = Hot w ith Tracer


0 Hem atom a = No Tracer

* Lymphocele: Usually found 4-8 weeks after surgery. The cause is a disruption o f
normal lymphatic channels during perivascular dissection. M ost are incidental and
don’t need intervention. I f they get huge, they can cause mass effect. This will look
like a photopenic area on the scan.

Hematoma Urinoma Lymphocele

Timing Early (< Month) Early (< Month) Late (> Month)

Blood Pool COLD - No Tracer COLD - No Tracer COLD - No Tracer

Delay COLD - No Tracer HOT with Tracer COLD - No Tracer

Vascular Com plications: Both arterial and venous throm bosis will result in no flow or
function. I f you suspect renal artery stenosis (m ost com m on at the anastom osis), you can do
a captopril study, with results sim ilar to RAS if there is a stricture.

544
structure

If you want to look at the renal cortex, you will w ant to use an agent that binds to the renal
cortex (via a sulfhydryl group). You have two m ain options with regard to tracer.

•Tc-DMSA: This is the more com monly used tracer. It binds to the renal cortex and is
cleared very slowly. C ritic a l o rg an is th e kid n ey (notice other renal tracers have the
bladder as their critical organ).

•Tc G H (glucoheptonate): This is less com m only used and although it binds to the
cortex, it is also filtered and therefore can be used to assess renal flow, the collecting
system, and the bladder. Critical organ is the bladder.

DMSA is the preferred cortical imaging agent in pediatrics, because it has a lower dose to the
gonads (even though its renal dose is higher than TcGH).

Indications for the exam:

A cute Pyelonephritis: Can appear as (a) fo c a l ill-defined area o f decreased uptake,


(b) m ultifocal areas o f decreased uptake, (c) diffuse decreased uptake - in an
otherwise norm al kidney.

o Scarring a n d M asses can also appear as fo c a l areas o f decreased uptake —


although scarring usually has volume loss.

* Column o f Bertin vs M ass: Simply put, the mass will be cold. The Colum n o f Bertin
(normal tissue) will be take up tracer.

The trick on DM SA is ju st like reading CX R or Chest CT - you need to know if it is acute


or chronic. The clinical history changes your DDx.

• Defects on DM SA with acute renal problem s = pyelo.


• D efects on D M SA with chronic renal problem s = scar (or mass).

545
Testicular
This hasn’t been done in the United States since 1968, therefore it is very likely to be on the
test. The study is basically a blood flow study. The prim ary clinical question is testicular
torsion ys other causes o f pain (epididymitis). The tracer used is sodium pertechnetate
(Na99mTc04). Oh, don’t forget to tape the penis out o f the w ay - tape it up, not down like is
required for the male residents on m am m ography rotations.

* Normal - Symmetric low level flow to the testicles.

* Acute (early) Torsion - Focal absence o f flow to the affected side (“nubbin sign”).

* Delayed (late) Torsion - Sometimes called a m issed torsion. The appearance is a


halo o f increased activity, with central photopenia.

* Testicular Abscess - Identical to delayed torsion - halo o f increased activity, with


central photopenia.

* Acute Epididym itis- Increased flow and blood pool to the affected side.

546
S E C T IO N 9:
_ P E T FOR C a n c e r
As mentioned before, 18-FDG is cyclotron produced and decays via beta positive emission to 18-0.
The positron gets emitted, travels a short distance, then collides with an electron producing two 511
keV photons which go off in opposite directions. The scanner is a ring and when the two photons
land 180 degrees apart at the same time the computer does math (which computers are good at) to
localize the origin.
A CT component is fused over the PET portion. This is done for two reasons:
(1) Anatomy - so you can see what the hell you are looking at.
(2) Attenuation Correction - Dense stuff will slow down the photons, and the CT allows for
correction of that. It also leads to errors, the classic one being a metallic pacemaker looks
bright hot on the corrected image (the computer overcorrected). This is a classic question.
The answer is look at the source images (uncorrected).
Some other technical trivia is that FDG enters the cell via a GLUT 1 transporter and is then
phosphorylated by hexokinase to FDG-6-Phosphate. This locks it in the cell. Normal bio distribution
is brain, heart, liver, spleen, GI, blood pool, salivary glands, and testes. The collecting system and
bladder (critical organ) will also be full of it, because that’s where it’s getting excreted.

When do you image? Following therapy; waiting 2-3 weeks for chemotherapy and 8-12 weeks
for radiation is typical. This avoids “stunning” - false negatives & inflammatory induced false positive.

Who do you image? The main utility is extent of disease, and distal metastatic spread. Local
invasion is tricky with a lot of things. Usually straight up CT or MRI is better for local invasion and
characterization. Doing a PET prior to biopsy (to help you select the best site) is also a good idea.

SUV: This is the standardized uptake value used I


to describe how much FDG something drinks (how j SUV- C o n c e n tra tio n at tim e T
metabolically active it is). The value is prone to all ~
kinds of issues — I talk more about this in the i (D o s e / B ody W e ig h t)
Radiology Physics War Machine. I

Values > 2.0 (some people say 2.5) are generally abnormal. This can be cancer or infection /
inflammation. Usually the higher the number the more aggressive the tumor. Lesions smaller than
1cm (some people say 7mm) are often too small to determine an SUV.

Insulin and Blood Glucose:


• High Blood Glucose (> 150-200): The more glucose the patient has, the more competition is
created for the FDG and you will have artificially low SUVs.
• Insulin: So why not just give the patient some insulin??? It will drive it all into the muscles. This
is a classic trick. PET with diffuse muscle uptake = Insulin Administration

Variable areas of uptake:


• Muscles (classic forearm muscle uptake in the nervous chair squeezing patient).
• Breast and ovaries in females at certain stages of the menstrual cycle.
• Thymus in younger patients (< 13) - and after chemotherapy “thymic rebound / hyperplasia”
• Lastly, brown fat around the neck, thorax, and adrenals (especially in a cold room).

547
Brown Fat /Brown Adipose Tissue (BAT):
You may remember from med school physiology that brown fat is something infants have to help
them keep warm. Although you lose most of it as you grow up, people do still have small amounts
( women > m en). The distribution is most common at the base of the neck / shoulders. It can also be
seen in the paraspinal fat along the thoracic spine. Both locations have a classic (testable)
appearance.
Brown Fat is More Common with - cold rooms and anxious patients.
Reducing Uptake From Brown Fat:
(1) Keep the room warm.
(2) Medications like propranolol (beta blocker), or reserpine - are classic. Diazepam is probably less
effective according to the literature — some people still give it.
(3) Diet: High fat and very low carb preparation diet the night before and morning of the PET
Hibernoma: This is a benign brown fat tumor - classically found at the shoulders and knees. They
tend to be very hot on PET, but have a characteristic well circumscribed appearance on CT with a
density between fat and muscle. Liposarcoma is the primary differential, so they are typically cut out.

Consequences of being fat:


• Fat people will have HIGHER SUV values, because the fat takes up less glucose
• You leave the house in high heels... and come back home in flip flops
• You go camping., and the bears have to hide their food

Ki67 Proliferation index: This isn’t something you see on a scan, but it is something that could be
used in a question stem - so it is good to know what it is. Ki67 is an antigen associated with cellular
proliferation. You’ve probably heard the pathologist ramble on about it in tumor board (while pointing at
blobs of red and blue dots). More Ki67 = More Aggressive Tumor

Breast Cancer: Inject FDG in the opposite side of the primarv breast cancer. Scan them supine
with their arms up. Screening with FDG is usually not done - it is very non-specific (so many false
positives). The most testable legit indication is probably a patient who needs an MRI for problem
solving - but can’t get one because of some contraindication (metal in the eyes, pacemaker, etc..).
Obviously you can (and often do) use it for staging and treatment response.

Tumors that are PET COLD Not Cancer but PET HOT

BAG (Adeno In Situ) - Lung Cancer Infection

Carcinoid / Neuroendocrine (Low/Intermediate Grade) Inflammation

RCC Ovaries in Follicular Phase

Peritoneal Bowel/Liver Implants Muscles

Anything Mucinous Brown Fat

Prostate Thymus

548
Special Situations - FDG PET Cancer Trivia
• Focal Thyroid Uptake - Requires Further Workup - might be cancer, might be nothing

• Diffuse Thyroid Uptake - Most often Autoimmune (Hashimoto) Thyroiditis

• RCC are COLD (usually), Oncocytomas are HOT

• COLD Ground Glass Nodule = Cancer, HOT Glass Nodule = Infection

• HCC is often cold (60%) - it has variable glucose-6- phosphatase and can’t trap the FDG

• Testicular cancers skip right to the retroperitoneum. The trivia is the seminomatous CA is FDG
hot, whereas non-seminomatous tends to be FDG COLD (or Luke Warm).

• While it’s OK for ovaries to be hot in functional (ovulating) young people. Grandma should
NOT have FDG up-take in her dried-up raisin ovaries. This is suspicious - next step Ultrasound.

• Metformin is classic for causing false positive bowel uptake. The uptake is typically intense and
diffuse. It tends to favor the colon (small bowel to a lesser extent). Most people suggest holding
it for 48 hours.

• Large Bowel: An isolated hot spot in the colon = colonoscopy. Villous adenomas and colon
cancers > 1cm can be FDG Avid.

• Neuroendocrine Tumors: If you see a neuroendocrine tumor on PET, it tends to be aggressive.

• Medullary Thyroid Cancer: The more aggressive the tumor, the more useful PET CT is. You can
trend this with calcitonin. If the calcitonin is > 1000 PET is sensitive, < 500 its usually shit.

• Pituitary Adenomas (benign ones) can be very hot on PET. These require a hormonal workup
(prolactin, etc..) and dedicated MRI.

• Adrenal Glands - mild uptake is normal. In general, an adrenal pathology is typically considered
malignant if the uptake is higher than the liver (used as an internal control). Although this does
not really hold up that well in my experience (especially since liver uptake is variable). For
example. Adrenal Adenomas can have moderate uptake (still usually not super hot). Comparison
to CT (HU < 10) is the best move to distinguish. Be careful with the lung cancers — remember
they like to met to the adrenals.

• Right Ventricle - The RV is not typically seen on PET unless it’s enlarged.

• Sarcoid - Mediastinal nodes will take up F-18 FDG in most cases of cardiac sarcoid. These can
be used for diagnostic biopsy (primary or after a failed endomyocardial biopsy).

• Lymphoma
• Most Lymphoma are Super Hot.

• Thymic Rebound vi’ Recurrent Lymphoma: Thymic Hyperplasia (or Rebound) can be “Warm”
on PET, but recurrent Lymphoma should be HOT. Also, Rebound tends to maintain the normal
thymus look (it sorta drapes over the heart). Lymphoma is round like a ball. A hot ball of
death, or a “great ball of fire”

• Extranodal marginal zone lymphomas, including the mucosa-associated lymphoid tissue


(MALT) marginal zone lymphoma, have been shown to have LOW Avidity for FDG.

549
S p ecial S itu atio n s - FDG PET C a n c e r Trivia
Gynecologic and Genitourinary

GYN PET

False Positives False Negatives

Physiologic FDG in the endometrium.


Two described peaks of FDG
Necrotic, Mucinous, Cystic, or
(1) Rag Week / IVIenstruation (first 4 days of
Low-grade Cancers — you gotta correlate with
cycle) and (2) Ovulation (~ day 14) and
US and/or MRI to avoid looking stupid.
menstruation may mimic disease.

Physiological uptake in the endometrium is 5 0% of RCC may not be FDG avid


usually diffuse. Cancer is usually more focal.

You w on’t see this in postmenopausal women

PhvsioloQic FDG in the ovaries. Small-volume peritoneal disease and small


Seen during ovulation lymph nodes — can be missed because they
are small, can also be missed because the
Typically appears as ovoid or a rim of activity adjacent bowel (which has physiological uptake)
with a photopenic center. masks them

You w on’t see this in postmenopausal women

Endometrial or ovarian FDG activity in the


postmenopausal = suspicious (next step
ultrasound)

Benign lesions:
• Uterine Fibroids
• Benign Endometriotic Cysts

Misregistration: Focal uptake in the ureter or the Perivesical disease - can be missed because of
bladder - if mapped incorrectly can simulate the high uptake in the adjacent bladder - look
disease. at source images (uncorrected)

Main strategy to reduce this artifact is to


minimize the time between the PET and CT
acquisitions

Vesicovaginal fistulas (common in advanced


disease) — can falsely elevate SUV values by
spilling urinary FDG excretion

550
S p ecial S itu atio n s - FDG PET C a n c e r T rivia
Osteosarcoma and Bone iVlisc...
• Rem ember secondary osteosarcomas (the ones from Pagets. Radiation, M ultiple
Chondromas, e tc ....) are the bad m other fuckers. They have by far the w orst outcome.

• Back in the stone ages - if you got diagnosed w ith an osteosarcom a they assum ed that
80% o f the time you had distal mets. This was prior to effective chem otherapy for
osteosarcom a... it was basically a death sentence. They do better now (little bit).

• Typically mets go to (a) second bone sites, and (b) the lungs. Rem em ber the classic
vignette o f a spontaneous pneum othorax in an osteosarcom a patient = lung met.

• With the m odem addition o f effective chemo, accurate staging and restaging actually
m atters - and this is w hy I’m ram bling on about osteosarcom a. PET m ay/could
actually play a key role in treatm ent and is therefore testable.

• Average SUV is used at some institutions for describing tum or metabolism. M ost
literature says you should use the SUV M ax for describing osteosarcom a - because it’s
a very heterogenous tumor.

• Multiple papers have shown a correlation betw een tum or grade and FDG uptake (SUV
values). H igher SUV M ax = Higher Tumor Grade.

• Baseline SUV Max is an independent and significant predictor o f overall survival


(more = bad). H aving said that P E T is not typically used fo r baseline staging.

• An im portant point is that especially with bone stuff, nukes is highly non-specific.
Rem ember earlier in the chapter I said several B9 bones lesions are hot on Tc-MDP?
Same thing with PET. Giant Cell is the classic exam ple. Lots o f cases o f GCTs have
higher SUV than sarcomas. Fibrous dysplasia is another classic that can be hot on
PET.

• There is no cutoff to tell the difference betw een a B9 bone lesion and a bone sarcoma.
Same deal with infection. Osteom yelitis can have very high FDG uptake.

• Important point = Because PET is so nonspecific, it is N O T used clinically to avoid


bio p sy .... but can sometimes guide biopsy.

• The most im portant prognostic factor = response to preoperative (neoadjuvant)


chemotherapy.

• There is a proven positive correlation o f FDG uptake and viable tum or tissue. In other
words, F'8-FDG PET can be used to evaluate the effectiveness o f neoadjuvant
chem otherapy.

• Rheum atoid A rthritis - Symmetric periarticular distribution suggests RA

551
S E C T IO N 10:
N o n -p e t for C a n c e r

In'!11 Octreoscan

>>>In Pentetreotide is the most com m only used agent for som atostatin receptor imaging. The
classic use is for carcinoid tumors, gastrinomas, paragangliom as, merkel cell tumors,
lymphoma, small cell lung cancer, m edullary thyroid cancer, and meningiomas.

Indiumin
Indium is produced in a cyclotron and decays with a 67 hour h alf life via electron capture.
It produces tw o p h o to p eak s at 173 keV and 247 keV. Just like Gallium, Inm in a liquid
will carry a +3 valence and behaves like Fe+s, with the capability o f forming strong bonds
with transferrin.

The most com mon application is to bind it to W BC,


although it can also be hooked to octreotide, or DTPA
for CNS imaging (cisternography). Basically, you can
hook indium to almost anything if you hook it first to
a strong chelator like DTPA. As a point o f trivia, you
need to isolate the W BCs prior to labeling because the
transferrin in the blood binds with greater affinity and
will out-com pete them.

Trivia: Meningiomas take up Octreotide (and Tc MDP).


As a point o f trivia, there are 5 som atostatin receptors but " 'I n Pentetreotide can only bind to
two o f them. The scan works because 80% o f neuroendocrine tum ors express these two
receptors.

Normal uptake is in the thyroid, liver,


gallbladder, spleen, kidneys, bladder, and
% %
GI tract. Imaging is done in early and
delayed phase. The advantage o f the early
phase (4 hours) is that the bowel activity is
absent. The delayed is done to clarify that
a. ».
the abdominal tracer is o f GI origin.
Meningioma - Hot on Octreotide and Tc MDP

552
MIBG

MIBG is an analog o f noradrenalin and is therefore taken up by adrenergic tissue. M IBG is


first line for tumors like pheochrom ocytom a, paragangliom a, and neuroblastoma. You can
have MIBG with either 1-123 or I -131. 1-123 is better because it has better imaging quahty.
1-131 is cheaper, and the long h alf life allows for delayed imaging.

Blocking the Thyroid Gland: The thyroid gland should be blocked, to prevent unintended
radiation to the gland from unbound 1-123 or 1-131. This is accom plished with Lugol’s Iodine
or Perchlorate. Sometimes y o u ’ll see ''SSKT' which is 5uper Saturated Potassium /odine

Biodistribution: Norm al in liver, spleen, colon, salivary glands. The adrenals m ay be faintly
visible. Note the kidneys are N OT seen.

Trivia: M IB G is s u p e rio r to M D P bone scan fo r n eu ro b la sto m a bone m ets. *Ifyou see a


skeleton on MIBG the answer is diffuse bone mets.

Trivia: M IBG is better for Pheo than In-111 Octreotide. M IBG is better than CT or MRI for
the extra-adenral Pheos.

Medication interaction with MIBG - High Yield Trivia

Certain medications interfere with the workings o f M IBG and must be held.
Medications include calcium -channel blockers, labetalol (other beta-blockers have no
effect), reserpine, tricychc antidepressants and sympathomim etics.

TH IS V5THAT: MIBG Scans

W h a t’s D ifferen t B etw e en


P atien t “A ” and “B” - w ho
both recen tly got IVIIBG
S c an s ?

P atien t B - Forgot to take


her Lugol’s !

So, s h e ju st co o k e d her
thyroid.

553
Gamesmanship - MIBG

Brown Fat = Just like you can see it on FDG PET (around the shoulders / traps) you can also
see it with MIBG. In fact, it’s the M OST COM M ON variant in I>23 M IBG bio distribution.

M echanism? Apparently brow n fat has sympathetic innervation.

Utility: The classic use for M IBG is Neuroblastom a. B u t.... if someone w anted to get sneaky
they could also show you a pheochrom ocytom a, paragangliom a, or carcinoid on MIBG.
Essentially any catecholamine producing tumor. See the chart on the next page for specifics.

Gamesmanship - Octreotide:

Octreotide M eclianistn Nitty Gritty: Octreotide works as an analog to somatostatin and will
bind to the same receptors as somastostain. W ell... sorta. Technically, there are 5 subtypes o f
somatostatin receptors and Octreotide will only bind to 2 o f them. As you can imagine, the
sensitivity o f the agent is going to depend on which receptors are expressed by the tumor.
Fortunately, m ost tumors express those 2 receptors.

Suspected Insulinoma: Some sources will say that Octreotide can trigger hypoglycem ia in
the setting o f an insulinoma. W hat to do about this seems to vary a lot on w ho you ask and
what you read. Some places will say to give them some IV solution with glucose before and
during the adm inistration o f Octreotide. O ther places will ju st say have D50 ready ju st in case.

What should yo u do i f both o f those options are choices on the exam? Simply read the m ind o f
the person who w rote the question and choose the correct answer.

Already on Octreotide Therapy: You will need to stop the treatm ent for 3 days prior to
giving the labeled agent.

554
M IB G Octreotide

Octreotide and MIBG are


inferior Superior
Carcinoid similar for detection of Liver
(Sensitivity 50-70% ) (Sensitivity 80-90% )
Mets

SH IT
Insulinoma Insulinoma is usually B9
(Sensitivity ~ 30% )

Good Gastrinoma is usually


Gastrinoma
(Sensitivity ~ 85% ) Malignant

Crap Crap
Non-Functional
(No Function, (No Function, FDG PET is the test of choice.
Islet Cell Tumor
No Receptors) No Receptors)

Superior For Non- Superior For Malignant FDG is also good - but
iVlaiignant (Adrenal) (Extra-Adrenal) Types Octreotide is cheaper - so
Pheo
Types (which is lil<e (which is a minority of people go that route first when
90% of them) them 10%) malignancy is suspected

Less Sensitive and


Less Specific (*on!y Superior
Paraganglioma
concentrates in the (Sensitivity ~ 95% )
functional subtypes)

Octreotide is still the first line


as a nukes agent. If it's
Medullary Inferior Superior -
negative (in the setting of
Thyroid CA (Sensitivity 30 %) (Sensitivity 50-80% )
rising calcitonin) the next step
is F D G -P E T

Superior Inferior
Neuroblastoma
(Sensitivity ~ 95% ) (Sensitivity ~ 64% )

555
Gallium

Gallium can be used for tumors. Remember, it’s very nonspecific with regards to infection,
inflammation, or tumor.

Trivia: During the Cretaceous Period (when many o f your attendings trained); G allium was
used to differentiate residual tum or vs fibrosis, scarring, necrosis e tc ... in patients with
H odgkin’s disease and M alignant Lymphoma

Trivia: If you are going to use Gallium for treatm ent m onitoring it’s im portant to have a pre­
treatment exam (to ensure that the tum or site is actually Gallium - avid).

ProstaScint

'In can be labeled to the antibody Capromab Pendetide (ProstaScint). Caprom ab is a


m onoclonal antibody which recognizes PSA m em brane antigen “PSM A ” - w hich is slightly
different that PSA. Pendetide is the chelating agent. In my opinion, the exam is shit and
doesn’t w ork well.

When do you do the test? If you have a rising PSA and negative bone scan. The purpose o f the
study is to look for mets outside the prostate bed (soft tissue mets). If they do not have distal
mets, they can be offered salvage therapy (radiation to the surgical bed). It’s im portant to not
obsess over the surgical bed, the real question is distal mets. H aving said that, the prostate bed
is best seen on the lateral between the bladder and penis.

Testable Trivia: ProstaScint will localize to soft tissue mets, NOT bone mets.

Testable Trivia: ProstaScint’s critical organ is the LIVER.

This confuses some people. Avoid this trap: Indium = WBC. It’s slang to say “Indium Scan”
and people in N ukes ju st think tagged W BC w ith Indium. B u t.... N ot all Indium is W BC.
Rem ember that Octreotide is actually labeled with Indium, and so is a bunch o f other stuff.

Bottom Line: F or Critical Organ Trivia


Indium ProstaScint = Liver
Indium W BC = Spleen
Indium Octreotide = Spleen

556
Sentinel Node Detection
A sentinel node is the node which receives afferent drainage directly from a prim ary cancer.
Surgeons w ant to know where these are at; especially w ith m elanom a and breast cancer. The
agent used for lym phoscintigraphy is 10-50 nm Tc99m su lfu r colloid.

Melanoma: Sentinel node mapping is done w hen you have a lesion betw een 1 m m -4 m m deep.
Less than 1 mm you are typically safe. M ore than 4 m m you are totally screw ed and it makes
no difference. The utility o f the test is to go after the ones in that m iddle zone (between
1-4 mm). Intradermal injection in 4 spots around the lesion / excision scar and im aging is done.

Breast Cancer: Cancer drains to the internal m am m ary chain nodes about 3% o f the time.
Knowing this, and w hich axillary node to go for first, can help avoid aggressive lym ph node
dissection. Injections can be done superficial or deep (into the pectoral muscle).

Size Matters

Does particle size m atter fo r sentinel node detection? Yes and No.

The idea is that particles have to be small enough to travel through the lymphatics. A
particle that is 500 nm will not go anywhere. So any answ er choice with 500 nm or larger
for a lymph node study is universally incorrect. N ow this is where you will read different
stuff (and send me nasty emails). Some sources say < 200 nm. This will probably work,
but you will be waiting all day for the study. The larger the particles the slow er the study.
More sources will say < 100 nm. This will work, but again slow study. There is at least
one paper out that advocates for using a 0.22 mm filter (available in most hospital
pharm acies because it is com monly used for sterilization o f hyperalim entation solutions).
If you do that you end up with particles that are 10-100 nm. If you use this 0.22 mm filter
m ethod you can routinely visualize lym phatic channels and sentinel nodes w ithin 30 mins
after administration.

Gamesmanship - H ow do yo u know w hat the question w riter does at his/her institution?


If it was me - 1 w ould pick a choice less than 100 nm. I f forced to choose between 100 and
5 0 - 1 would pick 50, and there is literature to back you up (not that y o u ’d have a chance to
defend yourself), but practically m ost places do filter the particles and use small ones.

Test Particle Size

Lymphoscintigraphy < 0.2 microns (< 200 nm)

VQ 10-100 microns (10,000 - 100,000 nm)

Liver Spleen “Unfiltered” - so all sizes big and small

557
Breast Specific Gamma Imaging

Tc99 Sestamibi will concentrate in a breast cancer 6 times more than normal background
breast tissue. It does pretty well, with the sensitivity supposedly near 90%. The technique is
to give 20-30 mCi o f Tc99 Sestamibi in the contralateral arm then image 20 mins later. A
foot injection is often done if you are going to image both breasts. You are supposed to use a
dedicated gam ma cam era that can mimic a m am m ogram and provide compression.

D oes Breast D ensity A ffect Uptake / D istribution ?

Nope. The distribution is homogeneous regardless of density. Having said that, hormonal fluctuation
can increase the background uptake.

When will background activity be lowest?

Around mid-cycle in prem enopausal women.

What are som e causes o f false positive studies ?

Fibroadenom a, fibrocystic change, or inflam m ation can give a false positive

What are som e causes o f false negative studies?

Lesions that are small (< 1 cm), or deep. Lesions located in the medial breast, and/or those
overlapping with heart activity.

What about lymph nodes?

You see lymph nodes on a “M IBI” scan - this is NOT norm al, it is concerning for mets.

558
S E C T I O N 1 1:
C a r d ia c

Myocardial Perfusion I SPECT:


Tc Sestamibi and Tc Tetrofosmin are the m ost com mon tracers. They w ork by crossing the cell
membrane and localizing in m itochondria (passive diffusion). They don’t redistribute (like
Thallium), giving better flexibility.

Sestamibi vi' Tetrofosmin - Tetrofosmin is cleared from the liver m ore rapidly and decreases the
chance o f a hepatic uptake artifact.

Thallium - This is historical with regard to cardiac imaging. It mimics potassium and crosses
the cell membrane first by distribution related to blood flow - second by delayed redistribution
(washout). W ashout is delayed in areas with poor perfusion.

Imaging Timing:

Tc studies (sestamibi and tetrofosm in) are done 30-90 mins after injection - allowing for
clearance from background

Thallium Sestam ibi and Tetrofosmin

Old N ew er

Crosses cell via N a/K pump Crosses cell via passive diffusion (localizes
in m itochondria)

Redistributes Does NOT redistribute

Imaging m ust be done im m ediately after Imaging typically done 30-90 mins after
injection injection to allow for background to clear

Lung/ H eart Ratio: Only done with Thallium. If there is more uptake in the lungs, this
correlates with m ulti-vessel disease or high grade LAD or LCX lesions.

General Principal: You will see less perfusion distal to an area o f vascular obstruction
(compared to normal myocardium). To improve sensitivity, the heart is stressed. Under stress
you need about 50% stenosis to see a defect (it needs to be like 90% w ithout stress).

559
Preparation: Patient shouldn’t eat for 4 hours prior to imaging (decreases GI blood flow). Patients
should (ideally) stop beta-blockers, calcium channel blockers, and long-acting nitrates for 24 hours
prior to the exam - as these meds mess with the sensitivity of the stress portion. There are reasons to
keep people on these meds (they might be getting risk stratification on medical therapy) - but I’d say
for the purpose of multiple choice just know that those medication classes mess with stress imaging
sensitivity.

Protocols: There are multiple ways to skin this particular cat. People will do two day exams; rest then
stress. People will do one day exam stress then rest. The advantage to doing stress first is that you can
stop if it’s normal. Typically the dosing is low for the rest and high for the stress.

Chemical Stress: If you can’t exercise, the modem trend is to give you Regadenoson (coronary
vasodilator) - which is a specific adenosine receptor agonist. It’s specific to a certain receptor having
less bronchospasm than conventional adenosine or dipyridamole. If they get bronchospasm anyway you
need to give them albuterol.

Known Left Bundle Branch Block: A known LBBB will make ECG stress testing non-diagnostic. So
diagnostic imaging (radionuclide myocardial perfusion) is typically the way to go. The important trivia
is:

LBBB Classic Artifact = False Positive Reversible Perfusion Defect at the Septum (anteroseptal
region). Supposedly this has something to do with the septum not relaxing correctly during diastolic
coronary filling (because the rhythm is not totally coordinated with the left sided block).

Pharmaceutic Choice = Adenosine or Dipyridamole is supposedly better for LBBB patients than
Dobutamine. The reason is Dobutamine increases the HR more, and the more rapid the HR, the worse
the septal relaxation stuff is. Dobutamine = More False Positives.

Findings:

Fixed D efect (seen on stress and rest) Scar (prior infarct)

Reversible D efect (seen on stress, better on Ischemia


rest)

Fixed D efect with Reversible D efect around it Infarct with peri-infarct ischem ia

Transient Ischemic Dilation From diffuse subendocardial hypoperfusion


(LV cavity is larger on stress) producing an apparent cavity dilation.
C orrelated w ith high risk disease (left m ain
or 3 vessel).

Fixed Cavity Dilation Dilated cardiom yopathy

Right Ventricular A ctivity on Rest I f has intensity sim ilar to LV then think
right ventricular hypertrophy

Lots o f splanchnic (liver and bowel) activity M eans you aren’t exercising hard enough -
not shifting enough blood out o f the gut.

560
THIS vs THAT: Stunned vs Hibernating Myocardium:
Stunned: This is the result of ischemia and reperfusion injury. It is an acute situation. The perfusion
will be normal, but contractility will be crap. It will get better after a few weeks.

Hibernating: This is a more chronic process, and the result of severe CAD causing chronic
hypoperfusion. You will have areas of decreased perfusion and decreased contractility even when
resting (just like scar). Don’t get it twisted, this is not an infarct. This tissue will take up FDG more
intensely than normal myocardium, and will also demonstrate redistribution of thallium. Perhaps
the easiest way to think about this is to imagine that the heart is trapped in another dimension and
possessed by a shadow demon - existing in a state of trans-dimensional living death.

Rapid Review
Ischemia = Will take up less tracer (relative to other areas) on stress, and the same amount of tracer
(relative to other areas) on rest. It’s not normal heart so it won’t contract well.
Scar = Won’t take up tracer on rest or stress (it’s dead Jim). It’s scar not muscle, so it won’t
contract normally either.
Stunned = The perfusion will be normal on both stress and rest, but the contractility is not normal.
Hibernating = Won’t take up tracer on rest or stress (it’s not dead, just asleep - like a bad soap
opera plot). The difference between hibernating muscle and scar is that the hibernating muscle will
take up FDG and redistribute thallium. The defect at rest will resolve / “redistribute” on delayed
thallium imaging. Remember thallium works with the Na/K pump - so cells need to be alive to
pump it in. A truly dead cell won’t have a functioning Na/K pump and therefore won’t be able to
redistribute / resolve the defect.

iVIUGA (Multigated Acquisition Scan):


This is an equilibrium radionucleotide angiogram with cardiac pool images taken after the tracer has
equilibrated to the intravascular space. Drugs like Adriamycin and Doxorubicin can be cardiac toxic.
Oncologists will alter treatment protocols when the LVEF drops (usually by 10%).

Huh? - It’s an angiogram using tagged RBCs. You time (gate) the exam to get pictures that can be used
to estimate the Ejection Fraction (and evaluate motion etc...)

These studies requires gating (the “G” in MUGA). The study is done using Tc 99 labeled RBCs, and
the objective is to calculate an EF (MUGA is more accurate than myocardial perfusion for LVEF).
Photopenic halo around the cardiac blood pool is a classic look for pericardial effusion. Regional wall
motion abnormality on a resting MUGA is usually infarct (could be stunned or hibernating as well).

The easiest way to ask a question about MUGA is also probably the most important practical pearl
(wow... I can’t believe I said that): _____________
Left Anterior Oblique
False Low EF: Screwed up LAO view can cause overlap of LV
(LAO) is the “best septal
with LA or RV or even great vessels - causing a false low EF. view” - and the one usually
Inclusion of the Left Atrium (which happens when it is enlarged) used to measure the LVEF.
the result of LA inclusion is inclusion of the LA counts — this A basic internal QA step
falsely lowers the EF. when reading these studies
is to confirm a good
False High EF: Wrong background ROI (over the spleen), will photopenic septum.
cause over subtraction of background and elevate the EF.

561
Misc Trivia:
R ubidium 82: This is a potassium analog (mechanism is N a/K pump). This is sim ilar to TI-201,
and can be used as a sim ilar agent. You can use it for PET myocardial perfusion, although it’s
not used in m ost places because o f cost limitations. Also, because o f the very short h alf life ( 75
seconds) it tends to give a dirtier image com pared to PET o f NH3.

I s a y m ade with a g e n e ra to r, y o u say Tc99 a n d R ubidium . *Rubidium is the only PET agent
made like this, so that instantly makes it a testable fact.

S h o rt Axis A n ato m y R eview - 1 always fo u n d this confusing:

“Anterior”

“Lateral”

“Septal”

“Inferior”

562
A rtifa c ts

Decreased activity in Check for ECG changes and


Breast Tissue anterior w all (m ay also wall motion. I f norm al, then
“Soft Tissue A ttenuation” affect septal and lateral - call it artifact. I f not sure
depending on body habitus) can repeat in prone position.

Check for ECG changes and


Left H emidiaphragm Decreased activity in
wall motion. I f normal, then
“Soft Tissue A ttenuation” inferior wall
call it artifact.

Increased activity in inferior


Liver Excretes Tc, so you
wall, can m ask true defect.
see it in the liver and bowel.
Subdiaphragm atic Can also mess with
Little bit o f exercise can be
Radiotracer A ctivity “norm alization” o f the
used to reduce GI blood
ventricle and m ake the rest
flow.
o f the LV look low.

You can repeat because


Patient M otion
Causes all kinds o f problem s tracer is fixed for around 2
(usually respiration)
hours

M isregistration Causes all kinds o f problem s

Seen more in exercise or


R eversible or Fixed Septal
Left Bundle Branch Block dobutam ine stress com pared
Defects , sparing the apex
to vasodilators

Look for m atching stress


and rest perfusion patterns
Norm al Apical Thinning N orm al variant w ith preserved w all function
to show you this is normal
(and not infarct)

563
Medication Trivia:

Mechanism Trivia

Inhibits the breakdow n o f


adenosine - which builds up.
Dipyridam ole N o caffeine
Adenosine is a potent
vasodilator.

N o caffeine

Side effects are worse than


with dipyridamole. Rare
Adenosine Vasodilator
side effect is AV block -
w hich will get better when
adenosine short h alf life
runs out.

Selective A 2 A - causes fewer


Regadenoson No caffeine
side effects

Patient Can NOT be on a


beta blocker.

Best used in patients who


Beta 1 agonist - acts like
cannot have Adenosine or
exercise by increasing heart
Dobutam ine D ipyridam ole. B etter in
rate and m yocardial
patients with COPD or
contraction.
Asthma, or who have taken
caffeine in the last 12 hours.

Avoid with LBBB

H alf life is shorter than


Am inophylline Antidote for A denosine D ipyridam ole - so must
continue to monitor.

564
SECTION 12:
Therapy

Treatment for Bone Pain:


There are currently three approved agents for bone pain associated w ith m etastatic disease
from breast and prostate cancer: (1) Sr^^.chloride, (2) Sm'53 EDTMP, and (3) Ra223-dichloride.

Why does cancer cause bone pain / bone problem s?

M etastatic disease leads to a tum or derived factor that increases osteolytic activity. You end up
with increased fracture risk, osteopenia, and hypercalcem ia o f malignancy.

Can the patient g et external radiation treatment with the therapy?

Yes, External radiation is not a contraindication and can be used w ith the therapy.

Absolute contraindications (for Sr a n d Sm) include: Pregnancy, Breastfeeding, and renal


Failure (GFR < 30). Patients with extensive bony mets (superscan) maybe sh o u ld n ’t be treated
either * controversial - a n d therefore not likely tested.

Strontium - (Metastron): It works by com plexing w ith the hydroxyapatite in areas where
bone turnover is the highest. It’s the oldest, and w orst o f the three agents. It is a pure beta
emitter. It has a high myelotoxicitiy, relative to new er agents and therefore isn’t really used.

Samarium - Smis3 (Quadram et): This is probably the second best o f the three agents;
“Samarium is a g o o d Samaritan. ” It works by com plexing w ith the hydroxyapatite in areas
where bone turnover is the highest. It is a beta decayer. The prim ary m ethod o f excretion is
renal. Unhke Sr89, about 28% o f the decay is via gam m a rays (103 kev) which can be used for
imaging. Does have some transient bone m arrow suppression (mainly throm bocytopenia and
leukopenia), but recovers faster than Sr.

S r8 9 (Metastron): S m i5 3 (Quadramet)

15-30% drops in platelet and W BC from pre­ 40-50%) drops in platelet and W BC from pre­
injection injection

8-12 weeks needed for full recovery 6-8 weeks needed for full recovery

565
Radium - Ra^23 (Xofigo): This is the m ost recent o f the three agents, and probably the best.
The idea is that Ra223 behaves in a sim ilar w ay to calcium. It is absorbed into the bone matrix
at the sites o f active bone mineralization. Its prim ary m echanism is the emission o f 4 alpha
particles, causing some serious double stranded DNA breaks.

Why is it the best?

(1) It’s an alpha em itter with a range shorter than Sr and Sm. This means less hem atologic
toxicity.

(2) A t least one trial actually showed a survival benefit in prostate CA.

(3) It has a long h alf life (11.4 days), allow ing for easy shipping.

What are the side effects?

Non-hematologic toxicities are generally more com m on than hem atologic ones; diarrhea,
fatigue, nausea, vomiting, and bone pain m ake the list.

Trivia: The general population is safe, as the gam m a effects are low. Soiled clothing and
bodily fluids should be handled with gloves, and clothes should be laundered separately. A 6
month period o f contraception is recom m ended although none o f this is evidence based (as per
usual).

Sr89 Smi53 Ra 223

Beta Emitter, w ith some


Pure Beta Em itter A lpha Em itter
im ageable gam m a rays

M ost Bone M arrow Toxicity


Less Bone M arrow Toxicity Least Bone M arrow Toxicity
(longest recovery).

Renal excretion Renal excretion GI excretion

Improves Survival (prostate


mets)

566
Yttrium-90
This can be used as a radioembolization method for unresectable liver tumors. This is a pure Beta
emitter that spares most of the adjacent normal liver parenchyma (as the maximum tissue penetration is
about 10 mm).

Prior to treatment with Y-90 the standard is to do a 99mTc MAA hepatic arterial injection. The primary
purpose of this injection is to look for a lung shunt fraction. This fraction needs to be < 10% under
ideal circumstances. You can still use Y-90 for 10-20% shunts but you need to decrease the dose.
Above 20% the risk of radiation pneumonitis is too large.

Particle Size: The optimal particle size is between 20-40 um, as this allows particles to trap in the
tumor nodules, but large enough to get stuck without totally obstructing. If you create a true
embolization the process actually doesn’t work as well because you need blood flow as a free radical
generation source.

Radiation Dose: The dose is typically 100-1000 Gy delivered. The current thinking is that lesions
require at least 70 Gy for monotherapy success.

Imaging: There are 175 Kev and 185 keV emissions you can use to image.

Trivia: The average half life is 2.67 days.

Radioimmune Therapy (RIT):


Monoclonal antibodies can be used with Indium"' , such as ibritumomab tiuxetan (Zevalin) for
refractory non-Hodgkin lymphoma treatment or as a first line treatment.

The idea is that you can give the antibody labeled with Indium'" for diagnostic evaluation of the tumor
burden and then if the biodistribution is ok you can give the antibody labeled with Y-90 for treatment.

Trivia: The antibody binds to the CD-20 receptors on B-cells.

What is considered altered distribution?

(1) Uptake in the lungs is more intense than the heart day one, or more intense than liver on day 2 & 3.

(2) Uptake in the kidneys more than the liver on day 3.

(3) Uptake in the bowel that is fixed, and/or more than the liver

(4) Uptake in the bone marrow > 25%

Trivia: Don’t give to patients with platelets less than lOOK

Most Common Side Effect? Thrombocytopenia and neutropenia (about 90% of cases).

Can you send them home post treatment? Dose to caretakers or persons near the patient is low, and they
can be released to the general population after treatment. Although some things like sleeping apart, no
kissing, etc... for about a week are still usually handed out.

Protocol: You need to first give rituximab to block the CD20 receptors on the circulating B cells and
those in the spleen to optimize biodistribution. Then you can give the In "' labeled antibody to assess for
altered biodistribution. If you suspect altered distribution you should get delayed full body imaging at
90-120 hours. If altered you shouldn’t treat. If ok, then blast’em.

567
S E C T IO N 13:
H igh Y ield

THIS v^THAT:- C ritic a l O rg a n VS T a r g e t O rg a n :


“Critical Organ”- A n organ that limits the dose of the radiopharmaceutical due to the increased
susceptibility of the critical organ for cancer. This may or may not be the “Target Organ.”

“Target Organ”- This is the organ you want the


tracer to accumulate in. It’s your organ of Why all the Controversy ?
interest.
You will read conflicting information for many
A few generalizations on Critical Organs: If of these. Apparently, there is a fair amount of
you are trying to figure it out, it’s the organ that bickering among PhDs on how / when to
the tracer is going to spend the most time in. calculate the dose to come up with these
For example, Gallium ends up in the bowel. Tc numbers.
RBC scans are going to end up passing through
To know which organ to pick on the test simply
the heart a lot, unless they are heat treated then
read the mind of the person who wrote the
the spleen eats them. Tc-MAG3s and DTPA is
question.
going to be the bladder, but DMSA (which
sticks to the kidney) is going to be the kidney.

Liver:
-In-ProstaScint
-M31 m IBG
-Sulfur Colloid (IV)

Gallbladder Wall:
. H ID A

Renal Cortex:
• T hallium
• DMSA

Proximal Colon
• Sulfur C olloid (oral)
• S estam ibi

Distal Colon:
Bladder: • G alliu m
• MAGS
. M23 m i b g
• M D P *some so u rce s
s a y bone

568
Tracer Analog Energy Physical Half Life

Tc - 99m “Low ” - 140 6 hours

Iodine -123 Iodine “Low ” - 159 13 hours

125 hours
Xenon - 133 “Low ” - 8 1
(biologic tl/2 30 seconds)

“Low ” - 135 (2%), 167


Thallium - 201 Potassium (8%), use 71 ^0‘H g 73 hours
daughter x-rays

“M edium ” - 173 (89%),


Indium -111 67 hours
247 (94%)

M ultiple; 93 (40% ), 184


Gallium - 67 Iron (20%), 300 (20% ), 393 78 hours
(5%)

Iodine -131 Iodine “H igh” - 365 8 days

Fluorine -18 Sugar “H igh” -511 110 mins

Cardiac Radionuclides
Treatment Radionuclides Half Life
Half Life

50.5 DAYS
Strontium 89 Rubidium 82 75 seconds
(14 days in bone)

Samarium 153 46 Hours N itrogen 13 10 mins

Radium - 223 11 Days

Yttrium 90 64 Hours

569
Blank on Purpose (for scribbles and notes)

570
Blank on Purpose (for scribbles and notes)

571
Made in the U SA
Colum bia, SC
04 February 2022

55320885R00311

55320885R00311

You might also like